Gen Math Q2

You might also like

Download as pdf or txt
Download as pdf or txt
You are on page 1of 519

General Mathematics

Quarter 2 – Module 1:
Simple and Compound
Interests

CO_Q2_General Mathematics SHS


Module 1
General Mathematics – Senior High School
Alternative Delivery Mode
Quarter 2 – Module 1: Simple and Compound Interests
First Edition, 2021

Republic Act 8293, section 176 states that: No copyright shall subsist in any
work of the Government of the Philippines. However, prior approval of the government
agency or office wherein the work is created shall be necessary for exploitation of such
work for profit. Such agency or office may, among other things, impose as a condition
the payment of royalties.

Borrowed materials (i.e., songs, stories, poems, pictures, photos, brand names,
trademarks, etc.) included in this module are owned by their respective copyright holders.
Every effort has been exerted to locate and seek permission to use these materials from
their respective copyright owners. The publisher and authors do not represent nor claim
ownership over them.

Published by the Department of Education


Secretary: Leonor Magtolis Briones
Undersecretary: Diosdado M. San Antonio

Development Team of the Module


Writers: Mary Grace D. Constantino and Raiza Ann E. Lipardo
Editors: Elizabeth D. Lalunio, Elizabeth B. Dizon, Anicia J. Villaruel and
Roy O. Natividad
Reviewers: Jerry Punongbayan, Diosmar O. Fernandez, Dexter M. Valle,
Jerome A. Chavez, Angelo S. Villanueva and Michelle R. Navarro
Illustrator: Hanna Lorraine Luna
Layout Artist: Roy O. Natividad, Sayre M. Dialola, Noel Rey T. Estuita and Argie L. Ty
Management Team: Francis Cesar B. Bringas
Job S. Zape Jr,
Ramonito Elumbaring
Reicon C. Condes
Fe M. Ong-ongowan
Hermogenes M. Panganiban
Phillip B. Gallendez
Josephine T. Natividad
Anicia J. Villaruel
Dexter M. Valle

Printed in the Philippines by ________________________

Department of Education – Region 4A CALABARZON

Office Address: Gate 2 Karangalan Village, Brgy. San Isidro Cainta, Rizal
Telefax: 02-8682-5773/8684-4914/8647-7487
E-mail Address: lrmd.calarbazon@deped.gov.ph
General Mathematics
Quarter 2 – Module 1:
Simple and Compound
Interests
Introductory Message

This Self-Learning Module (SLM) is prepared so that you, our dear learners,
can continue your studies and learn while at home. Activities, questions,
directions, exercises, and discussions are carefully stated for you to understand
each lesson.

Each SLM is composed of different parts. Each part shall guide you step-by-
step as you discover and understand the lesson prepared for you.

Pre-tests are provided to measure your prior knowledge on lessons in each


SLM. This will tell you if you need to proceed on completing this module or if you
need to ask your facilitator or your teacher’s assistance for better understanding of
the lesson. At the end of each module, you need to answer the post-test to self-
check your learning. Answer keys are provided for each activity and test. We trust
that you will be honest in using these.

In addition to the material in the main text, Notes to the Teacher are also
provided to our facilitators and parents for strategies and reminders on how they
can best help you on your home-based learning.

Please use this module with care. Do not put unnecessary marks on any
part of this SLM. Use a separate sheet of paper in answering the exercises and tests.
And read the instructions carefully before performing each task.

If you have any questions in using this SLM or any difficulty in answering
the tasks in this module, do not hesitate to consult your teacher or facilitator.

Thank you.
What I Need to Know

Hello my dear learners!


Are you familiar with the cliché that says, “Nothing is permanent except change”? A
big change happens to all of us. No one expected this, but we have to live with it.
Instead of taking it negatively let’s do our part to achieve a positive result.
Have you met the subject matters about interest, loan, savings, finances, expenses,
and investment when you were in Junior High or even in Elementary? Have you
heard your parents, relatives, classmates, teachers, friends, or even your neighbor
talking about these matters? You can actually encounter those not only in school
but anywhere you go like your home, market, malls, banks, and other public places.
So here we go to the next module about Math of Investment. I hope you learned the
previous module for logical sequence.
This lesson helps you understand thoroughly what is simple and compound
interest. It will let you decide correctly on how and where to save money. Nowadays
savings is very important, it is like saying “If seems that if you do not have savings
you are out or you don’t belong” and it will then make you analyse whether to save
in simple or compound on your favor, as a borrower or an investor.

Are you ready now? Let’s begin working on this module. Open your heart and mind
and have focus. Good Luck!

The module is composed of two lessons namely:


 Lesson 1 – Simple Interest
 Lesson 2 – Compound Interest
After going through this module, you are expected to:
1. illustrate simple and compound interests; and
2. distinguish between simple and compound interests.

1
CO_Q2_General Mathematics_SHS
Module 1
What I Know

Dear students, I know that this is a new lesson for you I need you to be patient in
answering this to test what you already know about the lesson. Good luck!
Choose the letter of the best answer. Write the chosen letter on a separate sheet of
paper.
1. What interest remains constant throughout the investment term?
a. simple c. annuity due
b. compound d. ordinary annuity

2. It is an interest computed based on the principal amount.


a. simple c. annuity due
b. compound d. ordinary annuity

3. What is the difference between simple and compound interest?


a. Simple yields higher interest than compound interest.
b. Simple interest has a shorter term than compound interest.
c. Simple interest is always better than compound interest.
d. Simple interest is computed based on the principal while
compound interest is computed based on the principal and also
on the accumulated past interests.
4. If you would like to invest money, which bank offer would you prefer if
you do not plan to withdraw your money in 2 years?
a. 5% simple interest per annum
b. 4% compounded interest per annum
c. 3% compounded interest semi-annually
d. 2% compounded interest quarterly
5. Which of the following statement is true about the borrower or debtor?
a. It is the amount of money borrowed or invested on the origin date.
b. It is the interest is computed on the principal and also on the
accumulated past interests
c. It refers to the person (or institution) who owes the money or avails of
the fund from the lender.
d. It refers to the person (or institution) who invests the money or makes
the funds available.

6. Which of the following statements is/are true?


I. Compound interest of a loan favors the borrower.
II. Simple interest remains constant throughout the investment term.
III. In compound interest, the interest from the previous year also earns
interest.
a. I only c. II and III
b. I and II d. I and III

2
CO_Q2_General Mathematics_SHS
Module 1
7. Which of the following formula can be used to solve for the simple interest?
a. I = Prt c. A= P(1 + rt)
Prt
b. SI = 100
d. All of the above

8. It is an amount after t years that the lender receives from the borrower on
the maturity date.
a. loan date c. maturity value
b. maturity date d. term

9. Which of the following describes time or term?


a. It is the date on which money is received by the borrower.
b. It is the amount paid or learned for the use of money.
c. It is the date of which the money borrowed or loan is to be completely
repaid
d. It is the amount of time in years the money is borrowed or invested;
length of time between the origin and maturity dates

10. In the formula, I= Prt, what is r?


a. revenue c. repaid
b. real value d. rate of interest

11. It is the amount of money borrowed or invested on the origin date.


a. future value c. maturity value
b. principal value d. repayment value

12. A person (or institution) who invests the money or makes the funds available.
a. Lender c. Both a and b
b. Creditor d. None of the choices.

13. It refers to an interest that is computed based on the principal and interest
accumulated every conversion period.
a. simple c. annuity due
b. compound d. ordinary annuity

14. It refers to the amount after � years that the lender receives from the borrower
on the maturity date?
a. present value c. interest
b. future value d. ordinary annuity

15.Which of the following formula can be used to solve for the compound interest?
r
a. A = P(1 + n )t c. C = P(1 + r)n − 1
r nt
b. A = P(1 + n
) d. Both a and b

3
CO_Q2_General Mathematics_SHS
Module 1
Lesson

1 Simple Interest

It’s quite difficult to pursue our dreams, especially during these trying times that
our country is experiencing the COVID-19 pandemic. Dream is just a dream until
you decide to make it happen. We can realize our dreams through perseverance,
patience, and determination. For now, continue to acquire new knowledge, develop
your skills, and cultivate your talents because these will be your weapon to succeed
in life.
I hope that you somehow encountered some math of investment terms like simple
interest, loans, savings, investments, maturity value, money, resources, and the
like. This lesson will help you understand simple interest. Different terminologies
about the simple interest that you can use for the succeeding lesson once you go
deeper on the problem solving about simple interest.

What’s In

Match Me! Reveal Me!


Match the terminologies in column B to its definition or statement in column A.
Write your answer on the blanks provided below the trivia question inside the box.
Your answer should reveal an important Filipino value that everyone should
possess. Finally, answer the guided questions that follow.

A B
1. It is the amount of time in years the money is S. Creditor
borrowed or invested.
2. It is the date on which the total amount borrowed R. Time or Term
with interest is to be completely repaid.
3. It refers to the person or institution that invests C. Origin/Loan Date
the money or makes the funds available.
4. It is the amount of money borrowed or invested on E. Principal
the origin date.
5. It is the amount after t years that the lender P. Maturity date
receives from the borrower on the maturity date.
6. It is the date on which money is received by the T. Maturity/Future
borrower. Value

4
CO_Q2_General Mathematics_SHS
Module 1
What Filipino value is shown when a person giving the greeting bows down
towards another person older that him or taking his hand or her hand and
pressing it on the forehead? This is an important Filipino value that the
younger generations should never forget and performed with the right hand
and saying "Mano po" to the elders.

_____ _____ _____ _____ _____ _____ _____


1 4 3 2 4 6 5

Guided Questions:

1. How will you define interest in your own words?


.

2. Based on the definition given will you able to know some important terms in
simple business math? If yes try to elaborate just one term.
.

3. Are all the terms given, important in finding simple interest? Justify your
answer. .

This module introduces a new chapter in General Mathematics. Specifically it


introduces Math of Investment others call it Business Mathematics but either
way it still includes finances, money, credit, investments, banking policies, and
other topics that are related to finances. I know that some of you have
encountered the words or terminologies mentioned above that would be a big
help to start.

Notes to the Teacher


You may suggest to the learners to use their gadgets at home and be
resourceful to search more information and different terms use for
math of investment and Business mathematics. Ask them what they
expect to learn in this subject since it is new to them. This will give you
an idea on how to develop your lesson.

5
CO_Q2_General Mathematics_SHS
Module 1
What’s New

What Am I!
Read and analyze! Choose your answers among the principal, simple interest,
interest rate and term. Write your answer on the blank provided for each number.
1. I am the _____________________ which is calculated by multiplying the
principal, the rate of interest and the time in years.
2. I am the _____________________, and simple interest is computed based on me
and the time in years.
3. I am the _______________________, usually I am in percent and also called as
the rate of increase of interest.
4. I am the _______________________, and I am the length of time between the
origin and maturity dates.

Simply Saving!

Using the idea that you learned in the What I am? activity, try to answer the
problem below. If you find difficulty solving the problem, that is fine. Continue
reading this module and soon you will find the appropriate solution.

A working student in one of the biggest fast-food restaurants in Lucena City wants
to save for the upcoming school year. He wants to deposit his money into a Filipino
owned bank so that even in a simple way he can help his fellow Filipino. Supposed
his monthly salary is ₱10,000.00 and it was deposited to an account that earns a
simple interest of 2.75% per annum. Find the simple interest after 6 months, one
year, and 18 months.

What is It

The Name Me activity previously summarizes the definition of the following terms:

Simple Interest ( �� ) – interest that is computed on the principal. The interest


remains constant throughout the term.

Lender or creditor – person (or institution) who invests the money or makes the
funds available

6
CO_Q2_General Mathematics_SHS
Module 1
Borrower or debtor – person (or institution) who owes the money or avails of the
funds from the lender

Origin or loan date – date on which money is received by the borrower


Repayment date or maturity date – date on which the money borrowed or loan
is to be completely repaid

Time or term (t) – amount of time in years the money is borrowed or invested;
length of time between the origin and maturity dates
Principal (P) – amount of money borrowed or invested on the origin date
Rate(r) – annual rate, usually in percent, charged by the lender, or rate of
increase of the investment
Interest (I) – amount paid or earned for the use of money
Maturity value or future value (F) –amount after t years that the lender
receives from the borrower on the maturity date
To solve the problem in the Simply Saving activity which is a common scenario
among Filipino working students wherein many can relate.

You can solve this problem using the simple interest formula
�� = ���
where:
Is = Simple Interest
P = Principal or amount invested or borrowed
r = simple interest rate
t = term of time in years
Here are the steps to find the simple interest:
Step 1: Identify the given and the unknown
P = ₱10,000.00
r = 2.75% or 0.0275
6 18
t =0.5 or 12
(6 months), 1 (1 year), 1.5 or 12
(18 months)
Is = ?
Step 2: Substitute the given to the formula
Is = Prt

For 6 months For 1 year


Is = (₱10,000.00) (0275) (0.5) Is = (₱10,000.00) (0275) (1)
= ₱137.50 = ₱275.00

For 18 months
Is = (₱10,000.00) (0275) (1.5)
= ₱412.50

7
CO_Q2_General Mathematics_SHS
Module 1
Notice that the time is divided into 12 since there are 12 months in a year. You will
also do the same if the given is in days, the divisor will be 360 for ordinary interest
or 365 if you are looking for exact interest. Don’t worry because it will be
indicated in the problem if you will compute for the ordinary or exact interests but
if not indicated always use the ordinary interest which consists of 360 days. In case
that the given time is in years then multiply it as is.
Now that you have the idea of how to solve simple interest study the example below.
Example
Problem Solving: Due to COVID-19 pandemic Miss Dada a female resident of Brgy.
May Pagkakaisa somewhere in Quezon Province thinks of a business that can
provide for her needs as well as the need of her neighbors so she can be of help
even in this trying time.
Since she doesn’t have money on hand, she decided to borrow from a bank as the
start-up capital of ₱50,000.00 at 7% simple interest rate payable within 5 years.
Compute for the interest yield.
Solution.
Simple Interest

Time Principal Interest Simple Interest Amount after (t) years


(t) (P) Rate (r) Solution Answer (Maturity Value)
50,000 + 3,500 =
1 50,000 7% (50,000)(0.07)(1) 3,500
53,500
50,000 7% (50,000)(0.07)(2) 50,000 + 7,000 =
2 7,000
57,000
50,000 7% (50,000)(0.07)(3) 50,000 + 10,500 =
3 10,500
60,500
50,000 7% (50,000)(0.07)(4) 50,000 + 14,000 =
4 14,000
64,000
50,000 7% (50,000)(0.07)(5) 50,000 + 17,500 =
5 17,500
67,500

Notice the simple interest remains constant throughout the year and you only
multiply with the term that you are computing. Therefore, if you are a borrower
make sure that the interest on your loan is not too high. Try to compute it first
before you apply for a loan. Compare first the interest and the term of the different
lending institutions before you decide to consider a loan or investment.

8
CO_Q2_General Mathematics_SHS
Module 1
What is It

Activity 1.1
Write the word TRUE if the statement is correct, otherwise write FALSE on a
separate answer sheet. If your answer is FALSE write the word or phrase that will
make the statement correct.

1. Simple interest changes throughout the investment term.


2. Simple interest computation will always be based on the original principal.
3. Interest is the amount of money invested or borrowed originally.
4. Simple interest is the product of the principal, rate of interest, and time.
5. In ordinary interest, the interest is computed based on 365 days.

Activity 1.2
Match the corresponding term/formula in column B with its definition in column A.
Write letter only on the blank provided for each number.
A B
___1. It is the amount that is charged a. �� = ���
for the privilege of borrowing
money.
___2. It refers to a person or institution b. Principal
who owes money.
___3. It is the amount of money c. � = �(1 + �)�
originally invested or borrowed.
___4. It is the formula for computing d. Lender/Creditor
simple interest.
___5. It refers to the person/institution e. Interest
who made the funds available.
f. Borrower/Debtor
Activity 1.3
Solve the following problems.
A couple with three children are peacefully living in a barangay near the City
proper. The husband is a government employee while the wife is a rug sewer. They
always remind their children about this: “Bilhin and kailangan at huwag
kailanganin ang hindi kayang bilhin”. Because of this teaching, the children open a
joint account to a certain cooperative to have a better investment. The total amount
they earned in selling rugs, was put in their account in the cooperative to help their
parents. Their total earnings amounting to ₱ 5,000.00 will earn an interest rate of
7.5% per year. Help them to compute for the simple interest earned and maturity
value, if their money will be invested in 3 years.

9
CO_Q2_General Mathematics_SHS
Module 1
What I Have Learned

A. Please read the sentences carefully and fill in the missing word/s by writing your
answer on the blanks provided.
1. Some who have the privilege of using a fund and promise to pay at some future
date is called ________________________________.
2. The amount that will be received at maturity date is the_____________________.
3. Simple interest is the interest computed on the the ______________________.

4. The date on which the loan amount is to be fully paid is called _____________.
5. ___________________is the amount calculated on the initial principal and with
fixed interest throughout the periods or term.

B. Give the formula to find the simple interest.

What I Can Do

Read and analyze the situation below then answers the question given.

A typhoon severely damaged a certain place in Quezon Province. The wealthiest


family in that area who happened to have large businesses have thought of a way
to help SME’s or the small business enterprises to recover, as well as those poor
families affected by the said typhoon. The family then decided to allot funds to lend
to those who are victims of calamity by giving a simple interest loan for only 10%
per annum. If the family was able to lend ₱1,500,000.00, how much interest would
be earned after 5 years? Use a model table discussed in this lesson to show the
simple interest earned. What do you think about Filipino values being shown in
this situation? Can you think of other means to help the community use simple
interest?

10
CO_Q2_General Mathematics_SHS
Module 1
Your output will be graded, according to this rubric.
Criteria 4 3 2 1
Comprehension Identify, analyze Identify, analyze Identify, Identify, analyze
and understand and understand analyze and and understand
all of the main most of the understand few of the main
issues in the main issues in some of the issues in the
study the study main issues in problem solving
the study
Accuracy of the With a complete With the correct With some data Incomplete and
solution and accurate answer but the missing to incorrect
solution. incomplete complete the solution.
solution solution,
Analysis Insightful and Thorough Seeming Incomplete
thorough analysis and analysis and analysis and
analysis and exploration of exploration of exploration of the
exploration of the questions the questions questions asked
the questions asked asked
asked
Filipino Values With at least With two With one Without Filipino
three Filipino Filipino values Filipino value value that related
values which are which are which is to the problem
significant and significant and significant and
related to the related to the related to the
problem problem problem

11
CO_Q2_General Mathematics_SHS
Module 1
Lesson

2 Compound Interest

This lesson will lead you to understand another type of earning interest, the
compound interest. If you understood the simple interest in the previous lesson,
this lesson will give you knowledge of how simple interest differs from compound
interest. This will lead you to compare your investment in the future from different
options. It will also help you to make wise decisions if you will apply for loans from
a bank to start your own business or if you need funds for emergency purposes.
Being financially literate is also a skill that will help you to become successful in
handling money and eventually lead you to a good life.

What’s In

Match Me! Reveal Me, for the second time!

Match the terminologies in column B to its definition or statement in column A.


Write your answer on the blanks provided below the trivia question inside the box.
Your answer should reveal one of the products that we should be proud of as a
Filipino. Finally, answer the guided questions that follow.

A B
1. It is the amount paid or earned for the use of O. Borrower/Debtor
money.
2. It refers to the person or institution who N. Compound Interest
owes money or avails of the funds from the
lender.
3. It is the interest computed on the principal T. Interest
and also on the accumulated past interest
4. It is the percentage of the principal that will C. rate of interest
be changed for a specified period of time.
5. It refers to maturity value. U. Future Value

12
CO_Q2_General Mathematics_SHS
Module 1
This is one of the fruit bearing trees found in the Philippines. We are also
second to the highest exporter of this product all over the world. It is also
traditionally called “the tree of life” because all the parts of this tree has
varied uses, from housing materials, novelty items up to food products.

_____ _____ _____ _____ _____ _____ _____


4 2 4 2 3 5 1

Guide Questions:

1. How will define compound interest?

2. Based on the definition given will able to know some important terms in
compound interest? If yes. name those terms.

The activity gives you a glimpse of what the lesson is all about. It is all about
compound interest and what makes this interest differ from the simple interest

What’s New

Be hospitable and helpful, please!


Read and analyze each item and write the word GENEROUS if your answer is true
and write HELPFUL if otherwise. These two words will remind you of Filipino
values, to be kind and generous even in the most difficult situation if you see
someone on difficult (false) situation.

1. Simple interest is computed using the formula Is = Prt.


2. Compound interest is computed on the principal and also on the accumulated
past interests.
3. An amount of ₱10,000.00 will yield more interest if it is invested in a bank that
offers a simple interest of 10% annually than a bank that offers a 10% compound
interest annually if it will be invested for three years.
4. Compound interest works best over a short period of time.
5. A borrower or debtor prefers compound interest rather than simple interest.
6. In compound interest, the interest from the previous years also earns interest.
7. It is better to deposit in a bank that offers simple interest than a bank that offers
compound interest.

13
CO_Q2_General Mathematics_SHS
Module 1
Are you confident about your answer? Are you generous enough to share your
answer and help your classmates to answer each item correctly? Or do you think
you still need help from your classmates? If you are still in doubt, don’t worry, the
next discussion will give your ideas of the correct answer.

What is It

Compound interest (�� ) is the interest computed on the principal and also on the
accumulated past interest, so compound interest is a way to earn money because
you don’t just earn using your original money, but also the interest you earned.

To give you a deeper concept of compound interest, reflect on the following


questions: Have you ever lend money to someone like a friend, sibling, or relative?
If so, would you let them pay more than, less than or just equal the amount that
you lent to them? Since you consider to help them, probably you will answer just
an equal amount is ok even though your money has been used for a period of time.
But, I’m sure some will answer that you should receive more than the amount they
borrowed and no one will say that you should receive less than the amount they
borrowed. I know you have your own reasons but let us see if compound interest
will change your view in life regarding loans or borrowings, savings, and investment.

Are you familiar with credit cards? We have what we call “Perma-Debt” which
means a continuous outstanding balance of a credit card where they pay the
monthly minimum that fits in their budget to lessen the burden of interest monthly
but tries to add some debt again on the following month so the debts never end. So
why did I tell you this? If you are a debtor, compound interest is not good for you.
Better yet pay your debt in full the soonest possible so that the burden of interest
will not be on your shoulder. Conversely, if you are an investor, compound interest
is your best buddy and it is better to invest in a long period of time for you to have
a greater return of your investment through interest earned. So, if you have the
means or a way to save and invest early, you must consider it as soon as possible
for you to gain more money in the future.
Now, that you already know how to solve simple interest. Study the example below
and compare this to the example given in Lesson 1. (Note: Same problem was given
here to compare the interest earned in simple and compound interests)
Example
Problem Solving: Due to COVID-19 pandemic Miss Dada a female resident of Brgy.
May Pagkakaisa somewhere in Quezon Province thinks of a business that can
provide for her needs as well as the need of her neighbors so she can be of help
even in this trying time.

14
CO_Q2_General Mathematics_SHS
Module 1
Since she doesn’t have money on hand, she decided to borrow from a bank with a
start-up capital of ₱50,000.00 at 7% interest rate compounded annually and
payable within 5 years. Compute for the interest yield.
Solution.
Compound Interest

Amount at Compound Interest Amount at the end of


Time Interest
the start of year t (Maturity
(t) Rate (r) Solution Answer
year t Value)
50,000 + 3,500 =
1 50,000 7% (50,000)(0.07)(1) 3,500
53,500
53,500 7% (53,500)(0.07)(1) 53,500 + 3,745 =
2 3,745
57,245
57,245 7% (57,245)(0.07)(1) 57,245 + 4,007.15 =
3 4,007.15
61,252.15
61,252.15 7% (61,252.15)(0.0 61,252.15 + 4,827.65
4 4,827.65
7)(1) = 66,079.80
66,079.80 7% (66,079.80)(0.0 66,079.80 + 4,625.59
5 4625.59
7)(1) = 70,705.39

Compound interest includes the interest from the current year and added on the
principal at the start of the following year, which means that the previous interest
earns interest as well, together with the principal until fully paid. So the interest
yielded on simple interest is lower than the compound interest. The amount at the
end of 5 years in simple interest is ₱67,500.00 while in compound interest it is
₱70,705.39. Therefore, if you are a borrower make sure that the interest on your
loan is not too high if you will find a lender that offers simple interest better grab it
that a lender who offers compound interest. On the other hand, if you are an
investor learn to invest your money to an interest that will yield higher returns like
compound interest. If you will be a lender in the future, I hope that you will not be
abusive regarding the interest, be reasonable, and act generously by helping others
who needed you the most.
Notice that the formula to find the future value in a compound interest is given by
� = �(1 + �)�
where:
� = future value
� = principal amount
� = compound interest rate
� = time or time in years
Also, to find the compound interest just deduct the principal (P) from the computed
future value (F). In the next module, you will encounter a situation where interest
will be compounded more than once a year.

15
CO_Q2_General Mathematics_SHS
Module 1
What’s More

Activity 2.1
Write the word TRUE if the statement is correct; otherwise write FALSE on a
separate answer sheet. If your answer is FALSE, write the word or phrase that will
make the statement correct.

1. Perma-debt decreases the amount of money available on hand to spend in the


future.
2. Compound interest works well if you save early for retirement or invest early.
3. The longer it takes for you to pay a debt the smaller the interest you pay.
4. If borrowers pay at least the minimum payment every month of their debt, their
must not be of good standing.
5. If you are planning to invest, compound interest is better than the simple
interest.

Activity 2.2

Your father asked you about investment and wanted to know the interest that will
be earned if he will invest ₱500,000.00 in a certain bank that offers an annual
compounding interest of 8% for 5 years.

Complete the table below to help your father.


Amount at Compound Interest
Interest Amount at the end of
Time (t) the start of
Rate (r) Solution Answer year t (Maturity Value)
year t
1 500,000 8% (500,000)(0.08)(1)
2 540,000 8% 43,200
8% (583,200)(0.08)(1) 583,200 + 46,656 =
3
629,856
8% 629,856 + 50,388.48 =
4 50,388.48
680,244.48
5 680,244.48 8% 54,419.56

16
CO_Q2_General Mathematics_SHS
Module 1
What I Have Learned

A. Please read the sentences carefully and fill in the missing word/s by writing your
answer on the line/s provided.
1. The amount calculated on the principal and accumulated interest from previous
periods on a deposit or loan is ___________________________.
2. Compound interest yields _______________ amount than simple interest.
3. If you are an investor, it is better to invest your money in ____________________.
4. If you are borrower it is better to borrow money in a lender who offers
__________________.
5. Debt using credit cards is an example of _________________.

B. Distinguish compound interest from a simple interest.

What I Can Do

Read and analyze the situation below then answers the question given.

The effect of COVID-19 pandemic has overwhelmed everyone. Nobody expected that
it will occur in the early year of the 21st century. It has affected the health,
economy, education, and lives of many people around the world. It is during these
hard times that we need to show our deepest concern not only to ourselves, and to
our family but our government as well. By simply following the safety precautions
against the coronavirus like staying at home, wearing face mask and shield
properly, and observing social distancing, we can show our support in fighting to
end this pandemic.

Now, everyone including you can help amidst pandemic or any kind of calamity if
you have the basic knowledge of value investing.

Assuming that somebody from your family is planning to borrow money from a
bank or institution to start a business. You are asked to help him to look for a
bank or lending institution that could offer the lowest interest rate. List all the
necessary information and possible lending institutions by researching within your
community. Identify the interest rate offered by different institutions and the rules
and regulations when it comes to payment. Compare their offers and decide on the
best lender where you can borrow funds. Submit a short report of your research.

17
CO_Q2_General Mathematics_SHS
Module 1
Here is the rubric that will serve as your guide to finish the task.
Criteria 4 3 2 1
Knowledge Demonstrate Demonstrate Demonstrate Demonstrate
about a systematic an a partial a little
compound understanding understanding understanding understanding
interest when deciding when deciding when deciding when deciding
with regards with regards with regards with regards
to value to value to value to value
investing investing investing investing
using using using using
compound compound compound compound
interest interest interest interest

Connections Independently Determines Some Requires


determines the effectiveness support when
the connections of evidence when determining
connections of math of determining the
math of investment to the connections of
investment to the decision connections of math of
the decision making using math of investment to
making using simple and investment to the decision
simple and compound the decision making using
compound interest making using simple and
interest simple and compound
compound interest
interest
Accuracy of With complete With With partially With
data and accurate considerable completeness incomplete
data to help completeness and accuracy and
with decision and accuracy of data to help inaccurate
making of data to help with decision data
with decision making
making

18
CO_Q2_General Mathematics_SHS
Module 1
Assessment

Multiple Choice. Choose the letter of the best answer. Write the chosen letter on a
separate sheet of paper.
1. Which interest is computed on the principal and then added to it?
a. simple c. annuity due
b. compound d. ordinary annuity
2. The simple interest formula is I = Prt. What does the t represent?
a. principle c. interest
b. time d. percent rate
3. All of the following are used to find simple interest, EXCEPT?
a. principal c. time
b. rate of interest d. future value
4. It is the amount after t years that the lender will receive from the borrower.
a. future Value c. interest
b. present Value d. principal
5. Which of the following describes rate (r)?
a. It is the amount of money borrowed or invested on the origin date.
b. It is the amount paid or earned for the use of money.
c. It is charged by the lender, or rate of increase of the investment
d. It is computed on the principal and also on the accumulated past
interests.
6. Rate of interest must be converted into ________ before substituting to the
formula of finding the interest.
a. fractions c. mixed numbers
b. decimals d. percentage
7. The principal in the formula I = Prt is ___________________.
a. the amount of money borrowed or deposited
b. the percent interest for his year
c. the amount taxed
d. the amount the bank owes you for being a customer at their bank
8. The interest from the previous year also earns interest, which of the following
describes the statement?
a. simple c. rate
b. compound d. time
9. Variable P in simple interest formula when you are the creditor stands for?
a. Original Amount borrowed c. Maturity Amount Borrowed
b. Original Amount invested d. Maturity Amount invested
10. The following statements are true EXCEPT ________.
a. The amount invested to a compounded interest yields to greater
maturity value than to a simple interest.
b. The amount borrowed from a simple interest yields to a smaller
maturity value than a compound interest
c. The simple interest yields interest only from initial principal amount.
d. The interest in a compound account yields interest only from initial
principal amount.

19
CO_Q2_General Mathematics_SHS
Module 1
11. Compound interest is favorable to the following except?
a. investor of a retirement fund
b. person who saves in bank
c. investor of stocks
d. borrower of a loan
12. Which of the following will yield a higher interest?
a. Investing in a bank that offers a simple interest rate of 10%
annually for 2 years
b. Investing in a cooperative that offers an interest rate of 10%
compounding annually for 2 years
c. Saving money in a piggy bank for 2 years
d. Both a and b
13. What formula can be used to find maturity (future) value of a compound
interest?
a. F = P + I c. Both a and b

b. F = (1 + �) d. None of the choices.
14. What interest is computed on the principal and also on the accumulated past
interests?
a. simple c. annuity due
b. compound d. ordinary Annuity
15. Which of the following can be used to find compound interest?
F
a. I = F – P c. P = (1+r)t)
b. F= P(1+r)t d. I = Prt

Additional Activities

Solve the following:

1. Supposed that a local farmer wants to borrow money from Landbank of the
Philippines to start the organic farming in his one (1) hectare of agricultural land.
The farmer needs ₱ 150,000.00 as start-up capital. The bank offers him 10%
interest rate compounded annually. Compute for the total amount to be paid
every year for 5 years. Show your answer in tabular form.
2. A private school teacher plans to apply for a housing loan in the Home Mutual
Development Fund or Pag-ibig. It offers her a loan amounting to ₱1,500,000.00,
considering all the rules and regulations regarding the policy with 6.5% interest
per annum payable within 15 years. Compare the maturity value if interest will
be paid using simple interest and/or compound interest compounded yearly.
Please use the model table shown in this module to compute and compare the
interest.

20
CO_Q2_General Mathematics_SHS
Module 1
Answer Key

21
CO_Q2_General Mathematics_SHS
Module 1
References
*DepED Material: General Mathematics Learner’s Material

General Mathematics Learner’s Material. First Edition. 2016. pp. 135-167

Faylogna, Frelie T., Calamiong, Lanilyn L., Reyes, Rowena C. General


Mathematics.Sta. Ana Manila: Vicarish Publications and Trading, Inc. 2017.
pp. 102-106

Oronce, Orlando.General Mathematics.Sampaloc Manila, Philippines. Rex


Bookstore, Inc. 2016.

22
CO_Q2_General Mathematics_SHS
Module 1
For inquiries or feedback, please write or call:

Department of Education - Bureau of Learning Resources (DepEd-BLR)

Ground Floor, Bonifacio Bldg., DepEd Complex


Meralco Avenue, Pasig City, Philippines 1600

Telefax: (632) 8634-1072; 8634-1054; 8631-4985

Email Address: blr.lrqad@deped.gov.ph * blr.lrpd@deped.gov.ph


General Mathematics
Quarter 2 – Module 2:
Interest, Maturity, Future,
and Present Values in Simple
and Compound Interests

CO_Q2_General Mathematics SHS


Module 2
General Mathematics – Senior High School
Alternative Delivery Mode
Quarter 2 – Module 2: Interest, Maturity, Future, and Present Values in Simple
First Edition, 2021

Republic Act 8293, section 176 states that: No copyright shall subsist in any work of
the Government of the Philippines. However, prior approval of the government agency or office
wherein the work is created shall be necessary for exploitation of such work for profit. Such
agency or office may, among other things, impose as a condition the payment of royalties.

Borrowed materials (i.e., songs, stories, poems, pictures, photos, brand names,
trademarks, etc.) included in this module are owned by their respective copyright holders.
Every effort has been exerted to locate and seek permission to use these materials from their
respective copyright owners. The publisher and authors do not represent nor claim ownership
over them.

Published by the Department of Education


Secretary: Leonor Magtolis Briones
Undersecretary: Diosdado M. San Antonio

Development Team of the Module


Writer: Ann Michelle M. Jolo
Editors: Elizabeth D. Lalunio, Elizabeth B. Dizon Anicia J. Villaruel, Roy O. Natividad
Reviewers: Fritz A. Caturay, Necitas F. Constante, Dexter M. Valle,
Jerome A. Chavez, Mary Grace D. Constantino and Jasmin J. Flores
Illustrator: Hanna Lorraine Luna and Diane C. Jupiter
Layout Artist: Roy O. Natividad, Sayre M. Dialola, Noel Rey T. Estuita and Argie L. Ty
Management Team: Francis Cesar B. Bringas
Job S. Zape, Jr.
Ramonito Elumbaring
Reicon C. Condes
Elaine T. Balaogan
Fe M. Ong-ongowan
Hermogenes M. Panganiban
Phillip B. Gallendez
Josephine T. Natividad
Anicia J. Villaruel
Dexter M. Valle

Printed in the Philippines by ________________________

Department of Education – Region 4A CALABARZON

Office Address: Gate 2 Karangalan Village, Brgy. San Isidro, Cainta, Rizal
Telefax: 02-8682-5773/8684-4914/8647-7487
E-mail Address: lrmd.calabarzon@deped.gov.ph
General Mathematics
Quarter 2 – Module 2:
Interest, Maturity, Future,
and Present Values in Simple
and Compound Interests
Introductory Message

This Self-Learning Module (SLM) is prepared so that you, our dear learners,
can continue your studies and learn while at home. Activities, questions, directions,
exercises, and discussions are carefully stated for you to understand each lesson.

Each SLM is composed of different parts. Each part shall guide you step-by-
step as you discover and understand the lesson prepared for you.

Pre-tests are provided to measure your prior knowledge on lessons in each


SLM. This will tell you if you need to proceed on completing this module or if you
need to ask your facilitator or your teacher’s assistance for better understanding of
the lesson. At the end of each module, you need to answer the post-test to self-check
your learning. Answer keys are provided for each activity and test. We trust that you
will be honest in using these.

In addition to the material in the main text, Notes to the Teacher are also
provided to our facilitators and parents for strategies and reminders on how they can
best help you on your home-based learning.

Please use this module with care. Do not put unnecessary marks on any part
of this SLM. Use a separate sheet of paper in answering the exercises and tests. And
read the instructions carefully before performing each task.

If you have any questions in using this SLM or any difficulty in answering the
tasks in this module, do not hesitate to consult your teacher or facilitator.

Thank you.
What I Need to Know

On your previous module, you have already illustrated and distinguished the
difference between simple and compound interests. A deeper analysis of this topic
will be the focus of this module wherein the relationships among the variables in
solving the simple and compound interest will be explored.

You will realize the importance of deriving a certain formula to compute the required
variable involved in interest. The connection among interest, principal or present
value, rate, time and maturity value will be reiterated. This topic will also revolve
around money and since it is talking about money you will learn more on how to
make decisions that concerns about it.

Are you now ready for the new lesson, if so you may proceed to this module and
have fun while learning.

The module is composed of two lessons, namely:


• Lesson 1 – Interest, Maturity, Future, and Present Values in Simple Interest
• Lesson 2 – Interest, Maturity, Future, and Present Values in Compound
Interest
After going through this module, you are expected to:
1. compute interest, maturity value, future value, and present value in simple
interest environment;
2. compute interest, maturity value, future value, and present value in compound
interest environment; and
3. derive the formula of simple and compound interest to compute the maturity,
future, and present value.

1
CO_Q2_General Mathematics SHS
Module 2
What I Know

Let’s find out how far you might already know about this topic by answering the
assessment below.
Choose the letter of the best answer. Write the chosen letter on a separate sheet of
paper.
1. Which of the following is the formula to find the simple interest?
a. 𝐼𝐼𝑠𝑠 = 𝑃𝑃𝑃𝑃𝑃𝑃
b. 𝐼𝐼𝑠𝑠 = 𝑃𝑃(1 + 𝑟𝑟)𝑡𝑡
c. 𝐼𝐼𝑠𝑠 = 𝐹𝐹 − 𝑃𝑃
𝐹𝐹
d. 𝐼𝐼𝑠𝑠 =
(1+𝑟𝑟)𝑡𝑡
2. What formula will be used to find the present value of simple interest?
a. 𝑃𝑃 = 𝐼𝐼𝑠𝑠 𝑟𝑟𝑟𝑟
𝐼𝐼𝑠𝑠
b. 𝑃𝑃 =
𝑟𝑟𝑟𝑟
𝑟𝑟𝑟𝑟
c. 𝑃𝑃 =
𝐼𝐼𝑠𝑠
𝐼𝐼 𝑟𝑟
d. 𝑃𝑃 = 𝑠𝑠
𝑡𝑡
3. If the investment amounting to ₱35,000 earned an interest of ₱2,500 how much
will be the maturity value?
a. ₱32,500
b. ₱37,500
c. ₱30,000
d. ₱40,000
4. Given that P = ₱5,250, r = 1.25% and t = 5 years, find the simple interest.
a. ₱32,812.50
b. ₱3,281.25
c. ₱328.13
d. ₱32.82
1
5. Given that P = ₱10,500, r = 4 % and t = 8 months, find the simple interest.
2
a. ₱315
b. ₱3,150
c. ₱3,780
d. ₱31,500
6. Which of the following is the formula to find the compound interest?
a. 𝐼𝐼𝑐𝑐 = 𝑃𝑃(1 + 𝑟𝑟)𝑡𝑡
b. 𝐼𝐼𝑐𝑐 = 𝐹𝐹 − 𝑃𝑃
𝐹𝐹
c. 𝐼𝐼𝑐𝑐 =
(1+𝑟𝑟)𝑡𝑡
𝑖𝑖 𝑚𝑚 𝑚𝑚𝑚𝑚
d. 𝐼𝐼𝑐𝑐 = 𝑃𝑃(1 + )
𝑚𝑚

2
CO_Q2_General Mathematics SHS
Module 2
7. What is the frequency of conversion if the annual rate is compounded quarterly?
a. 2 c. 4
b. 3 d. 12
8. If the interest rate is 5% compounded monthly, what is the interest rate per
conversion period?
a. 2.5% c. 1.25%
b. 1.67% d. 0.42%
9. Given that 𝐼𝐼𝑠𝑠 = 750, r = 5% and t = 3 years, what is the principal or present
value?
a. ₱50
b. ₱500
c. ₱5.000
d. ₱50,000
10. If F = ₱25,000 and P = ₱20,000, how much is the compound interest?
a. ₱45,000
b. ₱25,000
c. ₱20,000
d. ₱5,000

For numbers 11 – 12, use the following values: F = 40,000, t = 3 ½ years, 𝑖𝑖 4 = 0.10
and m= 4
11. What is the present value?
a. ₱30, 000.25
b. ₱28, 309.09
c. ₱25, 307.15
d. ₱20, 432.75
12. How much is the compound interest?
a. ₱9,999.75
b. ₱11,690.91
c. ₱14,692.85
d. ₱19,567.25
13. If the maturity value is ₱23,000 and the compound interest is ₱3,500, how
much is the present value?
a. ₱26,500
b. ₱23,000
c. ₱19,500
d. ₱15,000

For numbers 14 – 15, use the following values: P = ₱15,000, 𝑖𝑖12 = 5%, t = 4 years, m
= 12.

14. How much is the maturity value?


a. ₱18, 313.43
b. ₱17, 413.37
c. ₱16, 313.37
d. ₱15, 413.43
15. How much is the compound interest?
a. ₱413.43
b. ₱1,313.37
c. ₱2,413.37
d. ₱3, 313.43

3
CO_Q2_General Mathematics SHS
Module 2
Lesson Interest, Maturity, Future,
1 and Present Values in
Simple Interest

Business transactions are common events we experienced in our daily lives like
banking, financing, marketing and pawning activities which involves withdrawals,
deposits, used of credit cards and others. When these transactions occur there are
two parties involved the lender and the borrower hence there involves a particular
amount which we call interest.

Everything that we have right now is just borrowed, our talents, jobs, and even our
lives. God is the sole person who does not charge interest from what He lent.
However, the money that we borrowed or loaned from others once paid earned an
interest which will be tackled in this module.

What’s In

For you to begin consider the previous lesson essential in obtaining success in this
module. In the last module, you differentiate simple and compound interest.
Simple interest is computed by multiplying the principal (P) by the rate (r) and the
length of time (t) (I=Prt) while compound interest is computed on the principal plus
the accumulated past interests.

Different terms related to simple and compound interest were also given emphasis
such as the lender or creditor which refers to the person who invests or makes funds
available and the borrower or debtor, which refers to the person who owes the
money.
Moreover, different terms essential in the interest formula were also explained such
as time or term, principal or present value, rate, and maturity value. However,
additional terms for compound interest were also given importance such as
frequency of conversion, nominal rate, and rate of interest for each conversion period.
These terms will be useful for you to have a better grasp of this module.

4
CO_Q2_General Mathematics SHS
Module 2
Other skills such as expressing percent to decimals is also necessary for example:

Express the following as decimal:

1. 12% - 0.12
2. 10.5% - 0.105
3. 300% - 3
1
4. 8 % - 0.85
2

Notes to the Teacher


Use of calculator in this module is allowed because it will help
them to easily compute what is asked, however reiterate to the
learners that to ensure the accuracy and precision of the solution
the use of correct formula is necessary. Also, inform them that in
other books different variables were used to represent the
components of interest formula, it will not affect the result as long
as the relationship with other variables is the same with the
relationship to be presented in this module.

What’s New

Read and analyze the situation below.


Let’s Save
Janice and Jamaica are both senior high school students. After class the two had a
conversation:
Janice: Wow, your cellphone is so cool! Is that new?

Jamaica: Thank you! And yes, I just bought it yesterday.


Janice: Did your mother give you money? What will you do with your old cellphone?

5
CO_Q2_General Mathematics SHS
Module 2
Jamaica: No, I saved up my allowance to accumulate enough money to buy this
phone. I am still using my old phone. I will use my new phone for my social media
accounts.

Janice: How much is that?

Jamaica: It is only ₱2,300 but it is already android so I can download different


applications like Tiktok, Snapchat and games . Therefore, I will not be bored!

Janice: Oh! We are almost the same, I also saved ₱2,300 (already) from my allowance

Jamaica: What (do you plan) will you buy from your savings?

Janice: My old cellphone is still working, so I think there is no need for me to buy a
new one. It will be better if I save this money.

Jamaica: But your money might get lost if you will not spend it?

Janice: I saw from an advertisement in Cooperative Bank that the minimum amount
to open an account for students is ₱2,000, and they call it Kid Savers.

Jamaica: Is there an advantage if you put the money on the bank?

Janice: According to the advertisement there will be 2.5% interest in a year. I will use
it in my college education.

Questions:

1. What is the dialogue all about?


___________________________________________________________
2. Do you think saving money is important? Explain your answer.
________________________________________________________________
2. Do you think it is wrong for Jamaica to buy a new cellphone?
___________________________________________________________
3. What can you say about Janice’s attitude towards money?
___________________________________________________________
4. How much will Janice save after two years?
___________________________________________________________
5. How do you value education?
___________________________________________________________

6
CO_Q2_General Mathematics SHS
Module 2
What is It

From the previous conversation, you can say that allowance is one of the sources of
funds of ordinary students. A person may decide on what they want to do with their
money. In the scenario, there are two kinds of students. One who saves to buy the
things they wanted and the other one saves to invest for her future. In doing so
investing money in the bank earns interest. In computing the simple interest and
other related components, the formula is

𝐼𝐼𝑠𝑠 = 𝑃𝑃𝑃𝑃𝑃𝑃

where:
Is = simple interest
P = principal or the amount invested or borrowed or present value
r = simple interest rate
t = time or term in years

The formula can be manipulated to obtain the following relationships:


The formula for finding the principal amount

𝐼𝐼𝑠𝑠
𝑃𝑃 =
𝑟𝑟𝑟𝑟

The formula for finding the rate

𝐼𝐼𝑠𝑠
𝑟𝑟 =
𝑃𝑃𝑃𝑃

The formula for finding the time


𝐼𝐼𝑠𝑠
𝑡𝑡 =
𝑃𝑃𝑃𝑃

7
CO_Q2_General Mathematics SHS
Module 2
To find the maturity (future) value, you can use either of the following:

𝐹𝐹 = 𝑃𝑃(1 + 𝑟𝑟𝑟𝑟) or 𝐹𝐹 = 𝑃𝑃 + 𝐼𝐼𝑠𝑠

where:
F = maturity (future) value
Is = simple interest
P = principal or the amount invested or borrowed or present value
r = simple interest rate
t = time or term in years
Let us take the following for example:
Example 1: Given: 𝑃𝑃 = ₱18, 500, 𝑟𝑟 = 0.03, 𝑡𝑡 = 5. Find simple interest (𝐼𝐼𝑠𝑠 )

Solution:
Use the formula of Simple Interest 𝐼𝐼𝑠𝑠 = 𝑃𝑃𝑃𝑃𝑃𝑃
Substitute the given to the formula 𝐼𝐼𝑠𝑠 = 18,500(0.03)(5)
Performing the operation 𝐼𝐼𝑠𝑠 = ₱2,775
Therefore, the simple interest is ₱2,775

Example 2: Given: 𝑃𝑃 = ₱20,000, 𝐼𝐼𝑠𝑠 = ₱4,000, 𝑡𝑡 = 4 . Find the rate (𝑟𝑟)


Solution:
𝐼𝐼
Use the formula in finding the rate 𝑟𝑟 = 𝑃𝑃𝑃𝑃𝑠𝑠
4000
Substitute the given to the formula 𝑟𝑟 = (20,000)(4)

Performing the operations r = 0.05


Express your answer in percent r = 5%

Therefore, the rate of interest is 5%.

Example 3: Given: 𝑃𝑃 = ₱40,000., 𝐼𝐼𝑠𝑠 = ₱700, 𝑟𝑟 = 7%. Find time (𝑡𝑡).

Solution:
𝐼𝐼
Use the formula in finding the time 𝑡𝑡 = 𝑃𝑃𝑃𝑃𝑠𝑠
700
Substitute the given to the formula 𝑡𝑡 = (40000)(0.07)

Performing the operations t = 0.25


Express your answer as unit of time t= ¼ year or 3 mos.

Therefore, the term or time in years is ¼ year or 3 months.

8
CO_Q2_General Mathematics SHS
Module 2
Example 4: Given: 𝑃𝑃 = ₱15,000, 𝑡𝑡 = 4 months, 𝑟𝑟 = 2%. Find maturity (future) value (𝐹𝐹).

Solution:

Use the formula of maturity value 𝐹𝐹 = 𝑃𝑃(1 + 𝑟𝑟𝑟𝑟)


1
Substitute the given to the formula 𝐹𝐹 = 15,000(1 + (0.02) � �)
3
4𝑚𝑚𝑚𝑚𝑚𝑚. 1
Hint: time must be expressed in years so 4 months will become = 𝑦𝑦𝑦𝑦𝑎𝑎𝑎𝑎
12𝑚𝑚𝑚𝑚𝑚𝑚 3

Performing the operations F = ₱15,100


Alternative Solution:
1
Solve first the simple interest 𝐼𝐼𝑠𝑠 = (15,000)(0.02)( )
3

Is = 100
Use the formula 𝐹𝐹 = 𝑃𝑃 + 𝐼𝐼𝑠𝑠 F = 15,000 +100
Therefore, the maturity value is ₱15,100

What’s More

Activity 1.1
1. If P = ₱4,500, r = 1.25% and t = 5 years, find the simple interest.

a. What formula will be used? _______________________

b. How are you going to express the rate in decimal? ______________

c. How much is the simple interest? ___________________________

2. If P = ₱5,000, = 2% and t = 8 mos., find the maturity value.

a. Which formula will you use? __________________________

b. How are you going to express the time in years? __________________

c. How much is the maturity value? _________________________

Activity 1.2
9
CO_Q2_General Mathematics SHS
Module 2
Find the value of the required components in each item.

1. Find the simple interest and maturity value if P = ₱13,000, r = 0.8% and t=
2 ½ years.

2. If Is = ₱625, r = 2% and t =3 years, find the present value

Activity 1.3
Complete the table below by finding the unknown.

Principal Rate Time Interest Maturity


Value
₱45,000 2% 2.5 years
1.2% 4 years ₱4,560
₱105,000 3% ₱114,450

Guide Questions:

For the first set of values


a. What unknown variable will you solve first?
b. What formula will you use to find the missing values?

For the second set of values


a. Which among the missing components can be solved using the given?
b. How will you solve the next missing component?

For the third set of values


a. What formula will you use to find the missing value?
b. Which component can be solved first using the given?

What I Have Learned

10
CO_Q2_General Mathematics SHS
Module 2
Read the paragraph carefully and supply the missing term/s in the blanks that will
make it logical.

In doing a business transaction one of the essential things to consider is the interest
because it will be the basis whether you have gained or lost. When the interest is
computed based on the principal it is called ______________ its formula is
______________. There are other components included in the simple interest formula
such as principal or present value, rate, time, and maturity value. In finding the
principal the formula ______________ will be used. Meanwhile the rate must be
expressed in ________ and in finding its value given the simple interest, principal and
time you will use the formula__________. Moreover, time should be expressed in
________. In a simple interest environment, the formula to be used in finding the time
is _____________. Lastly maturity value can be obtained by adding the __________ and
_________ or if the simple interest is not given you can use the formula
___________________.

What I Can Do

Money Matters
Read and analyze the situation below, then answer the question that follows.

You are a new accounts clerk in Lucena Metropolitan Bank where you met Mr. and
Mrs. Smith who are planning for the education of their children in the future. You
introduce to them the advantage of time deposit having the following features:

Option A: 1.10% interest annually in 3 years,


Option B: 1.25% interest annually in 5 years
Option C: 1.75% interest annually in 8 years

The couple has an initial amount of ₱50,000 to be deposited. To help the couple in
deciding the terms of their investment make a proposal by completing the table
below.

PROPOSAL

PRINCIPAL TIME RATE INTEREST AMOUNT


₱50,000
₱50,000

11
CO_Q2_General Mathematics SHS
Module 2
₱50,000

Aside from the amount to be yielded at the end of each term what are the other
factors that you can consider?
List down the possible questions that you will ask to the couple to determine those
factors.
______________________________________________________________________________
______________________________________________________________________________
Is there any disadvantage in choosing each term? What are those?
______________________________________________________________________
______________________________________________________________________
What advice or tips can be given to the couple in choosing the term of interest?
_______________________________________________________________________
_______________________________________________________________________

Rubrics for the Task


Categories Excellent Fair Poor
3 2 1

Relevance of the Excellent Some Little to no


advice understanding in understanding in understanding in
creating advice for creating advice for creating advice for
choosing a plan choosing a plan choosing a plan

Interview Skills Excellent set of A fair set of A poor set of


questions that questions that questions that
elicit the answer to elicit the answer to elicit the answer to
help in the help in the help in the
decision making decision making decision making
process is given process is given process is given

Accuracy of The computations The computations There is no


Solution made are all made have flaws attempt in making
correct computation

12
CO_Q2_General Mathematics SHS
Module 2
Lesson Interest, Maturity, Future,
2 and Present Values in
Compound Interest
The previous lesson reiterates the importance of simple interest in a certain
transaction. Interest is a natural event in a business transaction, however not all
interest is classified as simple interest some of them are considered compound
interest.
This lesson will delve into compound interest and the different components involved
in it such as present value and maturity value.

What’s In

For you to begin considering the lesson on the previous module which is essential
in obtaining success in this lesson. Compound interest is not like a simple interest
wherein only the principal is considered in the computation this type of interest
considers the principal and the accumulated past interest. The frequency of
conversion, nominal rate, and rate of interest for each conversion period will also
play an important role in this lesson.
In the previous lesson the maturity value is computed using the formula 𝐹𝐹 = 𝐼𝐼𝑠𝑠 + 𝑃𝑃
where F is the future value , Is is the simple interest and P the present value or the
Principal and to find the principal or present value given the interest and maturity
value the formula 𝑃𝑃 = 𝐹𝐹 − 𝐼𝐼𝑠𝑠 can be employed.
Let us take the following example.
Example 1: Given: P = ₱35,000 and Is = ₱ 4,000, find F.
Solution: F

F = ₱35,000 + ₱4,000 = ₱39,000

Example 2: Given: F = ₱50,000 and P = ₱45,000, find Is.


Solution:

Is = ₱50,000 – ₱45,000= ₱5,000


Example 3: Given Is = ₱2,000 and F = ₱23,000., fnd P.

Solution:
P = ₱23,000 - ₱2,000 b= ₱21,000

13
CO_Q2_General Mathematics SHS
Module 2
What’s New

Read and analyze the situation below.

Let’s Save

Michael is planning to apply for a loan in a Cooperative Bank, and he is already


aware of the terms and conditions of payment for his loan. When he is about to pass
his application form and compare his computation with the bank’s payment terms
he notices some discrepancies.

Michael’s Computation Computation from the bank

Amount of Loan: ₱100,000 Amount of Loan: ₱100,000


Interest rate: 3% Interest rate: 3%
Due Date: After 3 years
Computation: Year 1 Year 2 Year 3
I = (100,000)(0.03(3) Int 3000 6090 9272.70

I = ₱9,000 Amt 103,000 106,090 109,272.70

Amount to be paid after 3 years


₱109,000

To be enlightened, he asked some explanations why they have different computations


and the bank gave him the detailed computation:

Initially at t = 0 ₱100,000
at t = 1 ₱100,000 (1.03) = ₱103,000
at t = 2 ₱103,000 (1.03) = ₱106,090
at t = 3 ₱106,090 (1.03) = ₱109,272.70

14
CO_Q2_General Mathematics SHS
Module 2
Questions
1. Is Michael’s computation correct?
____________________________________________________________

2. Is the bank’s computation fair? Why?


_____________________________________________________________

3. How much is the difference in the total amount to be paid between Michael’s
computation and the bank’s computation?
______________________________________________________________
4. Why do you think the bank’s computation yielded more interest?
______________________________________________________________

5. Do you think the bank committed an error in the computation of the amount
to be paid?
______________________________________________________________

6. If the term of payment will be longer what do you think will happen between
the difference of the amount to be paid in Michael’s computation and the
bank’s computation?
_________________________________________________________________

7. If you are Michael and you follow the computation made by the bank do you
think there is a way to lessen the amount to be paid at the end of 3 years?
How?
________________________________________________________________
________________________________________________________________

15
CO_Q2_General Mathematics SHS
Module 2
What is It

Notice that there is a difference between the computation of Michael and the bank
concerning the amount to be paid for the loan. Michael used simple interest to find
the amount to be paid for the loan for three years. While the bank computed first the
interest for the first year and added it to the loan amount, then the resulting amount
becomes the basis for computing the total amount to be paid for the second year,
and it follows the same pattern for the third year. Interest plays a major role in
computation because it became one of the factors in determining the amount to be
paid for the succeeding years. In such a case, we call that compound interest. To
better understand of what compound interest is, the following formulas will be
considered.
To find the compound interest, which is compounded annually the formula to find
the maturity value is:

𝐹𝐹 = 𝑃𝑃(1 + 𝑟𝑟)𝑡𝑡
where:
F = maturity (future) value
P = principal or present value
r = interest rate
t = term or time in years

To find the compound interest use the formula:

𝐼𝐼𝑐𝑐 = 𝐹𝐹 − 𝑃𝑃

where:
𝐼𝐼𝑐𝑐 = compound interest
P = principal or present value
F = maturity (future) value
To find the present value or principal of the maturity value F due in t years the
formulas are:

𝐹𝐹
𝑃𝑃 = 𝑃𝑃 = 𝐹𝐹(1 + 𝑟𝑟)−𝑡𝑡
(1 + 𝑟𝑟)𝑡𝑡 or

16
CO_Q2_General Mathematics SHS
Module 2
Example 1: Given: P= ₱18,500, r = 3% and compounded annually for 3 years, find
the maturity value (F) and the compound interest (Ic ).
Solution:
Use the formula of maturity value 𝐹𝐹 = 𝑃𝑃(1 + 𝑟𝑟)𝑡𝑡

Substitute the given to the formula 𝐹𝐹 = 18,500(1 + 0.03)3

Performing the operations F = ₱20,215.45

Apply the formula of compound interest 𝐼𝐼𝑐𝑐 = 𝐹𝐹 − 𝑃𝑃

Substitute the value of F that you get and P 𝐼𝐼𝑐𝑐 = 20,215.45 − 18500

Performing the operations 𝐼𝐼𝑐𝑐 = ₱1,715.45

Therefore, the maturity value is ₱20,215.45 and the compound interest


₱1,715.45

Example 2: Given F = ₱15,000, r = 2% compounded annually for 4 years, find the


present value (P).
Solution:
𝐹𝐹
Use the formula in finding the present value 𝑃𝑃 =
(1+𝑟𝑟)𝑡𝑡

15000
Substitute the given to the formula 𝑃𝑃 =
(1+0.02)4

Performing the operations P= ₱13,857.68

Therefore, the present value is ₱13,857.68


Compounding More Than Once a Year
In the examples above the interest are compounded annually, however, there are
cases that interest are compounded more than once a year so in this case additional
terms must be clarified such as:
Frequency of conversion (m) - number of conversion period in one year
Conversion or interest period – time between successive conversions of interest
Total number of conversion periods (n)
n = mt = (frequency of conversion) 𝑥𝑥 (time in years)
Nominal rate (𝒊𝒊𝒎𝒎 ) - annual rate of interest or interest rate per year
𝑖𝑖 (𝑚𝑚) 𝑎𝑎𝑎𝑎𝑎𝑎𝑎𝑎𝑎𝑎𝑎𝑎 𝑟𝑟𝑟𝑟𝑟𝑟𝑟𝑟 𝑜𝑜𝑜𝑜 𝑖𝑖𝑖𝑖𝑖𝑖𝑖𝑖𝑖𝑖𝑖𝑖𝑖𝑖𝑖𝑖
Rate (j) of interest for each conversion period 𝑗𝑗 = =
𝑚𝑚 𝑓𝑓𝑓𝑓𝑓𝑓𝑓𝑓𝑓𝑓𝑓𝑓𝑓𝑓𝑓𝑓𝑓𝑓 𝑜𝑜𝑜𝑜 𝑐𝑐𝑐𝑐𝑐𝑐𝑐𝑐𝑐𝑐𝑐𝑐𝑐𝑐𝑐𝑐𝑐𝑐𝑐𝑐

17
CO_Q2_General Mathematics SHS
Module 2
Study the table below.
Situations m N 𝑖𝑖 𝑚𝑚 j
2% compounded Annually m =1, t = 3 𝑖𝑖1 = 0.02 𝑖𝑖 𝑚𝑚
𝑗𝑗 =
annually for 3 𝑚𝑚
m=1 n = (1)(3)=3
years 0.02
𝑗𝑗 = = 0.02
1

2% compounded Semi – m = 2, t = 3 𝑖𝑖 2 = 0.02 0.02


𝑗𝑗 = = 0.01
2
semi – annually annually
n = (2)(3)=6
for 3 years
m=2
2% compounded Quarterly m = 4, t = 3 𝑖𝑖 4 = 0.02 0.02
𝑗𝑗 = = 0.005
4
quarterly for 3
m=4 n =(4) (3)=12
years
2% compounded Monthly m = 12, t = 3 𝑖𝑖12 = 0.02 0.02
𝑗𝑗 =
12
monthly for 3 = 0.0016�
m = 12 n = (12)(3) = 36
years
2% compounded Daily m = 365, t = 3 𝑖𝑖 365 = 0.02 0.02
𝑗𝑗 =
365
daily for 3 years
m = 365 n = (365(3) =1095

Since the rate for each conversion period is represented by j, then in t years, interest
is compounded mt times. Thus, the formula of Maturity Value for interest
compounding m times a year is:

𝐹𝐹 = 𝑃𝑃(1 + 𝑗𝑗)𝑛𝑛

where:
F = maturity value
P = present value
𝑖𝑖 (𝑚𝑚)
j=
𝑚𝑚
n = mt
Meanwhile, the formula in finding the present value given the maturity value is:

𝐹𝐹
𝑃𝑃 =
(1 + 𝑗𝑗)𝑛𝑛

18
CO_Q2_General Mathematics SHS
Module 2
Let us take the following for example:
Example 3: Given 𝑃𝑃 = ₱50,000.00, 𝑖𝑖 4 = 0.03, 𝑚𝑚 = 4, 𝑡𝑡 = 4, find F and Ic .
Solution:
Use the formula of maturity value 𝐹𝐹 = 𝑃𝑃(1 + 𝑗𝑗)𝑛𝑛

Solve for n and j 𝑛𝑛 = 𝑚𝑚𝑚𝑚; 𝑛𝑛 = 4(4); 𝑛𝑛 = 16


𝑖𝑖 (4) 0.03
𝑗𝑗 = ; 𝑗𝑗 = ; 𝑗𝑗 = 0.0075
𝑚𝑚 4

Substitute the values of the known variables 𝐹𝐹 = 50,000(1 + 0.0075)16


Performing the operations 𝐹𝐹 = 56,349.61
Use the formula of compound interest 𝐼𝐼𝑐𝑐 = 𝐹𝐹 − 𝑃𝑃

Substitute the values of F and P 𝐼𝐼𝑐𝑐 = 56,349.61 − 50,000

Performing the operation 𝐼𝐼𝑐𝑐 = 6,349.61

Therefore, the maturity value is ₱56,349.61 and the compound interest is


₱6,349.61
Example 4: Given = ₱45,000.00, 𝑖𝑖 2 = 0.02, 𝑚𝑚 = 2, 𝑡𝑡 = 4, find Ic .

Solution:
𝐹𝐹
Use the formula for Present value 𝑃𝑃 =
(1+𝑗𝑗)𝑛𝑛

Solve for n and j 𝑛𝑛 = 𝑚𝑚𝑚𝑚; 𝑛𝑛 = 2(4); 𝑛𝑛 = 8


𝑖𝑖 (2) 0.02
𝑗𝑗 = ; 𝑗𝑗 = ; 𝑗𝑗 = 0.01
𝑚𝑚 2
45000
Substitute the values of the known variables 𝑃𝑃 =
(1+0.01)8

Performing the operations 𝑃𝑃 = 41,556.75

Use the formula for compound interest 𝐼𝐼𝑐𝑐 = 𝐹𝐹 − 𝑃𝑃

Substitute the values of F and P 𝐼𝐼𝑐𝑐 = 45,000 − 41,556.75

Performing the operation 𝐼𝐼𝑐𝑐 = 3,443.25

Therefore, the present value is ₱41,556.75 and the compound interest is


₱3443.25

19
CO_Q2_General Mathematics SHS
Module 2
What’s More

Activity 2.1
Read each statement and answer the question that follows.
1. If P = ₱85,500, and r = 1.25% compounded monthly for 1 year, find the
compound interest.

What is the first component that should be computed? ____________________

What formula will be used? _______________________

How much is the maturity value? _____________________________


How will you find the simple interest? ______________

How much is the simple interest? ___________________________

2. If F = ₱50,000 with the rate 1.5% compounded quarterly for 5 years find the
present value and compound interest

Which should be find first present value or compound interest?


__________________________

How are you going to express the rate in decimal? ___________________

What formula will you use in finding the present value? ____________

How much is the present value? _________________________

How will you find the compound interest? _________________

How much is the compound interest? ______________________________

20
CO_Q2_General Mathematics SHS
Module 2
Activity 2.2

Find the value of the required components in each item.

1. Find the compound interest and maturity value if P = ₱43,000, with a rate of
5% is compounded semi-annually for 6 years.

2. Find the compound interest and present value if F = ₱105,000 with a rate of
2.5% is compounded quarterly for 3 years

Activity 2.3
Complete the table below by finding the unknown.
Present Nominal Interest Interest Time in Total Compound Maturity
Value rate (i(m)) compounded per years number of Interest Value
conversion conversions
period (j) (n)
60,000 3% quarterly 3
6% semi- 4 40,000
annually

Guide Questions:
For the first set of values
a. What unknown variable will you solve first?
b. How are you going to find j?
c. What will be n?
d. How much is the maturity value?
e. How much is the compound interest?
For the second set of values
a. What unknown variable will you solve first?
b. How are you going to find j?
c. What will be n?
d. How much is the maturity value?
e. How much is the compound interest?

Activity 2.4
Complete the table below by finding the unknown.
Present Nominal Interest Interest Time in Total Compound Maturity
Value rate (i(m)) compounded per years number of Interest Value
conversion conversions
period (j) (n)
78,000 4% quarterly 6
12% monthly 3 50,000

21
CO_Q2_General Mathematics SHS
Module 2
What I Have Learned

I. Fill in the blanks to make the statements true.


1. In finding the total number of conversion period you will multiply __________ by
_____________.

2. To get the rate of interest for each conversion period the annual rate of interest
will be ________ by the frequency of conversion.

2. The formula in getting the compound interest given the present value and
maturity value is ____________________
3.
4. The formula in getting the present value in a compound interest environment is
___________________

5. The formula __________________ is used to get the maturity value in a compound


interest environment.

II. In not more than three sentences explain how to compute the compound interest.
______________________________________________________________________________

22
CO_Q2_General Mathematics SHS
Module 2
What I Can Do

Money Matters
Read and analyze the situation below then answer the questions that follow.
Suppose you are a regular employee in a finance company that offers a loan payable
through salary deduction. One of its privileges is availing a loan that offers a 5%
interest compounded annually for 1 to 5 years. You are thinking to apply for a loan
but you wanted to analyze if you can shoulder the monthly payment. Make a loan
schedule and fill up an application form that will help you decide the amount of loan
that you will make
Loan Schedule
Loan Amount Maturity Value Monthly
t=1 t=2 t=3 t=4 t=5 Payment
₱10,000
₱20,000
₱30,000
₱40,000
₱50,000
₱100,000
₱150,000
₱200,000
₱250,000

23
CO_Q2_General Mathematics SHS
Module 2
Loan Application Form
Name: ________________________________________________________

Age:________________________ Date of Birth: ________________

Occupation: __________________________________________________

Monthly Income:______________________________________________

Sources of Funds:_____________________________________________

Loan Amount:_________________________________________________

Other Sources of Funds: ______________________________________

Monthly Payment of Loan:_____________________________________

Please answer the following questions truthfully:


1. Do you have an existing loans from other banks?
_________________________________________________________

2. What is the purpose of the loan?


_________________________________________________________
3. Can your salary cover the monthly payment of the loan? If not
do you have other resources to pay the monthly payment?
______________________________________________________

Rubrics for Scoring


Categories Excellent Fair Poor
3 2 1
Relevance of the Excellent Some Little to no
advice understanding in understanding in understanding in
creating advice for creating advice for creating advice for
choosing a plan choosing a plan choosing a plan
Planning The goal set is The goal set is The goal set is not
achievable and hard to achieve achievable and
realistic not realistic
Accuracy of The computations The computations There is no
Solution made are all made have flaws attempt in making
correct computation

24
CO_Q2_General Mathematics SHS
Module 2
Assessment

Multiple Choice. Choose the letter of the best answer. Write the chosen letter on a
separate sheet of paper.
1. Which of the following described to formula 𝐼𝐼𝑠𝑠 = 𝑃𝑃𝑃𝑃𝑃𝑃 ?
a. simple interest
b. compound interest
c. present value
d. maturity value
2. What will you get if you divide the simple interest by the product of the rate
and time?
a. present value of compound interest
b. present value of simple interest
c. maturity value of compound interest
d. maturity value of simple interest

3. If the investment amounting to ₱45,000 earned an interest of ₱3,500 how


much will be the maturity value?
a. ₱41,500
b. ₱48,500
c. ₱40,000
d. ₱25,000

4. Given that P = ₱15,250, r = 3.25% and t= 3 years, find the simple interest.
a. ₱24,868.80
b. ₱14,868.80
c. ₱1,486.88
d. ₱148.69

1
5. Given that P = ₱20,820, r = 2 % and t = 9 months, find the simple interest.
4
a. ₱ 351.34
b. ₱ 3513.38
c. ₱ 4,216.05
d. ₱ 42160.50

6. Which of the following is the formula to find the maturity value of the
compound interest?
a. 𝐹𝐹 = 𝑃𝑃(1 + 𝑟𝑟)𝑡𝑡
b. 𝐹𝐹 = 𝐹𝐹 − 𝑃𝑃
𝐹𝐹
c. 𝐹𝐹 =
(1+𝑟𝑟)𝑡𝑡
𝑖𝑖 (𝑚𝑚) 𝑚𝑚𝑚𝑚
d. 𝐹𝐹 = 𝑃𝑃(1 + )
𝑗𝑗

25
CO_Q2_General Mathematics SHS
Module 2
7. What is the frequency of conversion if the annual rate is compounded
monthly?
a. 2
b. 3
c. 4
d. 12

8. If the interest rate is 12% compounded quarterly, what is the interest rate per
conversion period?
a. 3%
b. 2.5%
c. 1.25%
d. 0.42%

9. Given that 𝐼𝐼𝑠𝑠 = 3,750, r = 2 ½ % and t = 3 years, what is the principal or present
value?
a. ₱500
b. ₱5,000
c. ₱50,000
d. ₱500,000

10. Given that F = ₱50,000 and P = ₱35,000 how much is the compound interest?
a. ₱15, 000
b. ₱25,000
c. ₱35,000
d. ₱45,000

For numbers 11 – 12, use the following values: F = 150,000, t = 6 years, 𝑖𝑖 2 = 5%


and m= 2

11. What is the present value?


a. ₱100,353.83
b. ₱111,533.38
c. ₱123,153.83
d. ₱132,531.38

12. How much is the compound interest?


a. ₱49646.17
b. ₱38466.62
c. ₱26846.17
d. ₱17468.62

13. If the maturity value is ₱50,000 and the compound interest is ₱3,500, how
much is the present value?

26
CO_Q2_General Mathematics SHS
Module 2
a. ₱53,500
b. ₱46,500
c. ₱46,000
d. ₱3,000

For numbers 14 – 15, use the following values: P=₱150,000, 𝑖𝑖 2 = 6%, t=4 years,
m=2

14. How much is the maturity value?

a. ₱190, 015.51
b. ₱179, 413.35
c. ₱167, 313.51
d. ₱159, 413.35

15. How much is the compound interest?


a. ₱9,413.35
b. ₱17,313.51
c. ₱29,413.35
d. ₱40,015.51

Additional Activities

27
CO_Q2_General Mathematics SHS
Module 2
Complete the table.
A.
Principal Rate Time Interest Maturity
Value
₱60,000 4% 3.5 years
2.2% 5 years ₱345
₱125,000 2% ₱135,000

B.

Present Nominal Interest Interest Time Total Compound Maturity


Value rate compounded per in number of Interest Value
(i(m)) conversion years conversions
period (j) (n)
₱80,000 2.1% quarterly 4
2% monthly 6 ₱70,000

Answer Key

28
CO_Q2_General Mathematics SHS
Module 2
Module 2
CO_Q2_General Mathematics SHS
29
General Mathematics Teacher's Guide. Pasig City: Department of Education, 2016.
2016.
General Mathematics Learner's Material. Pasig City: Department of Education,
References
What's More
Activity 1.1
1. ₱281.25
2. ₱5066.67
Activity 1.2
1. I =₱260, F = ₱13,260
2. ₱10,416.67
Activity 1.3
P r t Is F
45,000 2% 2 2250 47250
1/2
95,000 1.2% 4 4560 99560
105000 3% 3 9450 114450
Activity 1.4
P r t Is F
60000 5% 3 9000 69000
40000 1 2 1200 41200
½%
20000 0.5% 9mos 75 21075 Assessment
Activity 2.1
What I 1. ₱1074.89 1. A
2. P=₱46393.68
Know Ic = 3606.32
2. B
Activity 2.2 3. B
1. A 4. C
1. F = ₱57,830.22
2. B I = ₱14,830.22 5. A
3. B 2. P=₱97,435.81 6. A
4. C I=₱7564.19 7. D
5. A Activity 2.3
8. A
6. A P i(m m j t n Ic F
) 9. C
7. C 60000 3% 4 0- 3 12 5628. 65628.41 10.A
8. A 0075 41
31576. 6% 2 0.03 4 8 8423. 40000 11.B
9. C 37 63 12.B
10.D Activity 2.4
13.B
11.B P i(m m j t n Ic F
) 14.A
12.B 78000 4% 4 0.0 6 24 21039 99039.3 15.D
13.C 1 .30 0
34946. 12 12 0.0 3 36 15053 50000
14.A 25 % 1 .75
15.D
Orines, Fernando B. Next Century Mathematics. Quezon, City: Phoenix Publishing
House, 2016.
Oronce, Orlando A. General Mathematics. Sampaloc, Manila: Re Bookstore, Inc.,
2017.

Santos, Durawin C.,Biason, Ma. Garnet P. Math Activated:Engage Yourself and Our
World General Mathematics. Makati, City: Salesiana Books by Don Bosco
Press, Inc., 2016.

30
CO_Q2_General Mathematics SHS
Module 2
For inquiries or feedback, please write or call:

Department of Education - Bureau of Learning Resources (DepEd-BLR)

Ground Floor, Bonifacio Bldg., DepEd Complex


Meralco Avenue, Pasig City, Philippines 1600

Telefax: (632) 8634-1072; 8634-1054; 8631-4985

Email Address: blr.lrqad@deped.gov.ph * blr.lrpd@deped.gov.ph


General Mathematics
Quarter 2 – Module 3:
Solving Problems Involving
Simple and Compound Interest

CO_Q2_General Mathematics SHS


Module 3
General Mathematics
Alternative Delivery Mode
Quarter 2 – Module 3: Solving Problems Involving Simple and Compound Interest
First Edition, 2021

Republic Act 8293, section 176 states that: No copyright shall subsist in any work of
the Government of the Philippines. However, prior approval of the government agency or office
wherein the work is created shall be necessary for exploitation of such work for profit. Such
agency or office may, among other things, impose as a condition the payment of royalties.

Borrowed materials (i.e., songs, stories, poems, pictures, photos, brand names,
trademarks, etc.) included in this module are owned by their respective copyright holders.
Every effort has been exerted to locate and seek permission to use these materials from their
respective copyright owners. The publisher and authors do not represent nor claim ownership
over them.

Published by the Department of Education


Secretary: Leonor Magtolis Briones
Undersecretary: Diosdado M. San Antonio

Development Team of the Module


Writer: Jerson D. Jolo
Editors: Elizabeth D. Lalunio, Elizabeth B. Dizon,Anicia J. Villaruel, Roy O. Natividad
Reviewers: Jerry Punongbayan, Necitas F. Constante, Fritz A. Caturay,
Celestina M. Alba, Jerome A. Chavez, Edna Adel,
Lirio G. Parale, Flora P. Segovia
Illustrators: Hanna Lorraine G. Luna, Diane C. Jupiter
Layout Artist: Roy O. Natividad, Sayre M. Dialola, Noel Rey T. Estuita,
Argie L. Ty, Jilky I. Bosque
Management Team: Francis Cesar B. Bringas
Job S. Zape, Jr.
Ramonito Elumbaring
Reicon C. Condes
Elaine T. Balaogan
Fe M. Ong-ongowan
Elias A. Alicaya Jr.
Gregorio A. Co Jr.
Gregorio T. Mueco
Herbert D. Perez
Lorena S. Walangsumbat
Jee-Ann O. Borines
Asuncion C. Ilao

Printed in the Philippines by ________________________

Department of Education – Region 4A CALABARZON


Office Address: Gate 2 Karangalan Village, Brgy. San Isidro, Cainta, Rizal
Telefax: 02-8682-5773/8684-4914/8647-7487
E-mail Address: lrmd.calabarzon@deped.gov.ph
General Mathematics
Quarter 2 – Module 3:
Solving Problems Involving
Simple and Compound Interest
Introductory Message
This Self-Learning Module (SLM) is prepared so that you, our dear learners,
can continue your studies and learn while at home. Activities, questions, directions,
exercises, and discussions are carefully stated for you to understand each lesson.

Each SLM is composed of different parts. Each part shall guide you step-by-
step as you discover and understand the lesson prepared for you.

Pre-tests are provided to measure your prior knowledge on lessons in each


SLM. This will tell you if you need to proceed on completing this module or if you
need to ask your facilitator or your teacher’s assistance for better understanding of
the lesson. At the end of each module, you need to answer the post-test to self-check
your learning. Answer keys are provided for each activity and test. We trust that you
will be honest in using these.

In addition to the material in the main text, Notes to the Teacher are also
provided to our facilitators and parents for strategies and reminders on how they can
best help you on your home-based learning.

Please use this module with care. Do not put unnecessary marks on any part
of this SLM. Use a separate sheet of paper in answering the exercises and tests. And
read the instructions carefully before performing each task.

If you have any questions in using this SLM or any difficulty in answering the
tasks in this module, do not hesitate to consult your teacher or facilitator.

Thank you.
What I Need to Know

You already determine on your previous modules the different characteristics and
qualities of simple and compound interest. You also determine the different formula
and relationship involve in the simple and compound interest environment which
will aid in computing the required components. In this module, you will practice your
ability to think critically in solving problems related to this topic

Furthermore, realization regarding the importance and value of money is seen


because the problems involve for you to learn this competency are all related to
business transactions. You will explore the different problems related to investment
and loans to different banks and lending companies. At the end of this module, it is
hoped that you will practice a better judgement on spending your money.

Are you now ready for the new lesson? If so, you may proceed to this module and
have fun while learning.

This module is composed of two lessons namely:

Lesson 1 – Solving Problems Involving Simple Interest

Lesson 2 - Solving Problems Involving Compound Interest

After going through this module, you are expected to:

1. solve problems involving simple and compound interest; and


2. apply the different formulas involve in simple and compound interest
environment in solving problems.

1 CO_Q2_General Mathematics SHS


Module 3
What I Know

Let’s find out how far you might already know about this topic by answering the
assessment below.

Choose the letter of the best answer. Write the chosen letter on a separate sheet of
paper.

For numbers 1- 5, use the situation below.

Michael borrowed money from Castañas Cooperative Bank at 1.75% simple


interest for 5 years to add to his funds for his new business. It was charged
₱ 5,250.00 for interest.

1. What formula will be used to determine the amount of borrowed money?


a. 𝐼𝐼𝑠𝑠 = 𝑃𝑃𝑃𝑃𝑡𝑡
𝐼𝐼𝑠𝑠
b. 𝑃𝑃 =
𝑟𝑟𝑟𝑟
c. 𝐼𝐼𝑐𝑐 = 𝐹𝐹(1 + 𝑗𝑗)𝑛𝑛
𝐹𝐹
d. 𝑃𝑃 =
(1+𝑟𝑟)𝑡𝑡

2. How much money will Michael add on his funds for his new business?
a. ₱ 459.38
b. ₱ 4,593.75
c. ₱ 6,000.00
d. ₱ 60,000.00

3. How much will he need to pay after 5 years?


a. ₱ 5,709.38
b. ₱ 9,843.75
c. ₱ 11,250.00
d. ₱ 65,250.00

4. If Michael borrowed the same amount of money payable for the same number
of years and Castañas Cooperative Bank offered a promo as part of its
anniversary celebration and make all the interest loans 1.25%, how much
interest must be paid?
a. ₱ 3,750.00
b. ₱ 37,500.00
c. ₱ 63,750.00
d. ₱ 70,000.00

2 CO_Q2_General Mathematics SHS


Module 3
5. How much will he need to pay after 6 years if he availed the promo?
a. ₱ 64,500.00
b. ₱ 65,250.00
c. ₱ 70,000.00
d. ₱ 74,500.00

For numbers 6 – 10 use the problem below.

Gerson Joseph has ₱ 300,000.00 to invest at 1.12% compounded quarterly.

6. What is the total number of compounding frequency if the investment matures


after 6 years?
a. 4
b. 6
c. 10
d. 24

7. How much is the interest rate per conversion period?


a. 1.12%
b. 0.19%
c. 0.28%
d. 0.05%

8. How much is the maturity value of his investment in 4 years?


a. ₱ 300,000.00
b. ₱ 313,725.96
c. ₱ 466,671.30
d. ₱ 467,476.38

9. How much is the compound interest after 4 years?


a. ₱ 3,000.00
b. ₱ 13,725.96
c. ₱ 16,671.30
d. ₱ 16,746.38

10. How much is the additional interest earned if instead of 4 years the investment
matures after 6 years?
a. ₱ 7,096.72
b. ₱ 13,725.96
c. ₱ 20,822.68
d. ₱ 34,548.64

3 CO_Q2_General Mathematics SHS


Module 3
For numbers 11- 15 use the problem below:

Jeanelle is planning to borrow money amounting to ₱ 250,000.00. Limbon


Rural Bank is offering 1.25% compounded semi – annually, while Buli
Cooperative Bank offers 1% compounded monthly.

11. How much will be the maturity value after 5 years if he borrows from Limbon
Rural Bank?
a. ₱ 266,071.86
b. ₱ 207,660.68
c. ₱ 367,071.88
d. ₱ 376,716. 66

12. How much will be the maturity value after 5 years if he borrows from Buli
Cooperative Bank?
a. ₱ 207,560.78
b. ₱ 262,812.30
c. ₱ 376,431.54
d. ₱ 386,221.32

13. What will be the interest after 5 years if he borrows from Limbon Rural Bank?
a. ₱ 126,716.66
b. ₱ 117,071.88
c. ₱ 42,339.32
d. ₱ 16,071.86

14. What will be the interest after 5 years if he borrows from Buli Cooperative
Bank?
a. ₱ 136,221.32
b. ₱ 126,431.54
c. ₱ 12,812.30
d. ₱ 2,812.53

15. Which bank offers the smaller interest?


a. Both
b. None
c. Limbon Rural Bank
d. Buli Cooperative Bank

4 CO_Q2_General Mathematics SHS


Module 3
Lesson
Solving Problems Involving
1 Simple Interest

One of the most common scenario in our daily lives is engaging ourselves in a
business transaction. It is all but natural because many actions that we undertake
involves money. Moreover, when we are talking about money we usually involve
ourselves with the different problems that need solutions especially when it comes
to the computation of interest and the total amount to be paid at the end of a
particular term. So, are you ready to systematically solve problems involving simple
interest?

What’s In

For you to begin, consider the previous lesson which is essential in this topic.

You already learned that in simple interest the basis of computation is the original
principal. There are also different formula to be used in finding the different
components in simple interest and these are the following:

𝐼𝐼𝑠𝑠 = 𝑃𝑃𝑃𝑃𝑃𝑃 ; simple interest

𝐹𝐹 = 𝑃𝑃(1 + 𝑟𝑟𝑟𝑟) or = 𝑃𝑃 + 𝐼𝐼𝑠𝑠 ; maturity value


𝐼𝐼𝑠𝑠
𝑃𝑃 = ; principal or present value
𝑟𝑟𝑟𝑟

where:

P= principal or present value


F = maturity value

r= rate of interest

t= time in years
Is = Simple Interest

5 CO_Q2_General Mathematics SHS


Module 3
Let us consider the following examples:

1. Find Is, if P = ₱ 4,500.00, t= 3 and r = 2.5% = 0.025 (in decimal form)

𝐼𝐼𝑠𝑠 = 4500(0.025)(3)

𝐼𝐼𝑠𝑠 = ₱ 337.50
The simple interest is ₱ 337.50

2. If P = ₱ 3,375.00, r= 1.2% and t= 2 years find F

𝐹𝐹 = 𝑃𝑃(1 + 𝑟𝑟𝑟𝑟)

𝐹𝐹 = 3375[1 + (0.012)(2)]
F= ₱ 3,456.00
The maturity value is ₱ 3,456.00

Notes to the Teacher


Advise the learners to use scientific calculators because of the
nature of the numbers that will be computed. Remind them as well
that there are some formulas that will be used in this module that
has a different representation of variables in other books. Both will
yield the same answer as long as they represent the same
components.

6 CO_Q2_General Mathematics SHS


Module 3
What’s New

This or That

Febie is planning to invest her money in a bank. She researched some banks which
will make a better offer. She narrowed down her choices to these two banks which
have the following offers:

Insurance Bank, a bank that accepts an initial deposit of

₱ 5,000.00 with a simple interest of 1.5% in 3 years.

Trust Bank, a bank that accepts an initial deposit of

₱ 5,000.00 with a simple interest of 1.75% for 6 years

Questions:

1. What are the factors that Febie can consider in choosing a bank?
________________________________________________________________________

2. What are the differences between the offers made by the two banks?
________________________________________________________________________

3. How much interest is offered by the Insurance Bank? Trust Bank?


_______________________________________________________________________

4. If you were Febie, which will you choose between the two banks? Why?
_______________________________________________________________________

5. If Febie will invest ₱ 5,000.00, how much will be its maturity value in Insurance
Bank? Trust Bank?
_______________________________________________________________________

6. If you were Febie, what will be your basis in choosing a bank? Which bank will
you choose?

_______________________________________________________________________

7 CO_Q2_General Mathematics SHS


Module 3
What is It

If you noticed in the activity, there are two banks that offer different condition in
earning interest. Insurance Bank offers a rate that is smaller compared to Trust
bank. However, the time covered by the Insurance Bank is shorter. If Febie will
deposit on Insurance bank her ₱5,000.00 savings will yield an interest of ₱225.00
after 3 years. On the other hand, if she chooses to deposit her money in Trust Bank
the interest that she will earn is ₱525.00 but this will only be realized after 6 years.
The choice for the bank where Febie will invest her money will depend upon her
prerogative if she wanted to have a long term investment she can choose Trust Bank
but if she thinks that she will need money in the near future she can invest in the
Insurance Bank. There are times that the amount of return is not the only factor to
consider in our choice since there are other components involved in the simple
interest formula and that is what we will explore in this lesson by solving problems
involving simple interest.

In solving problems involving simple interest, we can follow the 4 –step rule of George
Polya.

George Polya’s 4 – Step Rule


Explore. This step involves careful reading, analyzing,
identifying the given and unknown facts in the problem and
expressing the unknown in terms of variables.
Plan. In this step, writing an equation that describes the
relationships between or among the variables is involved.
Solve. This step requires working out with the written
equation and other number relations to determine the
required quantities that answer the question in the problem.
Check. The final step that employs the use of other
approaches to examine the appropriateness of the answer.

8 CO_Q2_General Mathematics SHS


Module 3
Let us take the following examples.

1. Mariel deposited ₱ 16,000.00 in a bank that offers a simple interest rate of


1.75%. If she placed the money for 7 years, how much interest will she earn?

2. Janice has a loan with an interest rate of 1.5%. The amount of interest is
₱ 2,250.00 for a period of 3 years. How much is her loan?

In solving the problems, we will use the steps provided.

1. Mariel deposited ₱ 16,000.00 in a bank that offers a simple interest rate of


1.75%. If she placed the money for 7 years, how much interest will she earn?

Explore. After reading and understanding the problem, it will be determined that
the required component is the simple interest and the following are the known
values:

P = ₱ 16,000.00

r = 1.75% or 0. 0175

t= 7 years

Plan. Since the value of simple interest is required, the formula 𝐼𝐼𝑠𝑠 = 𝑃𝑃𝑃𝑃𝑃𝑃 will be
applied.

Solve: Using the formula we will arrive at:

𝐼𝐼𝑠𝑠 = 16,000(0.0175)(7)

𝐼𝐼𝑠𝑠 = ₱ 1,960.00

Check. As for checking, we can use other formulas related to simple interest and
substitute the value of simple interest. If the obtained value is any of the given facts,
then you obtain the correct answer.

Mariel earned ₱ 1,960.00 interest.

9 CO_Q2_General Mathematics SHS


Module 3
2. Janice has a loan with an interest rate of 1.5%. The amount of interest is
₱ 2,250.00 for a period of 3 years. How much is her loan?

Explore. The known values are

𝐼𝐼𝑠𝑠 = ₱ 2,250.00

r = 1.5% or 0.015

t = 3 years

The required value is the Principal or present value

Plan. Since the problem requires the present value or the principal the formula to
be used is

𝐼𝐼𝑠𝑠
𝑃𝑃 =
𝑟𝑟𝑟𝑟

Solve: Using the formula, we will arrive at:

𝐼𝐼𝑠𝑠
𝑃𝑃 =
𝑟𝑟𝑟𝑟

2,250
𝑃𝑃 =
(0.015)(3)

𝑃𝑃 = ₱ 50,000

Check. As for checking, the formula in finding the simple interest can be used.

Thus, the amount of her loan is ₱ 50,000.00

10 CO_Q2_General Mathematics SHS


Module 3
What’s More

Independent Practice 1

How much did Josie invest if the maturity value is ₱38,500.00 and the
simple interest computed is at 1.12% for 6 years?

Guide Questions

1. What is required in the problem? _________________________________

2. What are the given facts? ________________________________________

3. What formula will be used to solve the problem? _________________

4. How much is the original investment? ____________________________

Independent Assessment 1

Solve the problem in a separate sheet of paper.

Jamie invested a certain amount of money in JDJ Savings Bank with a


2.5% interest rate. He received ₱ 2,250.00 as interest after 2 years from his
investment. How much did he invest?

Marinel received ₱ 1,450,500.00 as her inheritance from her parents.


She deposited the said amount in a time deposit with 1% interest rate per
annum, how much money will be accumulated after 7 years?

11 CO_Q2_General Mathematics SHS


Module 3
What I Have Learned

Write a synthesis journal from the different activities and discussion you
encountered from this module. You can recall past experiences and real-life
problems and how you can solve and apply it in the future.

Synthesis Journal

What I did What I learned How I can use it

12 CO_Q2_General Mathematics SHS


Module 3
What I Can Do

Making Money Happen


You are an investor who is aiming to accumulate a money amounting to ₱100,000.00
at the end of a specific year. Make plans of the amount to be invested, terms and
rate of investment for simple interest then choose which between the plans is more
realistic for a starting investor.

Components Plan 1 Plan 2


Present Value
Rate
Time
Simple Interest
Maturity Value

Plan Chosen: ________________________________________________________

Reason for Choice: ___________________________________________________

____________________________________________________

Rubric for Scoring:

Categories Excellent Fair Poor


3 2 1

Decision Making Excellent decision Fair decision Poor decision


Skills making skills is making skills is making skills is
fostered. fostered. fostered.

Planning The goal set is The goal set is The goal set is not
achievable and difficult to achievable and
realistic. achieve. not realistic.

Accuracy of The computations The computations There is no


Solution made are all made have flaws. attempt in
correct. making
computation .

13 CO_Q2_General Mathematics SHS


Module 3
Lesson
Solving Problems Involving
2 Compound Interest
There are instances wherein the borrower was surprised because the amount of the
outstanding balance was changed when they failed to pay their monthly dues. This
happens because instead of simple interest the lender use compound interest in
computing the maturity value. As a borrower or investor, we must be aware with the
different conditions stated in any transactions we engaged in. Our ability to be
critical in whatever transactions that we will do, will be of great help in this topic
which is solving problems involving compound interest.

What’s In

Before starting this lesson, consider the previous module which is essential in this
topic.

You already learned that in compound interest the basis of computing the interest
is not only the principal but also the accumulated interest until such term.

There is also different formula in getting the different components of the compound
interest and they are the following:

𝐹𝐹 = 𝑃𝑃(1 + 𝑟𝑟)𝑡𝑡

𝐹𝐹 = 𝑃𝑃(1 + 𝑗𝑗)𝑚𝑚𝑚𝑚 ; maturity value for compounding more than once a year

𝐼𝐼𝑐𝑐 = 𝐹𝐹 − 𝑃𝑃 ; Compound Interest

𝐹𝐹
𝑃𝑃 = ; present value
(1+𝑗𝑗)𝑚𝑚𝑚𝑚

𝑖𝑖 (𝑚𝑚)
j=
𝑚𝑚

14 CO_Q2_General Mathematics SHS


Module 3
Where:
F= maturity value
Ic = compound interest
P= present value or principal amount
t = time in years
r = rate of interest
j= rate of interest for each conversion period
m=frequency of conversion
𝑖𝑖 (𝑚𝑚) =annual rate of interest
n= frequency of conversion x time in years
Consider the example below:

If P= ₱ 20,500.00, r= 2% and compounded annually for 3 years, find Ic and F


𝐹𝐹 = 𝑃𝑃(1 + 𝑟𝑟)𝑡𝑡
𝐹𝐹 = 20,500(1 + 0.02)3
F= ₱ 21,754.76
𝐼𝐼𝑐𝑐 = 𝐹𝐹 − 𝑃𝑃
𝐼𝐼𝑐𝑐 = 𝐹𝐹 − 𝑃𝑃
𝐼𝐼𝑐𝑐 = ₱ 1,254.76
The maturity value is ₱ 21,754.76 and the compound interest is ₱ 1,254.76

Notes to the Teacher


Advise the learners to use scientific calculators because of the
nature of the numbers that will be computed. Remind them as
well that there are some formulas that will be used in this module
that have different representations of variables in other books.
Both will yield the same answer as long as they represent the
same components.

15 CO_Q2_General Mathematics SHS


Module 3
What’s New

Which is Which?

Miguel wanted to apply for a loan in a lending company. The company offers different
terms of loan which are as follows:

₱ 50,000.00 loan at 1.15% interest compounded semi-annually for


3 years

₱ 50,000.00 loan at 1.25% interest compounded annually for 3


years

Questions:

1. What are the factors that Miguel can consider in choosing the terms of his loan?
________________________________________________________________________

2. What are the differences between the two terms of loans?


________________________________________________________________________

3. How much interest will a ₱ 50,000.00 loan at 1.15% yield? How much is the
interest of ₱ 50,000.00 loan at 1.25%?
_______________________________________________________________________

4. If you were Miguel, which between the two terms will you choose? Why?
_______________________________________________________________________

16 CO_Q2_General Mathematics SHS


Module 3
What is It

In the previous activity, the type of interest that was used is compound In the
previous activity, the type of interest that was used is compound interest. However,
there are different terms and conditions that were used. For Miguel to choose wisely
the terms he will use for his loan application, there is a need for him to analyze the
given conditions first. If he will choose to apply for a loan with 1.15% interest
compounded semi - annually for 3 years he will need to pay an interest amounting
to ₱ 1,749.99. However, if he chooses to apply a loan at 1.25%interest compounded
annually for 3 years he will need to pay ₱ 1,898.54. So, interest wise it is better to
apply for a loan with 1.15% interest compounded semi – annually for 3 years.

In choosing the terms, it is important to analyze first the component and the
conditions presented so that you can arrive in wise decision. This is also the things
that you must remember in solving problems involving compound interest.

In solving problems involving compound interest, we can follow the 4 –step


rule of George Polya.

George Polya’s 4 – Step Rule


Explore. This step involves careful reding, analyzing,
identifying the given and unknown facts in the problem and
expressing the unknown in terms of variables.
Plan. In this step writing, an equation that describes the
relationships between or among the variables is involved.
Solve. This step requires working out with the written
equation and other number relations to determine the required
quantities that answer the question in the problem.
Check. The final step that employs the use of other
approaches to examine the appropriateness of the answer.

17 CO_Q2_General Mathematics SHS


Module 3
Let us take the following examples:

1. Jasmin borrowed money from a bank at the rate of 1.89% compounded semi
– annually. If the amount she borrowed is ₱ 13,000.00, how much interest will
she need to pay at the end of 6 years?

2. Arthur wants to set aside an amount to be invested in a fund earning 1.02%


compounded quarterly, if he wants to accumulate ₱ 250,000.00 in 4 years,
how much must he set aside?

Solving the problems, we will use the steps provided.

1. Jasmin borrowed money from a bank at the rate of 1.89% compounded semi
– annually. If the amount she borrowed is ₱ 13,000.00, how much interest
will she need to pay at the end of 6 years?

Explore. The known values are

P = ₱ 13,000.00

𝑖𝑖 (2) =0.0189

m=2

t=6

mt=12
𝑖𝑖 2 0.0189
𝑗𝑗 = = = 0.00945
2 2

The required variable is the compound interest

Plan. Since the problem suggest that it is compounding more than once a year the
formulas:

𝐹𝐹 = 𝑃𝑃(1 + 𝑗𝑗)𝑚𝑚𝑚𝑚 and 𝐼𝐼𝑐𝑐 = 𝐹𝐹 − 𝑃𝑃 will be employed.

There is a need to find first the maturity value because it is an important element in
the compound interest formula.

18 CO_Q2_General Mathematics SHS


Module 3
Solve: Using the formula we will arrive at:

𝐹𝐹 = 𝑃𝑃(1 + 𝑗𝑗)𝑚𝑚𝑚𝑚

𝐹𝐹 = 13,000(1 + 0.00945)12

𝐹𝐹 = 14,553.29

After finding the maturity value, we can solve for the compound interest:

𝐼𝐼𝑐𝑐 = 𝐹𝐹 − 𝑃𝑃
𝐼𝐼𝑐𝑐 = 14,553.29 − 13,000

𝐼𝐼𝑐𝑐 = ₱ 1,553.29

Check. As for checking, the formula in finding the present value can be used.

Thus, the compound interest is ₱ 1,553.29

2. Arthur wants to set aside an amount to be invested in a fund earning 1.02%


compounded quarterly, if he wants to accumulate ₱ 250,000.00 in 4 years,
how much must he set aside?

Explore. The known values are

F = ₱ 250,000.00
𝑖𝑖 (4) =0.0102
m=4
𝑖𝑖 4 0.0102
j= = = 0.00255
4 4
t=4
mt=16

The required value is the principal or present value

19 CO_Q2_General Mathematics SHS


Module 3
Plan. Since the problem suggest that it is compounding more than once a year
the formulas:

𝐹𝐹 = 𝑃𝑃(1 + 𝑗𝑗)𝑚𝑚𝑚𝑚 and 𝐼𝐼𝑐𝑐 = 𝐹𝐹 − 𝑃𝑃 will be employed.

There is a need to find first the maturity value because it is an important element
in the compound interest formula.

Solve: Using the formula we will arrive at:

𝐹𝐹 = 𝑃𝑃(1 + 𝑗𝑗)𝑚𝑚𝑚𝑚

250,000 = 𝑃𝑃(1 + 0.00255)(4)(4)

250,000 = 𝑃𝑃(1.00255)16

250,000 𝑃𝑃(1.00255)16
=
(1.00255)16 (1.00255)16

𝑃𝑃 = 240,017.74

After finding the maturity value, we can solve for the compound interest:

𝐼𝐼𝑐𝑐 = 𝐹𝐹 − 𝑃𝑃

𝐼𝐼𝑐𝑐 = 250,000 − 240,017.74

𝐼𝐼𝑐𝑐 = ₱ 9,982.26

Check. As for checking, the formula in finding the present value can be used.

Thus, the compound interest is ₱ 9,982.2

20 CO_Q2_General Mathematics SHS


Module 3
What’s More

Independent Practice 1

How much must Michelle set aside and invest in a fund earning 2.1%
compounded quarterly if she wants to accumulate ₱ 50,000.00 in 5 years?

1. Explore: ______________________________________

2. Plan: ________________________________________

3. Solve: ________________________________________

4. Check: _______________________________________

Independent Assessment 1

How much should be deposited in a bank paying 1.15% compounded semi –


annually to accumulate an amount of ₱ 80,000.00 in 5 years?

21 CO_Q2_General Mathematics SHS


Module 3
What I Have Learned

Write a synthesis journal from the different activities and discussion you
encountered from this module. You can recall past experiences and real-life
problems and how you can solve and apply it in the future.

Synthesis Journal

What I did What I learned How I can use it

22 CO_Q2_General Mathematics SHS


Module 3
What I Can Do

You wanted to put up a business and planning to apply for a loan for your starting
capital. The lending company where you inquire offers Flexi Loan for new
entrepreneurs but the type of interest to be used is compound interest. Flexi Loan is
a program wherein the new entrepreneur will propose the term of loan and payment
subject for the approval of the management. Suppose you are a new entrepreneur,
make a proposal for your loan using a compound interest environment.

LOAN PROPOSAL

Name: _________________________________________________________________
Business to put up: ____________________________________________________
Brief Description of Business: ___________________________________________
Source of Funds: ________________________________________________________
Amount of Loan: _______________________________________________________
Nominal rate: __________________________________________________________
Frequency of Conversion: _______________________________________________
Time in years: ________________________________________________________
Compound Interest: __________________________________________________
Maturity Value: _______________________________________________________
Terms of Payment (Monthly, Quarterly, Semi Annually, Annually):
______________________________________________________________________
Amount of Payment per term: ___________________________________________
Co Maker: ______________________________________________________________

23 CO_Q2_General Mathematics SHS


Module 3
Rubric for Scoring:

Categories Excellent Fair Poor


3 2 1

Decision Making Excellent decision Fair decision Poor decision


Skills making skills is making skills is making skills is
fostered. fostered. fostered.

Planning The goal set is The goal set is The goal set is not
achievable and difficult to achievable and
realistic. achieve. not realistic.

Accuracy of The computations The computations There is no


Solution made are all made have flaws. attempt in
correct. making
computation.

24 CO_Q2_General Mathematics SHS


Module 3
Assessment

Choose the letter of the best answer. Write the chosen letter on a separate sheet of
paper.

For numbers 1- 5, use the situation below.


Jenny invested money from Lucena Rural Bank at 1.05% simple interest for 3 years.
It was charged ₱ 472.50 for interest.

1. What formula will be used to determine the amount invested?


a. 𝐼𝐼𝑠𝑠 = 𝑃𝑃𝑃𝑃𝑃𝑃
𝐼𝐼𝑠𝑠
b. 𝑃𝑃 =
𝑟𝑟𝑟𝑟
c. 𝐼𝐼𝑐𝑐 = 𝐹𝐹(1 + 𝑗𝑗)𝑛𝑛
𝐹𝐹
d. 𝑃𝑃 =
(1+𝑟𝑟)𝑡𝑡

2. How much money did Jenny invest?


a. ₱ 1,500.00
b. ₱ 15,000.00
c. ₱ 150,000.00
d. ₱ 600,000.00

3. How much is her accumulated money after 3 years?


a. ₱ 1,947.50.00
b. ₱ 15,472.50.00
c. ₱ 150,472.50.00
d. ₱ 600,472.50.00

4. If Jenny invested the same amount of money at Lucena Cooperative Bank who
is offering an interest rate of 1.5% for 3 years how much interest will she gain?
a. ₱ 325.00
b. ₱ 475.00
c. ₱ 675.00
d. ₱ 825.00

25 CO_Q2_General Mathematics SHS


Module 3
5. How much will she accumulate in 3 years if she invested in Lucena
Cooperative Bank?
a. ₱ 15,325.00
b. ₱ 15,425.00
c. ₱ 15,625.00
d. ₱ 15,675.00

For numbers 6 – 10 use the problem below.

Joy has ₱ 100,000.00 to invest at 1.11% compounded monthly.

6. What is the total number of compounding frequency if the investment matures


after 6 years?
a. 6
b. 12
c. 18
d. 72

7. How much is the interest rate per conversion period?


a. 0.0925%
b. 0.925%
c. 9.25%
d. 92.5%

8. How much is the maturity value of his investment in 4 years?


a. ₱ 104,537.90
b. ₱ 113,725.96
c. ₱ 166,671.30
d. ₱ 167,476.38

9. How much is the compound interest after 4 years?


a. ₱ 4,537.90
b. ₱ 13,725.96
c. ₱ 66,671.30
d. ₱ 67,746.38

26 CO_Q2_General Mathematics SHS


Module 3
10. How much is the additional interest earned if instead of 4 years the investment
matures after 6 years?
a. ₱ 6,883.49
b. ₱ 4,537.90
c. ₱ 5,822.68
d. ₱ 2,345.59

For numbers 11- 15 use the problem below:

Shirley is planning to invest ₱ 150,000.00. San Jose Rural Bank is offering 1.15%
compounded semi – annually, while Taysan Cooperative Bank offers 1.25%
compounded monthly.

11. How much will be the maturity value after 5 years if he borrows from San Jose
Rural Bank?
a. ₱ 158,851.63
b. ₱ 207,660.68
c. ₱ 367,071.88
d. ₱ 376,716.66

12. How much will be the maturity value after 5 years if he borrows from Taysan
Cooperative Bank?
a. ₱ 159,668.97 c. ₱ 376,431.54
b. ₱ 262,812.30 d. ₱ 386,221.32

13. What will be the interest after 5 years if he borrows from San Jose Rural Bank?
a. ₱ 26,716.66
b. ₱ 17,071.88
c. ₱ 12,339.32
d. ₱ 8,851.63

14. What will be the interest after 5 years if he borrows from Taysan Cooperative
Bank?
a. ₱ 36,221.32
b. ₱ 16,431.54
c. ₱ 9,668.97
d. ₱ 2,812.53

27 CO_Q2_General Mathematics SHS


Module 3
15. Which bank offers the smaller interest?
a. Both
b. None
c. San Jose Rural Bank
d.Taysan Rural Bank

Additional Activities

Solve the following problems on compound interest.

1. James invested ₱150,000.00 at 2.5% interest compounded semi – annually.


Find the maturity value if he invests for (a) 4 years? (b) 8 years? (c) How much
is the additional interest earned due to the longer time?

2. Jenny is planning to deposit ₱17,000.00 Quezon Metropolitan Bank is offering


7.5% compounded semi-annually for 5 years while Quezon Premier Bank is
offering 7% compounded monthly for 5 years. Which bank should she deposit
her money?

28 CO_Q2_General Mathematics SHS


Module 3
Module 3
CO_Q2_General Mathematics SHS 29
What's More
What I Know Lesson 1
Independent Practice 1
Assessment
1. B 1. B
2. D Principal Amount/Present
2. B
3. D Value
3. B
4. A F= ₱38,500, r = 1.12% or 4. C
5. A 0.0112 5. D
6. D t = 6 years 6. D
7. C F = P(1 + rt) 7. A
8. B 8. A
9. B ₱36,075.71
9. A
10. A Independent Assessment 1 10. D
11. A ₱45,000.00 11. A
12. B ₱1,552,035 12. A
13. D Lesson 2 13. D
14. C Independent Practice 1 14. C
15. D Explore: The known values are 15. C
F = ₱50,000.00
i = 2.1% or 0.021
m=4
t=5
j=
Plan: The required variable is the
Principal or the present value. The
formula is
Solve: ₱45,028.59
Check: Use the formula
Independent Assessment 1
₱75,542.19
Answer Key
References
General Mathematics Learner's Material. Pasig City: Department of Education,
2016.
General Mathematics Teacher's Guide. Pasig City: Department of Education, 2016.
Orines, Fernando B. Next Century Mathematics. Quezon, City: Phoenix Publishing
House, 2016.
Oronce, Orlando A. General Mathematics. Sampaloc, Manila: Re Bookstore, Inc.,
2017.
Santos, Durawin C.,Biason, Ma. Garnet P. Math Activated:Engage Yourself and Our
World General Mathematics. Makati, City: Salesiana Books by Don Bosco
Press, Inc., 2016.

30 CO_Q2_General Mathematics SHS


Module 3
For inquiries or feedback, please write or call:

Department of Education - Bureau of Learning Resources (DepEd-BLR)

Ground Floor, Bonifacio Bldg., DepEd Complex


Meralco Avenue, Pasig City, Philippines 1600

Telefax: (632) 8634-1072; 8634-1054; 8631-4985

Email Address: blr.lrqad@deped.gov.ph * blr.lrpd@deped.gov.ph


General Mathematics
Quarter 2 – Module 4:
Simple and General Annuities

CO_Q2_General Mathematics SHS


Module 4
General Mathematics – Senior High School
Alternative Delivery Mode
Quarter 2 – Module 4: Simple and General Annuities
First Edition, 2021

Republic Act 8293, section 176 states that: No copyright shall subsist in any work
of the Government of the Philippines. However, prior approval of the government agency or
office wherein the work is created shall be necessary for exploitation of such work for profit.
Such agency or office may, among other things, impose as a condition the payment of
royalties.

Borrowed materials (i.e., songs, stories, poems, pictures, photos, brand names,
trademarks, etc.) included in this module are owned by their respective copyright holders.
Every effort has been exerted to locate and seek permission to use these materials from
their respective copyright owners. The publisher and authors do not represent nor claim
ownership over them.

Published by the Department of Education


Secretary: Leonor Magtolis Briones
Undersecretary: Diosdado M. San Antonio

Development Team of the Module


Writers: Dennis E. Ibarrola
Editors: Elizabeth D. Lalunio, Elizabeth B. Dizon Anicia J. Villaruel and Roy O. Natividad
Reviewers: Fritz A. Caturay, Necitas F. Constante, Dexter M. Valle,
Jerome A. Chavez, January Regio and Maribel D. Millares
Illustrator: Hanna Lorraine Luna, Diane C. Jupiter and Meryll C Calvendra
Layout Artist: Roy O. Natividad, Sayre M. Dialola, Noel Rey T. Estuita and Argie L. Ty
Management Team: Francis Cesar B. Bringas
Job S. Zape, Jr.
Ramonito Elumbaring
Reicon C. Condes
Elaine T. Balaogan
Fe M. Ong-ongowan
Hermogenes M. Panganiban
Phillip B. Gallendez
Josephine T. Natividad
Anicia J. Villaruel
Dexter M. Valle

Printed in the Philippines by ________________________

Department of Education – Region 4A CALABARZON

Office Address: Gate 2 Karangalan Village, Brgy. San Isidro, Cainta, Rizal
Telefax: 02-8682-5773/8684-4914/8647-7487
E-mail Address: lrmd.calabarzon@deped.gov.ph
General Mathematics
Quarter 2 – Module 4:
Simple and General Annuities
Introductory Message

This Self-Learning Module (SLM) is prepared so that you, our dear learners,
can continue your studies and learn while at home. Activities, questions,
directions, exercises, and discussions are carefully stated for you to understand
each lesson.

Each SLM is composed of different parts. Each part shall guide you step-by-
step as you discover and understand the lesson prepared for you.

Pre-tests are provided to measure your prior knowledge on lessons in each


SLM. This will tell you if you need to proceed on completing this module or if you
need to ask your facilitator or your teacher’s assistance for better understanding of
the lesson. At the end of each module, you need to answer the post-test to self-
check your learning. Answer keys are provided for each activity and test. We trust
that you will be honest in using these.

In addition to the material in the main text, Notes to the Teacher are also
provided to our facilitators and parents for strategies and reminders on how they
can best help you on your home-based learning.

Please use this module with care. Do not put unnecessary marks on any
part of this SLM. Use a separate sheet of paper in answering the exercises and tests.
And read the instructions carefully before performing each task.

If you have any questions in using this SLM or any difficulty in answering
the tasks in this module, do not hesitate to consult your teacher or facilitator.

Thank you.
What I Need to Know

One of the dreams of every Filipino family is to purchase a house where they
can comfortably live. However, most often they cannot afford to purchase such
property in cash. One of the options to realize their dreams is to purchase it on
an installment basis. In this situation, the concept of annuities is being used.

In this learning module, learners should be able to investigate and analyze


problems involving simple and general annuities. This module was designed
and written with you in mind. It is here to help you identify and distinguish
simple and general annuities.

After going through this module, you are expected to:


1. define terms that are related to simple and general annuities;
2. illustrates and distinguishes simple and general annuities; and
3. represent annuities to real-life situations.

1
CO_Q2_General Mathematics SHS
Module 4
What I Know

Let’s find out how far you might already know about this topic! Please take this
challenge! Have Fun!

Multiple Choice. Choose the letter of the best answer. Write the chosen letter on
a separate sheet of paper.

1. What is referred to as a fixed sum of money paid to someone at regular


intervals, subject to a fixed compound interest rate?
a. compound interest c. annuity
b. simple interest d. annuity certain

2. What type of annuity is illustrated when the interest conversion or


compounding period is equal or the same as the payment interval?
a. annuity certain c. simple annuity
b. annuity uncertain d. general annuity

3. What type of annuity is illustrated when the interest conversion or


compounding period is unequal or not the same as the payment interval?
a. annuity certain c. simple annuity
b. annuity uncertain d. general annuity

4. What do you call the fixed payment and interest over a specified time?
a. annuity payment c. due payment
b. interest payment d. general payment

5. Which is an example of annuities?


a. interest c. bills
b. rent d. savings

6. What do you call the type of annuity in which the payments are made at
the end of each payment interval?
a. annuity certain c. ordinary annuity
b. contingent annuity d. general annuity

7. What do you call to the type of annuity which payments begin and end at
definite times?
a. contingent annuity c. simple annuity
b. ordinary annuity d. annuity certain

2
CO_Q2_General Mathematics SHS
Module 4
8. What do you call the type of annuity in which the payments extend over
an indefinite length of time?
a. contingent annuity c. general annuity
b. annuity certain d. ordinary annuity

9. A payment made at the end of each month for money borrowed that
charges 1.05% interest compounded quarterly. What type of annuities is
being represented?
a. simple c. ordinary
b. general d. annuity due

10. What type of annuity is represented by a deposit of ₱ 6,000.00 that is


made at the end of every three months to an account that earns 5.6%
interest compounded quarterly?
a. simple c. ordinary
b. general d. annuity due

11. Which of the following situations is an example of a simple annuity?


a. ₱ 2,000 deposited every six months for 2 years at 8% per year
compounded monthly
b. ₱ 2,000 deposited every month for 2 years at 8% per year
compounded annually
c. ₱ 2,000 deposited every month for 2 years at 8% per year
compounded monthly
d. ₱ 2,000 deposited every quarter for 2 years at 8% per year
compounded monthly
12. Which of the following situations is an example of a general annuity?
a. ₱ 5,000 deposited every month for 5 years at 8% per year
compounded annually
b. ₱ 5,000 deposited every six months for 5 years at 8% per year
compounded semi-annually
c. ₱ 5,000 deposited every three months for 5 years at 8% per year
compounded quarterly
c. ₱ 5,000 deposited every year for 5 years at 8% per year
compounded annually

3
CO_Q2_General Mathematics SHS
Module 4
For numbers 13-15. Given the cash flow of an annuity below.

Payment ₱ 1,000 ₱ 1,000 ₱ 1,000 ₱ 1,000 ₱ 1,000

Period 0 1 2 3 4 5
(Year)
₱ 1,000

₱ 1,000(1.05)1

₱ 1,000(1.05)2

₱ 1,000(1.05)3

₱ 1,000(1.05)4

13. What is the rate of interest?


a. 1.05% c. 0.05%
b. 5% d. 0.5%

14. What is the term of the payment?


a. 5 years c. 6 years
b. 4 years d. 1000 years

15. What is the sum of all the payments of the given annuity?
a. ₱ 5,000 c. ₱ 5,225.36
b. ₱ 5,455.82 d. ₱ 5,525.63

4
CO_Q2_General Mathematics SHS
Module 4
Lesson
Simple and General
1 Annuities

This lesson will tackle first the definition and types of annuities and the
difference between simple and compound annuities. There are types of annuity
according to correspondence of payment intervals with interest periods

Simple Annuity - the payment interval is also the same as the interest period.

General Annuity refers to an annuity where the length of the payment interval
is not the same as the length of the interest compounding period

What’s In

REMEMBER ME
As the saying goes, “A person who does not remember where he came from will
never reach his destination”. This saying is very popular, passing from one
generation to another generation. For you to remember the previous lesson,
here are some exercises to refresh your mind.

Activity 1
Answer each of the following.

1. If ₱40,000 is invested for 6 years at 5% compounded quarterly.


A. Give the value of the following using the formula � = � 1 + � � �ℎ��� � =


��� � = ��
1. P=______________
2. r=______________
3. i=_______________
4. n=______________

B. Find the compound amount.

Solution:

Answer:

5
CO_Q2_General Mathematics SHS
Module 4
C. Find the compound interest

Solution:

Answer:

2. ₱ 20,000 is invested for 15 years at 5 % compounded semi-annually.


A. Give the value of each variable using the formula � = � 1 + � �

�ℎ��� � = ��� � = ��

1. P=____________
2. r=_____________
3. i=_____________
4. n=_____________

B. Find the compound amount.

Solution:

Answer:

C. Find the compound interest.

Solution:

Answer:

6
CO_Q2_General Mathematics SHS
Module 4
What’s New

WISE DECISION

Mr. and Mrs. De Dios are planning to buy a new house despite of their limited
budget. They seek advice from a bank for them to produce enough amount for
the down payment of the house they have chosen.

According to the bank if ₱ 20,000 will be invested at the end of each year for 5
years in an account that pays interest at 10% compounded annually the couple
will be able to have the amount for down payment.

If there are 3 houses for sale offering a 10% down payment of the price which
among these 3 houses is the best choice?
House A – ₱850,740
House B – ₱1,221,020
House C – ₱2,110,000

For you to be able to determine the total amount of investment for the period of
5 years, you need to complete the table below.

Calculate the amount to which the first to the fourth period ₱ 20,000.00 will
grow using the compound interest formula.

Period Amount in Exponential Form Amount in Pesos

1 ₱20,000(1.01)4 ₱ 29,282 [No space]

2
3
4
5 ₱20,000 ₱ 20,000
Total =

Note: The ₱ 20,000 deposited at the end of the first year will draw interest for 4
years, so it will amount to ₱20,000(1.01)4 . The ₱20,000 deposited at the end of
the second year will draw interest for 3 years so it will amount to ₱20,000(1.01)3 ,
and so on.

7
CO_Q2_General Mathematics SHS
Module 4
1. What pattern will be reflected in the 2nd column of the table? Discuss.
___________________________________________________________________________
___________________________________________________________________________
___________________________________________________________________________
___________________________________________________________________________
___________________________________________________________________________
2. Using the pattern found in number 1, calculate the sum of the amounts in
the third column.

3. Given the answer in number 2, what will be the best house suited to the
investment made by Mr. and Mrs. De Dios? Explain.
___________________________________________________________________________
___________________________________________________________________________
___________________________________________________________________________
___________________________________________________________________________
___________________________________________________________________________

What is It

The activity in the What’s New involves fixed payment/investment for each
period (₱ 20,000 at the end of each year) and fixed compound interest rate over
a specified time; that in this case is 10% compounded annually for 5 years.

If the payment for each period is fixed and the compound interest rate is fixed
over a specified time the payment is called an annuity payment. Accounts
associated with streams of annuity payments are called annuities.

Annuity - a sequence of payments made at equal (fixed intervals or periods of


time.
The following are examples of annuities:
 Rental payment
 Monthly pensions
 Monthly payment for car loan
 Educational plan

8
CO_Q2_General Mathematics SHS
Module 4
Annuities may be classified in different ways, as follows.
Annuities
According to payment Simple Annuity - an General Annuity - an
interval and interest annuity where the annuity where the
period payment intervals is the payment intervals is not
same as the interest the same as the interest
period period
According to time of Ordinary Annuity (or Contingent Annuity -
payment Annuity Immediate) - a an annuity in which the
type of annuity in which payments extend over
the payments are made an indefinite (or
at the end of each indeterminate) length of
payment interval time
According to duration Annuity Certain - an Contingent Annuity -
annuity in which an annuity in which the
payments begin and end payments extend over
at definite times an indefinite (or
indeterminate) length of
time

In the activity, the payment is made at the end of each period. Such annuity is
called an ordinary annuity.

Each payment in an annuity is called the periodic payment (R).

The time between the successive payments dates of an annuity is called the
payment interval.

The time between the first payment interval and last payment interval is called
term of the annuity (t).

The sum of the future values of all the payments to be made during the entire
term of the annuity is the future value or the amount of an annuity (F).

The sum of the present values of all payments to be made during the entire
term of the annuity is called the present value of n annuity (P).

Annuities may be illustrated using a time diagram. The time diagram for an
ordinary annuity is given below.

Time Diagram for an n-Payment Ordinary Annuity


R R R R R… R

0 1 2 3 4 5 n

9
CO_Q2_General Mathematics SHS
Module 4
Simple Annuity - the payment interval is also the same as the interest period.

Example 1. ₱ 50,000 deposited every year for 5 years at 8% per year


compounded annually.

Solution: Notice that ₱ 50,000 was deposited every year and it is


compounded annually. Since the compounding period is similar to
the payment interval, then this is a type of simple annuity.

The cash flow of the given situation can be illustrated in the time diagram below:

Payment ₱ 50,000 ₱ 50,000 ₱ 50,000 ₱ 50,000 ₱ 50,000

Period 0 1 2 3 4 5

The future value of all the payments at the end of term (t=5).

Payment ₱ 50,000 ₱ 50,000 ₱ 50,000 ₱ 50,000 ₱ 50,000

Period 0 1 2 3 4 5
(Year)
₱ 50,000

₱ 50,000(1.08)1

₱ 50,000(1.08)2

₱ 50,000(1.08)3

₱ 50,000(1.08)4

Add all the future values obtained from the previous step.
₱ 50,000.00 = ₱ 50,000
1
₱ 50,000(1.08) = ₱ 54,000
₱ 50,000(1.08)2 = ₱ 58,320
₱ 50,000(1.08)3 = ₱ 62,985.60
₱ 50,000(1.08)4 = ₱ 68,024.45
Total ₱ 293,330.05 ₱ 293,330.05

10
CO_Q2_General Mathematics SHS
Module 4
General annuity refers to an annuity where the length of the payment interval
is not the same as the length of the interest compounding period.

Examples of General annuity:


1. Monthly installment of a car, lot or house with an interest rate that is
compounded annually.
2. Paying a debt semi-annually when the interest is compounded monthly.

Example 2: Find the amount of annuity of ₱700 every 6 months (½ year) for 12
years if interest is 6% compounded monthly.

Solution: Here, the payment interval (½ year) is different than the


interest period (annual). This is a general annuity.

We must match the interest period to the payment interval.

We must find the semi-annual rate that is equivalent to 6%,


compounded monthly.

Step 1: Using the formula � = � 1 + � � , find the value of ₱1 invested at


6%, compounded monthly after 1 year.
12
� = 1 1 + 0.005 Note: I = 0.06/12 = 0.005, n = 12

= 1.061677812

Step 2: Let the equivalent ½ year rate be i% (Note the equivalent yearly
rate would be 2i%). Now find the value of ₱1 invested at i% per ½ year
after 1 year.
2
�=1 1+� Note: n = 2, the number of times interest is
compounded

Step 3: These two amounts must be equal. Hence


2
1+� = 1.061677812

1+ i = 1.061677812 = 1.030377509

i= 0.030377509 or 3.0377509% (per ½ year)

Therefore, the equivalent ½ year rate is 3.0377509%

11
CO_Q2_General Mathematics SHS
Module 4
Now, we can illustrate the cash flow:

Payment ₱ 700 ₱ 700 ₱700 … ₱ 700

Period 0 1 2 3 … 24

The future value of all the payments at the end of term (t=24).

Payment ₱ 700 ₱ 700 ₱ 700 … ₱ 700

Period 0 1 2 3 … 24
(Year)
₱ 700

₱ 700(1.030377509)21

₱ 700(1.030377509)22

₱ 700(1.030377509)23

Add all the future values obtained from the previous step.
= ₱ 700 + ₱ 700(1.030377509)+ ₱ 700(1.030377509)2 +⋯+ ₱ 700(1.030377509)23
= ₱ 24,212.83

12
CO_Q2_General Mathematics SHS
Module 4
What’s More

Activity 1.1
Read each problem carefully and tell whether each of the given information
describes a simple annuity or general annuity. Complete the diagram that
follows. Have Fun!

a. Monthly payments of ₱ 3,000 for 4 years with interest rate of 3% compounded


monthly.

Type of Annuity: ________________

Class flow in the time diagram:

Payment …

Period

0 1 2 3 …

b. Quarterly payment of ₱ 5,000 for 10 years with interest rate of 2%


compounded semi-annually.

Type of Annuity: ________________

13
CO_Q2_General Mathematics SHS
Module 4
Diagram:

Payment …

Period 0 1 2 3 …

Activity 1.2
Identify the type of annuity in the given situations below then show the
illustration of the cash flow.
a. Monthly payments of ₱ 2,000 for 5 years with interest rate of 12%
compounded annually.

b. Yearly payment of ₱15,000 for 10 years with interest rate of 8%


compounded annually.

14
CO_Q2_General Mathematics SHS
Module 4
What I Have Learned

A. Please read the sentences carefully and fill in the missing word/s by writing
your answer on the line/s provided.
1. A sequence of payments made at equal (fixed) intervals or periods of time
is called ____________.
2. Each payment in an annuity is called ______________.
3. The time between the successive payments dates of an annuity is called
the ____________.
4. In simple annuity the payment interval is also the same as the
_______________.
5. In general annuity the interest conversion or compounding period is
____________ with the payment interval.

B. Give at least three examples of real-life situations in your locality that shows
annuity.
___________________________________________________________________________
___________________________________________________________________________
___________________________________________________________________________
___________________________________________________________________________
___________________________________________________________________________
C. Enumerate and describe the different types of annuities.

___________________________________________________________________________
___________________________________________________________________________
___________________________________________________________________________
___________________________________________________________________________
___________________________________________________________________________
___________________________________________________________________________

D. Being aware with the two types of annuities help you in different
transactions that you will encounter in the future. What are the advantages
and disadvantages of these annuities?
___________________________________________________________________________
___________________________________________________________________________
___________________________________________________________________________
___________________________________________________________________________
___________________________________________________________________________
___________________________________________________________________________
___________________________________________________________________________
___________________________________________________________________________

15
CO_Q2_General Mathematics SHS
Module 4
What I Can Do

Read and analyze the situation below then answers the question given.

Your mom asked your opinion if she will be joining the cooperative in her office.
She wants to know the amount of money she will be receiving after 6 months
and wanted to buy something in December. The cooperative wants her to
contribute ₱1,000 per month beginning in June 2020 which will earn 3%
compounded monthly. How much will be the future value of your mom’s
contribution at the end of December 2020?

Tasks:
You need to prepare a report showing a cash flow diagram on the total amount
of money your mother will earn at the end of December. At the end of your
report, write a conclusion stating your opinion to help your mom to decide.

Cash Flow Diagram

Conclusion:
______________________________________________________________________________
______________________________________________________________________________
______________________________________________________________________________

Rubrics:
5-States a conclusion with complete and appropriate justification, based on a
reasonable interpretation of the data.
4-States a conclusion with enough justification, based on a reasonable
interpretation the data.
3-States a conclusion with some justification, based on a reasonable
interpretation of the data.
2-States a conclusion on a reasonable interpretation of the data.
1-The conclusion is based on an unreasonable interpretation of the data.

16
CO_Q2_General Mathematics SHS
Module 4
Assessment

Multiple Choice. Choose the letter of the best answer. Write the chosen letter on
a separate sheet of paper.

1. Which of the following refers to the fixed sum of money paid to someone
at regular intervals and subject to a fixed compound interest?
a. annuity c. compound interest.
b. simple interest d. annuity certain

2. If the interest conversion or compounding period is equal or the same


with the payment interval, what type of annuity is illustrated?
a. annuity certain c. simple annuity
b. annuity uncertain d. general annuity

3. If the interest conversion or compounding period is unequal or not the


same as the payment interval, which type of annuity will be used?
a. annuity certain c. simple annuity
b. general annuity d. annuity uncertain

4. Which of the following is NOT an example of annuities?


a. pension c. car loan
b. educational plan d. deposit

5. Each payment in an annuity.


a. interest payment c. loan payment
b. periodic payment d. cash payment

6. The time between the successive payment dates of annuities.


a. payment interval c. periodic payment
b. payment due d. term

7. The interval between the beginning of the first payment period and the
end of the last period.
a. period c. term
b. due d. interval

8. The sum of all the payments to be made during the entire term of the
annuity.
a. future value c. loan value
b. present value d. interest value

17
CO_Q2_General Mathematics SHS
Module 4
9. What type of annuity is represented if the payment is made at the end of
each month for money borrowed that charge 0.15% interest compounded
quarterly?
a. simple c. ordinary
b. general d. annuity due

10. What type of annuity is represented by a deposit of ₱10,000.00


that is made at the end of every three months to an account that
earns 2.6% interest compounded quarterly?
a. simple c. ordinary
b. general d. annuity due

11. Which of the following situations is NOT an example of simple


annuity?
a. ₱1,500 deposited every month for 15 years at 10% per year
compounded annually
b. ₱1,500 deposited every month for 15 years at 10% per year
compounded monthly
c. ₱1,500 deposited every six months for 15 years at 10% per year
compounded semi-annually
d. ₱1,500 deposited every three months for 15 years at 10% per year
compounded quarterly

12. Which of the following situations is an example of a general annuity?


a. ₱5,000 deposited every month for 5 years at 8% per year
compounded annually
b. ₱5,000 deposited every six months for 5 years at 8% per year
compounded semi-annually
c. ₱5,000 deposited every three months for 5 years at 8% per year
compounded quarterly
d. ₱5,000 deposited every year for 5 years at 8% per year
compounded annually

18
CO_Q2_General Mathematics SHS
Module 4
For numbers 13-15. Given the cash flow of an annuity below.

Payment ₱ 8,000 ₱ 8,000 ₱ 8,000 ₱ 8,000

Period 0 1 2 3 4
(Year)
₱ 8,000

₱ 8,000(1.10)

₱ 8,000(1.10)2

₱ 8,000(1.10)3

13. What is the rate of interest?


a. 1.10% c. 0.10%
b. 10% d. 0.01%

14. What is the term of the payment?


a. 10 years c. 10 months
b. 4 years d. 4 months

15. What is the sum of all the payments of the given annuity?
a. ₱ 30,218 c. ₱ 32,000
b. ₱ 31,718 d. ₱ 37,128

19
CO_Q2_General Mathematics SHS
Module 4
Additional Activities

1. Why is it that many Filipinos do not save or save very little? Research the
top five external factors or barriers to saving. Your answers should be
supported by recent data. Cite your sources for the data gathered.

2. Choose one topic from the choices shown below. Ask someone how the
payment/ investment has been made. Make research on some strategies on
how to reduce or gain interest in the chosen topic.
a. Insurance
b. Time deposit
c. Educational plan
d. Housing Loan
e. Retirement Plan
f. Investment

20
CO_Q2_General Mathematics SHS
Module 4
Module 4
CO_Q2_General Mathematics SHS
21
What's More
Activity 1.1
1. Simple Annuity
₱3,000 ₱3,000 ₱3,000… ₱3,000
1 2 3 … 48
₱3,000
₱3,000(1.0025)45
₱3,000(1.0025)46
₱3,000(1.03)47
2. Compound Annuity
₱5,000 ₱5,000 ₱5,000 … ₱5,000
1 2 3 … 40
₱5,000
₱5,000(1.0066227)37
₱5,000(1.0066227)38
₱5,000(1.0066227)39
Activity 1.2
1. Compound Annuity
₱2,000 ₱2,000 ₱2,000 … ₱2,000
1 2 3 … 60
₱2,000
₱2,000(1.009488)57
₱2,000(1.009488)58
₱2,000(1.009488)59
2. Simple Annuity
₱1,500 ₱1,500 ₱1,500… ₱1,500
1 2 3 … 10
₱3,000
₱3,000(1.08)7
₱3,000(1.08)8
₱3,000(1.08)9
Answer Key
References
Alday, Eward M., Batisan, Ronaldo S., and Caraan, Aleli M. General
Mathematics. Makati City: Diwa Learning Systems Inc., 2016. 62-69.

Oronce, Orlando. General Mathematics. Quezon City: Rex Bookstore, Inc., 2016.
40-52.

General Mathematics Learner’s Material. First Edition. 2016. pp. 168-172

*DepED Material: General Mathematics Learner’s Material

22
CO_Q2_General Mathematics SHS
Module 4
For inquiries or feedback, please write or call:

Department of Education - Bureau of Learning Resources (DepEd-BLR)

Ground Floor, Bonifacio Bldg., DepEd Complex


Meralco Avenue, Pasig City, Philippines 1600

Telefax: (632) 8634-1072; 8634-1054; 8631-4985

Email Address: blr.lrqad@deped.gov.ph * blr.lrpd@deped.gov.ph


General Mathematics
Quarter 2 – Module 5:
Future and Present Values
of Simple and General Annuities

CO_Q2_General Mathematics SHS


Module 5
General Mathematics – Senior High School
Alternative Delivery Mode
Quarter 2 – Module 5: Future and Present Values of Simple and General Annuities
First Edition, 2021

Republic Act 8293, section 176 states that: No copyright shall subsist in any work of
the Government of the Philippines. However, prior approval of the government agency or office
wherein the work is created shall be necessary for exploitation of such work for profit. Such
agency or office may, among other things, impose as a condition the payment of royalties.

Borrowed materials (i.e., songs, stories, poems, pictures, photos, brand names,
trademarks, etc.) included in this module are owned by their respective copyright holders.
Every effort has been exerted to locate and seek permission to use these materials from their
respective copyright owners. The publisher and authors do not represent nor claim ownership
over them.

Published by the Department of Education


Secretary: Leonor Magtolis Briones
Undersecretary: Diosdado M. San Antonio

Development Team of the Module


Writers: Jea Aireen Charimae M. de Mesa
Editors: Elizabeth D. Lalunio, Elizabeth B. Dizon, Anicia J. Villaruel, Roy O. Natividad
Reviewers: Jerry Punongbayan, Necitas R. Constante, Celestina M. Alba, Jerome
A. Chavez, Azalea A. Gallano and Maria Leonor L. Justarez
Illustrator: Hanna Lorraine Luna, Diane C. Jupiter and Meryll C. Calvendra
Layout Artist: Roy O. Natividad, Sayre M. Dialola, Noel Rey T. Estuita, Argie L. Ty
Management Team: Francis Cesar B. Bringas
Job S. Zape, Jr.
Ramonito Elumbaring
Reicon C. Condes
Elaine T. Balaogan
Fe M. Ong-ongowan
Hermogenes M. Panganiban
Phillip B. Gallendez
Josephine T. Natividad
Anicia J. Villaruel
Dexter M. Valle

Printed in the Philippines by ________________________

Department of Education – Region 4A CALABARZON

Office Address: Gate 2 Karangalan Village, Brgy. San Isidro, Cainta, Rizal
Telefax: 02-8682-5773/8684-4914/8647-7487
E-mail Address: lrmd.calabarzon@deped.gov.ph
General Mathematics
Quarter 2 – Module 5:
Future and Present Values
of Simple and General Annuities
Introductory Message
This Self-Learning Module (SLM) is prepared so that you, our dear learners,
can continue your studies and learn while at home. Activities, questions, directions,
exercises, and discussions are carefully stated for you to understand each lesson.

Each SLM is composed of different parts. Each part shall guide you step-by-
step as you discover and understand the lesson prepared for you.

Pre-tests are provided to measure your prior knowledge on lessons in each


SLM. This will tell you if you need to proceed on completing this module or if you
need to ask your facilitator or your teacher’s assistance for better understanding of
the lesson. At the end of each module, you need to answer the post-test to self-check
your learning. Answer keys are provided for each activity and test. We trust that you
will be honest in using these.

In addition to the material in the main text, Notes to the Teacher are also
provided to our facilitators and parents for strategies and reminders on how they can
best help you on your home-based learning.

Please use this module with care. Do not put unnecessary marks on any part
of this SLM. Use a separate sheet of paper in answering the exercises and tests. And
read the instructions carefully before performing each task.

If you have any questions in using this SLM or any difficulty in answering the
tasks in this module, do not hesitate to consult your teacher or facilitator.

Thank you.
What I Need to Know

This module was designed and written with you in mind. It is here to help you
compute the future and present value of simple and general annuities. The scope
of this module permits it to be used in many different learning situations. This
modules aims to help you on how to be practical specifically in spending for your
needs whether be it in cash or in installment basis. It is important that you apply
the skills you will learn here in real life situations. Good luck!
This module is composed of two lessons namely:
Lesson 1 - Future and Present Values of Simple Annuity
Lesson 2 - Future and Present Vlaues of General Annuity

After going through this module, you are expected to:


1. identify the given facts and choose the proper formula to be used in
computing for the future and present value of simple and general
annuities; and
2. compute the future value and present value of simple and general
annuities.

1
CO_Q2_General Mathematics SHS
Module 5
What I Know

Let’s find out how far you might already know about this topic! Please take this
challenge! Have Fun!

Choose the letter of the best answer. Write the chosen letter on a separate sheet
of paper.

1. What do you call a sequence of payments made at equal time


periods?
a. Annuity
b. Simple Interest
c. Principal
d. Compound Interest

2. What is a type of annuity where the payment interval is the same as the
interest period?
a. General Annuity
b. Simple Annuity
c. Simple Interest
d. Compound Interest
3. What is the type of annuity in which the payment interval is not the same
as the interest period?
a. General Annuity
b. Simple Annuity
c. Simple Interest
d. Compound Interest

4. What is equal to the down payment (if there is any) plus the present
value of the installment payments?
a. Downpayment
b. Cash Price
c. Markup
d. Markdown

5. Installment payment for an appliance at the end of each month with


interest compounded monthly is an example of what type of annuity?
a. General Annuity
b. Simple Annuity
c. Ordinary Annuity
d. Annuity Due

2
CO_Q2_General Mathematics SHS
Module 5
6. Installment payment for an appliance at the beginning of each month with
interest compounded monthly is an example of what type of annuity?
a. General Annuity
b. Simple Annuity
c. Ordinary Annuity
d. Annuity Due

7. Which is NOT an example of Annuity?


a. House and Lot payment
b. Tuition Fee
c. Life Insurance
d. Installment basis of paying a car

8. What is the sum of present values of all the payments to be made during the
entire term of the annuity?
a. Present Value of an Annuity
b. Future Value of an Annuity
c. Periodic Payment
d. Term of an Annuity

9. What is the time between the first payment interval and last payment interval?
a. Periodic Payment
b. Term of an Annuity
c. Payment Interval
d. Present Value of an Annuity

10. What is the amount of each payment?


a. Future Value of an Annuity
b. Periodic Payment
c. Present Value of an Annuity
d. Term of an Annuity

11. What is the sum of future values of all the payments to be made during the
entire term of the annuity?
a. Periodic Payment
b. Payment Interval
c. Present Value of an Annuity
d. Future Value of an Annuity

3
CO_Q2_General Mathematics SHS
Module 5
12. What is the time between successive payments ?
a. Periodic Payment
b. Term of an Annuity
c. Time
d. Payment interval

13. Which is NOT an example of Installment basis purchase?


a. Pension
b. Car
c. House
d. Cellphone

For item numbers 14-15, refer to the following:


Suppose you would like to save ₱ 1,500 at the end of each month, for a year, in
a bank that gives 2.5% compounded monthly.

14. What type of annuity is present in the situation above?


a. Simple Annuity Due
b. General Annuity Due
c. Simple Annuity Immediate
d. General Annuity Immediate

15. What is the interest rate per annum in decimal form?


a. 2.5
b. 0.25
c. 0.025
d. 0.0025

4
CO_Q2_General Mathematics SHS
Module 5
Lesson
Future and Present Values
1 of Simple Annuity

Have you ever tried getting an appliance by applying in a Home Credit, Flexi
Finance, or through credit cards? If yes, you will realize that this lesson is the
direct application of this. With this, you can also see or compute how lending
institutions really work. You will also realize if they are really worth it.

What’s In

Most of us have had the experience of making a series of fixed payments over a
period of time—such as rent or car payments—or receiving a series of payments
for a period of time, such as interest from a bond or CD. These recurring or
ongoing payments are technically referred to as "annuities" (not to be confused
with the financial product called an annuity, though the two are related).

There are several ways to measure the cost of making such payments or what
they're ultimately worth. Here's what you need to know about calculating
the present value or future value of an annuity.

Definition

Simple Annuity is a type of annuity in which the payment period is the same
as the interval period (conversion period).

Example: Monthly payment where interest is compounded monthly


• Payment period refers to the time between successive period of
annuity.
• Term refers to time from the start of the first payment up to the last
payment.
• Periodic Payment (R) is the size of each annuity payment

5
CO_Q2_General Mathematics SHS
Module 5
Notes to the Teacher
Regular payments, such as the rent on an apartment or interest on a bond,
are sometimes referred to as "annuities."
In ordinary annuities, payments are made at the end of each time period. With
annuities due, they're made at the beginning.
The future value of an annuity is the total value of payments at a specific point
in time. The present value is how much money would be required now to
produce those future payments.

6
CO_Q2_General Mathematics SHS
Module 5
What’s New

How much will I earn?

You decided to join a Kabataan Savers Club which aims for financial growth of
the youth nowadays. If you pay ₱1,000.00 at the end of each month for 5 months
on account that pays interest at 12% compounded monthly, how much money
will you have after 5 months?

Guide Questions:

1. What type of annuity is present in the problem?


2. What were the given presented?
3. What formula should you use to solve the problem?
4. What would be the answer in the problem?

7
CO_Q2_General Mathematics SHS
Module 5
What is It

The future value of an annuity is the total accumulation of the payments and
interest earned. The present value of an annuity is the principal that must be
invested today to provide the regular payment of an annuity.

Present Value Future Value


1 − (1 + 𝑖𝑖)−𝑛𝑛 (1+𝑖𝑖)𝑛𝑛 −1
𝐹𝐹 = 𝑅𝑅 ∗ � �
𝑃𝑃 = 𝑅𝑅 ∗ � � 𝑖𝑖
Simple Annuity 𝑖𝑖
Where Where
P-Present Value
R-periodic payment F-Future Value or Amount
R -interest rate per period; R-Periodic Payment
where i=r/m r-interest rate per period
i-annual rate where i =r/m
m-number of conversion i-annual rate
period in a year m-number of conversion
n-total number of period in a year
conversion n-total number of conversion
periods periods
n = t(m) n = t(m)
t-number of years t-number of years

8
CO_Q2_General Mathematics SHS
Module 5
In this case, with the example presented above we can answer the following
questions. (1) Since the interest conversion is equal or the same as the payment
interval so we will use simple annuity.

(2) Identifying the given facts, we have:

P = ₱1,000.00

i(12) =r/m= 0.12/12 =0.01 t= 5 months/12

n = t* m= (5 months/12) *12=5

(3) Since we will find the amount of money after 5 months, we will use the
(1+𝑖𝑖)𝑛𝑛 −1
formula: 𝐹𝐹 = 𝑅𝑅 ∗ � �
𝑖𝑖

1000[(1+0.01)5 −1]
(4) Then substituting all the given facts, we will obtain: 𝐹𝐹 =
0.01

Future Value = ₱5,101.01

Example 1.B

Contrast in calculating the future value, a present value (PV) tells you how
much money would be required now to produce a series of payments in the future,
again assuming a set interest rate.

Using the same example above, the given is as follows:

P =?

i(12) = 0.12/12 =0.01 t= 5 months/12

n = t*m =(5/12)*12= 5

Since we are looking for the present value, we use the formula
1 − (1 + 𝑖𝑖)−𝑛𝑛
𝑃𝑃 = 𝑅𝑅 ∗ � �
𝑖𝑖
By substituting it in the formula, we will obtain

1000[1 − (1 + 0.01)−5 ]
𝑃𝑃 =
0.01
Present Value = ₱4,853.43

As you can notice, future value is higher than the present value. This is because
of the time value of money—the concept that any given sum is worth more now
than it will be in the future because it can be invested in the present.

9
CO_Q2_General Mathematics SHS
Module 5
Definition. The cash value or cash price is equal to the down payment (if there
is any) plus the present value of the installment payments.

Example 1.C

Mr Angeles paid ₱200,000.00 as a downpayment for a car. The remaining


amount is to be settled by paying ₱ 16,200.00 by the end of each month for 5
years. If interest is 10.5% compounded monthly, what is the cash price of his
car?

To solve this, let us identify the given:

Downpayment= ₱200,000.00 R= periodic payment


0.105
i = r/m = 0.00875
12

n= t*m=5*12=60 years

R= ₱16,200.00

Obtain the present value of the car by plugging the given in our formula

1 − (1 + 0.00875)−60
𝑃𝑃 = 16,200 ∗ � �
0.00875

The present value of the car is ₱ 753,702.20

To get the cash value, simply add the obtained present value and the
downpayment made, so;

CV= 200,000 + 753,702.20

The total cash value of the car is ₱ 973,702.20.

10
CO_Q2_General Mathematics SHS
Module 5
Example 1.D

Mr. Edgar borrowed from his friend ₱ 200,000.00 He promised to pay the amount
plus its interest by an equal amount of money each year for 3 years. What must
be his annual payment if they agreed on an interest of 10% compounded
annually?

This example is different from the examples presented above. This time, you are
going to compute the Regular periodic payment. We will be manipulating the
formula of present value to obtain the formula for the periodic payment.
𝑃𝑃
𝑅𝑅 =
1 − (1 + 𝑖𝑖)−𝑛𝑛
𝑖𝑖
Given:

P= ₱ 200,000.00

i= 0.10

n= 3

Substituting the values to our formula, we get,


200,000
𝑅𝑅 =
1 − (1 + 0.1)−5
0.1
Mr Edgar must pay ₱ 80,422.96 every year.

11
CO_Q2_General Mathematics SHS
Module 5
What’s More

Read each problem carefully and answer each question to solve the problem. Have
Fun!

Practice Activity 1:

Find the Present Value (P) and the Future Value (F) of quarterly
payments of ₱ 2,000.00 for 5 years with interest rate of 8% compounded
quarterly.

Hint: State the given. Identify which formula to be used. Substitute the
value to the formula.
1−(1+𝑖𝑖)−𝑛𝑛 (1+𝑖𝑖)𝑛𝑛 −1
𝑃𝑃 = 𝑅𝑅 ∗ � � 𝐹𝐹 = 𝑅𝑅 ∗ � �
𝑖𝑖 𝑖𝑖

Independent Activity 1

Find the Present Value (P) and the Future Value (F) of semi-annual
payments of ₱ 8,000.00 for 12 years with interest rate of 12%
compounded semi-annually.

Practice Activity 2:

How much should you invest in a fund each year paying 2% compounded
annually to accumulate ₱ 100,000.00 in 5 years?

12
CO_Q2_General Mathematics SHS
Module 5
Hint: State the given. Identify which formula to be used. Substitute the
value to the formula.

𝑃𝑃
𝑅𝑅 =
1 − (1 + 𝑖𝑖)−𝑛𝑛
𝑖𝑖

Independent Activity 2

Starting on her 30th birthday, a woman will invest an amount every year
on her birthday in an account that grows at an annual rate of 7%. What
should be the amount invested should she want her fund to
₱ 300,000.00 just before her 65th birthday?

Practice Activity 3

The value of a car requires a ₱ 169,000.00 cash downpayment and a


monthly payment of ₱ 12,000.00 If money is computed at 10%
compounded monthly, how much is the cash price of the car payable in
5 years?

Hint: State the given. Identify which formula to be used. Substitute the
value to the formula.
CV = Downpayment + Present Value

Independent Activity 3

The buyer of a lot pays ₱ 50,000.00 cash and ₱ 10,000.00 every month for
10 years. If money is 8% compounded monthly, how much is the cash price
of the lot?

13
CO_Q2_General Mathematics SHS
Module 5
What I Have Learned

A. Write down the formula need to solve the following.

1. Present Value of a Simple Annuity


______________________________________________________.

2. Future Value of a Simple Annuity


______________________________________________________.

3. Regular Periodic Payment


______________________________________________________.

4. Cash Value/Price
______________________________________________________.

B. This lesson will help you to become more practical through considering your
financial situation before involving yourself with any type of investments or
loans. It is important to know the amount that you may get after several periods
of time. Cite any situation where you apply practical decision in terms of
financial concern.
___________________________________________________________________________
___________________________________________________________________________
___________________________________________________________________________

14
CO_Q2_General Mathematics SHS
Module 5
What I Can Do

Let’s be practical!

A new brand of cell phone is for sale in SM Lucena at ₱ 13,499.00 in cash or in


instalment terms, ₱ 2,500.00 each month for the next 6 months at 9%
compounded monthly.

1. If you were the buyer, what do you prefer? Cash or installment?


2. Why did you choose cash? Installment?
3. What is the advantage of choosing cash basis?
4. What is the advantage of choosing installment basis?

Submit on a long bond paper an essay containing your answer on the questions
above. You should note that the computation must be included in the front page
of your paper. The essay must consist of at least 5 sentences and a maximum of
10 sentences on why you must choose your payment options. Justify your answer
with the computations you had and your own personal reasons.

15
CO_Q2_General Mathematics SHS
Module 5
Below is the rubrics for you to be guided in this activity.

Areas of 4 pts 3 pts 2 pts 1 pt


Assessment
Computations Computations Computations Computations Erroneous and
are free of error are slightly are slightly inaccurate
and is accurate. inaccurate and inaccurrate but computations.
close to correct not close to
answers. correct
answers.
Ideas Presents ideas Presents ideas Ideas are too Ideas are
in an original in a consistent general. vague or
manner. manner. unclear.

Organization Strong and Organized Some No


organized beg/mid/end organization; organization;
beg/mid/end attempt at a lack
beg/mid/end beg/mid/end

Word Choice Sophisticated Nouns and Needs more Little or no use


use of nouns verbs make nouns and of nouns and
and verbs make essay verbs. verbs.
the essay very informative.
informative.

Sentence Sentence Sentence Sentence No sense of


Structure structure structure is structure is sentence
enhances evident; limited; structure or
meaning; flows sentences sentences need flow.
throughout the mostly flow. to flow.
piece.

16
CO_Q2_General Mathematics SHS
Module 5
Lesson
Future and Present Values
2 of General Annuity

In this course, you will solve all sorts of general annuity problems. A general
annuity is an annuity where the payments do not coincide with the interest
periods. You will be able to see that it is very easy to deal with general annuities
once an equivalent interest rate is determined with that equivalent rate being
compounded as often as the payments are made.

What’s In

Why do you need to know investments, bonds, stocks, interests? Why is there a
need to invest your hard earned money?

Having the knowledge in basic concepts in business mathematics or the


mathematics of investment may help you decide whether to use that credit card
for a 5% interest compounded monthly or a simple interest for a period of 6
months. Some topics might shed light on which banks would give a higher
interest rate for your savings.

For you to begin, consider the lesson on the previous module which is essential
in obtaining success in this lesson. General annuity is not like a simple annuity.
Unlike simple annuity, general annuity deals with different payment period and
interval period. Here, you have to convert the rate first before substituting the
values to the formula.

Definition

General Annuity is a type of annuity in which the payment period is not the
same as the interval period (conversion period). You will be able to easily identify
this .

17
CO_Q2_General Mathematics SHS
Module 5
Example:

A four-year lease agreement between Alfred and Thrifty Mall Inc. (TMI) indicates
that, Alfred pays TMI ₱100,000.00 at the end of every year if the agreed interest
rate is 5% compounded quarterly

In this example, the payment period is a whole year. However, the interest period
is quarterly or every 3 months. Hence, the annuity is a general annuity

Notes to the Teacher

In general ordinary annuities, payments are made at the end of each time
period. With annuities due, they're made at the beginning.
The terms used in simple annuity have the same meaning in general annuity.

18
CO_Q2_General Mathematics SHS
Module 5
What’s New

What is the present value of an annuity of ₱ 2,000.00 payable annually for 9 years
if the money is worth 5% compounded quarterly.

Guide Questions:

1. What type of annuity is present in the problem?


2. What were the given presented?
3. What formula should you use to solve the problem?
4. What would be the answer in the problem?

19
CO_Q2_General Mathematics SHS
Module 5
What is It

The future value of an annuity is the total accumulation of the payments and
interest earned. The present value of an annuity is the principal that must be
invested today to provide the regular payment of an annuity.

1 − (1 + 𝑖𝑖)−𝑛𝑛 (1+𝑖𝑖)𝑛𝑛 −1
𝐹𝐹 = 𝑅𝑅 ∗ � �
𝑃𝑃 = 𝑅𝑅 ∗ � � (1+𝑖𝑖)𝑏𝑏 −1
General Annuity (1 + 𝑖𝑖)𝑏𝑏 − 1
Where
Where
R-regular payment
R-regular payment
r-interest rate per period;
r-interest rate per period;
where i=r/m
where i=r/m
i-annual rate
i-annual rate
m-number of conversion
m-number of conversion
period in a year
period in a year
n-total number of conversion
n-total number of
periods
conversion
n = t(m)
periods
t – number of years
n = t(m)
t – number of years , where p is the
number of months in a
, where p is the
payment interval and c is the
number of months in a
number of months in a
payment interval and c is
compounding period.
the number of months in a
compounding period.

20
CO_Q2_General Mathematics SHS
Module 5
Example 2.A

If you will notice, the interest conversion period is different from the payment
interval. Since this is the case, we will refer to general annuity.
Consider the following given,
P = ₱ 2,000.00

n = 9(4) = 36

or 0.0125

c=3

p = 12

Using the formula in getting the present value of general annuity we will obtain:
1−(1+𝑖𝑖)−𝑛𝑛
𝑃𝑃 = 𝑅𝑅 ∗ � � then substituting the given facts, we will obtain
(1+𝑖𝑖)𝑏𝑏 −1

we will obtain ₱ 14,155.99 as the present value


of general annuity.

Example 2.B

Looking for the future value of the example above, let us use the given.

P = ₱ 2,000.00

n = 9(4) = 36

or 0.0125

c=3

p = 12

(1+𝑖𝑖)𝑛𝑛 −1
Using the formula for future value fo general annuity: 𝐹𝐹 = 𝑅𝑅 ∗ � �
(1+𝑖𝑖)𝑏𝑏 −1

And substituting the given to the formula, we will have

(1 + 0.0125)36 − 1
𝐹𝐹 = 2000 � �
(1 + 0.0125)4 − 1

Future Value is ₱ 22,139.17

21
CO_Q2_General Mathematics SHS
Module 5
What’s More

Read each problem carefully and answer each question to solve the problem. Have
Fun!

Practice Activity 1

Payment of ₱ 500.00 is made at each year for 10 years. Interest has a


nominal rate of 8% convertible quarterly. Find the present and the future
value.

Hint: State the given. Identify which formula will be used. Substitute the
value to the formula.
(1+𝑖𝑖)𝑛𝑛 −1 1−(1+𝑖𝑖)−𝑛𝑛
𝐹𝐹 = 𝑅𝑅 ∗ � � 𝑃𝑃 = 𝑅𝑅 ∗ � �
(1+𝑖𝑖)𝑏𝑏 −1 (1+𝑖𝑖)𝑏𝑏 −1

Independent Activity 1

Annual payments of ₱1,000.00 at the end of each term for 8 years with
interest rate of 6% compounded quarterly. Find the present and the
future value.

22
CO_Q2_General Mathematics SHS
Module 5
What I Have Learned

A. Write down the formula needed to solve the following.

1. Present Value of a General Annuity


______________________________________________________.

2. Future Value of a General Annuity


______________________________________________________.

B. Write down the differences and similarities between Simple Annuity and
General Annuity.
___________________________________________________________________________
___________________________________________________________________________
___________________________________________________________________________
___________________________________________________________________________
___________________________________________________________________________
___________________________________________________________________________
___________________________________________________________.

23
CO_Q2_General Mathematics SHS
Module 5
What I Can Do

Ask someone borrowing from a five-six (Bumbay) money lender. How much will
be charged if you want to loan ₱ 5,000.00, payable in one year? Determine the
interest rate per period and the annual interest rate. Is it a good loan term? Why
or why not?
Submit on a long bond paper an essay containing your answer on the questions
above. You should note that the computation must be included in the front page
of your paper. Then essay must consist of at least 5 sentences and a maximum
of 10 sentences on why you must chose your answer. Justify your answer with
the computations you had and your own personal reasons.

Below is the rubrics so that you are guided in this activity.

Areas of 4 pts 3 pts 2 pts 1 pt


Assessment
Computations Computations Computations Computations Erroneous
are free of error are slightly are slightly and
and is accurate. inaccurate and inaccurrate but inaccurate
close to correct not close to computations
answers. correct answers.

Ideas Presents ideas Presents ideas Ideas are too Ideas are
in an original in a consistent general. vague or
manner manner unclear.

Organization Strong and Organized Some No


organized beg/mid/end organization; organization;
beg/mid/end attempt at a lack
beg/mid/end beg/mid/end

Word Choice Sophisticated Nouns and Needs more Little or no


use of nouns verbs make nouns and use of nouns
and verbs make essay verbs. and verbs.
the essay very informative.
informative.
Sentence Sentence Sentence Sentence No sense of
Structure structure structure is structure is sentence
enhances evident; limited; structure or
meaning; flows sentences sentences need flow
throughout the mostly flow. to flow.
piece.

24
CO_Q2_General Mathematics SHS
Module 5
Assessment

Multiple Choice. Choose the letter of the best answer. Write the chosen letter on
a separate sheet of paper.

1. What is the present value of the simple annuity of ₱ 5,000.00 payable semi-
annually for 10 years if money is worth 6% compounded semi-annually?
a. ₱ 74,387.37
b. ₱ 67,200.42
c. ₱ 81,600.96
d. ₱ 34,351.87

2. With the same given in number 1, what is the accumulated amount (future
value)?
a. ₱ 74,387.37
b. ₱ 67,200.42
c. ₱ 81,600.96
d. ₱ 134,351.87

For number 3-6, refer to the problem below.


Mr. Michael’s monthly insurance premium is ₱ 500.00, payable at the end of
each month. His policy matures 20 years later, after which he can withdraw all
his payments plus the interest earned. If the money is worth 15% compounded
monthly, how much does he expect to withdraw on the maturity of his policy?

3. What is being asked in the problem?


a. Cash Value
b. Regular Periodic payment
c. Future Value
d. Present Value

4. What is the total conversion period of the insurance policy?


a. 20
b. 12
c. 120
d. 240

5. What is the annual interest rate of the insurance policy converted to decimal?
a. 0.15
b. 0.015
c. 0.125
d. 0.0125

25
CO_Q2_General Mathematics SHS
Module 5
6. How much does he expect to withdraw on the maturity of his policy?
a. ₱ 800,519.00
b. ₱ 798,716.74
c. ₱ 748,619.74
d. ₱ 543,519.84

7. Find the present value of an annuity of ₱ 10,000.00 payable semi-annually for


5 years if money is worth 6% per year compounded quarterly.
a. ₱ 120,640.00 c. ₱ 230,145.19
b. ₱ 171,686.39 d. ₱ 286,640.00

For number 8 and 9, refer to the problem below.


A high school student would like to save ₱ 50,000 for his graduation. He will be
depositing on his savings every month for 5.5 years and interest is at 0.25%
compounded monthly?

8. What is the interest rate per month converted to decimals?


a. 0.25
b. 0.025
c. 0.0025
d. 0.00025

9. How much should he deposit in his bank to get ₱ 50,000 after 5.5 years?
a. ₱ 752.46
b. ₱ 762.46
c. ₱ 764.52
d. ₱ 765.42

26
CO_Q2_General Mathematics SHS
Module 5
For number 10 and 11, refer to the situation below.
The buyer of a car pay ₱300,000.00 cash as downpayment and is going to pay
₱12,000.00 monthly for 5 years if money is worth 12% compounded monthly.

10. How much more will he be disbursing (present value) after paying the
downpayment amount?
a. ₱ 529,360.04
b. ₱ 539,460.09
c. ₱ 539,730.07
d. ₱ 589,733.43

11. What is the cash value of the car?


a. ₱ 829,360.04
b. ₱ 839,460.09
c. ₱ 839,730.07
d. ₱ 889,733.43

12. Find the future value of an annuity of ₱ 10,000.00 payable quarterly for 3
years if money is worth 12% compounded monthly.
a. ₱ 33,596.60
b. ₱ 33,695.60
c. ₱ 33,965.60
d. ₱ 33,956.60

13. What is the present value of an annuity of ₱ 5,000.00 payable quarterly for
10 years If money is worth 5% per year compounded annually?
a. ₱ 345,590.98
b. ₱ 345,950.98
c. ₱ 349,550.98
d. ₱ 394,055.98

14. Find the future value of an annuity of ₱10,000.00 payable quarterly for 5
years if money is worth 12% compounded monthly.
a. ₱ 64,395.55
b. ₱ 64,935.55
c. ₱ 69,435.55
d. ₱ 69,534.55

15. Find the present value of an annuity of ₱20,000.00 payable semi-annually for
5 years if money is worth 6% per year compounded quarterly.
a. ₱120,640.00
b. ₱ 145,022.80
c. ₱ 170,408.33
d. ₱186,640.00

27
CO_Q2_General Mathematics SHS
Module 5
Additional Activities

Solve the following:

A. Mr. Barela’s monthly amortization is ₱3,500.00, payable at the end of each


month. His house and lot will be fully paid after 20 years. If the amount of
house and lot is 5% compounded monthly, how much is its maturity value?
Write your computation on the box provided below.

B. An cellphone is for sale at either (a) ₱15,999.00 cash or, (b) on terms,
₱1,499.00 each month for 12 months. Money is 9% compounded monthly.
Which is lower, the cash price or the present value of the installment
terms? Explain. Write your computation on the box provided below.

Explanation:

28
CO_Q2_General Mathematics SHS
Module 5
Module 5
CO_Q2_General Mathematics SHS
29
What's More
Practice Activity 1
P=₱32,702.87
F= ₱48,594.74
Independent Activity 1
P= ₱100,402.90
F= ₱406,524.60
Practice Activity 2
₱19,215.84
Independent Activity 2
₱2,028.21
Practice Activity 3
₱733,784.43
What I Know Assessment
1. A 1. A
2. B Independent Activity 3 2. D
3. A 3. C
₱874,214.81
4. B 4. D
5. C 5. D
6. D Practice Activity 4 6. C
7. C 7. B
8. A P= ₱7,327.48 8. C
9. B 9. A
F= ₱3,318.54
10. B 10.B
11. D 11. B
12. D Independen Activity 4 12. C
13. A 13. C
14. C P= ₱6,176.42 14. A
15. C 15. B
F= ₱9,946.04
Answer Key
References
Alday, E. M., Batisan, R. S., & Caraan, A. M. (2016). General Mathematics.
Makati City: Diwa Learning Systems Inc.
Cox, J. (2020, February 19). ThoughtCo. Retrieved June 10, 2020, from
https://www.thoughtco.com/essay-rubric-
2081367#:~:text=An%20essay%20rubric%20is%20a,organized%20into%
20one%20convenient%20paper.

*DepED Material: General Mathematics Learner’s Material

General Mathematics Learner’s Material. First Edition. 2016. pp. 199-220

HEYFORD, S. C. (2020, April 30). Investopedia. Retrieved June 9, 2020, from


https://www.investopedia.com/retirement/calculating-present-and-
future-value-of-annuities/

Oronce, Orlando.General Mathematics.Sampaloc Manila, Philippines. Rex


Bookstore, Inc. 2016.

30
CO_Q2_General Mathematics SHS
Module 5
For inquiries or feedback, please write or call:

Department of Education - Bureau of Learning Resources (DepEd-BLR)

Ground Floor, Bonifacio Bldg., DepEd Complex


Meralco Avenue, Pasig City, Philippines 1600

Telefax: (632) 8634-1072; 8634-1054; 8631-4985

Email Address: blr.lrqad@deped.gov.ph * blr.lrpd@deped.gov.ph


General Mathematics
Quarter 2 – Module 6:
Fair Market Value
of a Cash Flow

CO_Q2_General Mathematics SHS


Module 6
General Mathematics
Alternative Delivery Mode
Quarter 2 – Module 6: Fair Market Value of a Cash Flow
First Edition, 2021

Republic Act 8293, section 176 states that: No copyright shall subsist in any work of
the Government of the Philippines. However, prior approval of the government agency or office
wherein the work is created shall be necessary for exploitation of such work for profit. Such
agency or office may, among other things, impose as a condition the payment of royalties.

Borrowed materials (i.e., songs, stories, poems, pictures, photos, brand names,
trademarks, etc.) included in this module are owned by their respective copyright holders.
Every effort has been exerted to locate and seek permission to use these materials from their
respective copyright owners. The publisher and authors do not represent nor claim ownership
over them.

Published by the Department of Education


Secretary: Leonor Magtolis Briones
Undersecretary: Diosdado M. San Antonio

Development Team of the Module


Writers: Geovanni S. Delos Reyes
Editors: Elizabeth D. Lalunio, Elizabeth B. Dizon,Anicia J. Villaruel, Roy O. Natividad
Reviewers: Fritz A. Caturay, Necitas F. Constante, Celestina M. Alba,
Jerome A. Chavez Shielamarie E. Arce, Carmela Ana A. Reforma,
Rafaela M. Merle
Illustrators: Hanna Lorraine G. Luna, Diane C. Jupiter
Layout Artist: Sayre M. Dialola, Roy O. Natividad, Noel Rey T. Estuita, Argie L. Ty,
Jilky I. Bosque
Management Team: Francis Cesar B. Bringas
Job S. Zape, Jr.
Ramonito Elumbaring
Reicon C. Condes
Elaine T. Balaogan
Fe M. Ong-ongowan
Elias A. Alicaya Jr.
Gregorio A. Co Jr.
Gregorio T. Mueco
Herbert D. Perez
Lorena S. Walangsumbat
Jee-Ann O. Borines
Asuncion C. Ilao

Printed in the Philippines by ________________________

Department of Education – Region 4A CALABARZON


Office Address: Gate 2 Karangalan Village, Brgy. San Isidro, Cainta, Rizal
Telefax: 02-8682-5773/8684-4914/8647-7487
E-mail Address: lrmd.calabarzon@deped.gov.ph
General Mathematics
Quarter 2 – Module 6:
Fair Market Value
of a Cash Flow
Introductory Message
This Self-Learning Module (SLM) is prepared so that you, our dear learners,
can continue your studies and learn while at home. Activities, questions, directions,
exercises, and discussions are carefully stated for you to understand each lesson.

Each SLM is composed of different parts. Each part shall guide you step-by-
step as you discover and understand the lesson prepared for you.

Pre-tests are provided to measure your prior knowledge on lessons in each


SLM. This will tell you if you need to proceed on completing this module or if you
need to ask your facilitator or your teacher’s assistance for better understanding of
the lesson. At the end of each module, you need to answer the post-test to self-check
your learning. Answer keys are provided for each activity and test. We trust that you
will be honest in using these.

In addition to the material in the main text, Notes to the Teacher are also
provided to our facilitators and parents for strategies and reminders on how they can
best help you on your home-based learning.

Please use this module with care. Do not put unnecessary marks on any part
of this SLM. Use a separate sheet of paper in answering the exercises and tests. And
read the instructions carefully before performing each task.

If you have any questions in using this SLM or any difficulty in answering the
tasks in this module, do not hesitate to consult your teacher or facilitator.

Thank you.
What I Need to Know

This module was written and developed for students to know how to calculate the
fair market value of a cash flow whether inflow or outflow that includes an annuity.
The topic to be discussed in this module includes comparing and computing the fair
market value of goods or properties which is essential in dealing with the next
chapter. The language used in this module is appropriate to the diverse
communication and language ability of the learners.

After going through this module, you are expected to:


1. calculate the fair market value of a cash flow stream
that includes an annuity;
2. compare the fair market value of cash flow; and
3. make a decision based on the market value of cash flow.

What I Know

Choose the letter of the best answer. Write your chosen letter on a separate sheet of
paper.

1. What is the term that refers to payment received or payments or deposits made?
a. annuity
b. cash flow
c. fair market value
d. general annuity

2. What is defined as an annuity where the length of the payment interval is not
the same as the length of the interest compounding period?
a. cash flow
b. fair market value
c. general annuity
d. general ordinary annuity

3. What is the formula to determine the present value of an ordinary annuity?


1−(1+𝑗𝑗)−𝑛𝑛
a. 𝑃𝑃 = 𝑅𝑅
𝑗𝑗
1+(1+𝑗𝑗)−𝑛𝑛
b. 𝑃𝑃 = 𝑅𝑅
𝑗𝑗
(1+𝑗𝑗)𝑛𝑛 −1
c. 𝑃𝑃 = 𝑅𝑅
𝑗𝑗
(1+𝑗𝑗)𝑛𝑛 +1
d. 𝑃𝑃 = 𝑅𝑅
𝑗𝑗

1 CO_Q2_General Mathematics SHS


Module 6
4. What is the other term for fair market value?
a. annuity
b. cash flow
c. economic value
d. future value

5. It refers to a single amount that is equivalent to the value of the payments stream
at that date.
a. annuity
b. cash flow
c. down payment
d. fair market value

6. It is a general annuity in which the periodic payment is made at the end of the
payment interval.
a. annuity
b. cash flow
c. general annuity
d. general ordinary annuity

7. What is the present value of quarterly payments of ₱ 2,000 for 5 years with an
interest rate of 8% compounded quarterly?
a. ₱ 32,702.87
b. ₱ 34,702.87
c. ₱ 33,702.87
d. ₱ 31,702.87

8. What is the formula to determine the future value of an ordinary annuity?


1−(1+𝑗𝑗)−𝑛𝑛
a. 𝐹𝐹 = 𝑅𝑅
𝑗𝑗

1+(1+𝑗𝑗)−𝑛𝑛
b. 𝐹𝐹 = 𝑅𝑅
𝑗𝑗

(1+𝑗𝑗)𝑛𝑛 −1
c. 𝐹𝐹 = 𝑅𝑅
𝑗𝑗

(1+𝑗𝑗)𝑛𝑛 +1
d. 𝐹𝐹 = 𝑅𝑅
𝑗𝑗

9. What do you call a specific time chosen to compare the time value of one or more
dated sums of money?
a. economic value
b. focal date
c. future value
d. present value

2 CO_Q2_General Mathematics SHS


Module 6
10. What is the future value of quarterly payments of ₱2,000 for 5 years with an
interest rate of 8% compounded quarterly?
a. ₱ 46,594.74
b. ₱ 47,594.74
c. ₱ 45,594.74
d. ₱ 48,594.74

11. Cash inflows can be represented by ___________ numbers?


a. even
b. negative
c. odd
d. positive

12. How much is the fair market value of shoes if a vendor sells it for ₱1,200.
Cedric offers to buy it at ₱1,000 and they agreed at the middle price?
a. ₱1,100
b. ₱1,150
c. ₱1,175
d. ₱1,050

(1+𝑗𝑗)𝑛𝑛 −1
13. What is R in the formula 𝐹𝐹 = 𝑅𝑅 ?
𝑗𝑗

a. future value
b. present value
c. regular payment
d. time

14. How much is the fair market value of the truck if it is originally sold at ₱ 250,000
and Dan offered to buy the truck at ₱ 225,000 to which the seller agreed?
a. ₱ 275,000
b. ₱ 225,000
c. ₱ 250,000
d. ₱ 237,500

15. Cash outflows can be represented by ___________ numbers.


a. even
b. negative
c. odd
d. positive

3 CO_Q2_General Mathematics SHS


Module 6
Lesson
Fair Market Value of a Cash
1 Flow

Fair market value is the price an asset can be sold in the open market when certain
conditions are met. The conditions are as follows: the parties involved are aware of
all the facts, are acting in their own interest, are free of any pressure to buy or sell,
and have ample time to make decisions. This module focuses on calculating the fair
market value of a cash flow stream on an annuity.

What’s In

Since our discussion in this module is anchored on the idea of general annuity, let
us recall some concepts from the previous lessons on the future and the present
values of a general annuity. In this way, it will be easier for you to understand the
lesson.

Let us start with the difference between a general annuity and general ordinary
annuity. A general annuity is defined as an annuity where the length of the payment
interval is not the same as the length of the interest compounding period, while the
general ordinary annuity is a general annuity in which the periodic payment is made
at the end of the payment interval.

Examples of a general annuity include monthly installment payment of a car, lot, or


house with an interest rate that is compounded annually and paying a debt semi-
annually when the interest is compounded monthly.

Let us also recall how to determine the future and present value of a general ordinary
annuity. The future value F and the present value P of a general ordinary annuity is
given by

(1+j)n −1 1−(1+j)−n
F=R and P=R
j j

where R is the regular payment


j is the equivalent interest rate per payment interval converted from the
interest rate per period; and
n is the number of payments.

4 CO_Q2_General Mathematics SHS


Module 6
Let us have the following examples to further deepen our understanding of the
previous lesson which is essential in tackling this module.
Example 1: Suppose Mrs. Remoto would like to save ₱3,000 at the end of each month
for six months, in a fund that gives 9% compounded monthly. How much is the
amount of future value of her savings after 6 months?

Solution:
Given: R = ₱ 3,000
term t = 6 months
interest rate per annum i(12) = 0.09
number of conversions per year m = 12
0.09
interest rate per period 𝑗𝑗 = = 0.0075
12

Find: amount (future value) at the end of the term, F

a.) The cash flow for this problem is given below:


3,000 3,000 3,000 3,000 3,000 3,000
0 1 2 3 4 5 6
b.) The future value of all the payments at the end of the term (t=6) is given
below:
3,000 3,000 3,000 3,000 3,000 3,000
0 1 2 3 4 5 6

3,000

3,000(1+0.0075)

3,000(1+0.0075)2

3,000(1+0.0075)3

3,000(1+0.0075)4

3.000(1+0.0075)5

c.) Adding all the future values obtained from the previous step.

3,000 = 3,000

3,000(1+0.0075) = 3,022.5

3,000(1+0.0075)2 = 3,045.16875

3,000(1+0.0075)3 = 3,068.00752

3,000(1+0.0075)4 = 3,091.01757

3,000(1+0.0075)5 = 3,114.20020

F =18,340.89

5 CO_Q2_General Mathematics SHS


Module 6
d.) Let us solve using the formula
(1+𝑗𝑗)𝑛𝑛 −1
𝐹𝐹 = 𝑅𝑅
𝑗𝑗

(1+0.0075)6 −1
𝐹𝐹 = 3,000
0.0075

𝐹𝐹 = 18,340.89

Example 2: Suppose Mrs. Remoto (in problem no.1) would like to know the present
value of her monthly deposit of ₱ 3,000 when interest is compounded monthly. How
much is the present value of her savings at the end of 6 months?

Solution:

Given: R = ₱3,000

term t = 6 months

interest rate per annum i(12) = 0.09

number of conversion per year m = 12


0.09
interest rate per period 𝑗𝑗 = = 0.0075
12

Find: Present value, P

a.) The cash flow for this problem is given below:

3,000 3,000 3,000 3,000 3,000 3,000


0 1 2 3 4 5 6

b.) Discount the payment of each period to the beginning of the term that is,
find the present value of each payment. Recall the formula:
𝐹𝐹 3,000
𝑃𝑃 = 𝑚𝑚𝑚𝑚 = = 3,000(1.0075)−𝑡𝑡
𝑖𝑖 𝑚𝑚 1.0075𝑡𝑡
�1 + �
𝑚𝑚

3,000 3,000 3,000 3,000 3,000 3,000

0 1 2 3 4 5 6

3,000(1.0075)-1

3,000(1.0075)-2

3,000(1.0075)-3

3,000(1.0075)-4

3,000(1.0075)-5

3,000(1.0075)-6

6 CO_Q2_General Mathematics SHS


Module 6
c.) Adding all the discounted payments to get the present value.

3,000(1.0075)-1 = 2,977.667

3,000(1.0075)-2 = 2,955.501

3,000(1.0075)-3 = 2,933.50

3,000(1.0075)-4 = 2,911.663

3,000(1.0075)-5 = 2,889.988

3,000(1.0075)-6 = 2,868.474

P =17,536.79

d.) Let us solve using the formula


1−(1+𝑗𝑗)−𝑛𝑛
𝑃𝑃 = 𝑅𝑅
𝑗𝑗

1−(1+0.0075)−6
𝑃𝑃 = 3,000
0.0075

𝑃𝑃 = 17,536.79

Examples 3 and 4 are left for you to try to find the future value (F) and present
value (P).

Example 3: In order to save for her high school graduation, Marie decided to save ₱
200 at the end of each month. If the bank pays 0.250% compounded monthly, how
much will her money be at the end of 6 years?

Example 4: Mr. Ribaya paid ₱ 200,000 as a down payment for the car. The remaining
amount is to be settled by paying ₱16,200 at the end of each month for 5 years. If
interest is 10.5% compounded monthly, what is the cash price of his car.

Notes to the Teacher


The present and future values of an annuity are also related by

𝐹𝐹
𝐹𝐹 = 𝑃𝑃 (1 + 𝑗𝑗)𝑛𝑛 and 𝑃𝑃 = (1+𝑗𝑗)𝑛𝑛

7 CO_Q2_General Mathematics SHS


Module 6
What’s New

CONCEPT MAPPING: Make a concept map from the given words below by arranging
the word into an idea and connect by either a word or a phrase.

(1 + 𝑗𝑗)𝑛𝑛 − 1
𝐹𝐹 = 𝑅𝑅
𝑗𝑗 Annuity
Future
Value

Fair General
Cash Market ordinary
Flow Value annuity

General Present
annuity 1 − (1 + 𝑗𝑗)−𝑛𝑛
𝑃𝑃 = 𝑅𝑅 Value
𝑗𝑗

Rubric for the above activity:


Excellent Good Fair Poor
Score
(10 points) (8 points) (6 points) (4 points)
The The
The presentation
presentation presentation The
was thoroughly
was well was somewhat presentation
Organization organized and
organized and easy
organized and was somewhat
to follow most of
in logical somewhat confusing.
the time.
formation. incoherent.
Shows lack of Shows no
Shows an Shows a few
understanding understanding
understanding misunderstandings
Content and of the topic’s of the topic’s
of the topic’s of the topic’s
content and content and
Concept content and no content and few
some many
misconceptions misconceptions are
misconceptions misconceptions
are evident. evident.
are evident. are evident.
All words Most words Some words
All words
accurately accurately accurately
accurately
Knowledge of connected and
connected and
connected and connected and
shows an shows a little shows no
Relationship understanding
shows some
understanding understanding
understanding of
among Concepts of the of the of the
the relationship
relationship relationship relationship
among them.
among them. among them. among them.

Total

8 CO_Q2_General Mathematics SHS


Module 6
What is It

Concept maps are visual representations of information. They can take the form of
charts, graphic organizers, tables, flowcharts, Venn diagrams, timelines, or T-charts.
Concept maps are especially useful for students who learn better visually, although
they can benefit any type of learner.

In the previous activity, you need to arrange the given words logically in the form of
either chart, organizers, flowcharts, etc. and connect the words by either word or
phrase to convey a meaningful idea. The concept you have formed will be essential
in the discussion of this topic.

After you finish the activity, try to answer the following:

1. Have you organized the words logically?


2. What idea or concept have you learned from the activity?
3. How can you define domain and range?

If you find difficulty in doing the activity, that is okay because after you read more
about this module on how to determine the fair market value of a cash flow stream,
you are free to go back to our activity and try to re-arrange the words in a logical
way.

Fair Market Value of a Cash Flow

Cash flow is a term that refers to payments received (cash inflows) or payments or
deposits made (cash outflows). Cash inflows can be represented by positive numbers
and cash outflows can be represented by negative numbers. It is also the amount of
cash and cash-equivalents being transferred into and out of the business.

The fair market value or economic value of cash flow (payment stream) on a
particular date refers to a single amount that is equivalent to the value of the
payments stream at that date. This particular date is called the focal date. In its
simplest sense, fair market value (FMV) is the price that an asset would sell for on
the open market.

Example 1. Mr. Ribaya received two offers on a lot that he wants to sell. Mr. Ocampo
has offered ₱ 50,000 and a ₱1 million lump-sum payment 5 years from now. Mr.
Cruz has offered ₱ 50,000 plus ₱ 40,000 every quarter for five years. Compare the
fair market values of the two offers if money can earn 5% compounded annually.
Which offer has a higher market value?

9 CO_Q2_General Mathematics SHS


Module 6
Given:
Mr. Ocampo’s Offer Mr. Cruz’s offer
₱ 50,000 down payment ₱ 50,000 down payment
₱ 40,000 every quarter for
₱ 1,000,000 after 5 years
five years

Find: Fair market value of each offer

Solution:

Let us illustrate the cash flows of the two offers using time diagrams.

Mr. Ocampo’s offer:

50,000 1 million
0 1 2 3 4 5
Mr. Cruz’s offer:

50,000
40,000 40,000 40,000 … 40,000
0 1 2 3 … 20

Choose a focal date and determine the values of the two offers at that focal
date. For example, the focal date can be the date at the start of the term.

Since the focal date is at t=0, compute for the present value of each other.

Mr. Ocampo’s offer: Since ₱ 50,000 is offered today, then its present value is still
₱50,000. The present value of ₱ 1,000,000 offered 5 years from now is:

𝑃𝑃 = 𝐹𝐹(1 + 𝑗𝑗)−𝑛𝑛

𝑃𝑃 = 1,000,000 (1 + 0.05)−5

𝑃𝑃 = ₱ 783,526.17

Fair Market Value (FMV) = Down payment + Present Value

= 50,000+ 783,526.17

FMV = ₱ 833,526.17

10 CO_Q2_General Mathematics SHS


Module 6
Mr. Cruz’s offer: We first compute for the present value of a general annuity with
quarterly payments but with annual compounding at 5%.

Solve the equivalent rate, compounded quarterly, of 5% compounded annually.

𝐹𝐹1 = 𝐹𝐹2
(4)(5) (1)(5)
𝑖𝑖 4 𝑖𝑖 1
𝑃𝑃 �1 + � = 𝑃𝑃 �1 + �
4 1

20
𝑖𝑖 4 0.05 5
�1 + � = �1 + �
4 1

𝑖𝑖 4
1+ = (1.05)1/4
4

𝑖𝑖 (4)
= (1.05)1/4 − 1
4

𝑖𝑖 (4)
= 0.01227
4

The present value of an annuity is given by


1 − (1+𝑗𝑗)−𝑛𝑛
𝑃𝑃 = 𝑅𝑅
𝑗𝑗

1− (1+0.1227)−20
𝑃𝑃 = 40,000
0.012272

𝑃𝑃 = ₱ 705,588.40

Fair Market Value (FMV) = Down payment + Present Value

= 50,000 + 705,588.40

(FMV) = ₱ 755.588.40

Hence, Mr. Ocampo’s offer has a higher market value. The difference between the
market values of the two offers at the start of the term is

833,526.17 – 755,588.40 = ₱ 77,937.77

Alternate solution (Focal date at the end of the term):

Mr. Ocampo’s offer:

The future value of ₱1,000,000 at the end of the term at 5% compounded annually
is given by

𝐹𝐹 = 𝑃𝑃 (1 + 𝑗𝑗)𝑛𝑛

𝐹𝐹 = 50,000 (1 + 0.05)5

𝐹𝐹 = 63,814.08

The fair market value of this offer at the end of the term is 63,814.08 plus ₱
1,000,000 amounting to ₱ 1,063,814.08.

11 CO_Q2_General Mathematics SHS


Module 6
Mr. Cruz’s offer:

The future value of this ordinary general annuity is given by


(1+𝑗𝑗)𝑛𝑛 −1
𝐹𝐹 = 𝑅𝑅
𝑗𝑗

(1+0.01227)20−1
𝐹𝐹 = 40,000
0.012272

𝐹𝐹 = 900,489.71
The future value of 50,000 at the end of the term is ₱63,814.08, which was already
determined earlier.

Fair Market Value (FMV) = 900,489.71 + 63,814.08 = ₱ 964,303.79

As expected, Mr. Ocampo’s offer still has a higher market value, even if the local
date was chosen to be at the end of the term. The difference between the market
values of the two offers at the end of the term is

1,063814.08 – 964,303.79 = ₱ 99,510.29

You can also check that the present value of the difference is the same as the
difference when the focal date was at the start of the term:

𝑃𝑃 = 99,510.29 (1 + 0.05)−5 = ₱ 77,968.92

Example 2. Company A offers ₱150,000 at the end of 3 years plus ₱300,000 at the
end of 5 years. Company B offers ₱25,000 at the end of each quarter for the next 5
years. Assume that money is worth 8% compounded annually. Which offers has a
better market value?

Given:

Company A Company B
₱150,000 at the end of 3 years ₱25,000 at the end of each
₱300,000 at the end of 5 years quarter for the next 5 years

Find: the fair market value of each offer

Solution:

Illustrate the cash flow of the two offers using time diagrams.

Company A offer:

150,000 300,000
0 1 2 3 4 5

Company B offer:

25,000 25,000 25,000 … 25,000


0 1 2 3 … 20

12 CO_Q2_General Mathematics SHS


Module 6
Suppose that selected focal date is the start of the term. Since the focal date is the
start of the term, compute for the present value of each offer.

Company A offer:

The present value of ₱ 150,000 three years from now is

𝑃𝑃1 = 𝐹𝐹 (1 + 𝑗𝑗)−𝑛𝑛

𝑃𝑃1 = 150,000 (1 + 0.04)−6

𝑃𝑃1 = ₱ 118,547.18

The present value of ₱300,000 five years from now is

𝑃𝑃2 = 𝐹𝐹 (1 + 𝑗𝑗)−𝑛𝑛

𝑃𝑃2 = 300,000 (1 + 0.04)−10

𝑃𝑃2 = ₱ 202,669.25

Fair Market Value (FMV) = P1 + P2

= 118,547.18 + 202,669.25

=₱321.216.43

Compute for the present value of a general annuity with quarterly payments but
with semi-annual compounding at 8%.

Solve the equivalent rate, compounded quarterly, of 8% compounded semi-


annually.

𝐹𝐹1 = 𝐹𝐹2
(4)(5) (2)(5)
𝑖𝑖 4 𝑖𝑖 2
𝑃𝑃 �1 + � = 𝑃𝑃 �1 + �
4 2

20
𝑖𝑖 4 0.08 10
�1 + � = �1 + �
4 2

1
𝑖𝑖 4
1 + = (1.04)(2)
4
1
𝑖𝑖 4
= (1.04)(2) − 1
4

𝑖𝑖 4
= 0.01980
4

The present value of an annuity is given by

1 − (1 + 𝑗𝑗) −𝑛𝑛
𝑃𝑃 = 𝑅𝑅
𝑗𝑗

1 – (1 + 0.01980) −20
𝑃𝑃 = 25,000
0.01980
𝑃𝑃 = ₱ 409,575.91

13 CO_Q2_General Mathematics SHS


Module 6
Therefore, Company B offer is preferable since its market value is larger.

Example 3. Kat received two offers for investment. The first one is ₱150,000 every
year for 5 years at 9% compounded annually. The other investment scheme is
₱12,000 per month for 5 years with the same interest rate. Which fair market
value between these offers is preferable?

Solution:

Let the focal point be the end of the term, and so, compute for the future
value.

First offer (ordinary annuity)

Given: R = ₱150,000, i(1) = 0.09, m=1, t=5, n=5 payments


(1+𝑗𝑗)𝑛𝑛 −1
𝐹𝐹 = 𝑅𝑅
𝑗𝑗

(1+0.09)5 −1
𝐹𝐹 = 150,000
0.09

𝐹𝐹 = ₱ 897,706.59

Second offer: (General Annuity)

Given: R = 12,000, i(1)=0.09, m = 1, n = (5)(12)=60 monthly payment interval

Convert 9% compounded annually to its equivalent interest rate for monthly


payment interval.
(12) 1
𝑖𝑖 (12) 𝑖𝑖 1
𝑃𝑃 �1 + � = 𝑃𝑃 �1 + �
12 1

(12)
𝑖𝑖 (12)
�1 + � = (1 + 0.09)
12

1
𝑖𝑖 (12)
= (1.09)(12) − 1
12

𝑖𝑖 (12)
= 𝑗𝑗 = 0.00721
12

Find the future value of this investment


(1+𝑗𝑗)𝑛𝑛 −1
𝐹𝐹 = 𝑅𝑅
𝑗𝑗

60
𝑜𝑜0𝑝𝑝𝑝𝑝𝑝𝑝𝑝𝑝
–1
𝑝𝑝𝑝𝑝[=𝑙𝑙1+0.00721)
𝐹𝐹 = 12,000 0.00721

𝐹𝐹 = ₱ 896,869.86

14 CO_Q2_General Mathematics SHS


Module 6
What’s More

Activity 1.1
Complete the table by computing the future value and present value of an ordinary
annuity given the following conditions.

Future Value Present


(F) Value (P)
Monthly payments of ₱ 3,000 for 4 years with
1. an interest rate of 3% compounded
quarterly.

Quarterly payments of ₱ 5,000 for 10 years


with an interest rate of 2% compounded
annually.
2.
Semi-annual payments of ₱105,000 with an
3. interest rate of 12% compounded annually
for 5 years.

Activity 1.2

Complete the table by filling up the columns with the required values.

Converted
R im m t n interest F P
rate (j)

Monthly payments of
₱ 2,000 for 5 years with an
1.
interest rate of 12%
compounded quarterly.
Quarterly payments of
₱ 15,000 for 10 years with
2.
an interest rate of 8%
compounded annually.
Semi-annual payments of ₱
150,000 with an interest
3.
rate of 6% compounded
annually for 6 years.

15 CO_Q2_General Mathematics SHS


Module 6
Activity 1.3

Answer the following problems:

1. The buyer of a car pays ₱ 169,000 cash and ₱ 12,000 every month for 5
years. If money is 10% compounded monthly, how much s the cash price
of the car?
2. The buyer of a lot pays ₱ 50,000 cash and ₱ 10,000 every month for 10
years. If money is 8% compounded monthly how much is the cash value
of the lot?

What I Have Learned

Fill in each blank with correct word or words to complete each statement.

1. ____________________ is defined as an annuity where the length of the payment


interval is not the same as the length of the interest compounding period while
_________________________ is a general annuity in which the periodic payment is
made at the end of the payment interval.

2. A _______________is a term that refers to payments received (_________________)


or payments or deposits made (___________________). Cash inflows can be
represented by ______________ numbers and cash outflows can be represented
by _____________ numbers. It is also the amount of cash and cash-equivalents
being transferred into and out of the________________.

3. The fair market value or __________________ of a cash flow (payment stream) on


a particular date refers to a single amount that is equivalent to the value of the
payments stream at that date. This particular date is called the _____________.

16 CO_Q2_General Mathematics SHS


Module 6
What I Can Do

Answer the problem given below by showing your complete solution and your
decision on a sheet of paper.

1. A television set is for sale at ₱ 13,499 in cash or on installment terms,


₱ 2,500 each month for the next 6 months at 9% compounded annually.
If you were the buyer, what would you prefer, cash or installment?

Assessment

Choose the letter of the best answer.

1. What is the other term for economic value?


a. fair market value
b. future value
c. general annuity
d. present value

(1+𝑗𝑗)𝑛𝑛 − 1
2. The formula 𝑅𝑅 is used to determine the __________ of an ordinary
𝑗𝑗
annuity.
a. annuity
b. cash flow
c. future value
d. present value

3. What is the future value of a semi-annual payments of ₱ 8,000 for 12 years


with interest rate of 12% compounded semi-annually?
a. ₱ 407,524.60
b. ₱ 408,524.60
c. ₱ 405,524.60
d. ₱ 406,524.60

4. Cash ___________ can be presented by positive numbers.


a. flows
b. inflows
c. outflows
d. value

17 CO_Q2_General Mathematics SHS


Module 6
5. How much is the original selling price of shoes being sell by the vendor if
Cedric offers to buy it at ₱ 1,000 and they agreed at its fair market value of
₱ 1,150?
a. ₱ 1,200
b. ₱ 1,100
c. ₱ 1,000
d. ₱1,300

6. It is the price an asset would sell for on the open market when certain
conditions are met.
a. annuity
b. cash flow
c. down payment
d. fair market value

7. What is the present value of an ordinary annuity having semi-annual


payments of ₱8,000 for 12 years with an interest rate of 12% compounded
semi-annually?
a. ₱ 110,402.90
b. ₱ 100,402.90
c. ₱ 105,402.90
d. ₱ 103,402.90

8. Cash ____________ can be represented by a negative number.


a. flows
b. inflows
c. outflows
d. value

9. It is an annuity where the length of the payment interval is not the same as
the length of the interest compounding period.

a. cash flow
b. fair market value
c. general annuity
d. general ordinary annuity

1−(1+𝑗𝑗)−𝑛𝑛
10. The formula 𝑅𝑅 is used to determine the __________ of an ordinary
𝑗𝑗
annuity.
a. future value
b. present value
c. real value
d. zero value

18 CO_Q2_General Mathematics SHS


Module 6
11. It is the amount of cash and cash-equivalents being transferred into and out
of the business.

a. cash flow
b. fair market value
c. general annuity
d. general ordinary annuity

12. What is the future value of an ordinary annuity having daily payments of ₱
50 for 30 days with an interest rate of 20% compounded daily?

a. ₱ 1,611.98
b. ₱ 1,511.98
c. ₱ 1,411.98
d. ₱ 1,311.98

13. A store sells a washing machine. Mark offers to give a down payment of
₱5,000 and pay ₱6,000 at the end of every 6 months for two years. Assuming
that the money compounds by 3% monthly. What is the economic value of
the washing machine?

a. ₱ 24,545.42
b. ₱ 23,126.31
c. ₱ 20,000
d. ₱ 28,126.31
(1+𝑗𝑗)𝑛𝑛 −1
14. What is n in the formula 𝐹𝐹 = 𝑅𝑅 ?
𝑗𝑗

a. future value
b. present value
c. number of payments
d. regular payment

15. What is the present value of an ordinary annuity having daily payments of
₱ 50 for 30 days with an interest rate of 20% compounded daily?

a. ₱ 1,587.33
b. ₱ 1,487.33
c. ₱ 1,387.33
d. ₱ 1,687.33

19 CO_Q2_General Mathematics SHS


Module 6
Additional Activities

Decide on the following problems by determining the fair market value. Show
your solutions in a sheet of paper together with your decision on the problem.

1. Investment in Sunrise Company is ₱ 100,000 at the end of 5 years plus


₱ 24,000 annually for 4 years afterwards. Investment in XYZ company offers
₱ 50,000 semi-annually ₱ 15,000 every 6 months after 6 years. Assume that
the money is worth 9% compounded annually, which investment is
preferable?

2. A motorcycle is for sale ₱ 60,500 cash or on installment terms ₱ 3,000 per


month for 2 years at 12% compounded annually. If you were the buyer, what
would you prefer, cash or installment?

20 CO_Q2_General Mathematics SHS


Module 6
Module 6
CO_Q2_General Mathematics SHS 21
What's More
Activity 1.1
F P
1. ₱152,379.36 ₱135,537.43
2. ₱220,630.08 ₱180,993.53
3. ₱1,372,987.26 ₱779,069.84
Activity 1.2
R im m t n j F P
1. ₱2,000 0.12 12 5 60 0.009902 ₱162,823.88 ₱90151.63
2. ₱15,000 0.08 4 10 40 0.019427 ₱894,851.52 ₱528,728.78
3. ₱150,000 0.06 2 6 12 0.029563 ₱2,123,527.28 ₱1,497,002.94
Activity 1.3
1. Cash Price = 169,000 + 564,784.43 = ₱733,784.43
2. Cash Value = 50,000 + 824,214.81 = ₱874,214.81
What I Know Assessment
1. b 1. a
2. c 2. c
3. a 3. d
4. c 4. b
5. d 5. a
6. d 6. d
7. a 7. b
8. c 8. c
9. b 9. c
10. d 10. d
11. d 11. a
12. a 12. b
13. c 13. d
14. b 14. c
15. b 15. b
Answer Key
References
Chen, James. 2020. Financial Analysis. February 21. Accessed June 5, 2020.
https://www.investopedia.com/terms/f/fairmarketvalue.asp.
Commission on Higher Education. 2016. Teaching Guide for Senior High School
General Mathematics. Quezon city: Commission on Higher Education.
n.d. "Concept Map Rubric." Accessed June 10, 2020.
https://www.nps.gov/grsm/learn/education/classrooms/upload/Concept-
Map-Scoring-Rubric.pdf.
DepEd - Bureau of Learning Resources. 2016. General Mathematics: Learner's
Material First Edition. Pasig City: Department of Education.
Ioncmaste. n.d. Mathematics of Finance Project. Accessed June 10, 2020.
https://sites.ualberta.ca/~ioncmast/financefolder/glossary.html#:~:text=Foc
al%20Date%3A%20A%20specific%20time,at%20an%20appropriate%20interes
t%20rate.
teachtogether. n.d. Simple Annuities. Accessed June 5, 2020.
http://teachtogether.chedk12.com/teaching_guides/view/29#.
Tuovila, Alicia. 2020. Corporate Finance and Accounting. March 5. Accessed June 5,
2020. https://www.investopedia.com/terms/c/cashflow.asp.
University of California at Chapel Hill. n.d. The Learning Center-UNC-Concept Maps.
Accessed June 10, 2020. https://learningcenter.unc.edu/tips-and-
tools/using-concept-maps/.

22 CO_Q2_General Mathematics SHS


Module 6
For inquiries or feedback, please write or call:

Department of Education - Bureau of Learning Resources (DepEd-BLR)

Ground Floor, Bonifacio Bldg., DepEd Complex


Meralco Avenue, Pasig City, Philippines 1600

Telefax: (632) 8634-1072; 8634-1054; 8631-4985

Email Address: blr.lrqad@deped.gov.ph * blr.lrpd@deped.gov.ph


General Mathematics
Quarter 2 – Module 7:
Deferred Annuity

CO_Q2_General Mathematics SHS


Module 7
General Mathematics – Senior High School
Alternative Delivery Mode
Quarter 2 – Module 7: Deferred Annuity
First Edition, 2020

Republic Act 8293, section 176 states that: No copyright shall subsist in any work of
the Government of the Philippines. However, prior approval of the government agency or office
wherein the work is created shall be necessary for exploitation of such work for profit. Such
agency or office may, among other things, impose as a condition the payment of royalties.

Borrowed materials (i.e., songs, stories, poems, pictures, photos, brand names,
trademarks, etc.) included in this module are owned by their respective copyright holders.
Every effort has been exerted to locate and seek permission to use these materials from their
respective copyright owners. The publisher and authors do not represent nor claim ownership
over them.

Published by the Department of Education


Secretary: Leonor Magtolis Briones
Undersecretary: Diosdado M. San Antonio

Development Team of the Module


Writers: Jenn Wynzel L. Derecho and Moahna Aura M. Mancenido
Editors: Elizabeth D. Lalunio, Elizabeth B. Dizon, Anicia J. Villaruel, Roy O. Natividad
Reviewers: Fritz A. Caturay, Necitas F. Constante, Dexter M. Valle, Jerome
A. Chavez, Edmund Actub, John Lester F. Guerero
Illustrator: Hanna Lorraine Luna and Diane C. Jupiter
Layout Artist: Roy O. Natividad, Sayre M. Dialola, Noel Rey T. Estuita, Argie L. Ty
Management Team: Francis Cesar B. Bringas
Job S. Zape, Jr.
Ramonito Elumbaring
Reicon C. Condes
Elaine T. Balaogan
Fe M. Ong-ongowan
Hermogenes M. Panganiban
Phillip B. Gallendez
Josephine T. Natividad
Anicia J. Villaruel
Dexter M. Valle

Printed in the Philippines by ________________________

Department of Education – Region IV-A CALABARZON

Office Address: Gate 2 Karangalan Village, Barangay San Isidro


Cainta, Rizal 1800
Telefax: 02-8682-5773/8684-4914/8647-7487
E-mail Address: region4a@deped.gov.ph
General Mathematics
Quarter 2 – Module 7:
Deferred Annuity
Introductory Message

This Self-Learning Module (SLM) is prepared so that you, our dear


learners, can continue your studies and learn while at home. Activities,
questions, directions, exercises, and discussions are carefully stated for you
to understand each lesson.

Each SLM is composed of different parts. Each part shall guide you
step-by-step as you discover and understand the lesson prepared for you.

Pre-tests are provided to measure your prior knowledge on lessons in


each SLM. This will tell you if you need to proceed on completing this module
or if you need to ask your facilitator or your teacher’s assistance for better
understanding of the lesson. At the end of each module, you need to answer
the post-test to self-check your learning. Answer keys are provided for each
activity and test. We trust that you will be honest in using these.

In addition to the material in the main text, Notes to the Teacher are
also provided to our facilitators and parents for strategies and reminders on
how they can best help you on your home-based learning.

Please use this module with care. Do not put unnecessary marks on
any part of this SLM. Use a separate sheet of paper in answering the exercises
and tests. And read the instructions carefully before performing each task.

If you have any questions in using this SLM or any difficulty in


answering the tasks in this module, do not hesitate to consult your teacher
or facilitator.

Thank you.
What I Need to Know

Previously, you learned how to solve simple and general annuity. So far, all
the problems on finding present and future values only dealt with a single cash flow
which is either invested at the start (for future value problems) or to receive at the
end (for present value problems). However, most of the financial events happening in
people’s lives rarely happen in just a single event. It is common for workers to receive
their salary twice a month or monthly, to pay loans, electricity, water, phone, and
other utility bills monthly, and likewise, be able to set aside savings regularly with
these normal routines in mind, it is then important to be able to set up a method to
efficiently compute the future and the present value of a regular stream of cash
flows.

This module will help you understand and explore deferred annuity or an
annuity whose payments do not necessarily start at the beginning or at the end of
the next compounding period. Example of which is the monthly pension that will
start after five years if a certain employee avails the five-year lump sum upon
retirement.

After going through this module, you are expected to:

1. calculate the present value and period of deferral of a deferred annuity; and

2. construct a time diagram for a deferred annuity.

1
CO_Q2_General Mathematics SHS
Module 7
What I Know

Let’s find out how far you might already know about this topic! Please take
this challenge! Have Fun!

Directions: Choose the letter of the best answer. Write the chosen letter on a separate

sheet of paper.

1. A credit card company offers a deferred payment option for the purchase of
any appliance. Rose plans to buy a smart television set with monthly payment
of ₱ 4,000.00 for two years. The payment will start at the end of 3 months. If
it is 10% compounded monthly, what is the period of deferral?
a. 1 c. 3
b. 2 d. 4

2. What are the actual payments should Rosa make in the situation in
question number 1?
a. 2 c. 24
b. 4 d. 26

3. What is the present value of the television set in the situation in question
number 1?

a. ₱ 30,519.20 c. ₱ 85,260.53
b. ₱ 78,716.74 d. ₱ 143,519.84

4. What is the period of deferral if monthly payment of ₱ 2,000.00 for 8 years will start

6 months from now?

a. 4 c. 6
b. 5 d. 9

2
CO_Q2_General Mathematics SHS
Module 7
5. Which of the following annuity below does not begin until a given time interval
has passed?

a. Simple Annuity c. Deferred Annuity

b. General Annuity d. Contingent Annuity

6. A new businessman’s debt is to be paid by regular payments of ₱ 25,000.00


paid at the end of every 3 months for 8 years for his products. The first
payment will be done after 3 years. How may periods of deferral are there?

a. 5 c. 11

b. 7 d. 12

7. A father decided to own a lot only as debt that is to be paid by regular


payments of ₱ 3500.00 paid at the end of every 2 months for 5 years. The first
payment is due at the end of one year. How many periods of deferral are there?
a. 2 c. 10
b. 5 d. 12

8. What is the present value of a deferred annuity of ₱ 30,000.00 at the end of


every year for 10 years if the first payment of a mother will be done at the end
of the second year, and the money is compounded at 3% annually?
a. ₱ 244,354.12 c. ₱ 278,520.14
b. ₱ 248,452.51 d. ₱ 291,245.18

9. What is the period of deferral if semi-annual payment of ₱ 12,000.00 for 10


years that will start 5 years from now?
a. 5 c. 10
b. 9 d. 11

10. Mariel purchased a smart television set through the credit cooperative of their
company. The cooperative provides an option for a deferred payment. Mariel
decided to pay after 2 months of purchase. Her monthly payment is computed
at ₱ 3,800.00 payable in 12 months. If the interest rate is 12% convertible
monthly, what is the period of deferral?
a. 1 c. 3
b. 2 d. 12

11. What is the present value of the television set in the situation in question
number 10?
a. ₱ 41,800.00 c. ₱ 42,345.84
b. ₱ 41,854.14 d. ₱ 46,816.00

3
CO_Q2_General Mathematics SHS
Module 7
12. An employee, preparing for his retirement 20 years from now, agreed to
pay an insurance company ₱ 20,000.00 every year until retirement in
exchange of yearly pension for 25 years. If the insurance company puts
the payment in an account providing 2.3% annual compound interest,
how much yearly pension can he receive?
a. ₱ 26,559.85 C. ₱ 31,254. 12
b. ₱ 29,457.63 D. ₱ 32,127.23

13. What is the period of deferral if an employee makes a semi-annual


payment employee of ₱ 11,500.00 for 10 years which will start 5 years
from now?
a. 20 c. 9
b. 10 d. 5

14. An agricultural loan is to be repaid quarterly for 5 years that will start at
the end of 2 years. If the interest rate is 6% converted quarterly, how much
is the loan if the quarterly payment is ₱ 10,000.00?
a. ₱ 154,694.03 c. ₱ 163,215.12
b. ₱ 157,455.08 d. ₱ 173,234.05

15. Ana converted her loan to light payments which gives her an option to pay
₱ 1,500.00 every month for 2 years. The first payment is due 3 months
from now. How much is the amount of the loan if the interest rate is 9%
converted monthly?
a. ₱ 29,245.60 c. ₱ 32,105.25
b. ₱ 29,725.55 d. ₱ 32,346.70

4
CO_Q2_General Mathematics SHS
Module 7
Lesson

7 Deferred Annuity

Learning new things like investigating, analyzing, and solving problems


involving simple and compound interests and simple and general annuities are
important to be financially literate. With these, you can make a wise decision about
the value of your money and how to invest your money for maximum benefits.

People are always looking for ways to supplement their existing financial
strategies, best investment, continuous growth, and ways to save for retirement for
longer periods. A deferred annuity is one tool that can serve just such a purpose.
Also, it is a contract with an insurance company that promises to pay the owner a
regular income, or a lump sum, at some future date. Investors often use deferred
annuities to supplement their other retirement income or benefits, such as Social
Security (retirement, disability, and supplemental benefits. If you have several years
until retirement, a deferred annuity could make sense for you.

A deferred annuity can be a long-term investment in which you invest a sum


of money, then receive payments several years down the line after the initial sum has
accrued interest. Also, it can be a contract with an insurance company that promises
to pay the owner a regular income, or a lump sum, at some future date. It come in
several types-fixed, indexed, and variable-which determine how their rate of return
is computed.

What’s In

Review the definition of terms:


Annuity. It is a sequence of equal payments (or deposits) made at a regular interval
of time.

Annuity Immediate or Ordinary Annuity. It is a type of annuity in which the


payments are made at the end of every period.
1−(1+𝑗𝑗)−𝑛𝑛
P=R� �
𝑗𝑗

5
CO_Q2_General Mathematics SHS
Module 7
Review on how to calculate the present value of an annuity immediate or ordinary
annuity:

Scenario 1. Suppose a senior high school student wants to purchase a cellular


phone for online learning. He decided to pay monthly for 1 year starting at
the end of the month. How much is the cost of the cellular phone if his monthly
payment is ₱ 2,500.00 and the interest rate is 9% compounded monthly?

Solution:

Given periodic payment R = 2,500.00

interest rate per annum i (12) = 0.09

term t = 1 year

number of conversion per year m = 12

Find: P

𝑖𝑖 (12)
The interest per period is 𝑗𝑗 = = 0.0075
𝑚𝑚

The number of payments is n = mt = 12(1) = 12

The present value of an ordinary annuity is

1−(1+𝑗𝑗)−𝑛𝑛 1−(1+0.0075)−12
P =R� � = 2500 � � = 28, 587.28
𝑗𝑗 0.0075

Thus, the cost of the cellular phone now is ₱ 28,587.28

6
CO_Q2_General Mathematics SHS
Module 7
What’s New

Recall the previous scenario. What if the senior high school student is
considering a different payment scheme to buy the cellular phone? In this scheme,
he has to pay ₱ 2,500.00 monthly for 1 year starting at the end of the fourth month.
If the interest rate is 9% converted monthly, how much is the cash value of the
cellular phone?

In this example, the senior high school student will start paying at the end of
the 4th month up to the end of the 15th month. The time diagram for his option is
given by:

2500 2500 … 2500

0 1 2 3 4 5 … 15

Now, how do we get the present value of this annuity?

Assume payments are also being made during the period of deferral;
Step 1 in other words, there are no skipped payments. The associate time
diagram is:

P =?
2500 2500 2500 2500 2500 … R

0 1 2 3 4 5 … 15

From the previous lesson, the present value P of the ordinary annuity is given by

1−(1+𝑗𝑗)−𝑛𝑛 1−(1+0.0075)−15
P =R� �= 2500 � �= 35, 342.49
𝑗𝑗 𝑗𝑗

Therefore, the present value P with assumed payments starting the first month up
to the fifteenth month is ₱ 35, 342.49

Step 2 Find the present value of the payments made during the period of
deferral. Based on the problem, the payments will start at the end of
the fourth month thus, there will be 3 payments during the period of
deferral.

1−(1+𝑗𝑗)−𝑛𝑛 1−(1+0.0075) −3
P=R� �= 2500� �= 7,388.89
𝑗𝑗 𝑗𝑗

Therefore, the present value P during the period of deferral is ₱ 7,388.89

7
CO_Q2_General Mathematics SHS
Module 7
Since the payments in the period of deferral are artificial payments,
Step 3 subtract the present value of these payments from present value P
with assumed payments starting the first month up to the fifteenth,
thus

₱ 35,342.49 – ₱ 7,388.89 = ₱ 27,953.60

Thus, the present value of the cellular phone is ₱27,953.60

Recall the present value P of scenario 1 in the first activity which is equal to
₱ 28,587.28. Comparing this to the present value in, What’s New activity which is
equal to ₱ 27,953.60. It is lower than the present value in the first because the
payment in the second scheme will be received on a later date.

Notes to the Teacher


The two payment schemes have the same number of
payments n and the same interest rate per period j. Their
main difference is the start payments. The first scheme
started at the end of the first interval which makes it an
ordinary annuity. The second scheme started on a later
date. This annuity is called deferred annuity.

8
CO_Q2_General Mathematics SHS
Module 7
What is It

Deferred annuities are series of payments, as they have already learned in the past
lessons on annuities but, will start on a later date.

Some examples of this type of annuity in real life.


1. If you will buy an appliance, some big stores or appliances center offers
deferred payment.

2. A credit card company is offering its clients to purchase today but to


start paying monthly with their choice of the term after 3 months.
3. A real estate agent is urging a house and lot buyer to purchase now and
start paying after 3 years when the housing unit is ready for occupancy.
4. A worker who has gained extra income now and wants to save his money
so that he can withdraw his money monthly starting on the day of his
retirement from work.

Deferred Annuity is an annuity that does not begin until a given time interval has
passed. It is a kind of annuity which payments (or deposits) starts in more than one
period from the present. Likewise, the first payment interval does not coincide with
the first interest period and it is put off to some later date.

It may also be considered as an insurance contract designed for long-term savings.


Unlike an immediate annuity, which starts annual or monthly payments almost
immediately, investors can delay payments from a deferred annuity indefinitely.
During that time, any earnings in the account are tax-deferred.

Period of Deferral is the time between the purchase of an annuity and the start of
the payments for the deferred annuity.

9
CO_Q2_General Mathematics SHS
Module 7
Time Diagram for a Deferred Annuity
Deferred annuity of n payment

R* R*… R* R R …… R

0 1 2 … k k+1 k+2 k+n


1st payment starts on the (k+ 1) th period
NO payment for k periods

In this time diagram, the period of deferral is 𝑘𝑘 because the regular


payments of R start at time k + 1.
The notation R* represents k “artificial payments,’’ each equal to
R, but are not actually paid during the period of deferral.

To determine the present value of a deferred annuity, find the present value
of all k + n payments (including the artificial payments), then subtract the present
value of all artificial payments.

To get the Present value of a deferred annuity we will follow the formula:

1−(1+𝑗𝑗)−(𝑘𝑘+𝑛𝑛) 1−(1+𝑗𝑗)−𝑘𝑘
P=R -R
𝑗𝑗 𝑗𝑗

Where:
R = Present Payment
j = Interest rate per period
n = number of actual payments
k = number of conversation periods in the deferral or number of
artificial payments

Example 1

Find the present value of 10 semi-annual payments of ₱ 2,000.00 each if the


first payment is due at the end of 3 years and money is worth 8% compounded
semi-annually.

Solution:
Consider an 8-year timeline for the illustration.
To visualize and find d, we have
1st payment
(P1)
P2 P3 P4 P5 P6 P7 P8 P9 P10

1 2 3 4 5 6 7 8
k = 5 (see circles with x)

10
CO_Q2_General Mathematics SHS
Module 7
Given: R = ₱ 2,000.00 t=5
r = 8% m=2

Find: P

Number of artificial payments: k=5


Number of actual payments: n = mt = (2)(5) = 10
𝑖𝑖 2 .08
Interest rate per period j = = = .04
𝑚𝑚 2

Using this formula

1−(1+𝑗𝑗)−(𝑘𝑘+𝑛𝑛) 1−(1+𝑗𝑗)−𝑘𝑘
P=R -R
𝑗𝑗 𝑗𝑗
1−(1+𝑗𝑗)−(5+10) 1−(1+𝑗𝑗)−5
P=R -R
𝑗𝑗 𝑗𝑗
1−(1+𝑗𝑗)−(5+10) 1−(1+𝑗𝑗)−5
= 2000 - 2000 or
0.04 0.04
1− (1+0.04)−(5+10) 1− (1+0.04)−5
= 2000 � − �
0.04 0.04

= ₱ 13,333.13

Example 2

Find the present value of a deferred annuity of ₱ 1,500.00 every 3 months for 8
years that is deferred 3 years if money is worth 6% converted or compounded
quarterly

Solution:
Given: R = ₱ 1500.00 t = 8 years Find: P
r = 6%, m= 4

Number of artificial payments: k = mt = (4)(3) = 12


Number of actual payments: n = mt = (4)(8) = 32
𝑖𝑖 4 .06
Interest rate per period: j = = = 0.015
𝑚𝑚 4

If you assume that there are payments in the period of deferral, there would be a
total of k + n = 12 + 32 = 44 payments
1st Payment (P1)
12 32

1 2 3 4 5 6 7 8 9 10 11

1−(1+𝑗𝑗)−(𝑘𝑘+𝑛𝑛) 1−(1+𝑗𝑗)−𝑘𝑘
P=R -R
𝑗𝑗 𝑗𝑗
1− (1+0.015)−(12+ 32) 1− (1+0.015)−12
= 1500 � − �
0.015 0.015
= ₱ 31,699.68

11
CO_Q2_General Mathematics SHS
Module 7
Example 3:

Find the period of deferral in each of the following deferral annuity problem
(one way to find the period of deferral is to count the number of artificial
payment (k)).

a. Monthly payment of ₱12,000.00 for 9 years that will start 6 months from now.

Consider the time diagram

Skip payment

0 1 2 3 4 5 6 7 8 9

Therefore, there are 5 skip payments

Answer: 5 months or 5 periods

b. Semi-annual payments of ₱7,500.00 for 15 years that will start 4 years from
now.

Skip payment

6m 1y 1y,6m 2y 2y, 6m …………. 3y, 6m 4y

Answer: 7 periods or 7 semi-annual intervals

12
CO_Q2_General Mathematics SHS
Module 7
What’s More

Directions: Read each problem carefully and answer each question to solve the
problem. Have Fun!

Activity 1.1

Find the period of deferral in each of the following deferral annuity problems (one
way to find the period of deferral is to count the number of artificial payment (k).
Make a diagram.

1. Payment of ₱ 3,000.00 every 3 months for 8 years that will start 6 years
Time Diagram Answer

2. Payment of ₱ 1,000.00 every other month for 2 years that will start after
3 years.
Time Diagram Answer

3. Payment of ₱ 700.00 every month for one (1) year that will start at the
end of the third month
Time Diagram Answer

4. Payment of ₱ 400.00 every 5 months for 3 years that will start at the
end of 5 years.
Time Diagram Answer

13
CO_Q2_General Mathematics SHS
Module 7
5. Semi-annual payment of ₱ 12,000.00 for twelve (12) years that will start
after 5 years.

Time Diagram Answer

Activity 1.2
Directions: Read each problem carefully about calculating the present value and
period of deferral of a deferred annuity and perform the appropriate
solutions to answer the problem.

1. A group of college students decided to invest the money they earned from
the fund-raising project. After 6 months from today, they want to withdraw
from this fund ₱ 10,000.00 quarterly for 1 year to fund for community
service. How much is the present total deposit if the interest rate is 4%
converted quarterly?
2. A company offers a deferred payment option for the purchase of any
furniture. Gladys plans to buy a dining table set with a monthly payment
of ₱ 4,000.00 for 2 years. The payment will start at the end of 3 months.
How much is the cash price of the dining set if the company will give 10%
compounded monthly?

14
CO_Q2_General Mathematics SHS
Module 7
What I Have Learned

A. Directions: Fill in the blanks below with the correct term to make each
statement true about calculating the present value and period of deferral of a
deferred annuity.
1. A deferred annuity is an annuity whose payments starts in more than
period from the ________.
2. Each payment in an annuity is called the ________.
3. A deferred annuity is an annuity whose first payment takes place at some
predetermined time ______.
4. In retirement planning, payments on income taxes are deferred until you
______ the money.

Answer the following questions about calculating the present value and period
of deferral of a deferred annuity.

1. What are the benefits of a deferred annuity?

_________________________________________________________________________
_____________________________________________________________________

2. Is a deferred annuity a good investment?

_________________________________________________________________________
_____________________________________________________________________

3. What is the difference of grace period and deferment?

________________________________________________________________________
____________________________________________________________________

B. Explain the basic types of deferred annuities: fixed, indexed, variable, and
longevity.

15
CO_Q2_General Mathematics SHS
Module 7
What I Can Do

Directions: Create your own loan plan with complete computations using the
applications of Annuity on a short bond paper. Explain using five
sentence paragraph the importance of annuity in your daily life on a
separate sheet of yellow pad paper.

Rubrics

Category Excellent Very Satisfactory Needs


Satisfactory Improvement

Content 100% of the answers 80-99% of the 60-79% of the Below 60% of the
Accuracy (20) and computations are answers and answers and answers and
correct. (20) computations are computations are computations are
correct. (17) correct. (14) correct. (11)

Presentation of Output is Output is Output is Output is


Output exceptionally presentable in acceptably distractingly
(20) presentable in terms terms of presentable messy and not
of creativity and creativity and though it may be presentable. (11)
neatness. (20) neatness. (17) a bit messy. (14)

Organization of All sentences are very All sentences are Some ideas are Ideas are
Ideas (10) clear, and the ideas clear, and the not clear and disorganized and
are very organized. ideas are inorganized but not clear. (4)
(10) organized. overall, ideas are
(8) acceptable. (6)

16
CO_Q2_General Mathematics SHS
Module 7
Assessment

Directions: Read the following questions about calculating the present value and
period of deferral of a deferred annuity and choose the correct answer by
writing the letter on a separate sheet of paper.

1. What is the present value of 10 semi-annual payments of ₱ 2,000.00 if the first


payment is due at the end of 3 years and money is worth 8% compounded semi-
annually?

a. ₱ 10,330.31 c. ₱ 15,841.12

b. ₱ 13,333.13 d. ₱ 17,332.25

2. Which of the following statements DOES NOT refer to annuities?

a. Annuities do not use the pooling technique to spread risk


b. An owner may change the annuity date, the beneficiary, or the

settlement option
c. Once the payout period begins, the annuitant receives periodic

payments
d. The accumulation period is the period prior to the annuitization date

For items number 3 to 4, study the situation below.

A farmer decided to sell his land and to deposit the fund in a bank. After
computing the interest, he learned that he may withdraw ₱ 390,500.00 yearly for
10 years starting at the end of 6 years when it is time for him to retire. How
much is the fund deposited if the interest rate is 5% converted annually?

3. What is the number of artificial payments?

a. 4 c. 6

b. 5 d. 10

4. What is the present value of the annuity for withdrawal?

a. ₱ 2,005,197.75 c. ₱ 2,362,595.82

b. ₱ 2,344,592.24 d. ₱ 4,215,120.15

17
CO_Q2_General Mathematics SHS
Module 7
5. Gladys borrows ₱ 400,000.00 at an interest rate of 4% per year compounded semi-
annually. She agreed to settle her loan by making 12 semi-annual payments at
the end of each six months. If the first payment is made at the end of 2 years, how
much is the periodic payment?

a. ₱ 80,756.35 c. ₱ 40,138.96

b. ₱ 76,348.25 d. ₱ 49,165.39

6. What is the period of deferral if semi-annual payments of ₱5,000.00 for 13 years

that will start 4 years from now?

a. 7 c. 9
b. 8 d. 10

7. Melwin availed a loan from the bank that gave him an option to pay

₱ 20,000.00 monthly for 2 years. The first payment is due after 4 months. If
the interest rate is 10% converted monthly, what is the period of deferral?
a. 1 c. 3
b. 2 d. 12

8. How many numbers of payments are to be done in the situation in question


number 7?

a. 16 c. 27

b. 24 d. none

9. What is the present value in the situation in question number 7?

a. ₱ 422,759.78 c. ₱ 525,215.16

b. ₱ 433,655.13 d. ₱ 625,322.14

10. What is the present value of a deferred annuity of ₱1,500.00 every 3 months for
8 years that is deferred 3 years if money is worth 6% converted or compounded
quarterly?

a. ₱ 15,339.25 c. ₱ 31,699.67

b. ₱ 18,231.34 d. ₱ 43,825.32

18
CO_Q2_General Mathematics SHS
Module 7
11. What type of deferred annuity in which a return is based on the performance
of a portfolio of mutual funds, or sub-accounts, chosen by the annuity owner.

a. fixed annuity c. indexed annuity


b. variable annuity d. longevity annuity

12. A car is to be purchased in monthly payments of ₱ 17,000.00 for 4 years


starting at the end of 4 months. How much is the cash value of the car if the
interest rate used is 12% converted monthly?
a. ₱ 626,571.56 c. ₱ 816,000.00
b. ₱ 657,915. 02 d. ₱ 913,920.05

13. A group of employees decided to invest a portion of their 13th-month pay.


After 3 months from today, they want to withdraw from this fund ₱5,000.00
monthly for 12 months to fund their travel tour that they decide to do every
month. How much is the total deposit now if the interest rate is 5% converted
monthly?

a. ₱ 52,544.17 c. ₱ 68,245.12

b. ₱ 57,922.41 d. ₱ 88,201. 05

14. Meghan purchased a laptop for the online class of her kids through the credit
cooperative for their company. The cooperative provides an option for a
deferred payment. Meghan decided to pay after 4 months of purchase. Her
monthly payment is computed as ₱3,500.00 payable in 12 months. How
much is the cash value of the laptop if the interest rate is 8% convertible
monthly?
a. ₱ 45,360.00 c. ₱ 39,441.14
b. ₱ 42,000.00 d. ₱ 36,225.15

15. Payments of ₱ 7,000.00 every 2 years for 10 years starting at the end of 6
years. What is the period of deferral?

a. 1 c. 3

b. 2 d. 10

19
CO_Q2_General Mathematics SHS
Module 7
Additional Activities

Directions: Write a journal about which is better in terms of buying a large purchase
or property – “saving money on bank or getting a loan”?

Rubrics

Presentation of Output is Output is Output is Output is


Output exceptionally presentable in acceptably distractingly
(20) presentable in terms terms of presentable messy and not
of creativity and creativity and though it may be presentable. (11)
neatness. (20) neatness. (17) a bit messy. (14)

Organization of All sentences are very All sentences are Some ideas are Ideas are
Ideas (30) clear, and the ideas clear, and the not clear and disorganized and
are very organized. ideas are inorganized but not clear. (21)
(30) organized. overall, ideas are
(27) acceptable. (24)

20
CO_Q2_General Mathematics SHS
Module 7
Module 7
CO_Q2_General Mathematics SHS
21
What I Know What's More Assessment
1. B Activity 1.1 1. B
2. C 2. A
3. C 1. 24 periods or 24 3 months interval 3. B
4. B 2. 18 periods or 18 2 months interval 4. C
5. C 3. 2 periods or 2 months intervals 5. C
6. D 4. 11periods or 11 5 months intervals 6. A
7. B 7. C
5. 10 periods or 10 half-year intervals
8. B 8. B
9. B 9. A
10. A Activity 1.2 10. C
11. C 1 Php38,633.32 11. B
12. A 2. Php 85, 260.00 12. A
13. C 13. B
14. A 14. C
15. D What I have learned 15. B
A1. Present
2. Periodic payment
3. k+1
4. withdraw
Answer Key
2
References
Chan, Miro, Quiming. General Mathematics. Viba Group Inc. 2016. Pp. 83-86

Dimasuay Lynie, Alcala Jeric, Palacio Jane. Enhanced Teacher’s Manual General
Mathematics for Senior High School. C & E Publishing, Inc. 2016 pp. 101-105

General Mathematics Learner’s Material. First Edition. 2016. pp. 199 - 204

Oronce, Orlando.General Mathematics.Sampaloc Manila, Philippines. Rex


Bookstore, Inc. 2016. Pp. 224-230

*DepED Material: General Mathematics Learner’s Material

Teach Together, Deferred Annuity, K-12 Teacher’s Resource Community. January


12, 2017
https://www.google.com/search?ei=GhzmXoqABPaSr7wPrKWpmAY&q=deferred+a
nnuity+teach+together&oq=deferred+annuity+teach+together&gs_lcp=CgZwc3ktYW
IQAzoECAAQRzoECAAQQzoCCAA6BggAEBYQHjoFCAAQzQI6CAghEBYQHRAeOgUI
IRCgAToHCCEQChCgAVCUPljBdWCRe2gBcAF4AIABtQWIAd8lkgELMC4zLjUuNi4xL
jGYAQCgAQGqAQdnd3Mtd2l6&sclient=psy-
ab&ved=0ahUKEwiK9bKSq4HqAhV2yYsBHaxSCmMQ4dUDCAw&uact=5

22
CO_Q2_General Mathematics SHS
Module 7
For inquiries or feedback, please write or call:

Department of Education - Bureau of Learning Resources (DepEd-BLR)

Ground Floor, Bonifacio Bldg., DepEd Complex


Meralco Avenue, Pasig City, Philippines 1600

Telefax: (632) 8634-1072; 8634-1054; 8631-4985

Email Address: blr.lrqad@deped.gov.ph * blr.lrpd@deped.gov.ph


General Mathematics
Quarter 2 – Module 8:
Stocks and Bonds

CO_Q2_General Mathematics SHS


Module 8
General Mathematics – Senior High School
Alternative Delivery Mode
Quarter 2 – Module 8: Stocks and Bonds
First Edition, 2020

Republic Act 8293, section 176 states that: No copyright shall subsist in any work of
the Government of the Philippines. However, prior approval of the government agency or office
wherein the work is created shall be necessary for exploitation of such work for profit. Such
agency or office may, among other things, impose as a condition the payment of royalties.

Borrowed materials (i.e., songs, stories, poems, pictures, photos, brand names,
trademarks, etc.) included in this module are owned by their respective copyright holders.
Every effort has been exerted to locate and seek permission to use these materials from their
respective copyright owners. The publisher and authors do not represent nor claim ownership
over them.

Published by the Department of Education


Secretary: Leonor Magtolis Briones
Undersecretary: Diosdado M. San Antonio

Development Team of the Module


Writer: January B. Regio
Editors: Elizabeth D. Lalunio, Elizabeth B. Dizon, Anicia J. Villaruel, Roy O. Natividad
Reviewers: Fritz A. Caturay, Necitas F. Constante, Dexter M. Valle, Jerome A.
Chavez, Dennis E. Ibarrola, Mary Ann C. Guiang and Moahna Aura M.
Mancenido
Illustrators: Hanna Lorraine G. Luna, Dianne C. Jupiter
Layout Artists: Sayre M. Dialola, Roy O. Natividad, Noel Rey T. Estuita, Argie L. Ty
Management Team: Francis Cesar B. Bringas
Job S. Zape, Jr.
Ramonito Elumbaring
Reicon C. Condes
Elaine T. Balaogan
Fe M. Ong-ongowan
Hermogenes M. Panganiban
Phillip B. Gallendez
Josephine T. Natividad
Anicia J. Villaruel
Dexter M. Valle

Printed in the Philippines by ________________________

Department of Education – Region IV-A CALABARZON

Office Address: Gate 2 Karangalan Village, Barangay San Isidro


Cainta, Rizal 1800
Telefax: 02-8682-5773/8684-4914/8647-7487
E-mail Address: region4a@deped.gov.ph
General Mathematics
Quarter 2 – Module 8:
Stocks and Bonds
Introductory Message

This Self-Learning Module (SLM) is prepared so that you, our dear learners,
can continue your studies and learn while at home. Activities, questions, directions,
exercises, and discussions are carefully stated for you to understand each lesson.

Each SLM is composed of different parts. Each part shall guide you step-by-
step as you discover and understand the lesson prepared for you.

Pre-tests are provided to measure your prior knowledge on lessons in each


SLM. This will tell you if you need to proceed on completing this module or if you
need to ask your facilitator or your teacher’s assistance for better understanding of
the lesson. At the end of each module, you need to answer the post-test to self-check
your learning. Answer keys are provided for each activity and test. We trust that you
will be honest in using these.

In addition to the material in the main text, Notes to the Teacher are also
provided to our facilitators and parents for strategies and reminders on how they can
best help you on your home-based learning.

Please use this module with care. Do not put unnecessary marks on any part
of this SLM. Use a separate sheet of paper in answering the exercises and tests. And
read the instructions carefully before performing each task.

If you have any questions in using this SLM or any difficulty in answering the
tasks in this module, do not hesitate to consult your teacher or facilitator.

Thank you.

CO_Q2_General Mathematics SHS


Module 8
What I Need to Know

Have you ever heard of stocks and bonds? Have you ever experienced to
borrow money from your parent or friend and had to pay it back with interest?

This module was designed and written for learners like you to describe how
stocks and bonds work. This lesson will also prepare you to be financially
independent and succeed in the near future.

In this topic, learners are expected to demonstrate an understanding of the


basic concepts of stocks and bonds. Learners should also be able to use appropriate
financial instruments involving stocks and bonds in formulating conclusions and
making decisions. After going through this module, you are expected to:

1. illustrate stocks and bonds; and


2. distinguish between stocks and bonds.

What I Know

Choose the letter of the best answer. Write the chosen letter on a separate sheet of
paper.

1. It is the money that is earned in trade or business after paying the costs of
producing and selling goods and services.
a. stock c. bond
b. profit d. investment

2. What is the other name for a bond's interest rate?


a. par value c. coupon rare
b. face value d. principal

3. A person or an entity that gives money or allocates capital with the expectation
of receiving financial returns.
a. entrepreneur c. politician
b. stockbroker d. investor

1 CO_Q2_General Mathematics SHS


Module 8
4. It is used to describe the ownership certificates of any company and is also
known as shares and equity.
a. bond c. stock
b. stockbroker d. commission

5. A business formed to manufacture or supply products or services for profit.


a. stock c. stock
b. company d. bond

6. An activity in which money is put at risk for the purpose of making a profit.
a. Invest c. buy
b. sell d. collect

7. What is a bond’s coupon?


a. the purchase price of a bond
b. the interest earned to be paid by the bond issuer
c. the value of a bond at its issue date
d. the value of a bond at its maturity date

8. What is a bond?
a. a certificate of ownership in a corporation, with the right to a share of
the earnings
b. a payment from an investor to a corporation for the rights to future
profits
c. a group of stocks sold together for a set price
d. a certificate representing a loan from an investor to a corporation or
government entity

9. Why would someone buy a bond instead of a stock?


a. It is a less risky investment.
b. To have ownership in a company
c. It can yield a higher return on investment.
d. To receive dividend payments

10. What is a stock dividend?


a. a capital gains
b. the price the stock is sold for
c. part of the company’s profits that is paid to owners
d. the price paid when stock is sold to an investment bank

11. When people buy stock on a stock market. They are considered as _________.
a. the people selling the stock receives the money.
b. the corporation loses money.
c. the corporation receives the money.
d. the people buying the stock receives the money.

2 CO_Q2_General Mathematics SHS


Module 8
12. Which of the following best defines liquidity?
a. Investing in several different assets with unrelated risks.
b. The expenses of negotiating and executing an exchange.
c. The ability to convert a store of value to cash.
d. The amount that will be repaid at the end of a bond’s term.

13. One-way people can earn money from stocks is by


a. selling the stock for a lower price than the price they paid for the stock.
b. buying stock from an investment banker.
c. selling the stock for the same price as they paid for the stock.
d. selling the stock for a higher price than the price they paid for the stock.

14. If the interest rate on bonds increases, which of the following is the most likely
result?
a. The supply of money decreases.
b. Bond prices increase.
c. The transaction costs associated with holding money increases.
d. People are willing to hold less wealth in the form of money.

15. All of the following are reasons to buy bonds EXCEPT:


a. Bonds generally have outperformed the stock market over the last 100
years.
b. Bonds pay out interest at set intervals, allowing people to live off the
income.
c. Bonds may outperform the stock market during certain periods of time.
d. Investing in bonds may generate less tax liability than investing in
stocks.

3 CO_Q2_General Mathematics SHS


Module 8
Lesson

1 Stocks and Bonds

Let’s say that it's time to invest your money. So, how exactly are you going to
allocate that money? After all, a well-diversified portfolio strategy is recommended
before you start to buy assets such as stocks and bonds. Indeed, stocks and bonds
are two of the most traded types of assets—each available for sale on several different
platforms or through a variety of markets or brokers. Here, we go over the primary
differences between stocks and bonds.

What’s In

Definition of Terms. Complete each statement by choosing the correct answer


in the given word box. Write the corresponding letter of each number on the blanks
to unlock the bible scripture.

Words to Treasure!

The L _ _ D is my sh _ _ _e r _ , I l _ _ k _othin_.
1 2 3 4 5 6 7 8 9 10

Psalm 23:1

a. stock market c. dividend d. stock


e. debtor or borrower h. interest g. bond
n. stockbroker o. market value p. principal
r. investor

1. The current price of a stock at which it can be sold is called _______________.


2. A person or institution who invests the money or make the funds available is
called _________________.
3. A person or institution who owes the money or avails of the funds from the
lender is called ___________________.
4. The amount of money borrowed or invested on the origin date is called
___________________.

4 CO_Q2_General Mathematics SHS


Module 8
5. The amount paid or earned for the use of money is called ___________________.
6. A stake of ownership in a company that is sold off in exchange for cash is
called _______________________.
7. An equity market where regular activities of buying, selling, and issuance of
shares of publicly-held companies take place is referred to as ________________.
8. A sum of money paid by a company to its shareholders out of its profits is
called ____________________.
9. An investment adviser who executes buy and sell orders for stocks and other
securities on behalf of clients is called as _______________________.
10. A debt that the company or entity enters into with the investor that pays the
investor interest on that debt is referred as ________________________.

Notes to the Teacher


This module will help you to teach your students the importance of
stocks and bonds and how will you positively influence them on
investment decisions and to be financially literate. You can also
prepare additional activities, resources, videos to feed additional
inputs as you may feel appropriate that will deepen the learners
understanding.

5 CO_Q2_General Mathematics SHS


Module 8
What’s New

Read carefully the lyrics of the song, you may sing the song in the tune of
“Row, row, row your boat” to emphasize how people can save with stocks and bonds.

Stocks and Bonds

Stocks, stocks, stocks are shares

That pays a dividend.

Gradually, gradually, gradually, gradually

Investing then you earn.

Stocks, stocks, stocks, they change

The price goes low or high.

Gradually, gradually, gradually, gradually

Money goes round and round.

Bonds, bonds, bonds are loans

That pay an interest

Gradually, gradually, gradually, gradually

Growing while you wait.

Questions to Ponder

1. Based on the given song, can you define stocks? How about bonds?
2. Distinguish the difference between stocks and bonds?
3. Between stocks and bonds, which do you prefer to invest your money? Why?

6 CO_Q2_General Mathematics SHS


Module 8
What is It

Stocks and bonds are the heartbeat of the economy. Much of the world's
business activity would be impossible without stocks and bonds. But whether you
trade on the Philippine Stocks Exchange, financial terms can always be confusing.
So, before you invest in a stock or a bond, you need to know - what is the difference?
And which one should you choose?

Stocks and bonds are certificates that are sold to raise money for starting a
new company or for expanding an existing company. They are also called securities,
and people who buy them are called investors.

STOCKS

Companies sell shares of ownership in their company to raise money to


finance operations, plan expansion, and so on. These ownership shares are called
stocks. The buyers of the stock (stockholders) receive stock certificates verifying
the number of shares of stocks they own. The two basic types of stocks are common
stock and preferred stock.

Common stockholders have voting rights. Preferred stockholders do not have


voting rights, but they receive preference over common stockholders in dividends
(payments from profit) and the company’s assets if the company goes bankrupt.
Stock Market provides an orderly trading place for stock wherein prices or market
value vary from day to day and within a day. Only stockbrokers who specialize to
work in the stock market can trade on the floor. The broker receives a commission
for the services of both buying and selling stocks.

BONDS

Sometimes companies raise money by selling bonds instead of stock. When


you buy a stock, you become a part-owner in the company. To raise money,
companies may not want to sell more stocks and thus dilute the ownership of their
current stock owners, so they sell bonds. A bond is a form of long-term investment
issued by a corporation or government where the purchaser becomes a creditor of
the company. It represents a promise from the company to pay the face amount to
the bond owner at a future date, along with interest payments at a stated rate. The
company, state or municipality that issues the bond is called the issuer.

The annual interest paid by the issuer to the lender (bond holder) on the bond
is referred to as the coupon. The coupon rate is the annual payout as percentage
of the bond’s par value. Bonds have two kinds of values. These are par value and

7 CO_Q2_General Mathematics SHS


Module 8
market value. The par value of the bond is the same as its face value while the
market value of a bond is the price at which the bond is being sold. It may be greater
than or less than the amount of the par value. If the market value is greater than
the par value, then the bond is selling at a premium. If the market value is less than
the par value, then the bond is selling at a discount.

Now I know!

Investors are always told to diversify their portfolios between stocks and
bonds, but what’s the difference between the two types of investments? Here's a look
at the difference between stocks and bonds on the most fundamental level.

Basis for
Stocks Bonds
Comparison
A form of equity instrument or A form of debt instrument or
raising money by allowing raising of money by borrowing
Definition
investors to be part owners of from investors.
the company.
Government Institutions,
Issuers Corporates Financial Institutions,
Companies etc.
Status of Shareholders are the owners of Bondholders are the lenders to
Holders the company. the company.
Profits earned by the company Interest payments are made in
Form of
are paid in the form of the form of Coupon Payments.
Returns
Dividends.
The risk level is high since it The risk level is relatively low
depends upon the performance since bondholders are prioritized
Risk Level
of the issuer, so no guaranteed for repayments.
returns.
Major Risks Market Risk, Business Risk Interest Rate Risk, Inflation Risk
Associated
Shareholders get the right to Bondholders get the preference
Additional
vote. in terms of repayment and on
Benefit
liquidation.
When interest rates fall When market interest rates
Market
significantly, the market stock decrease, the market value of an
Value
value rises. existing bond increases.

8 CO_Q2_General Mathematics SHS


Module 8
What’s More

Group the following characteristics of stocks and bonds and write on the space
provided.

 offers fixed interest rate


 makes profit by dividend
 debt instrument
 equity instrument
 own a small piece of the company
 sold by Government and financial institution
 lower risk with lower reward
 higher risk but with higher reward
 market value varies everyday
 has a maturity date

Stocks Bonds

What I Have Learned

Complete the following statements by writing the correct word or words.

1. When a company goes to sell a _______________________, they decide to


sell a certain amount of shares of ownership in their company that they
will give up in exchange for cash from investors.

2. A stock is a security in that company that can also be referred to as


equity or a ________________________.

3. Those who own ___________________ stock in a company typically have


voting rights in shareholder's meetings and may even receive dividends,
while ______________________ stock owners do receive dividends but
don't always receive voting rights.

9 CO_Q2_General Mathematics SHS


Module 8
4. _____________________ are fixed-income investments, which operate
from a fixed interest rate and a fixed amount of time wherein the
company, government, or other will repay the money plus the interest.

5. A ________________________ represents a collection of shares in a


company which is entitled to receive a fixed amount of dividend at the
end of relevant financial year, whereas ________________________ is
associated with debt raised by the company from outsiders which carry
a fixed ratio of return each year and can be earned as they are generally
for a fixed period of time.

What I Can Do

Perform the task below.


Suppose you have savings in the bank that you want to invest in stocks and
bonds instead of setting up in a new business. Write one to two paragraphs
discussing what method you can use to make the investment and explain the reasons
for your decision.

Content Area Rubric


 Use of the skills of evaluation, analysis, and synthesis is
apparent.
3  Sound reasoning is employed.
 Appropriate and accurate specific examples are cited and
Explained.
 Use of the skills of synthesis and analysis is apparent.
 Reasoning employed is on the inferential level.
2
 Appropriate examples are cited and explained, however, some
inaccurate information is included.
 Use of literal skills is apparent.
 Reasoning employed is on the concrete level.
1
 Some examples may be cited, may attempt to be explained,
and inaccurate information is included.
 There is little or no evidence of any apparent skills.
0  There is little or no evidence of any reasoning employed.
 Examples, if cited, are inaccurate or inappropriate.

10 CO_Q2_General Mathematics SHS


Module 8
Assessment

Choose the letter of the best answer. Write the chosen letter on a separate sheet of
paper.
1. What is another term for stock?
a. bond c. debt instrument
b. debenture d. equity instrument
2. It is a type of stock for which stockholders get first choice in distributed profits.
a. common stock c. face value stock
b. stock market d. preferred stock
3. Another term for a bond’s face value.
a. par value c. maturity
b. coupon d. final payment
4. A bond that pays all of its interest and principal at the bond’s maturity date.
a. bond fund c. coupon bond
b. par-value fund d. zero-coupon bond
5. Which financial assets carries the most risk?
a. bond c. savings deposits
b. stock d. checking deposits
6. Which is the LEAST risky investment?
a. stocks c. Philippines treasury bonds
b. corporate bond d. mutual funds
7. A person or agent who trades for you and charges a fee or commission for
executing buys and sells of stocks through a stock exchange.
a. stockbroker c. entrepreneur
b. stockholder d. politician
8. What is stock portfolio?
a. The online tool used to track stock prices.
b. A list of all the stocks you own.
c. The document that you receive for purchasing stock.
d. A group of stocks that you can purchase at one time on a stock
exchange.
9. Why do people buy stocks?
a. There is no chance of a loss.
b. They expect to earn a return.
c. The government encourages them to buy stock.
d. They are guaranteed interest payment each year.
10. What is the best explanation of a bond?
a. It is an ownership interest in a company.
b. It is an equity or share in a company.
c. It represents a corporate or government debt obligation.
d. It is a debt instrument.

11 CO_Q2_General Mathematics SHS


Module 8
11. Diversifying can occur by
a. buying different stocks and bonds in different industries.
b. buying similar stocks and bonds in the same industries.
c. buying similar stocks and bonds in different industries.
d. buying different stocks and bonds in the same industries.
12. Which best defines the risk of a financial asset?
a. Investing in several different assets with unrelated risks.
b. The amount that will be repaid at the end of a bond’s term.
c. The probability that an asset will lose value.
d. The uncertainty that an asset might gain or lose value.
13. Why would someone choose to put money in stocks as opposed to a savings
account that earns interest?
a. They are guaranteed a return in stocks.
b. There is a potential to earn more money in the stock market.
c. They are guaranteed a return in a savings account.
d. There is a potential to earn more money in the savings account.
14. What happens to the price and interest rate of a bond if the demand for that
bond increases?
a. Price increases; interest rate is unaffected.
b. Price is unaffected; interest rate is unaffected.
c. Price increases; interest rate decreases.
d. Price decreases; interest rate decreases.
15. Several years ago, Company A issued bonds to raise funds so that it could buy
equipment. Those bonds were purchased by the Bank of the East. However,
the Bank of the East has decided that it doesn’t want to have any assets in
the form of bonds, so it is selling off all the bonds that it owns. Which of the
following is most likely to be the result of this action?
a. The default risk of the bonds will increase.
b. Bond prices will increase.
c. The face value of the bonds will decrease.
d. Interest rates will increase.

Additional Activities

Journal Writing
Write a journal on the saying “Do not put all your eggs in one basket” by
relating the lesson on stocks and bonds.

12 CO_Q2_General Mathematics SHS


Module 8
Module 8
CO_Q2_General Mathematics SHS 13
Assessment What I Have
1. d
Learned
2. d
1. stock
3. a
4. c
2. share
5. b 3. common,
6. c preferred
7. a 4. bond
8. b 5. stock, bond
9. b
10.c
11.a
12.d
13.b
14.c
15.c
What’s More What's In What I Know
Stocks: 1. o 1. b
2. r 2. c
 makes profit by dividend 3. e 3. d
 equity instrument 4. p 4. c
 own a small piece of the 5. h 5. b
company 6. d 6. a
 higher risk but with higher 7. a 7. b
reward 8. c 8. d
 market value varies 9. n 9. a
everyday 10.g 10.c
11.a
Bonds: 12.c
13.d
 offers fixed interest rate 14.d
 debt instrument 15.a
 sold by Government and
financial institution
 lower risk with lower
reward
 It has a maturity date
Answer Key
References

Aunzo, Rodulfo, Flores Maricar, Gagani Ray Ferdinand M, and Quennie Ypanto.
2016. General Mathematics Activity-based, Scaffolding of Student . Quezon
City: C&E Publishing, Inc.

2016. General Mathematics Learner’s Material . Meralco Avenue, Pasig City,


Philippines 1600: Lexicon Press Inc.

2016. General Mathematics Teacher’s Guide. Meralco Avenue, Pasig City,


Philippines 1600: Lexicon Press Inc.

Oronce, Orlando A. 2016. General Mathematics. Sampaloc, Manila: Rex Bookstore,


Inc.

14 CO_Q2_General Mathematics SHS


Module 8
For inquiries or feedback, please write or call:

Department of Education - Bureau of Learning Resources (DepEd-BLR)

Ground Floor, Bonifacio Bldg., DepEd Complex


Meralco Avenue, Pasig City, Philippines 1600

Telefax: (632) 8634-1072; 8634-1054; 8631-4985

Email Address: blr.lrqad@deped.gov.ph * blr.lrpd@deped.gov.ph


General Mathematics
Quarter 2 – Module 9:
Different Markets
for Stocks and Bonds

CO_Q2_General Mathematics SHS


Module 9
General Mathematics – Senior High School
Alternative Delivery Mode
Quarter 2 – Module 9: Stocks and Bonds
First Edition, 2020

Republic Act 8293, section 176 states that: No copyright shall subsist in any work of
the Government of the Philippines. However, prior approval of the government agency or office
wherein the work is created shall be necessary for exploitation of such work for profit. Such
agency or office may, among other things, impose as a condition the payment of royalties.

Borrowed materials (i.e., songs, stories, poems, pictures, photos, brand names,
trademarks, etc.) included in this module are owned by their respective copyright holders.
Every effort has been exerted to locate and seek permission to use these materials from their
respective copyright owners. The publisher and authors do not represent nor claim ownership
over them.

Published by the Department of Education


Secretary: Leonor Magtolis Briones
Undersecretary: Diosdado M. San Antonio

Development Team of the Module


Writers: Arvin A. Asnan
Editors: Elizabeth D. Lalunio, Elizabeth B. Dizon, Anicia J. Villaruel, Roy O. Natividad
Reviewers: Fritz A. Caturay, Necitas F. Constante, Celestina M. Alba, Jerome
A. Chavez, Ann Michelle M. Jolo, Charito N. Laguador and Moahna Aura M.
Mancenido
Illustrator: Hanna Lorraine Luna, Diane C. Jupiter and Michael A. Alonzo
Layout Artist: Roy O. Natividad, Sayre M. Dialola, Noel Rey T. Estuita, Argie L. Ty
Management Team: Francis Cesar B. Bringas
Job S. Zape, Jr.
Ramonito Elumbaring
Reicon C. Condes
Elaine T. Balaogan
Fe M. Ong-ongowan
Hermogenes M. Panganiban
Phillip B. Gallendez
Josephine T. Natividad
Anicia J. Villaruel
Dexter M. Valle

Printed in the Philippines by ________________________

Department of Education – Region IV-A CALABARZON

Office Address: Gate 2 Karangalan Village, Barangay San Isidro


Cainta, Rizal 1800
Telefax: 02-8682-5773/8684-4914/8647-7487
E-mail Address: region4a@deped.gov.ph
General Mathematics
Quarter 2 – Module 9:
Different Markets
for Stocks and Bonds
Introductory Message

This Self-Learning Module (SLM) is prepared so that you, our dear learners,
can continue your studies and learn while at home. Activities, questions, directions,
exercises, and discussions are carefully stated for you to understand each lesson.

Each SLM is composed of different parts. Each part shall guide you step-by-
step as you discover and understand the lesson prepared for you.

Pre-tests are provided to measure your prior knowledge on lessons in each


SLM. This will tell you if you need to proceed on completing this module or if you
need to ask your facilitator or your teacher’s assistance for better understanding of
the lesson. At the end of each module, you need to answer the post-test to self-check
your learning. Answer keys are provided for each activity and test. We trust that you
will be honest in using these.

In addition to the material in the main text, Notes to the Teacher are also
provided to our facilitators and parents for strategies and reminders on how they can
best help you on your home-based learning.

Please use this module with care. Do not put unnecessary marks on any part
of this SLM. Use a separate sheet of paper in answering the exercises and tests. And
read the instructions carefully before performing each task.

If you have any questions in using this SLM or any difficulty in answering the
tasks in this module, do not hesitate to consult your teacher or facilitator.

Thank you.

CO_Q2_General Mathematics SHS


What I Need to Know

Have you ever heard stocks and bonds? Have you ever experienced to borrow
money from your parent or friend and had to pay it back with interest?

This module was designed and written for learners like you to describe how
stocks and bonds work. This lesson will also prepare you to be financially
independent and succeed in the near future.

In this topic, learners are expected to demonstrate understanding of basic


concepts of stocks and bonds. Learners should also be able to use appropriate
financial instruments involving stocks and bonds in formulating conclusions and
making decisions. After going through this module, you are expected to:
1. solve problems involving stock valuation,
2. solve for the face value and redemption value of bonds, and
3. solve for the bond premium and discount on bonds.

1
CO_Q2_General Mathematics SHS
Module 9
What I Know

Choose the letter of the best answer. Write the chosen letter on a separate sheet of
paper.

1. The interest rate the bond issuer will use in computing the interest payment
which is usually expressed in percent.
a. Coupon rate
b. Dividend
c. Dividend Rate
d. Maturity Date

2. The ratio of the annual dividend per share and the market value per share.
a. Coupon Amount
b. Dividend Rate
c. Market Value
d. Stock yield ratio

3. The periodic interest payment that the bondholder receives during the time
between the purchase date and maturity date.
a. Coupon Amount
b. Coupon Rate
c. Face Value
d. Market Value

4. The rate per coupon payment period; denoted by r.


a. Coupon Amount
b. Coupon Rate
c. Face Value
d. Market Value

5. A certain financial institution declared a ₱ 30,000,000.00 dividend for the


common stocks. If there is a total of 700,000.00 shares of common stocks,
how much is the dividend per share?
a. ₱ 42.86
b. ₱ 52.86
c. ₱ 62.86
d. ₱ 72.86

2
CO_Q2_General Mathematics SHS
Module 9
6. A certain corporation declared a 3% dividend on a stock with a par value of
₱500.00 Mrs. Lingan owns 200 shares of stocks with a par value of ₱ 500.00
How much is the dividend she received?
a. ₱ 2000.00
b. ₱ 2500.00
c. ₱ 3000.00
d. ₱ 3500.00

For numbers 7,8 and 9.


Corporation A, with a current market value of ₱ 52.00, give a dividend of ₱8
per share of its common stock. Corporation B, with a current market value of ₱ 95.00,
give a dividend of ₱ 12.00 per share.
7. Find the stock yield ratio for Corporation A.
a. 12.38%
b. 13.38%
c. 14.38%
d. 15.38%

8. What is the stock yield ratio for Corporation B?


a. 11.63%
b. 12.63%
c. 13.63%
d. 14.63%

9. In which corporation will you invest your money? Why?


a. Corporation A, because each peso will earn you more if you invest in A
than in B.
b. Corporation A, the lower the share the higher the stock yield ratio.
c. Corporation B, because each peso will earn you more if you invest in B
than in A.
d. Corporation B, the lower the stock yield ratio the more you earn.

10. Determine the amount of the semi-annual coupon for a bond with a face value
of ₱ 300,000.00 that pays 10% payable semi-annual for its coupon.
a. ₱ 12,500.00
b. ₱ 15,000.00
c. ₱ 17,500.00
d. ₱ 20,000.00

For numbers 11, 12 and 13.


Suppose that a bond has a face value of ₱ 100,000.00 and its maturity date is
10 years from now. The coupon rate is 5% payable semi-annually. Assuming that
the annual market rate is 4%.

3
CO_Q2_General Mathematics SHS
Module 9
11. Find the amount of the semi-annual coupon.
a. ₱ 2500.00
b. ₱ 2750.00
c. ₱ 3000.00
d. ₱ 3250.00

12. Find the present value of ₱ 100,000.00


a. ₱ 67,556.42
b. ₱ 62,452.81
c. ₱ 57,332.21
d. ₱ 52,112.54

13. Find the fair price of the given bond.


a. ₱ 102,445.12
b. ₱ 104,182.56
c. ₱ 106,912.89
d. ₱108, 512.43

14. A bank declared a dividend of ₱ 27.00 per share for the common stock. If the
common stock closes at ₱ 93.00, how large is the stock yield ratio on this
investment between purchase date and the maturity date?
a. 0.26
b. 0.29
c. 0.32
d. 0.35

15. Determine the amount of semi-annual coupon paid for a 3% bond with a face
value of ₱ 100,000.00 which matures after 8 years. How many coupons are
paid?
a. 10 times
b. 13 times
c. 16 times
d. 19 times

4
CO_Q2_General Mathematics SHS
Module 9
Lesson
Different Markets for Stocks
1 and Bonds

Many of us deal with stocks and bonds (or want to!) but have no idea what are
their worth. Maybe you want to try investing. You may want a better understanding
of your investments. Or, if you work for a big corporation, your employer may sell
shares, and you need to determine whether it’s worth the price. Either way, it is good
to know the difference between stocks and bonds and have a basic understanding of
how they are valued.

What’s In

It’s Puzzle Time!

Good day learners. Before we start our new learning experience today, let us have
an activity. There are 10 words from the puzzle hidden vertically, horizontally and
diagonally. Write the words below using the clue then give a meaningful definition
for each word.

A T O W G L V O T O L E I M E Q
I F I S S U E R X G I H K A U D
D F A E S T B D B W I O G R A O
I G I S O U O R K R E A H K I N
S M C T A Z N C A S C J R E C I
C V L O I H F I K D A U P T X R
O I H C M L K M I B P N Q V E M
U E Q K S M B V L R R Y E A U L
N H K E A W I F J F L O U L L D
T S E O A D O S V Z O T K U A O
Y C H Y E P R I I W P O C E V A
O B C N I K D C B O S I H O R R
Q A D U R D E O B O N D A M A A
E C O U P O N R A T E U E J P N
U Z J A O G W T A X I V F O L E

5
CO_Q2_General Mathematics SHS
Module 9
1. S = ______________________________________________________.

2. B = ______________________________________________________.

3. C = ______________________________________________________.

4. M = ______________________________________________________.

5. P = ______________________________________________________.

6. I = ______________________________________________________.

7. C = ______________________________________________________.

8. S = ______________________________________________________.

9. D = ______________________________________________________.

10. D = ______________________________________________________.

From the previous activity, these terms have something to do with the new
lesson. These will help you understand the different markets for stocks and bonds
and it will be easier for you to use and apply these terms on the computations
regarding our next topic.

Notes to the Teacher


This module will help you to teach your students the
importance of stocks and bonds and how will you
positively influence them on investment decisions and to
be financially literate. You can also prepare additional
activities, resources, videos to feed additional inputs as
you may feel appropriate that will deepen the learners
understanding of the lesson.

6
CO_Q2_General Mathematics SHS
Module 9
What’s New

This is an activity that will test your decision prowess. Given 2 scenarios
below, answer questions below.

Scenario 1.

Your grandparents gave you ₱ 175,000.00 on your 16th birthday. You were instructed
to invest the money so that the earnings can be used to pay for your tuition fee in
college. Having heard about the risks and rewards of the stock market from your
parents, you become interested in buying stocks in a particular company. Below are
the options given to you by your parents:

Option1: Company ABC’s selling stock is ₱ 1,500.00 per share that will have
a dividend of ₱ 200.00 per year. The stock can be sold after two years
at ₱ 2,000.00 and the market requires a rate of return of 15%.

Option2: Company XYZ’s selling stock is ₱ 1,000.00 per share that will have
a dividend of ₱ 180.00 per year. The stock can be sold after two years
at ₱ 2,000.00 and the market requires a rate of return of 7%.

In which company will you invest your money? Why?

__________________________________________________________________________________
__________________________________________________________________________________
__________________________________________________________________________________

Scenario 2.

Suppose that you withdraw ₱ 1,500 from your savings account. Over the
weekend, several people want to borrow money from you. Read the stories and decide
to whom you will lend your money.

Story1. Your younger sister is having a garage sale. She needs cash to make
change for the day. She will sell you a bond for ₱ 1,000.00. You will
keep the bond for a week, and she promised to pay you back.

Story2. Your older brother has a small business idea but doesn’t have any
money. He wants to borrow your ₱ 1,500.00 and promised to pay you
back ₱ 1,800.00 in four weeks.

Story3. Your best friend at school, whom you know very-well, wants to borrow
your ₱ 1,500.00 to buy cupcakes. She plans to sell at a higher price
and promised to pay you back ₱ 1,700.00 in two weeks.

Whose bonds will you buy? Why?

__________________________________________________________________________________
__________________________________________________________________________________
__________________________________________________________________________________

7
CO_Q2_General Mathematics SHS
Module 9
What is It

Before we proceed to our next activity, let us recall some different terms about stocks.

Definition of Terms in Relation to stocks


Stocks – share in the ownership of a company

Dividend – share in the company’s profit

Dividend Per Share – ratio of the dividends to the number of shares


Stock Market – a place where stocks can be bought or sold. The stock
market in the Philippines is governed by the Philippine Stock Exchange
(PSE).
Market Value – the current price of a stock in which it can be sold

Stock Yield Ratio – ratio of the annual dividend per share and the
market value per share. Also called current stock yield.
Par Value – the per share amount as stated on the company certificate.
Unlike market value, it is determined by the company and remains stable
over time.

Stock valuation is considered to have high value if the stockholder gets higher
dividend at present and in the future, otherwise the stock is considered to have a low
value. The computation of the stock value is necessary in order to know if the stock
market value is cheap or expensive at a given time.

Example 1:

A certain financial institution declared a ₱ 30,000,000 dividend for the common


stocks. If there is a total of 700,000 shares of common stocks, how much is the
dividend per share?

Given: Total Dividend = ₱ 30,000,000.00

Total Shares = 700,000

Find: Dividend per share

8
CO_Q2_General Mathematics SHS
Module 9
Solution:

Total Dividend
Dividend per share =
Total Shares

30,000,000
=
700,000

= 42. 86

Therefore, the dividend per share is ₱ 42.86

Example 2:

A certain corporation declared a 3% dividend on a stock with a par value of ₱ 500.00


Mrs. Lingan owns 200 shares of stocks with a par value of ₱ 500.00 How much is
the dividend she received?
Given: Dividend percentage = 3%
Par Value = ₱ 500.00
Number of Shares = 200
Find: Dividend
Solution:
The dividend per share is: ₱ 500 x 0.03 = ₱15
Since there are 300 shares, the total dividend is: ₱15/share x 200 shares =
₱ 3,000.00
In summary,
Dividend = (Dividend Percentage) x (Par Value) x (No. of Shares)
= (0.03)(500)(200)
= 3,000
Thus, the dividend is ₱ 3,000.00

Example 3:
Corporation A, with a current market value of ₱ 52.00, give a dividend of ₱ 8.00 per
share of its common stock. Corporation B, with a current market value of ₱ 95.00,
give a dividend of ₱ 12.00 per share. Use the stock yield ratio to measure how much
dividends shareholders are getting in relation to the amount invested.

Solution.
Given: Corporation A:
Dividend per share = ₱ 8
Market Value = ₱ 52
Find: Stock yield ratio

9
CO_Q2_General Mathematics SHS
Module 9
dividend per share
Stock yield ratio =
market value
𝟖𝟖
=
𝟓𝟓𝟓𝟓

= 0.1538 = 15.38%
Given: Corporation B
Dividend per share = ₱ 12.00
Market Value = ₱ 95.00
Find: Stock yield ratio

dividend per share


Stock yield ratio =
market value
12
=
95

= 0.1263 = 12.63%
Corporation A has a higher stock-yield-ratio than Corporation B. thus, each
peso will earn you more if you invest in corporation A than in corporation B. If all
other things are equal, then it is wiser to invest in Corporation A.
As example 3 shows, the stock yield ratio can be used to compare two or more
investments.
Definition of Terms in Relation to bonds.

Bond – interest-bearing security which promises to pay (1) stated amount


of money on the maturity date, and (2) regular interest payments called
coupons.
Coupon – periodic interest payment that the bondholder receives during
the time between purchase date and maturity date; usually receive semi-
annually.

Coupon Rate – the rate per coupon payment period; denoted by r.

Price of a bond – the price of the bond at purchase time; denoted by P.


Par Value or Face Value – the amount payable on the maturity date;
denoted by F.

If P = F, the bond is purchase at par.


If P < F, the bond is purchased at a discount.
If P > F, the bond is purchased at premium.

Term of a Bond – fixed period of time (in years) at which the bond is
redeemable as stated in the bond certificate; number of years from the
time of purchased to maturity date.
Fair Price of a Bond – present value of all cash inflows to the bondholder.

10
CO_Q2_General Mathematics SHS
Module 9
Example 4: Determine the amount of the semi-annual coupon for a bond with a
face value of ₱300,000.00 that pays 10% payable semi-annual for its coupon.

Given: Face Value F = 300,000

Coupon Rate = 10%

Find: Amount of the semi-annually coupon

Solution:

Annual Coupon amount: 300,000(0.10) = 30,000

1
Semi-Annually Coupon Amount: 30,000 � � = 15,000
2

Thus, the amount of semi-annual coupon is ₱ 15,000.00

Note: The coupon rate is used only for computing the coupon amount,
usually paid semi-annually. It is not the rate at which money grows. Instead
current market conditions are reflected by the market rate, and it will be
used to compute the present value of future payments.

Example 5:

Find the value of a ₱ 20,000.00 bond with interest of 14% compounded semi-
annually which is redeemable at 108% in 5 years that yields the purchaser 6%
converted semi-annually.

Solution:

Finding the purchase price of the bond requires us to determine first the
coupon payment and the redemption value of the bond. Using the information given
in the problem, substitute the values to the respective formulas. Thus, we have

Fv = Face Value = ₱ 20,000

𝑟𝑟𝑏𝑏 = bond rate = 0.14

m = number of conversion period per year = 2

t = term = 5 years

11
CO_Q2_General Mathematics SHS
Module 9
𝑟𝑟𝑦𝑦 = yield rate = 0.06

0.14
b = periodic bond rate = = 0.07
2

n = total number of conversion period = 5(2) = 10

0.06
i = periodic rate = = 0.03
2

Substitute these values to compute the following:

Coupon payment: c p = (20,000)(0.07) = ₱ 1,400

Redemption Value: Rv = (20,000)(1.08) = ₱ 21,600

Purchase Price:

−n 1 − (1 + i ) − n 
p = Rv (1 + i ) + c p  
 i 

1 − (1 + 0.03) −10 
p = (21,600)(1 + 0.03) −10 + 1,400  = ₱ 28,014.71
 0.03 

Therefore, the investor who pays ₱28,014.71 is investing the money.

12
CO_Q2_General Mathematics SHS
Module 9
What’s More

Activity 1.1. Complete the table below by supplying the correct dividend, shares
and dividend per share.

Dividend per
Dividend Shares
share
1 ₱ 10,000.00 750
2 1,000 ₱ 15.25
3 ₱ 2,000,000.00 ₱ 200.00

Activity 1.2. Complete the table below by supplying the correct dividend, shares
and dividend per share.

Dividend
Par Value Shares Dividend
Percentage
1 5% ₱ 700.00 200
2 ₱ 500.00 50 ₱12,000.00
3 3% 100 ₱ 3000.00
4 7.5% ₱ 350.00 ₱ 5000.00

Activity 1.3. Solve the following problems.

1. A financial institution declares a dividend of ₱ 75,000,000.00 for its common


stock. Suppose there are 900,000 shares of common stock, how much is the
dividend per share?

2. ABC Corporation gives ₱ 38.00 dividend per share for its common stock. The
market value of stock is ₱ 10800. Determine the stock yield ratio.

3. A bank declares a dividend of ₱27.00 per share for the common stock. If the
common stock closes at ₱ 93.00, how large is the stock yield ratio on this
investment?

4. Find the amount of the semi-annual coupon for a ₱ 200,000.00 bond which pays
5% convertible semi-annually coupons.

5. Determine the amount of semi-annual coupon paid for a 3% bond with a face
value of ₱ 100,000.00 which matures after 8 years. How many coupons are paid?

13
CO_Q2_General Mathematics SHS
Module 9
What I Have Learned

Complete the following statements by writing the correct word or phrase.

1. _____________________ is the share in the company’s profit.

2. The ratio of the dividends to the number of shares is the _______________________.


3. _____________________ is the per share amount as stated on the company
certificate. Unlike market value, it is determined by the company and remains
stable over time.
4. _____________________ is ratio of the annual dividend per share and the market
value per share. Also called current stock yield.

5. The rate per coupon payment period; denoted by r is ______________________.


6. The present value of all cash inflows to the bondholder is ____________________.

7. The price of the bond at purchase time; denoted by P is _____________________.

8. A certain financial institution declared a ₱ 10,000,000.00 dividend for the common


stocks. If there is a total of 300,000 shares of common stocks, then the dividend
per share is ______________.

9. Corporation Gen. Math, with a current market value of ₱ 95.00, give a dividend of
₱ 15.00 per share of its common stock. The stock yield ratio is _______________.

10. The amount of the semi-annual coupon for a bond with a face value of ₱
100,000 .00 that pays 15% payable semi-annual for its coupon is
________________

14
CO_Q2_General Mathematics SHS
Module 9
Reflection. Going back to our first problem, it is time to review our answers. You
can now decide correctly after this learning experiences.

Scenario 1.
Your grandparents gave you ₱ 175,000.00 on your 16th birthday. You were
instructed to invest the money so that the earnings can be used to pay for your
tuition fee in college. Having heard about the risks and rewards of the stock
market from your parents, you become interested in buying stocks in a
particular company. Below are the options given to you by your parents:

Option1: Company ABC’s selling stock is ₱ 1,500.00 per share that will have a
dividend of ₱ 200.00 per year. The stock can be sold after two years at ₱2,000.00
and the market requires a rate of return of 15%.

Option2: Company XYZ’s selling stock is ₱ 1,000.00 per share that will have a
dividend of ₱ 180.00 per year. The stock can be sold after two years at ₱2,000.00
and the market requires a rate of return of 7%.

In which company will you invest your money? Why?


_____________________________________________________________________________
_____________________________________________________________________________
_____________________________________________________________________________
_____________________________________________________________________________
_____________________________________________________________________________

Scenario 2.

Suppose that you withdraw ₱ 1,500.00 from your savings account. Over the
weekend, several people want to borrow money from you. Read the stories and
decide to whom you will lend your money.

Story1. Your younger sister is having a garage sale. She needs cash to make
change for the day. She will sell you a bond for ₱ 1,000.00 You will keep the
bond for a week, and she promised to pay you back.

Story2. Your older brother has a small business idea but doesn’t have any
money. He wants to borrow your ₱ 1,500.00 and promised to pay you back ₱
1,800.00 in four weeks.

Story3. Your best friend at school, whom you know very-well, wants to borrow
your ₱ 1,500.00 to buy cupcakes. She plans to sell at a higher price and
promised to pay you back ₱1,700.00 in two weeks.

Whose bonds will you buy? Why?


_____________________________________________________________________________
_____________________________________________________________________________
_____________________________________________________________________________
_____________________________________________________________________________
_____________________________________________________________________________

15
CO_Q2_General Mathematics SHS
Module 9
What I Can Do

Perform the task below.


Stocks and bonds are efficient ways to help businesses grow. For businesses,
alternative ways to raise money for growth, such as saving and reinvesting its profits,
can take a very long time. Create a poster showing the importance of stocks and
bonds for you as an individual, for the community and for the nation.

Rubrics for Poster-making

Description 5 4 3 2 1

The poster All One of the


includes all required required
The poster Several
Competence required elements elements
includes all required
Required elements as are is not
required elements
Elements well as included included
elements. are missing.
additional on the on the
information poster. poster.

All graphics
All graphics are related All Graphics
are related to to the topic graphics do not Graphics do
the topic and and most of are related relate to not relate to
make it it are easier to the the topic. the topic.
easier to to topic. One or Several
Graphics
understand. understand. Some two borrowed
Relevance
All borrowed All borrowed borrowed graphics do
graphics borrowed graphics graphics not have a
have a graphics have a have a source
source have a source source citation.
citation. source citation. citation.
citation.

The poster
The poster
The poster is is
The poster The is
exceptionally acceptably
is attractive poster is distractingly
attractive in attractive
in terms of attractive messy or
Attractiveness terms of in terms of
design, though it very poorly
design, design,
layout, and may be a designed. It
layout, and layout,
neatness. bit messy. is not
neatness. and
attractive.
neatness.

16
CO_Q2_General Mathematics SHS
Module 9
Assessment

Choose the letter of the best answer. Write the chosen letter on a separate sheet of
paper.
1. The periodic interest payment that the bondholder receives during the time
between purchase date and maturity date.
a. Coupon Amount
b. Coupon Rate
c. Face Value
d. Market Value

2. A certain corporation declared a 4.5% dividend on a stock with a par value of


₱ 400.00. Mr. Reyes owns 100 shares of stocks with a par value of ₱ 400.00
How much is the dividend she received?
a. ₱ 1500.00
b. ₱ 1600.00
c. ₱ 1700.00
d. ₱ 1800.00

3. A certain financial institution declared a ₱ 80,000,000.00 dividend for the


common stocks. If there is a total of 500,000 shares of common stocks, how
much is the dividend per share?
a. ₱ 140.00
b. ₱ 150.00
c. ₱ 160.00
d. ₱ 170.00

4. It is the rate per coupon payment period; denoted by r.


a. Coupon Amount
b. Coupon Rate
c. Face Value
d. Market Value

5. It is the interest rate the bond issuer will use in computing the interest
payment, usually expressed in percent.
a. Coupon Date
b. Dividend
c. Dividend Rate
d. Maturity Date

6. It is the ratio of the annual dividend per share and the market value per share.
a. Coupon Amount
b. Dividend Rate
c. Market Value
d. Stock yield ratio

17
CO_Q2_General Mathematics SHS
Module 9
7. A bank declared a dividend of ₱ 35.00 per share for the common stock. If the
common stock closes at ₱ 98.00, how large is the stock yield ratio on this
investment?
a. 0.36
b. 0.39
c. 0.63
d. 0.66
8. Determine the amount of semi-annual coupon paid for a 3% bond with a face
value of ₱ 100,000.00 which matures after 8 years. How many coupons are
paid?
a. 10 times
b. 13 times
c. 16 times
d. 19 times

For numbers 9, 10 and 11.


Corporation M, with a current market value of ₱ 65.00, give a dividend of
₱ 11.00 per share of its common stock. Corporation N, with a current market value
of ₱ 75.00, give a dividend of ₱ 15.00 per share.

9. Find the stock yield ratio for Corporation M.


a. 14.56%
b. 15.76%
c. 16.92%
d. 17.08%
10. What is the stock yield ratio for Corporation N?
a. 18%
b. 20%
c. 22%
d. 24%
11. In which corporation will you invest your money? Why?
a. Corporation M, because each peso will earn you more if you invest in
M than in N.
b. Corporation M, the lower the share the higher the stock yield ratio.
c. Corporation N, because each peso will earn you more if you invest in N
than in M.
d. Corporation N, the lower the stock yield ratio the more you earn.

12. Determine the amount of the semi-annual coupon for a bond with a face value
of ₱ 450,000.00 that pays 12% payable semi-annual for its coupon.
a. ₱ 27,000.00
b. ₱ 24,000.00
c. ₱ 21,000.00
d. ₱ 18,000.00

18
CO_Q2_General Mathematics SHS
Module 9
For numbers 13, 14 and 15.
Find the value of a ₱ 50,000.00 bond with interest of 12% compounded semi-
annually which is redeemable at 105% in 3 years that yields the purchaser 8%
converted semi-annually.

13. Find the amount of the semi-annual coupon.


a. ₱ 2500.00
b. ₱ 2750.00
c. ₱ 3000.00
d. ₱3250.00
14. Find the redemption value of ₱ 50,000.00
a. ₱ 52,500.00
b. ₱ 55,200.00
c. ₱ 56,500.00
d. ₱ 59,200.00
15. Find the fair price of the given bond.
a. ₱ 55,319.56
b. ₱ 57,217.92
c. ₱ 59,034.12
d. ₱ 61,491.41

19
CO_Q2_General Mathematics SHS
Module 9
Additional Activities

Perform what is being asked in the following:

1. What will you pay today for a stock that is expected to make a ₱ 45.00 dividend
in one year if the expected dividend rate is 5% and you require a 12% return
on your investment?

2. XYZ Company’s preferred stock is selling for ₱ 60.00 a share. If the required
return is 8%, what will the dividend be two years from now?

3. Your broker is trying to sell you a stock with a current market price of
₱ 2,160.00 The stock’s last dividend was ₱ 53.25, and earnings and dividends
are expected to increase at a constant growth rate of 10%. Is the stock fairly
valued if the return is 13%? Explain why or why not.

20
CO_Q2_General Mathematics SHS
Module 9
Module 9
CO_Q2_General Mathematics SHS
21
Assessment What’s More What I Know
1. A 1. C
2. D Activity 1.1 2. D
3. C 1. ₱13.33 3. A
4. B 4. B
2. ₱15,250.00
5. C 5. A
6. D 3. 10,000 6. C
7. A 7. D
8. C Activity 1.2 8. B
9. C 1. ₱7,000.00 9. A
10. B 10. B
2. 48%
11. C 11. A
12. A 3. ₱1000.00 12. A
13. C 4. ₱190.48 13. D
14. A 5. 14. B
15. B Activity 1.3 15. C
1. ₱83.33
2. 0.35%
3. 0.29
4. ₱5,000.00
5. 16 times
Answer Key
References

Aoanan, Grace O., Plarizan, Ma. Lourdes P., Regidor, Beverly T., Simbulas, Lolly J.
General Mathematics for Senior High School. Quezon City: C&E Publishing,
Inc. 2016.

Buzon, Olivia N., Lapinid, Minie Rose C., Nivera, Gladys C. Geometry: Patterns and
Practicalities. Makati City: Salesiana Books by Don Bosco Press. 2007

Orines, Fernando B., Esparrago, Mirla S., Reyes, Nestor V. Advanced Algebra,
Trigonometry, and Statistics. Second Edition/ Orines, Fernando B. Quezon
City: Pheonix Publishing House. 2008.

22
CO_Q2_General Mathematics SHS
Module 9
For inquiries or feedback, please write or call:

Department of Education - Bureau of Learning Resources (DepEd-BLR)

Ground Floor, Bonifacio Bldg., DepEd Complex


Meralco Avenue, Pasig City, Philippines 1600

Telefax: (632) 8634-1072; 8634-1054; 8631-4985

Email Address: blr.lrqad@deped.gov.ph * blr.lrpd@deped.gov.ph


General Mathematics
Quarter 2 – Module 10:
Market Indices
for Stocks and Bonds

CO_Q2_General Mathematics SHS


Module 10
General Mathematics
Alternative Delivery Mode
Quarter 2 – Module 10: Market Indices for Stocks and Bonds
First Edition, 2021

Republic Act 8293, section 176 states that: No copyright shall subsist in any work of
the Government of the Philippines. However, prior approval of the government agency or office
wherein the work is created shall be necessary for exploitation of such work for profit. Such
agency or office may, among other things, impose as a condition the payment of royalties.

Borrowed materials (i.e., songs, stories, poems, pictures, photos, brand names,
trademarks, etc.) included in this module are owned by their respective copyright holders.
Every effort has been exerted to locate and seek permission to use these materials from their
respective copyright owners. The publisher and authors do not represent nor claim ownership
over them.

Published by the Department of Education


Secretary: Leonor Magtolis Briones
Undersecretary: Diosdado M. San Antonio

Development Team of the Module


Writers: Baby Jane B. Agudo
Editors: Elizabeth D. Lalunio, Elizabeth B. Dizon,Anicia J. Villaruel, Roy O. Natividad
Reviewers: Fritz A. Caturay, Necitas F. Constante, Celestina M. Alba,
Jerome A. Chavez ,Shiela Marie Arce, Carmela Ana A. Reforma,
Rafaela M. Merle
Illustrators: Hanna Lorraine G. Luna, Diane C. Jupiter
Layout Artist: Sayre M. Dialola, Roy O. Natividad, Noel Rey T. Estuita, Argie L. Ty,
Jilky I. Bosque
Management Team: Francis Cesar B. Bringas
Job S. Zape, Jr.
Ramonito Elumbaring
Reicon C. Condes
Elaine T. Balaogan
Fe M. Ong-ongowan
Elias A. Alicaya Jr.
Gregorio A. Co Jr.
Gregorio T. Mueco
Herbert D. Perez
Lorena S. Walangsumbat
Jee-Ann O. Borines
Asuncion C. Ilao
Printed in the Philippines by ________________________

Department of Education – Region 4A CALABARZON


Office Address: Gate 2 Karangalan Village, Brgy. San Isidro, Cainta, Rizal
Telefax: 02-8682-5773/8684-4914/8647-7487
E-mail Address: lrmd.calabarzon@deped.gov.ph
General Mathematics
Quarter 2 – Module 10:
Market Indices
for Stocks and Bonds
Introductory Message
This Self-Learning Module (SLM) is prepared so that you, our dear learners,
can continue your studies and learn while at home. Activities, questions, directions,
exercises, and discussions are carefully stated for you to understand each lesson.

Each SLM is composed of different parts. Each part shall guide you step-by-
step as you discover and understand the lesson prepared for you.

Pre-tests are provided to measure your prior knowledge on lessons in each


SLM. This will tell you if you need to proceed on completing this module or if you
need to ask your facilitator or your teacher’s assistance for better understanding of
the lesson. At the end of each module, you need to answer the post-test to self-check
your learning. Answer keys are provided for each activity and test. We trust that you
will be honest in using these.

In addition to the material in the main text, Notes to the Teacher are also
provided to our facilitators and parents for strategies and reminders on how they can
best help you on your home-based learning.

Please use this module with care. Do not put unnecessary marks on any part
of this SLM. Use a separate sheet of paper in answering the exercises and tests. And
read the instructions carefully before performing each task.

If you have any questions in using this SLM or any difficulty in answering the
tasks in this module, do not hesitate to consult your teacher or facilitator.

Thank you.
What I Need to Know

Previously, you learned how to illustrate stocks and bonds and distinguish between
stocks and bonds, and how to solve problems involving stocks and bonds.

Get ready and focus on the world of investing.

After going through this module, you are expected to:


1. analyze the different market indices for stocks and bonds.

What I Know

Let’s find out how far you know about this topic! Please take this challenge! Have
Fun! Choose the letter of the best answer. Write the chosen letter on a separate
sheet of paper.

1. Which of the following defines a stock market index?


a. It is a measure of a portion of the stock market.
b. It is the value of the index.
c. Dividend per share last year.
d. Net change between the two last trading days.

For items 2 – 8, refer to the table below.

52- 52-
VOLUME NET
WK- WK- STOCK HIGH LOW DIV CLOSE
(100s) CHG
HIGH LOW
64.30 61.00 ABC 63.00 60.00 .70 3050 64.20 0.20
98.00 82.00 DEF 98.00 80.00 .30 700 98.20 -0.2
88.00 84.50 GHI 87.55 83.50 .50 1000 87.50 0.20
74.00 80.50 JKL 72.55 70.50 .30 900 72.50 0.20

2. What is the lowest price for the past 52 weeks of stock DEF?
a. ₱82
b. ₱84
c. ₱88
d. ₱98

1 CO_Q2_General Mathematics SHS


Module 10
3. During the past 52 weeks, which stocks in the table sold at the highest
price?
a. ABC
b. DEF
c. GHI
d. JKL

4. What was the closing price the day before the last trading day of stock GHI?
a. ₱87.60
b. ₱87.50
c. ₱87.40
d. ₱87.30

5. During the past 52 weeks, which stocks in the table sold at the lowest price?
a. ABC
b. DEF
c. GHI
d. JKL

6. How many shares of stock ABC were traded?


a. 305,000
b. 405,000
c. 505,000
d. 605,000

7. Which stocks have the lowest shares were traded for the day?
a. ABC
b. DEF
c. GHI
d. JKL

8. What is the net change between the two last trading days of stock JKL?

a. −0.20
b. 0.20
c. 0.30
d. 0.50

For items 9 – 13, refer to the table below.


Bid Ask/Offer
Size Price Price Size
150 365,100 22.6000 22.8000 22,000 1
12 71,600 22.5500 22.9000 185,600 4
40 546,500 22.5000 22.1500 4,100 1

9. What is the total number of trader who wishes to buy a total of 365,100
shares at ₱22.60 per share?
a. 120
b. 130
c. 140
d. 150

2 CO_Q2_General Mathematics SHS


Module 10
10. How many trader/s is willing to sell his/her 22,000 shares for ₱22.80 per
share?
a. 6
b. 4
c. 2
d. 1

11. What is the total number of trader who wishes to buy a total of 546,500
shares at ₱22.50 per share?
a. 12
b. 40
c. 150
d. 160

12. How many trader/s is willing to sell his/her 185,600 shares for ₱22.90 per
share?
a. 6
b. 4
c. 2
d. 1

13. What is the total number of trader who wishes to buy a total of 71,600
shares at ₱22.55 per share?
a. 12
b. 40
c. 150
d. 160

14. It refers to the number of individual buy orders and the total number of
shares they wish to buy.
a. ask size
b. bid size
c. ask price
d. bid price

15. It can be a standard by which investors can compare the performance of


their stocks.
a. stock index
b. market index
c. market price
d. stock price

3 CO_Q2_General Mathematics SHS


Module 10
Lesson
Market Indices for Stocks
1 and Bonds

Learning how to invest your money is indispensable for your future. One of the
primary benefits of investing in the stock market is the chance to grow your money.

You have learned from your previous modules on how to find the dividend per share,
determine the stock yield ratio and compare two or more investments. This module
will help you analyze the different market indices for stocks and bonds and interpret
the theory of efficient markets.

What’s In

Let us start your journey by recalling the previous lessons you already learned about
stocks and bonds.

Activity 1

Study the given table and calculate the gain or loss for each item and record your
answer in the fourth column under “Change in Price”. In the fifth column, write
whether the change in price is a gain or loss.

Item Price Bought Price Sold Change in Price Gain or Loss

1 ₱1,552.50 ₱1,580.60
2 ₱468.90 ₱449.80
3 ₱345.70 ₱390.90
4 ₱1,298.20 ₱1,250.50
5 ₱456.30 ₱600.00

4 CO_Q2_General Mathematics SHS


Module 10
Activity 2

Multiply the Number of Items by the Price per Item to calculate the Total
Cost of the Items.

Number of
Item Price Per Item Total Cost of the Items
Items
1 600 ₱85.60
2 150 ₱80.50
3 65 ₱325.50
4 450 ₱165.00
5 70 ₱123.00

Activity 3

Read each problem below and answer the questions that follow:

Jhom owns 85 shares of common stock, worth ₱500 per value. If the corporation
1
declares a 6 % dividend, (a) what is the total dividend that Jhom should get?
4

__________________________________________________________________________________
__________________________________________________________________________________
__________________________________________________________________________________
__________________________________________________________________
(b) Did Jhom make the right decision to have a share in the stock market? Yes or
No? Explain why.

__________________________________________________________________________________
__________________________________________________________________________________
__________________________________________________________________________________
__________________________________________________________________

Notes to the Teacher


To be able to arrive in an accurate and similar answer, the teacher
must advise the learners to recall the formula in different markets
for stocks and bonds.

5 CO_Q2_General Mathematics SHS


Module 10
What’s New

Now, you already know how to find the dividend per share and determine the stock
yield ratio. Also, you already learned how to compare the two or more investments. I
am confident that you are now ready for the new lesson.

Stock on You!

The table below shows how information about stocks can be presented (values are
hypothetical).

52-
52-WK- VOLUME NET
WK- STOCK HIGH LOW DIV CLOSE
HIGH (100s) CHG
LOW
74.30 68.00 AAA 70.00 69.50 .70 9050 70.25 0.10
115.00 82.00 BBB 113.90 112.30 .30 10700 113.3 -0.20
88.00 84.50 CCC 87.55 83.50 .50 500 87.50 0.20
57.19 35.80 DDD 56.00 54.20 .28 10200 56.20 -0.15
31.60 23.25 EEE 30.00 27.80 1.70 1500 30.00 0.30

A. Use the Stocks for AAA to answer the following questions:

1. Give the highest and lowest prices for the past 52 weeks.
2. Give the highest and lowest prices for AAA shares yesterday.
3. What was the dividend per share last year?
4. What was the closing price on the last trading day?
5. What was the closing price the day before the last trading day?

B. Use the table to answer the following questions:

1. During the past 52 weeks, which stocks in the table sold at the highest
price? At the lowest price?
2. How many shares of EEE were traded?
3. Which stocks have the highest and lowest shares were traded for the
day?

6 CO_Q2_General Mathematics SHS


Module 10
What is It

In the previous activity, first, you need to analyze the information included in the
stock table. To find the highest price for the past 52 weeks, you need to look under
the heading 52-WK-HIGH. The price is given as 74.30. Thus, the highest selling price
of the stock in the past 52 weeks was ₱74.30.

To find the lowest price for the past 52 weeks, you need to look under the heading
52-WK-LOW. The price is given as 68. Thus, the lowest selling price for a share of
AAA stock for the past 52 weeks was ₱68.00. To find the highest and lowest prices
for the shares yesterday, look under the heading HIGH and LOW we have the
numbers 70 and 69.50 respectively. Thus, the highest and lowest prices for AAA
shares were ₱70. and ₱69.50. Looking at the heading DIV, we have the entry 0.70.
Thus, the dividend per share last year was 0.70.

Going back to the table, to find the closing price in the last trading day look under
the heading CLOSE, the number given is 70.25. Thus, when the stock exchange
closed in the last trading, the price was ₱70.25. The closing price the day before the
last trading day is the difference between CLOSE and NETCHG, thus we have ₱70.25
– ₱0.10 = ₱70.15.

To answer the following questions in letter B you need to compare all the information
or data in the table. During the past 52 weeks, the BBB stock sold at the highest
price ₱115 and EEE stock sold at the lowest price ₱31.60. To find the total shares
were traded of EEE look under the heading VOLUME, referring to sales volume and
the number given was 1500. In this case stock EEE sold 1,500 shares of 100 which
is equal to 150,000 shares. The BBB stock has the highest shares were traded with
10,700 shares of 100 which is equal to 1,070,000 and CCC stock was the lowest
shares were traded with 500 shares of 100 which is equal to 50,000.

For better understanding, study the definition of the related terms involving market
indices for stocks and bonds and analyze the given examples.

7 CO_Q2_General Mathematics SHS


Module 10
Definition:

The measure of a portion of the stock market is called the stock market index.

One of the examples is the PSE Composite Index or PSEi. It is composed of 30


companies carefully selected to present the general movement of market prices.

The up and down movement in percent change over time can indicate how the index
is performing.

Sector indices, indices that representing a particular sector.


Examples: financial institutions, industrial corporations, holding firms, service
corporations, mining/ oil, property.

The stock index can be a standard by which investors can compare the performance
of their stocks. A financial institution may want to compare its performance with
those others. This can be done by comparing it with the “financials” index.

Stock Index Tables

Stock indices are reported in the business section of magazines or newspapers, as


well as online. Here is the example of the stock index table that shows how a list of
index values is typically presented.

Value – it refers to the value of index

Chg – change of the index value from the previous trading day (the difference
between the value today and value yesterday)

%Chg – ratio of Chg to Val (Chg divided by Val)

8 CO_Q2_General Mathematics SHS


Module 10
Stock Tables

It is necessary to know and how to read the data or information in stocks and it
can be presented in the table. (values are hypothetical)

52- 52-
VOL NET
WK- WK- STOCK HIGH LOW DIV CLOSE
(100s) CHG
HIGH LOW
74.30 68.00 AAA 70.00 69.50 .70 9050 70.25 0.10
115.00 82.00 BBB 113.9 112.30 .30 10700 113.30 -0.20
88.00 84.50 CCC 87.55 83.50 .50 500 87.50 0.20
57.19 35.80 DDD 56.00 54.20 .28 10200 56.20 -0.15
31.60 23.25 EEE 30.00 27.80 1.70 1500 30.00 0.30

We will use the headings and the first row of the table to explain each column
and this serves as an example.

STOCK • The heading STOCK is a three-letter symbol the company using for
AAA trading. Example AAA

52- • The heading 52–WK–HIGH is the highest selling price of the stock
WK- in the past 52 weeks. The highest price at which AAA stock traded
HIGH during the past 52 weeks was ₱74.30.
74.3

52-
• The heading 52–WK–LOW is the lowest selling price of the stock in
WK-
the past 52 weeks. The lowest price at which AAA stock traded
LOW
during the past 52 weeks was ₱68
68

HIGH • The heading HIGH is the highest selling price of the stock on the
70 last trading day. The selling price of AAA stock in the last trading
day was ₱70.

LOW • The heading LOW is the lowest selling price of the stock on the last
69.5 trading day. The selling price of AAA stock in the last trading day
was ₱69.50.

DIV • The heading DIV is the dividend per share last year. The dividend
.70 per share last year of AAA stock was 0.70

• The heading VOL is the number of shares (in hundreds) traded in


VOL
the last trading day. In this case, stock AAA sold 9050 shares of 100
9050
which is equal to 905,000 shares.

9 CO_Q2_General Mathematics SHS


Module 10
CLOSE • The heading CLOSE is the closing price on the last trading days.
The closing price of AAA stock in the last trading day was ₱70.25.
70.25

NET • The heading NETCHG is the net change between the last two trading
CHG days. In the case AA, the net change is 0.10. The closing price the
0.10 day before the last trading day is ₱70.25 – 0.10 = ₱70.15

Buying or Selling Stocks

To sell his stock is another way of a shareholder to earn an income. A broker may be
used to buy or sell stocks. Nowadays, most of the transactions are done by making
a phone call to a registered broker or by logging on to a reputable online platform.
Those with accounts in online trading platforms may often encounter a table such
as the following.

Bid Ask/Offer
Size Price Price Size
122 354,100 21.60 21.80 20,000 1
9 81,700 21.55 21.90 183,500 4
42 456,500 21.50 22.15 5,100 1
2 12,500 21.45 22.25 11,800 4
9 14,200 21.40 22.30 23,400 6

In the table, the terms mean the following:

Bid Size – the number of individual buy orders and the total number of shares they
wish to buy.

Bid Price – the price that buyers are willing to pay for the stock

Ask Price - the price that the sellers of the stock are willing to sell the stock

Ask Size – how many individual sell orders have been placed in the online platform
and the total number of shares these sellers wish to sell.

Look at the table above, the first row under Bid means that there are a total of 122
traders who wish to buy a total of 354,100 shares at ₱21.60 per share. On the other
hand, the first row under Ask means that just one trader is willing to sell his/her
20,000 shares at a price of ₱21.80 per share.

10 CO_Q2_General Mathematics SHS


Module 10
Bond Market Indices

A bond market index is a measure of a portion of a bond market. A bond is a form


of long – term investment issued by a corporation or government where the purchaser
becomes a creditor of the company. A bondholder is the one who holds a corporation
or government bond.

The main platform for bonds or fixed-income securities in the Philippines is the
Philippine Dealing and Exchange Corporation (or PDEx). Unlike stock indices that
are associated with virtually every stock market in the world, bond market indices
are far less common. Other than certain regional bond indices that have sub-indices
covering the Philippines, our bond market does not typically compute a bond market
index. Instead, the market rates produced from the bond market are interest rates
that may be used as benchmarks for other financial instruments.

The Bond Market and Government Bonds

Government bonds are sold out to banks and other brokers and dealers every
Monday by Bureau of Treasury. It depends on their terms (or tenors), these bonds
are also called treasury bills (t – bills), treasury notes (t-notes), or treasury bonds (t-
bonds). On the day right after the auction, the news agencies are normally reported
the resulting coupon rates and the total amount sold for these bonds. Since these
bond transactions involve large amounts, these bonds are usually limited to banks,
insurance firms, and other financial institutions. The banks may then resell these
bonds to their investors.

Although the coupon rate for bonds is fixed, bond prices fluctuate because they are
traded among investors in what is called the secondary market. These prices are
determined by supply and demand, the prevailing interest rates, as well as other
market forces. As the price of the bond may increase or decrease, some investors
may choose to sell back to banks the bonds they acquired before their maturity to
cash in their gains even before maturity.

Despite the fact that bond investing is considered safer than stock investing, there
is still some risk involved. The most extreme scenario is a default by the issuer. In
this case, the investor can lose not only the coupons but even the money invested in

11 CO_Q2_General Mathematics SHS


Module 10
the bond. Bond investors should thus be aware of the financial condition of the issuer
of the bond and prevailing market conditions.

Example 1

Consider the stock for FFF and GGG to answer the questions that follow.

52- 52-
YLD VOL NET
WK- WK- STOCK HIGH LOW DIV CLOSE
% (100s) CHG
HIGH LOW
75 69 FFF 71.00 67.90 3.50 2.5 190 72.30 0.30
118 100 GGG 115.20 114.30 1.40 3.1 10500 115.00 -0.10

For stocks FFF and GGG answer item 1-6

1. Give the highest and lowest prices for the past 52 weeks.
2. Give the highest and lowest prices shares yesterday.
3. What was the dividend per share last year?
4. What was the annual percentage yield last year?
5. What was the closing price in the last trading day?
6. What was the closing price the day before the last trading day?
7. During the past 52 weeks, which stocks in the table sold at the highest
price? At the lowest price?
8. How many shares of GGG were traded?
9. Which stocks have the lowest shares were traded for the day?

Answers:
For Stock FFF:

1. 52-WK-HIGH = ₱75
52-WK-HIGH = ₱69
2. Highest Price = ₱71
Lowest Price = ₱67.90
3. Dividend per share = ₱3.50
4. YLD% = 2.5%
5. Closing Price = ₱72.30
6. Closing Price (the day before the last trading day) = ₱72.30 - ₱0.30 = ₱72

12 CO_Q2_General Mathematics SHS


Module 10
For Stock GGG:

1. 52-WK-HIGH = ₱118
52-WK-HIGH = ₱100
2. Highest Price = ₱115.20
Lowest Price = ₱114.30
3. Dividend per share = ₱1.40
4. YLD% = 3.1%
5. Closing Price = ₱115.00
6. Closing Price (the day before the last trading day) = ₱115.00 + ₱0.10 =
₱115.10
7. During the past 52 weeks, the GGG stock sold at the highest price ₱118 and
FFF stock sold at the lowest price ₱75.
8. The total shares were traded of look under the heading volume, referring to
sales volume and the number given was 10,500. In this case stock GGG sold
10,500 shares of 100 which is equal to 1,050,000 shares.
9. The FFF stock has the lowest shares were traded with 190 shares of 100 which
is equal to 19,000 shares.

Example 2

Rachel owns 2,000 shares of a company A at ₱980 per share. She also owns
15 000 shares for company B at ₱250 per share. In which company is the total value
of her share greater? Explain how you know which is greater.

To calculate the total value of her share:

Company A
No. of shares x Selling Price = Total Price
2,000 x ₱980 = ₱1,960,000
Company B

No. of shares x Selling Price = Total Price


15,000 x ₱250 = ₱3,750,000

Therefore, the greater value of her share is in company B with 15,000 at ₱250
per share, its total value is ₱3,750,000. To find which company will give a greater
value of share multiply the number of shares to its selling price then compare the
final answer.

13 CO_Q2_General Mathematics SHS


Module 10
What’s More

Activity 1.1

A. Consider the stock for the first company to answer the questions that follow.

52-
52-WK- STOC VOL YLD NET
WK- HIGH LOW DIV CLOSE
HIGH K (100s) % CHG
LOW
74.30 68.00 HHH 70.00 69.50 .70 9050 2.6 70.25 0.10
115.00 82.00 III 113.90 112.30 .30 10700 2.5 113.30 -0.20
88.00 84.50 JJJ 87.55 83.50 .50 500 3.1 87.50 0.20
57.19 35.80 KKK 56.00 54.20 .28 10200 4.2 56.20 -0.15
31.60 23.25 LLL 30.00 27.80 1.70 1500 2.0 30.00 0.30

1. Give the highest and lowest prices for the past 52 weeks.

2. Give the highest and lowest prices for shares yesterday.

3. What was the dividend per share last year?

4. What was the annual percentage yield last year?

5. What was the closing price on the last trading day?

6. What was the closing price the day before the last trading day?

B. Use the stock table above to answer the following questions.

7. During the past 52 weeks, which stocks in the table sold at the highest

price? At the lowest price?

8. How many shares of KKK were traded?

9. Which stocks have the lowest shares were traded for the day?

10. Which stocks have the highest shares were traded for the day?

14 CO_Q2_General Mathematics SHS


Module 10
Activity 1.2

True or False: Read each statement below carefully. Place a T on the line if you
think a statement is TRUE. Place an F on the line if you think the statement is
FALSE.

_________1. Stocks are shares in the ownership of a company.

_________2. A stock market index is a measure of a portion of the stock market.

_________3. The up and down movement in percent change over time can

indicate how the index is performing.

_________4. Bid Size is the price that buyers are not willing to pay for the stock.

_________5. Bid size is the number of individuals who buy orders and the total

number of shares they wish to buy.

_________6. Buying and selling stocks can be done by making phone calls to a
registered broker or by logging on to a reputable online trading
platform.

_________7. A stock market index is a measure of a portion of the bond market.

_________8. Stock indices are far less common than bond market indices.

Activity 1.3
Solve the following problems.

1. Nicco owns 5,000 shares of a company at ₱780 per share. He also owns
25,000 shares for another company at ₱260 per share. In which company is
the total value of her share greater? Explain how you know which is greater.
___________________________________________________________________________
___________________________________________________________________________
___________________________________________________________________________

2. On July 1, 2019, Aldrin bought 400 shares of ABC stocks at ₱450 per share.
He sold half of his shares on August 3, 2019, for ₱470. How much money did
he gain or lose on the investment?
___________________________________________________________________________
___________________________________________________________________________
___________________________________________________________________________

15 CO_Q2_General Mathematics SHS


Module 10
What I Have Learned

A. Please read the sentences carefully and fill in the missing word/s by writing your
answer on the space provided for.

1. A _________________ is a measure of a portion of the stock market.

2. The ___________________ can be a standard by which investors can compare

the performance of their stocks.

3. The highest selling price of the stock in the past 52 weeks is represented as

_________________________on a stock table.

4. The highest selling price of the stock in the last trading day weeks is

represented as _________________________on a stock table.

5. A ____________________ is a measure of a portion of a bond market.

6. _________________ is the price that the buyers are willing to pay for the stock.

7. _________________ is the price that the sellers of the stock are willing to sell the

stocks.

8. __________________ tells how many individual sell orders have been placed in

the online platform and the total number of shares these sellers wish to sell.

B. If you know the number of shares you bought and the price per share, how can

this be used in calculating the total cost of your investment?

_______________________________________________________________________________

_______________________________________________________________________________

_______________________________________________________________________________

_______________________________________________________________________________

16 CO_Q2_General Mathematics SHS


Module 10
What I Can Do

Using the latest newspaper, or available online information choose


a stock table for a company of your choice and explain the meaning
of the data that can be found on the table.

Your output will be graded using this rubric.

Score Criteria
Highly The output demonstrates a complete understanding of interpreting
Proficient a stock table. The data in each column on the table was discussed
(90-100%) with complete, accurate, and clear explanations. It is also error-
free, without ambiguity, and grammatically correct.
Proficient The output demonstrates a considerable understanding of
(80-89%) interpreting a stock table. The data in each column on the table
was discussed with complete explanations. There are 1 to 2
grammatical errors.
Developing The output demonstrates some understanding of interpreting a
(70-79%) stock table. The data in each column on the table was discussed
with minimal explanations. There are more than 2 to 5
grammatical errors.
Beginning The output demonstrates a limited understanding of interpreting a
(60-69%) stock table. The data in each column on the table was not
discussed clearly. There are more than 5 grammatical errors.

Assessment

Choose the letter of the best answer. Write the chosen letter on a separate sheet of
paper.

1. It is a measure of a portion of the stock market


a. stock market
b. stock market index
c. value of the index
d. dividend

17 CO_Q2_General Mathematics SHS


Module 10
For items 2 – 8, refer to the table below.
52- 52-
VOLUME NET
WK- WK- STOCK HIGH LOW DIV CLOSE
(100s) CHG
HIGH LOW
65 62.50 LMN 64.00 60.00 .10 350 64.20 0.30
87 85.00 OPQ 86.30 85.00 .30 700 87.00 -0.20
94 92.50 RST 93.55 92.25 .40 15000 93.50 0.10
79 70.50 XYZ 78.55 76.50 .20 9300 78.50 0.40

2. What is the lowest price for the past 52 weeks of stock RST?
a. ₱62.50
b. ₱70.50
c. ₱85
d. ₱92.50

3. During the past 52 weeks, which stocks in the table sold at the highest
price?
a. RST
b. XYZ
c. OPQ
d. LMN

4. What was the closing price the day before the last trading day of stock XYZ?
a. ₱87.60
b. ₱87
c. ₱78.10
d. ₱78.50

5. During the past 52 weeks, which stocks in the table sold at the lowest price?
a. LMN
b. OPQ
c. RST
d. XYZ

6. How many shares of stock OPQ were traded?


a. 70
b. 700
c. 7,000
d. 70,000

7. Which stock has the lowest shares were traded for the day?
a. XYZ
b. LMN
c. OPQ
d. RST

8. What is the net change between the two last trading days of stock XYZ?
a. −0.20
b. 0.10
c. 0.30
d. 0.40

18 CO_Q2_General Mathematics SHS


Module 10
For items 9 – 12, refer to the table below.

Bid Ask/Offer
Size Price Price Size
360 364,200 23.6000 23.8000 22,000 2
11 61,600 23.5500 23.9000 188,600 5
56 537,500 23.5000 23.1500 4,100 1

9. What is the total number of trader who wishes to buy a total of 537,500
shares at ₱23.50 per share?
a. 11
b. 56
c. 140
d. 350
10. How many trader/s is willing to sell his/her 22,000 shares for ₱23.80 per
share?
a. 6
b. 5
c. 4
d. 2
11. What is the total number of trader who wishes to buy a total of 61,600
shares at ₱23.55 per share?
a. 11
b. 14
c. 56
d. 360
12. How many trader/s is willing to sell his/her 188,600 shares for ₱23.90 per
share?
a. 1
b. 2
c. 5
d. 11
13. What is the total number of trader who wishes to buy a total of 364,200
shares at ₱23.600 per share?
a. 11
b. 56
c. 140
d. 360
14. It refers to how many individual sell orders have been placed in the online
platform and the total number of shares these sellers wish to sell.
a. ask size
b. bid size
c. ask price
d. bid price
15. It refers to the price that the sellers of the stock are willing to sell the stock.
a. ask size
b. bid size
c. bid price
d. ask price

19 CO_Q2_General Mathematics SHS


Module 10
Additional Activities

Using stock table for a company of your choice in What I


Can Do. Answer the following questions.

A. Use the Stocks for the first company to answer the following questions:

1. Give the highest and lowest prices for the past 52 weeks.

2. Give the highest and lowest prices for shares yesterday.

3. What was the dividend per share last year?

4. What was the closing price on the last trading day?

5. What was the closing price the day before the last trading day?

B. Use the table and answer the following questions:

6. During the past 52 weeks, which stocks in the table sold at the highest

price? At the lowest price?

7. How many shares of EEE were traded?

8. Which stocks have the highest and lowest shares were traded for the

day?

20 CO_Q2_General Mathematics SHS


Module 10
Module 10
CO_Q2_General Mathematics SHS 21
What I Know What's More Assessment
1. a Activity 1.1 1. b
2. a 2. d
3. b 1. 52-WK-HIGH = ₱74.30 3. a
4. d 52-WK-HIGH = ₱68.00 4. c
5. a 2. Highest Price = ₱70.00 5. a
6. a Lowest Price = ₱69.5 6. d
7. b 3. Dividend per share = ₱0.70 7. b
8. b 4. YLD% = 2.6% 8. d
9. d 5. Closing Price = ₱70.25 9. b
10.d 6. Closing Price (the day before the last trading 10. d
11. b day) = ₱70.25 - ₱0.10 = ₱70.15 11. a
12. b B. 12. c
13. a 7. Highest Price: stock III 13. d
14. b Lowest Price: stock LLL 14. a
15. a 8. 1,020,000 15. d
9. Stock JJJ
10. Stock III
Activity 1.2
1. T
2. T
3. T
4. F
5. T
6. T
7. T
8. F
Activity 1.3
1. The greater value of her share is
the second company with 250
shares at ₱260.00 per share
2. Aldrin gained ₱4,000 on his investment.
Answer Key
References
Oronce, Orlando. General Mathematics. Sampaloc Manila, Philippines. Rex
Bookstore, Inc. 2016.

General Mathematics Learner’s Material. First Edition. 2016. pp. 217 - 224

*DepED Material: General Mathematics Learner’s Material

http://www.pse.com.ph/stockMarket/home.html

22 CO_Q2_General Mathematics SHS


Module 10
For inquiries or feedback, please write or call:

Department of Education - Bureau of Learning Resources (DepEd-BLR)

Ground Floor, Bonifacio Bldg., DepEd Complex


Meralco Avenue, Pasig City, Philippines 1600

Telefax: (632) 8634-1072; 8634-1054; 8631-4985

Email Address: blr.lrqad@deped.gov.ph * blr.lrpd@deped.gov.ph


General Mathematics
Quarter 2 – Module 11:
Business and Consumer Loans

CO_Q2_General Mathematics SHS


Module 11
General Mathematics
Alternative Delivery Mode
Quarter 2 – Module 11: Business and Consumer Loans
First Edition, 2020

Republic Act 8293, section 176 states that: No copyright shall subsist in any work of
the Government of the Philippines. However, prior approval of the government agency or office
wherein the work is created shall be necessary for exploitation of such work for profit. Such
agency or office may, among other things, impose as a condition the payment of royalties.

Borrowed materials (i.e., songs, stories, poems, pictures, photos, brand names,
trademarks, etc.) included in this module are owned by their respective copyright holders.
Every effort has been exerted to locate and seek permission to use these materials from their
respective copyright owners. The publisher and authors do not represent nor claim ownership
over them.

Published by the Department of Education


Secretary: Leonor Magtolis Briones
Undersecretary: Diosdado M. San Antonio

Development Team of the Module


Writer: Rey Mark R. Queaño
Editors: Elizabeth D. Lalunio, Elizabeth B. Dizon, Anicia J. Villaruel, Roy O. Natividad
Reviewers: Jerry Punongbayan, Diosmar O. Fernandez, Dexter M. Valle, Jerome
A. Chavez, Jea Airee Carimae M . De Mesa, Dr. Anicia J. Villaruel,
Marilou M. Tierra and Moahna Aura M. Mancenido
Illustrator: Hanna Lorraine Luna, Diane C. Jupiter and Michael A. Alonzo
Layout Artist: Roy O. Natividad, Sayre M. Dialola and Noel Rey T. Estuita
Management Team: Francis Cesar B. Bringas
Job S. Zape, Jr.
Ramonito Elumbaring
Reicon C. Condes
Elaine T. Balaogan
Fe M. Ong-ongowan
Hermogenes M. Panganiban
Phillip B. Gallendez
Josephine T. Natividad
Anicia J. Villaruel
Dexter M. Valle

Printed in the Philippines by ________________________

Department of Education – Region IV-A CALABARZON

Office Address: Gate 2 Karangalan Village, Barangay San Isidro


Cainta, Rizal 1800
Telefax: 02-8682-5773/8684-4914/8647-7487
E-mail Address: region4a@deped.gov.ph
General Mathematics
Quarter 2 – Module 11:
Business and Consumer Loans
Introductory Message
This Self-Learning Module (SLM) is prepared so that you, our dear learners,
can continue your studies and learn while at home. Activities, questions, directions,
exercises, and discussions are carefully stated for you to understand each lesson.

Each SLM is composed of different parts. Each part shall guide you step-by-
step as you discover and understand the lesson prepared for you.

Pre-tests are provided to measure your prior knowledge on lessons in each


SLM. This will tell you if you need to proceed on completing this module or if you
need to ask your facilitator or your teacher’s assistance for better understanding of
the lesson. At the end of each module, you need to answer the post-test to self-check
your learning. Answer keys are provided for each activity and test. We trust that you
will be honest in using these.

In addition to the material in the main text, Notes to the Teacher are also
provided to our facilitators and parents for strategies and reminders on how they can
best help you on your home-based learning.

Please use this module with care. Do not put unnecessary marks on any part
of this SLM. Use a separate sheet of paper in answering the exercises and tests. And
read the instructions carefully before performing each task.

If you have any questions in using this SLM or any difficulty in answering the
tasks in this module, do not hesitate to consult your teacher or facilitator.

Thank you.
What I Need to

Previously, you have learned about the difference between stocks and bonds.
We said that stocks are the ownership shares you buy from a company while
bonds are a form of long-term investment issued by a corporation or
government where the purchaser becomes a creditor of the company. You now
also have the background on how to describe different markets for stocks and
bonds as well as the skills in analyzing different market indices for stocks and
bonds.

Now, you are about to begin another module in General Mathematics. This
module was designed and was written with you in mind. It is here to help you
learn the concepts of business loans and consumer loans. The scope of this
module permits it to be used in many different learning situations. The
language used recognizes the diverse vocabulary level of students. The lessons
are arranged to follow the standard sequence of the course. But the order in
which you read them can be changed to correspond with the textbook you are
now using.

Are you ready for the new lesson? Fasten your seatbelt and focus on the
world business and consumer loans.

After going through this module, you are expected to:

1. define loan and its related terms;

2. illustrate business loan and consumer loan; and

3. distinguish business loan from consumer loan.

1
CO_Q2_General Mathematics SHS
Module 11
What I Know

Let’s find out how far you might already know about this topic! Please take
this challenge! Have Fun!

Directions. Choose the letter of the best answer. Write the chosen letter on a

separate sheet of paper.

1. What do you call the money borrowed that may be used for a business

purpose?

a. business loan

b. collateral

c. consumer loan

d. mortgage

2. What do you call an asset used to secure the loan that may be real estate

or other investments?

a. business loan

b. collateral

c. consumer loan

d. mortgage

3. What is the term coined as the time it takes to pay the entire loan?

a. collateral

b. credit report

c. due date

d. term of the loan

2
CO_Q2_General Mathematics SHS
Module 11
4. Which of the following is money lent to an individual for personal or family

purposes?

a. business loan

b. collateral

c. consumer loan

d. mortgage

5. What do you call a person who guarantees to pay for someone else’s

financial obligation if the borrower fails to do so?

a. amortization

b. collateral

c. guarantor

d. mortgage

6. Which of the following is not an example of a business loan?

a. cash flow loans

b. credit cards

c. invoice financing

d. micro loans

7. Which of the following is not an example of a consumer loan?

a. asset-based financing

b. car loans

c. mortgages

d. student loans

8. Which of the following statements about the interest rate of consumer and

business loans are correct?

a. The business loan has higher interest than a consumer loan.

b. Consumer loan has a higher interest rate than a business loan.

c. Consumer loan and business loan are always equal in interest rates.

d. The business loan has no interest rate, unlike consumer loan.

3
CO_Q2_General Mathematics SHS
Module 11
9. Which of the following statements about terms of payment for consumer

and business loans are correct?

a. The business loan has a longer term of payment than a consumer

loan.

b. Consumer loan has a longer term of payment than a business loan.

c. Consumer and business loans are always equal in terms of payment.

d. Consumer loan has no specific term of payment, unlike a business

loan.

10. Which of the following is not a required document if you are applying for

a business loan.

a. certificate of employment

b. company’s financial statement

c. credit report

d. income tax return

For 11-15. Identify the following situations and write a if it is a business loan,
b if the situation is a consumer loan, c if the situation is both a business loan
and a consumer loan, or d if the situation is neither a business loan nor a
consumer loan.

______11. Mr. Tuazon wants to have another branch of his car repair shop.
He decided to apply for a loan that he can use to pay for the
rentals of the new branch.

______12. Mrs. Morales runs a trucking business. She wants to buy four
more trucks for the expansion of her business. She applied for a
loan in a bank worth ₱5,000,000.00

______13. Margarita decided to take her family for a vacation. To cover the
expenses, she decided to apply for a loan.
______14. John Rey decided to purchase a condominium unit near his
workplace. He got a loan worth ₱3,000,000.00

______15. Mr. Malonzo renovated his house for ₱100,000.00 This was made
possible because of an approved loan worth ₱85,000.00

4
CO_Q2_General Mathematics SHS
Module 11
Lesson Illustrating and
1 Distinguishing Business
and Consumer Loans
According to Max of the movie The Adventures of Sharkboy and
Lavagirl, “Dream a better dream, then work to make it real”. If you have a big
dream, you should have an extra bigger persistence to achieve that dream. If
you want to buy something big and expensive, you have to save for it. If you
want a car, a house, or if you want to start your own business, you have to
deal with it diligently. Right?

But in this world of modern business, you have the option to achieve
these dreams or buy what you want instantly. By just paying a certain amount
at a certain period and interest, you can have the money. Yes, you are right,
we can apply for loans.

And to know more about loans, welcome to this module. This lesson
will give you insights on the basic concepts of loans specifically on business
and consumer loans. At the end of this module, you should be able to decide
wisely on the appropriateness of business and consumer loans and its proper
utilization. Are you all ready?

What’s In

Before we dig into the main topic of this module - the business and
consumer loans, let us have first the background on the concepts of LOANS.

The basic concepts of loans are very familiar to all of us. Through loans,
any individual can receive assistance in addressing financial needs from
different lending corporations. Loans are not just limited to borrowing money
in a bank to finance business expansion, but also covers the use of credit
card, amortizing appliances and many more.

5
CO_Q2_General Mathematics SHS
Module 11
Activity:

Directions. The following statements describe the basic terms related to the
topic of this module that you are most likely familiar with. Rearrange the
jumbled letters to reveal the answer. Write your answer in the given space.

_________1. It is a financial obligation of paying someone a certain amount for

the use of his or her money. (ONAL)

_________2. It is the amount paid to borrow money (STEINERT)

_________3. It is the amount of money that the loan applicant requests to

borrow. (ALPNIRICP)

_________4. It is the payment or interest level used to determine a borrower's

monthly payment. (TEAR)

_________5. It is the maturity or length of time over which a loan extends, for

instance one year, five years, or more. (EMRT)

6
CO_Q2_General Mathematics SHS
Module 11
What’s New

Read the story of two best friends, analyze their situation, and try to
answer the questions below.

Best Friends Forever


Bessy and Conny are childhood best friends since elementary. Even
their parents are also friends and treat each other as family. The two girls
went to same school up to college but with different courses. Conny took
up Architecture while Bessy got into Business Administration. But even
though they were not in the same college, they see to it that they still have
time to each other especially at times that they need someone to lean on
and a shoulder to cry on.
Because of their dedication, hard work and innate intelligence, there
is no doubt that both of them graduated as Cum Laude. Bessy and Conny
have big dreams. They are dreaming not only for themselves but for their
family. The Filipino culture of close family ties was very evident among
them.
One day, after graduation Bessy and Conny talk to each other and
discuss their plans.
“Hi sister, how are you today” Bessy said.
“I’m good sister, how about you? Conny asked.
“I’m also fine sis” Bessy said.
“So, what’s our plan sis? Now that we graduated from college, it’s
our turn to help our family”, Conny said.
“Yeah, I am planning to expand our family business so that I could
practice my degree at the same time I could help my parents” Bessy said.
“That’s a good plan. I also want to practice my degree in
Architecture. That’s why I want to build a new house for my family. I want
to give my parents their dream house” Conny replied.
“We really have big dreams sis, but we are just starting, how can we
do that?” Bessy asked.
“Well, my uncle is working at North-South bank and they offer loans
that we can pay while working with our plans.” Conny explained.
“I think that is a good idea sis. We will be working hard to make a
good payment for that loan at the same time we could start with what we
are dreaming of.” Bessy said.

7
CO_Q2_General Mathematics SHS
Module 11
The next morning, Bessy and Conny went to the bank to apply for
their loans. After talking to the bank manager, accomplishing all the forms
and submitting all the requirements, their loans were approved.
“How your application sis” Bessy asked.
“It’s okay, they asked me for a collateral and I submitted an income
tax return and a bank statement. Then they give me 10 years as the term
of my loan at 6.5% annual interest” Conny explained.
“Hmm, I see. Well, I was also asked for a collateral so I gave the title
of our house. They also require the owners of our business who happened
to be my parents as guarantors. I was also asked to submit credit report,
income tax returns and our business’ financial statement. They only give
me 5 years to pay my loan at 10% interest rate.” Bessy also explained.
Even though they applied for different loans, good thing that it was
both approved. And the two promised to make the best out of the money
lent to them to start their dreams.

Questions:
1. Who among them applied for a loan easier? Why do you say so?
2. What were the requirements asked for the approval of their loans?
3. In what way do you think Bessy’s loan is different from Conny’s
loan?
4. Do you think applying for a loan is good or bad?
5. If you will apply for a loan, what will be your plan so that you will
never fail in paying it?

8
CO_Q2_General Mathematics SHS
Module 11
What is It

The story of Bessy and Conny aside from it’s a story of best friends, it
is also a story of two different kinds of loans - the business loan and
consumer loan.
A business loan is what Bessy applied for. A business loan is money
lent specifically for a business purpose. It may be used to start a business or
to have a business expansion. The business loan includes microloans, invoice
financing, mezzanine financing, bank loans, asset-based financing, cash flow
loans, as well as business cash advances. On the other hand, Conny applied
for a consumer loan which is money lent to an individual for personal or
family purposes. Consumer loan includes credit cards, mortgages, home
equity lines of credit, refinances, auto loans, student loans, as well as personal
loans.
While reading the story, you also encountered the following terms:
• Collateral – assets used to secure the loan. It may be real estate or other
investments
• Term of the loan – time to pay the entire loan
• Guarantor – a person who guarantees to pay for someone else’s financial
obligation if the borrowers fail to do so.

Business Loan versus Consumer Loan


Term of Business Loan Consumer Loan
Reference
Collateral May require collateral (real May also require a
estate, other investment, collateral
equipment, fixtures or
furniture)
Guarantor Require the business owner Does not usually
to sign as guarantor require a guarantor
Documents The lendee has to submit a The lendee may require
credit report, income tax a credit report, bank
returns and the company's statements, and
financial statement. income tax return. If
he/she is employed, a
certificate of
employment and
employee pay slips will
be submitted.
Term of the Loan Shorter-term Longer-Term
Interest rate Higher interest Lower interest rate

9
CO_Q2_General Mathematics SHS
Module 11
Examples:

1. Andrea, a hardworking transwoman who recently finished her short course


on hair and makeup is planning to put up her small salon. She was advised
by her friends to apply for a bank loan. Which loan will she apply? A
business loan or a consumer loan?

Answer: Andrea needs a business loan since she is planning to use the

money to start her salon business.

2. Mr. Zion is an environmentalist who is planning to conduct a mangrove


planting at Brgy. Barra. He wants to cover all the expenses for this activity,
but he realized that his money is not enough. Hence, he needs to borrow
money to finance his advocacy. Is he going to apply for a business loan or
a consumer loan?

Answer: Mr. Zion will apply for a consumer loan since the money that he

will borrow is for the finances of his activity.

10
CO_Q2_General Mathematics SHS
Module 11
EXm

What’s More

Activity 1.1
Directions. Identify whether the following statements are TRUE or FALSE.
Write T if the underlined word/s is/are correct for the statement, otherwise
write the correct word/s to make the statement correct.

______________1. Business loans are money used to finance a business or to


start a new business.
______________2. Credit cards and car loans are example of business loans.
______________3. Guarantors are the assets used to secure a loan which
include real-states or investments.
______________4. Consumer loan has higher interest rate than business loan.
______________5. Consumer loan has longer term of payment than business
loan.

Activity 1.2
Directions. Identify whether the following illustrates a business loan or a
consumer loan. Write BL if the situation shows business loan and CL if it
states a consumer loan.

1. Mr. Manzano plans to have a barbershop. He wants to borrow some money


from the bank to buy the equipment and furniture for the
barbershop.
Answer: __________________

2. Margareth is planning to take up Engineering as her college course. To


support her, Mr. And Mrs. Alvar, her parents, borrowed money from the
bank for her financial needs.
Answer: __________________

3. Because of his love for our country, Mr. Razo always conduct an
Independence Day presentation to his barangay. However, because of
COVID - 19, he got a problem with his business and need extra money to
finance his advocacy. He realized that he needs to borrow money from the
bank to continue his annual Independence Day activity.
Answer: __________________

4. Mr. Bonifacio wants to renovate their 100-year old ancestral house in


Sariaya, Quezon which was also identified as a historical house in the
province. He wants to repair the ceiling in preparation for the rainy season.
He will borrow some money from the bank to finance this plan.
Answer: __________________

11
CO_Q2_General Mathematics SHS
Module 11
5. Mr. Santiago owns the Kakanin food cart business. He wants to put another
food cart on a new mall in Tayabas City. He decided to have a loan
amounting to ₱100,000.00 to establish the new business.
Answer: __________________

6. In preparation for the online learning of her children due to the COVID-19
pandemic, Mrs. Castro decided to buy laptops for them and got an internet
connection. But because of short in budget, she needed to borrow money
amounting to ₱80,000.00 for this school preparation.
Answer: __________________

7. Instead of working as a Manager in a certain company, Mr. De Guzman


decided to leave his job and put up an internet cafe instead. To finance his
business, he borrowed money from a Kabayan Bank.
Answer: __________________

8. Because Anna believed that Filipinos are truly honest, she decided to put
up an Honesty Bookstore in front of Excellence National High School.
Everyone can get what they need and pay for it without anyone in the
cashier. But before she can own this shop, she borrowed ₱1,000,000.00
from the bank to finance the expenses in putting up this bookstore.
Answer: __________________

9. Yzzyr excelled in Science and even won various competitions relative to


Applied Science. At the back of her achievements are her supportive
parents. Before she invented the COVID-19 scanning robot, her parents
need to finance her project with a big amount. To help her, they even
borrowed money from the bank just for her to finish her invention.
Answer: __________________

10. Xandria really loves her family. They supported her in all her endeavors.
As return of gratitude, she promised them that they will make a tour to
great places in the world. To realize her promise, and to help her finance
their tour, she applied for a personal loan amounting to ₱200,000.00 to
Bank XY.
Answer: __________________

12
CO_Q2_General Mathematics SHS
Module 11
What I Have Learned

A. Directions. Using your own words, please complete the following


statements to reveal what you have learned to the topic. Write your
answers on the given space.
1. Business loan is a money
______________________________________________________________________
_____________________________________________________________________.

2. Consumer loan is intended for


______________________________________________________________________
_____________________________________________________________________.

3. Collateral is
______________________________________________________________________
_____________________________________________________________________.

4. Guarantor is the person


______________________________________________________________________
_____________________________________________________________________.

5. Term of loan is about


__________________________________________________.
B. Directions. Using the Venn Diagram, write words or statements that will
compare
business from consumer loans.

Business Loan Consumer Loan

13
CO_Q2_General Mathematics SHS
Module 11
What I Can Do

To see whether you imbibe the idea of business loan and consumer
loan, please do any of the following application activities.
A. Comic strip
Make a 4-panel comic strip showing a scenario in a bank where a person
is doing a business loan, while the other one is a consumer loan.
B. Short story
Write a short story of a family who applied for either a business loan or a
consumer loan showing how they struggle but, in the end, become
successful because of the loan they got.
C. Jingle or Rap
Make a short jingle or a rap illustrating and distinguishing business loan
and consumer loan. (You may also submit a video to your teacher if your
facilities allowed.)
Please be guided by these rubrics.
Comics Strip
Category 4 3 2 1
Basic Comic strip Comic strip Comic strip is Comic strip
Elements contains all contains all missing one is missing
(Title, four elements in a elements basic element. two or more
completed creative, basic
panels) organized form. elements.
Content Comic strip Shows the Shows the Just show
clearly shows ideas of ideas of difference
the ideas of business business and between
business and and consumer consumer
consumer loans consumer loans but with and
including all the loans but missing two or business
elements with more elements loans
(documents etc.) missing one (documents without
element etc.) including
(documents the
etc.) elements.
Illustrations Strip contains Strip Strip contains Strip
four completed contains three contains
(colored) four completed two or fewer
drawings in a completed (colored) completed
creative, (colored) drawings. (colored)
organized drawings. drawings.
format.
Grammar There are no There are no There are no There are
more than two more than more than five or more
errors. three errors. four errors. errors.

14
CO_Q2_General Mathematics SHS
Module 11
Short Story
Very Needs
Good Fair
Category Good Improvement
(11–15) (6–10) (1–5)
(16–20)

Content: Originality of ideas;


Appropriateness of the ideas of
business and consumer loans;
Development of characters and plot
Other points of interest, e.g. use of
dialogue, point of view, irony,
symbolism, theme
Organization: Plot structure, e.g.
orientation, complication, climax,
resolution; Coherence and cohesion
of ideas; Paragraphing; Other
points of interest, e.g. flashback,
story within a story, transitions,
twist

Language and style: Range and


appropriateness of vocabulary; 
Range and accuracy of grammar
structures; Accuracy of spelling,
capitalization, punctuation;
Appropriateness of tone and
register ; Use of narrative writing
techniques, e.g. varying short and
long sentences, appealing to the five
senses, withholding details to
create suspense; Use of literary
devices, e.g. alliteration,
onomatopoeia, simile, metaphor,
personification

Jingle/Rap Making
4 3 2 1
Lyrics The lyrics The lyrics are The lyrics are The lyrics
are catchy catchy and somewhat are not
and he/she he/she catchy and catchy and
discusses discusses the he/she lack the
the concepts concepts of discusses discussion
of business business and some the of the
and consumer concepts of concepts of
consumer loans well. business and business

15
CO_Q2_General Mathematics SHS
Module 11
loans well consumer and
and loans. consumer
complete. loans.
Melody Contains 3 Contains 2 Contains 1 Lack of
or more qualities of quality of an qualities of
qualities of an effective effective an effective
an effective jingle/rap: jingle/rap: jingle/rap:
jingle/rap: short, short, short,
short, catchy, catchy, catchy,
catchy, simple or simple or simple or
simple or repetitive. repetitive. repetitive.
repetitive.
Presentation Excellent Some Little Lacks
creativity; creativity; creativity; creativity;
very well most parts few parts not well
through out appeared to appeared to through out
have been be well
well throughout.
throughout.

Assessment

Directions. Choose the letter of the best answer and write it on a separate
sheet of paper.

1. What do you call the loan that intends to spend for personal, family or
household purposes?

a. business loan

b. collateral

c. consumer loan

d. mortgage

16
CO_Q2_General Mathematics SHS
Module 11
2. What do you call the loan that is usually made to fund a company’s
operating expenses and other financial needs?

a. business loan

b. collateral

c. consumer loan

d. mortgage

3. What term is given to a person who guarantees to pay for someone else’s
financial obligation if the borrowers fail to do so?

a. amortization

b. collateral

c. guarantor

d. mortgage

4. Which of the following is an example of a business loan?

a. auto loans

b. credit cards

c. mezzanine financing

d. mortgages

5. Which of the following is an example of a consumer loan?

a. asset-based financing

b. car loans

c. invoice financing

d. micro loans

17
CO_Q2_General Mathematics SHS
Module 11
6. Which of the following statement about the interest rate of consumer and

business loans is correct?

a. The business loan has higher interest than a consumer loan.

b. Consumer loan has a higher interest rate than a business loan.

c. Consumer and business loans are always equal in interest rates.

d. The business loan has no interest rate, unlike consumer loan.

7. Which of the following statement about terms of payment for consumer

and business loans is correct?

a. The business loan has a longer term of payment than a consumer


loan.

b. Consumer loan has a longer term of payment than a business loan.

c. Consumer and business loans are always equal in terms of payment.

d. Consumer loan has no specific term of payment, unlike a business


loan.

8. Which of the following is not a required document if an employee is applying

for a consumer loan?

a. certificate of employment

b. company’s financial statement

c. credit report

d. employee’s pay slip

For 9-15. Identify the following situations and write a if it is a business loan,
b if the situation is a consumer loan, c if the situation is both a business loan
and a consumer loan, or d if the situation is neither a business loan nor a
consumer loan.

18
CO_Q2_General Mathematics SHS
Module 11
____9. Mr. Enriquez owns a lemon juice cart business. He wants to put
another juice cart on a new mall in the other city. He decided to have
a loan to establish a new business.

____10. Mr. Gozon wants to have another branch of his barbershop. He


decided to apply for a loan amounting to ₱100,000.00 that he can
use to pay for the rentals of the new branch.

____11. Andrew has a computer shop. He owns 10 computers. He decided to


borrow some money from the bank to buy 10 more computers.

____12. Ann decided to take her family for a vacation in Palawan, Cebu and

Bohol. To cover the expenses, she decided to apply for a loan

amounting to ₱100,000.00

____13. Mrs. Amor runs a recruitment agency. She wants to put up another

office tin the province. She applied for a loan in a bank worth

₱2,000,000.00

____14. Mr. Agoncillo renovated his house for Php 90,000. This was made
possible because of an approved loan worth ₱85,000.00

____15. Mark decided to apply for a car loan since his work is very far from
his residence. He got a car loan worth ₱1,500,000.00

19
CO_Q2_General Mathematics SHS
Module 11
Additional Activities

If ever you come across in either business or consumer loans, give your
thoughts on the following statements showing how to manage loans effectively
(Security Bank Team).

1. Borrow for the right reasons.


___________________________________________________________________________
___________________________________________________________________________
___________________________________________________________________________
_________.

2. Borrow only for what you need.


___________________________________________________________________________
___________________________________________________________________________
___________________________________________________________________________
_________.

3. Know how much you can afford to pay.


___________________________________________________________________________
___________________________________________________________________________
___________________________________________________________________________
_________.

4. Choose the right loan facility.


___________________________________________________________________________
___________________________________________________________________________
___________________________________________________________________________
_________.

5. Monitor your cash flow, specially your spending.


___________________________________________________________________________
___________________________________________________________________________
___________________________________________________________________________
_________.

20
CO_Q2_General Mathematics SHS
Module 11
Module 11
CO_Q2_General Mathematics SHS
21
What is in
1. LOAN 2. INTEREST 3. PRINCIPAL 4. RATE 5. TERM
What's More What I Know Assessment
1. a 1. c
Independent Practice 2. b 2. a
3. d 3. c
1. T 4. c
4. c
2. Consumer Loan 5. c 5. b
3. Collateral 6. b 6. a
4. Lower 7. b
7. a
5. T 8. b
8. a
9. b 9. a
Independent Assessment 10. a 10. a
1. a 11. a
11. a
2. b 12. b
12. a
3. b 13. b 13. a
4. b 14. b 14. b
5. a 15. b
15. b
6. b
7. a
8. a
9. a
10. b
Answer Key
References
Books:
General Mathematics Learner's Material. Pasig City, Philippines: Department of Education -
Bureau of Learning Resources, 2016.

Orines, Fernando B. Next Century Mathematics 11 General Mathematics. Quezon City:


Phoenix Publishing House, 2016.

Oronce, Orlando, A. General Mathematics, 1st Edition. Sampaloc, Manila: Rex Book Store,
2016.

Online Sources:

https://www.slideshare.net/reycastro1/basic-concept-of-business-and-
consumer-loans?from_action=save

http://www.dscoopemea.org/consumer-loan-and-business-loan-how-are-
they-different

https://www.securitybank.com/blog/how-to-manage-debts-loans/

https://www.edb.gov.hk/attachment/en/curriculum-
development/resource-support/net/assessment

https://www.scribd.com/doc/252358606/Writing-a-Jingle-Rubric-2

22
CO_Q2_General Mathematics SHS
Module 11
For inquiries or feedback, please write or call:

Department of Education - Bureau of Learning Resources (DepEd-BLR)

Ground Floor, Bonifacio Bldg., DepEd Complex


Meralco Avenue, Pasig City, Philippines 1600

Telefax: (632) 8634-1072; 8634-1054; 8631-4985

Email Address: blr.lrqad@deped.gov.ph * blr.lrpd@deped.gov.ph


General Mathematics
Quarter 2 – Module 12:
Solving Problems on Business
and Consumer Loans

CO_Q2_General Mathematic SHS


Module 12
General Mathematics
Alternative Delivery Mode
Quarter 2 – Module 12: Solving Problems on Business and Consumer Loans
First Edition, 2021

Republic Act 8293, section 176 states that: No copyright shall subsist in any work of
the Government of the Philippines. However, prior approval of the government agency or office
wherein the work is created shall be necessary for exploitation of such work for profit. Such
agency or office may, among other things, impose as a condition the payment of royalties.

Borrowed materials (i.e., songs, stories, poems, pictures, photos, brand names,
trademarks, etc.) included in this module are owned by their respective copyright holders.
Every effort has been exerted to locate and seek permission to use these materials from their
respective copyright owners. The publisher and authors do not represent nor claim ownership
over them.

Published by the Department of Education


Secretary: Leonor Magtolis Briones
Undersecretary: Diosdado M. San Antonio

Development Team of the Module


Writers: Bayani A. Quitain
Editors: Elizabeth D. Lalunio, Anicia J. Villaruel, Roy O. Natividad
Reviewers: Jerry Punongbayan, Diosmar F. Fernandez, Celestina M. Alba,
Shielamarie E. Arce, Carmela Ana A. Reforma, Rafaela M. Merle
Illustrator: Hanna Lorraine Luna, Diane C. Jupiter
Layout Artist: Roy O. Natividad, Sayre M. Dialola, Argie L. Ty, Glynis P. Aviles
Management Team: Francis Cesar B. Bringas
Job S. Zape, Jr.
Ramonito Elumbaring
Reicon C. Condes
Elaine T. Balaogan
Fe M. Ong-ongowan
Elias A. Alicaya Jr.
Gregorio A. Co Jr.
Gregorio T. Mueco
Herbert D. Perez
Lorena S. Walangsumbat
Jee-Ann O. Borines
Asuncion C. Ilao

Printed in the Philippines by ________________________

Department of Education – Region 4A CALABARZON

Office Address: Gate 2 Karangalan Village, Brgy. San Isidro, Cainta, Rizal
Telefax: 02-8682-5773/8684-4914/8647-7487
E-mail Address: lrmd.calabarzon@deped.gov.ph
General Mathematics
Quarter 2 – Module 12:
Problems on Business and
Consumer Loans
Introductory Message
This Self-Learning Module (SLM) is prepared so that you, our dear learners,
can continue your studies and learn while at home. Activities, questions, directions,
exercises, and discussions are carefully stated for you to understand each lesson.

Each SLM is composed of different parts. Each part shall guide you step-by-
step as you discover and understand the lesson prepared for you.

Pre-tests are provided to measure your prior knowledge on lessons in each


SLM. This will tell you if you need to proceed on completing this module or if you
need to ask your facilitator or your teacher’s assistance for better understanding of
the lesson. At the end of each module, you need to answer the post-test to self-check
your learning. Answer keys are provided for each activity and test. We trust that you
will be honest in using these.

In addition to the material in the main text, Notes to the Teacher are also
provided to our facilitators and parents for strategies and reminders on how they can
best help you on your home-based learning.

Please use this module with care. Do not put unnecessary marks on any part
of this SLM. Use a separate sheet of paper in answering the exercises and tests. And
read the instructions carefully before performing each task.

If you have any questions in using this SLM or any difficulty in answering the
tasks in this module, do not hesitate to consult your teacher or facilitator.

Thank you.
What I Need to Know

In previous lessons, you learned how to solve simple and compound interests. In the
most recent lesson, you studied the basic concepts of business and consumer loans.
You understand definitions such as annuities, interests, loans and collaterals. In
this lesson, you will study the application of business and consumer loans in real-
life situations. To better appreciate this module let us first consider the following
situations:
(a) Mario came from a poor family. His parents have no stable jobs. But due to
Mario’s determination, he excelled and was able to graduate college. After
being permanent for 2 years in his work, Mario dreamt of providing a simple
and decent shelter for his family. What do you think Mario should do?
(b) Cathy finished Senior High School in her hometown. She took a TVL strand in
dressmaking, However, because of financial problems she was not able to
continue in college. She decided to help her mother in their small business of
dressmaking. Their business increased in production due to her hardwork.
Subsequently, Cathy decided to venture in a larger scale and wished to export
their Filipiniana-style products. But this would need a bigger capital. What do
you think Cathy should do?
(c) During his STEM high school years, Albert was fond of experimenting on RC
motors. In his engineering years in college, he creatively thought of
experimenting about perpetual motion that could provide energy to machines.
However, he was short of finances to start this project which is primarily
Filipino-made. What do you think Albert should do?
Well, you should probably answer all three situations by finding money. Generally,
it is correct. But what are the options to do that? One of the options can be found in
this module. So, study diligently this module because these are all situations in
which you might find yourself in the future. So, by studying this module you may
be given an idea of different decisions you can make to fulfill your dream.

After going through this module, you are expected to solve problems on business and
consumer loans (amortization and mortgage).

1 CO_Q2_General Mathematic SHS


Module 12
What I Know

Let’s find out how far you might already know about this topic! Please take this
challenge! Have Fun!

Read and analyze each item carefully. Choose the letter that corresponds to your
answer for each statement and write it on a separate answer sheet.

1. It is a loan from a bank or other financial institution that helps a borrower


purchase a home.
a. collateral b. mortgage c. mortgagee d. balance

2. It is the process of paying off debt with regular payments made over time.
a. capital b. insurance c. mortgage d. amortization

3. The mortgagor is the one who _______ a mortgage.


a. borrows b. lends c. gives d. transacts

4. The remaining debt at a specified time is called _____________.


a. outstanding balance c. consumer debt
b. foreign debt d. maintaining balance

5. Fill in the blank to make the statement true:


It is your _________ to repay a mortgage loan when you buy a house.
a. duty b. responsibility c. obligation d. task

6. Mr. Santos borrowed ₱1,500,000.00 to be repaid in full after 5 years with an


interest rate of 6% per annum. What is the amount of the principal?
a. 5 b. 6% c. ₱2,007,338.00 d. ₱1,500,000.00

7. For the purchase of his car worth ₱400,000.00, Nilo requested the buyer Lito to
pay him 30% down payment. How much will Lito pay Nilo for the down
payment?
a. ₱100,000.00 b. ₱280,000.00 c. ₱120,000.00 d. ₱150,000.00

8. A student loan of ₱50,000.00 was applied by Rica to support her final year in
college payable in 60 consecutive months after her graduation. In how many
years will she need to pay fully her student loan?
a. 3 years b. 5 years c. 7 years d. 10 years

9. Magda purchased a condominium worth ₱2,000,000.00. However, the bank


demands a 25% down payment. How much is the mortgaged amount?
a. ₱500,000.00 b. ₱1,500,000.00 c. ₱2,000,000.00 d. ₱2,500,000.00

2 CO_Q2_General Mathematic SHS


Module 12
10. A motorcycle seller offers John a 12% down payment. How much would be the
mortgaged amount if the motorcycle is sellable at ₱120,000.00?
a. ₱300,000.00 b. ₱225,600.00 c. ₱200,000.00 d. ₱105,600.00

11. Anita borrowed ₱500,000.00 from the bank to renovate her house. The
effective rate of interest is 6%. How much is to be paid if the loan is to be paid
in 3 years?
a. ₱595,508.00 b. ₱530,000.00 c. ₱470,000.00 d. ₱561,800.00

12. Belle applied in a financing company for a ₱200,000.00 worth of business loan
to expand her computer business. Suppose she is required by the company to
pay this in 3 years, what is the amount to be paid if the effective rate of interest
is 7%
a. ₱171,735.00 b. ₱228,980.00 c. ₱245,008.60 d. ₱224,720.00

13. Mr. Arevalo borrowed from a bank to purchase a car worth ₱1,200,000.00. If
he has to pay ₱30,000.00 monthly for 5 years, how much will he pay for the
total interest?
a. ₱600,000.00 b. ₱500,000.00 c. ₱400,000.00 d. ₱300,000.00

14. Ms. Binibini obtained a loan of ₱100,000.00 for a 5-year mortgage. How much
will be its total interest if his monthly payment is ₱5,000.00?
a. ₱400,000.00 b. ₱200,000.00 c. ₱500,000.00 d. ₱300,000.00

15. Aling Puring, through VIP Financing Inc, purchased a building for her new
merchandise. It costs ₱2,500,000.00. She paid the financing company
₱500,000.00 as down payment. She plans to pay the remaining balance by
paying monthly for 10 years with an interest rate of 12% monthly. How much
would be her monthly payment?
a. ₱28,000.50 b. ₱50,000.00 c. ₱30,000.00 d. ₱28,694.19

3 CO_Q2_General Mathematic SHS


Module 12
Lesson Solving Problems on
1 Business and Consumer
Loans
In your previous lesson, you learned about the basic concepts of business and
consumer loans. In that module, you were able to learn the definition of different
terms such as consumer loans, business loans, collaterals, etc. In this module, you
are going to expand and apply that knowledge in solving real-life situations
concerning business and consumer loans, specifically, amortization and mortgage.
This may seem new to you but as you go along through this module, it would be
easier for you to sense that the contents of this module have wider applications in
our daily lives. Basic definitions of amortization and mortgage, as well as mortgagee
and mortgagor, will be tackled in this module. Problem-solving that entails the
calculation of mortgage amount, total amount of interest, monthly payments,
outstanding balance, are just some of the keywords that you will encounter in this
module. So, relax and focus and who knows, you may be able to use this in the near
future.

The lesson and activities in this module will explain what you have to know about
real-life situations concerning consumer and business loans.

After going through this module, you are expected to solve problems on business and
consumer loans (amortization and mortgage).

What’s In

In your most recent lesson, you learned the basics of consumer and business loans.
Moreover, in previous lessons, you were taught how to calculate simple and
compound interests as well as annuities. As a review, ready yourself in doing this
first activity.

Activity 1
Based on your knowledge of the previous lessons, try solving the following problems:
1. How much interest is charged when ₱100,000.00 is borrowed for 6 months at
an annual simple interest rate of 12%?
2. Find the maturity value and the compound interest if ₱15,000 is compounded
annually at an interest rate of 3% in 3 years.
3. Rina started to deposit ₱3,000.00 quarterly in a fund that pays 2%
compounded quarterly. How much will be in the fund after 5 years?

4 CO_Q2_General Mathematic SHS


Module 12
Activity 2
Identify which of the following statements belong to either consumer or business
loans. Write the number of each statement to its corresponding column inside the
table below.
1. Maria wants to borrow money from a financing agency for her college
education.
2. Mang Pedro would like to improve his auto-repair shop. He went to the bank
to borrow money to finance his plan to buy some equipment.
3. Rico finds it hard to travel to and from his workplace. So, he filed a loan from
a bank for him to purchase a motorcycle.
4. Mr. Ramsy would like to open a computer shop, but he was short of capital.
So, he went to the bank and file for a loan to purchase computers and other
equipment.
Consumer Loans Business Loans

What’s New

Most people think about the security and comfort of their families. This security and
comfort include the provision of a house and in case of a far workplace by having a
personal car or motorcycle. However, a large amount of money is needed to buy them
in straight cash. Recently, the government implemented a program called the Balik-
Probinsya, Balik-Pag-asa program that will provide resources for those who will be
coming back to provinces from Manila to start a new life. This may mean having
something to provide for the stability of a family such as a house or maybe a
motorcycle in less amount. A house to provide for the family shelter while a
motorcycle may be necessary as a means of transportation to report to work. But
despite the help the government may provide, this still seems difficult to have if you
have no cash on hand. So, you will resort to what we call a loan. In this module, we
will call it a mortgage.

Questions:
1. What do you understand about the word “loan” or “mortgage”?
___________________________________________________________________________
2. Do you think anyone can file for a loan from a bank or any financial
institution? _______________________________________________________________
3. Enumerate some government institutions or private agencies wherein you can
apply for a mortgage of a house or any vehicle?
___________________________________________________________________________

5 CO_Q2_General Mathematic SHS


Module 12
4. Which do you think is more proper: to buy a motorcycle, using a
straight cash or using a mortgage? Why?
_____________________________________________________________________
5. Do you think it is appropriate to pay an obligation such as a loan or
mortgage? Why?
_____________________________________________________________________

What is It

You have noticed from the beginning of this module that you were given situations
that have real-life applications. Again, this module is about solving problems
involving consumer and business loans. A loan is something borrowed, especially a
certain amount, that you are obligated to pay fully at a certain period with interest.
So, for instance a bank may lend you a capital of ₱500,000.00 for your intended
business but you must pay it fully at a certain period of time for instance in 3 years
with 12% interest per annum. The following examples will help you with amortization
and mortgage. But before that, let us learn some terms which can guide you as you
study this module:

Amortization is the process of paying off debt with regular payments made over time.
Mortgage is a loan from a bank or other financial institutions that help a borrower
purchase a home or a car.
Mortgagor is the one who borrows a mortgage.
Mortgagee is the one who lends a mortgage.
Collaterals are assets that can secure a loan.
Fixed-rate mortgage is a type of mortgage where interest remains constant.
Obligation is what parties may do under a contract or terms of agreement.
Outstanding balance is any remaining debt at a specified time.

Let us now go to the computation process.

Example 1
Mario came from a poor family. His parents have no stable jobs. But due to Mario’s
determination, he excelled and was able to graduate college. After being permanent
for 2 years in his work, Mario dreamt of providing a simple and decent shelter for his
family. He went to the bank and applied for ₱1,000,000.00 loan. The bank approved
his application with the following terms: 10% down payment, 12% total interest, and
must be paid in full after 5 years.
a. How much is his mortgaged amount?
b. How much should he have paid after 5 years?

6 CO_Q2_General Mathematic SHS


Module 12
Solution:
(a) Down payment = (down payment rate) x (amount of loan applied)
= (0.10) (₱1,000,000.00)
= ₱100,000.00
Thus,
Mortgaged amount = (amount of loan applied) – (down payment)
= ₱1,000,000.00 – ₱100,000.00
= ₱ 900,000.00

Therefore, Mario has a mortgaged amount of ₱900,000.00.

(b) Given P = ₱ 900,000.00, j = 0.12, n = 5


F = 𝑃𝑃(1 + 𝑗𝑗)𝑛𝑛 = 900,000(1+0.12)5 = ₱1,586,107.51

Therefore, Mario should have paid ₱1,586,107.51 after 5 years.

Example 2
Cathy finished Senior High School in her hometown. She took a TVL strand in
dressmaking, However, because of financial incapacity, she was not able to continue
in college. She decided to help her mother in their small business of dressmaking.
Their business increased in production due to her hard work. Subsequently, Cathy
decided to venture on a larger scale and wished to export their Filipiniana-style
products. But this would need a bigger capital. So, she decided to go to a financing
institution, Phiram Inc., to borrow ₱500,000.00. If her monthly payment was
₱12,000.00 on a 5-year mortgage, how much was the total amount of interest?

Solution:
Given P = ₱500,000.00, monthly payment = ₱12,000.00
Total Amount = (₱12,000.00) (12 months) (5 years) = ₱720,000.00
Total Amount of Interest = Total Amount – Amount of mortgage
= ₱720,000.00 – ₱500,000.00 = ₱220,000.00
Therefore, the total amount of interest is ₱220,000.00

Example 3

During his STEM high school years, Albert was fond of experimenting on RC motors.
In his engineering years in college, he creatively thought of experimenting about
perpetual motion that could provide energy to machines. However, he was short of
finances to start this project which is primarily Filipino-made. AG Resibo Company
approached Albert, and offered to lend him a capital of ₱2,000,000.00 to be paid in
5 years with 10% monthly interest. Moreover, there is an outright deduction of
100,000 from his applied loan.
(a) How much is his monthly payment?

7 CO_Q2_General Mathematic SHS


Module 12
Solution:
𝑖𝑖 12 0.10
(a) Given P = ₱1,900,000.00 i12 = 0.10 j= = = .00833;
12 12
n = mt = (12)(5) = 60

Use the formula:


1−(1+𝑗𝑗)−𝑛𝑛
P=R[ ]
𝑗𝑗
In finding R, Regular (monthly) payment, we use:
1−(1+𝑗𝑗)−𝑛𝑛
R=P/[ ]
𝑗𝑗
By substitution we have,
1−(1+.00833)−60
R = ₱1,900,000.00 / [ ]
.00833
R = ₱40,365.65
Therefore, the monthly payment is ₱40,365.65.

What’s More

Read each problem carefully and answer each question to solve the problem.

Activity 1.1
1. A business loan of ₱200,000 is to be repaid in full after 2 years. What is the
amount to be paid if the effective rate of interest is 10%? (Hint: F = 𝑃𝑃(1 + 𝑗𝑗)𝑛𝑛 )
2. Johna purchased a condominium worth ₱3,500,000. However, the bank
demands a 10% down payment. How much is the mortgaged amount?
3. A motorcycle seller requires Justo ₱5,000 down payment. How much would
be the mortgaged amount if the motorcycle is sellable at ₱120,000?

Activity 1.2
Solenn decided to venture in an online business and opted to go to a financing
institution to borrow ₱50,000. If her monthly payment is ₱1,500 on a 5-year
mortgage, how much is the total amount of interest?

Activity 1.3
PC Yaw Inc. approached Rudy and offered to lend him a capital for his computer
business amounting to ₱300,000 to be paid in 3 years with 12% monthly interest.
How much would be his monthly payment?

8 CO_Q2_General Mathematic SHS


Module 12
What I Have Learned

A. Again, this module is about solving problems in consumer and business loans.
A loan is something borrowed, especially a certain amount, that you are
obligated to pay fully at a certain period with interest. Fill each blank with
correct term/s which are related to consumer and business loans.
____________ is the process of paying off debt with regular payments made over
time.
____________ is a loan from a bank or other financial institutions that helps a
borrower purchase a home or a car.
____________ is the one who borrows a mortgage.
____________ is the one who lends a mortgage.
____________ are assets that can secure a loan.

B. Enumerate the different formulas that you should know to solve problems
involving consumer and business loans.

What I Can Do

1. Create your own or similar real-life situation where consumer and business
loans are applied. Conduct a thorough search by looking for any Philippine
bank or institution that offer any loans. Attach the screenshot of your search.
2. In a bond paper, present the problem and write the current interest rate being
charged by your chosen bank or institution. Compute for the following:
(a) Mortgaged Amount (c) Total amount paid
(b) Monthly installment (d) Total amount of interest

Note that your grade will be according to the criteria below: Accuracy of research
data (50%), Accuracy of solution (40%), and Applicability to the current situation
(10%).

9 CO_Q2_General Mathematic SHS


Module 12
Assessment

Read and analyze each item carefully. Choose the letter that corresponds to your
answer for each statement and write it on a separate answer sheet.

1. These are assets used to secure a loan.


a. collateral c. mortgagee
b. mortgage d. balance

2. It is a type of mortgage where the interest remains constant throughout the term
of the loan.
a. capital c. fixed-rate mortgage
b. insurance d. amortization

3. The mortgagee is the one who _______ in a mortgage.


a. borrows c. gives
b. lends d. transacts

4. Ms. Cuyo borrowed a loan of ₱500,000.00 to be repaid in full after 3 years with
an interest rate of 12% per annum. What is its interest rate in decimal form?
a. 5000 c. 0.12%
b. 12% d. 0.12

5. Joey borrowed ₱50,000.00 from a financing institution. His final loan amount is
₱48,000.00 due to the deduction of the down payment. How much is the down
payment?
a. ₱1,000.00 c. ₱48,000.00
b. ₱2,000.00 d. ₱50,000.00

6. Mr. Santos borrowed ₱2,500,000.00 to be repaid in full after 10 years with an


interest rate of 10% per annum. What is the amount of the principal?
a. 10 c. ₱2,500,000.00
b. 10% d. ₱2,750,000.00

7. For the purchase of her motorcycle worth ₱200,000.00, Mila requested the buyer,
Lita, to pay him 25% down payment. How much will Lita pay Mila for the down
payment?
a. ₱50,000.00 c. ₱150,000.00
b. ₱100,000.00 d. ₱180,000.00

8. A student loan of ₱30,000.00 was applied by Bebot to support her final year in
college payable in 3 consecutive years after her graduation. In how many months
will she need to pay fully her student loan?
a. 12 months c. 36 months
b. 24 months d. 60 months

10 CO_Q2_General Mathematic SHS


Module 12
9. A ₱20,000,000.00 worth of beach resort was bought by Mr. Sy through ABC
Bank. However, the bank demands a 20% down payment. How much is the
mortgaged amount?
a. ₱400,000.00 c. ₱4,000,000.00
b. ₱800,000.00 d. ₱16,000,000.00

10. A car dealer offers Ms. Garcia a brand-new car worth ₱420,000.00 with a 15%
down payment. How much would be the mortgaged amount?
a. ₱100,000.00 c. ₱200,000.00
b. ₱125,600.00 d. ₱357,000.00

11. Patty borrowed ₱800,000.00 from the bank to develop her piggery. The effective
rate of interest is 9%. How much is to be paid if the loan is to be paid in 4 years?
a. ₱1,129,265.00 c. ₱1,223,089.00
b. ₱1,170,000.00 d. ₱1,361,800.00

12. A ₱600,000.00 worth of business loan was applied by Basilio from XYZ Bank to
expand his poultry business. Suppose he is required by the company to pay
this in 5 years, what is the amount to be paid if the effective rate of interest is
8%?
a. ₱746,265.00 c. ₱895,008.00
b. ₱881,596.80 d. ₱924,720.00

13. Mr. Alvarez borrowed from a bank to purchase a farm lot worth ₱1,800,000.00.
If he has to pay ₱20,000.00 monthly for 9 years, how much will he pay for the
total interest?
a. ₱320,000.00 c. ₱360,000.00
b. ₱340,000.00 d. ₱380,000.00

14. Ms. Diamante obtained from a financing company a loan of ₱3,000,000.00 to


expand her jewelry business for a 6-year mortgage. How much will be its total
interest if his monthly payment is ₱50,000.00?
a. ₱200,000.00 c. ₱4000,000.00
b. ₱300,000.00 d. ₱600,000.00

15. Mang Akoy, through Alang-Ala Financing Inc. (AFI), purchased a grocery store
for her spouse that costs ₱75,000.00. He paid AFI ₱10,000.00 as down
payment. He plans to pay the remaining balance by paying monthly for 3 years
with an interest rate of 7% monthly. How much would be his monthly payment?
a. ₱3,000.50 c. ₱2,632.33
b. ₱2,807.82 d. ₱2,007.21

11 CO_Q2_General Mathematic SHS


Module 12
Additional Activities

Solve the following:

1. A business loan worth ₱500,000.00 is to be repaid in quarterly installment in 3


years. How much is the quarterly payment if money is worth 10% converted
quarterly?

2. A newly married couple decided to buy a brand-new car. The net amount of the
loan is ₱800,000.00. They plan to amortize the loan in a monthly installment for 3
years, if money is worth 12% convertible monthly, how much is the monthly
installment?

3. A business loan worth ₱500,000.00 is to be repaid in quarterly installment in 3


years. How much is the quarterly payment if money is worth 10% converted
quarterly?

4. A newly married couple decided to buy a brand-new car. The net amount of the
loan is ₱800,000.00. They plan to amortize the loan un monthly installment for 3
years, if money is worth 12% convertible monthly, how much is the monthly
installment?

12 CO_Q2_General Mathematic SHS


Module 12
Module 12
CO_Q2_General Mathematic SHS 13
What I Know What's More Assessment
1. B Activity 1.1 1. A
2. D 2. C
3. A 1. ₱ 242,000 3. B
4. A 2. ₱ 3,150,000 4. D
5. C 3. ₱ 115,000 5. B
6. D 6. C
7. C Activity 1.2 7. A
8. B 1. ₱ 40,000 8. C
9. B 9. D
10.D Activity 1.3 10. D
11.A 1. ₱ 9,964.293 11. A
12.C 12. B
13.A 13. C
14.B 14. D
15.D 15. D
Answer Key
References
DepEd BLR. 2016. General Mathematics, first ed., DepEd Philippines

Khan Academy. 2020. Mortgage interest rates. https://www.khanacademy.org/


economics-finance-domain/core-finance/housing/mortgages-tutorial/v/
mortgage-interest-rates

14 CO_Q2_General Mathematic SHS


Module 12
For inquiries or feedback, please write or call:

Department of Education - Bureau of Learning Resources (DepEd-BLR)

Ground Floor, Bonifacio Bldg., DepEd Complex


Meralco Avenue, Pasig City, Philippines 1600

Telefax: (632) 8634-1072; 8634-1054; 8631-4985

Email Address: blr.lrqad@deped.gov.ph * blr.lrpd@deped.gov.ph


General Mathematics
Quarter 2 – Module 13:
Propositions

CO_Q2_General Mathematics SHS


Module 13
General Mathematics – Senior High School
Alternative Delivery Mode
Quarter 2 – Module 13: Propositions
First Edition, 2021

Republic Act 8293, section 176 states that: No copyright shall subsist in any work of
the Government of the Philippines. However, prior approval of the government agency or office
wherein the work is created shall be necessary for exploitation of such work for profit. Such
agency or office may, among other things, impose as a condition the payment of royalties.

Borrowed materials (i.e., songs, stories, poems, pictures, photos, brand names,
trademarks, etc.) included in this module are owned by their respective copyright holders.
Every effort has been exerted to locate and seek permission to use these materials from their
respective copyright owners. The publisher and authors do not represent nor claim ownership
over them.

Published by the Department of Education


Secretary: Leonor Magtolis Briones
Undersecretary: Diosdado M. San Antonio

Development Team of the Module


Writer: Azalea A. Gallano
Editors: Elizabeth D. Lalunio, Anicia J. Villaruel, Roy O. Natividad,
Reviewers: Jerry Punongbayan, Diosmar O. Fernandez, Dexter M. Valle, Edmund Actub,
Karen T. Salvanera and Moahna Aura M. Mancenido
Illustrators: Hanna Lorraine Luna, Diane C. Jupiter
Layout Artist: Roy O. Natividad, Sayre M. Dialola, Argie L. Ty and Janeth D. Bendal
Management Team: Francis Cesar B. Bringas
Job S. Zape, Jr.
Ramonito Elumbaring
Reicon C. Condes
Elaine T. Balaogan
Fe M. Ong-ongowan
Hermogenes M. Panganiban
Philip B. Gallendez
Josephine T. Natividad
Anicia J. Villaruel
Dexter M. Valle

Printed in the Philippines by ________________________

Department of Education – Region IV-A CALABARZON

Office Address: Gate 2 Karangalan Village, Brgy. San Isidro, Cainta, Rizal
Telefax: 02-8682-5773/8684-4914/8647-7487
E-mail Address: lrmd.calabarzon@deped.gov.ph
General Mathematics
Quarter 2 – Module 13:
Propositions
Introductory Message
This Self-Learning Module (SLM) is prepared so that you, our dear learners,
can continue your studies and learn while at home. Activities, questions, directions,
exercises, and discussions are carefully stated for you to understand each lesson.

Each SLM is composed of different parts. Each part shall guide you step-by-
step as you discover and understand the lesson prepared for you.

Pre-tests are provided to measure your prior knowledge on lessons in each


SLM. This will tell you if you need to proceed on completing this module or if you
need to ask your facilitator or your teacher’s assistance for better understanding of
the lesson. At the end of each module, you need to answer the post-test to self-check
your learning. Answer keys are provided for each activity and test. We trust that you
will be honest in using these.

In addition to the material in the main text, Notes to the Teacher are also
provided to our facilitators and parents for strategies and reminders on how they can
best help you on your home-based learning.

Please use this module with care. Do not put unnecessary marks on any part
of this SLM. Use a separate sheet of paper in answering the exercises and tests. And
read the instructions carefully before performing each task.

If you have any questions in using this SLM or any difficulty in answering the
tasks in this module, do not hesitate to consult your teacher or facilitator.

Thank you.
What I Need to Know

This module will help you understand the concepts of propositions. To be specific,
you will learn how to illustrate a proposition as you go through its pages.

The module is composed of one lesson, namely:


• Lesson 1 – Propositions

After going through this module, you are expected to:


1. define proposition;.
2. illustrate a proposition; and
3. identify whether a sentence is a proposition or not.

What I Know

Choose the letter of the best answer. Write the chosen letter on a separate sheet of
paper.

1. Which of the types of sentences qualifies a proposition?


a. declarative
b. imperative
c. exclamatory
d. interrogative

2. Which of the following denotes a proposition?


a. a bold letter
b. a small letter
c. a capital letter
d. an italicized letter

3. What is the truth value of a false proposition?


a. True
b. False
c. Either true or false
d. Cannot be determined

1 CO_Q2_General Mathematics SHS


Module 13
4. Which of the following shows a correct proposition?
a. a: Rest recharges.
b. b: Do not ever quit.
c. c: What does failure teach?
d. d: Stepping forward is a good start.

For items 5-10, identify the only proposition among the choices.

5.
a. Alas!
b. Come and see it.
c. Where do you come from?
d. It is a hot day yet it brings comfort.

6.
a. Really?
b. Awesome!
c. Calm down to avoid commotion.
d. Obedience is a good trait of a member.

7.
a. What a bright smile!
b. Is the laugh too loud?
c. Brush your teeth after eating.
d. Enamel covers the outer surface of our teeth.

8.
a. Don’t pick flowers.
b. Does nature teach us?
c. Try smelling a sampaguita flower.
d. Sampaguita is our national flower and is lovely.

9.
a. Mix them.
b. Which hue of green do you like?
c. You can make violet if you mix blue and red.
d. Can you give a pair of complementary colors?

10.
a. That is too much!
b. Take your meds on time.
c. Do I have to drink my medication?
d. If one is overdosed with drugs, then the body may be harmed.

2 CO_Q2_General Mathematics SHS


Module 13
11. Which makes “I am excited!” a non-proposition?

a. It is bias.
b. It is nonsense.
c. It is not declarative.
d. It is a false statement.

12. Which makes “The capital city of the Philippines is Manila.” a


proposition?
a. It is a true statement.
b. It is a simple sentence.
c. It involves proper nouns.
d. It begins with the predicate.

13. Is “Can I add soy sauce in Adobo?” a proposition?


a. No
b. Yes
c. Sometimes
d. Given is insufficient

14. Which does not make “You will get orange in a mixture of red and
blue.” a proposition?

a. It is false.
b. It is declarative.
c. It is a statement.
d. It gives a command.

15. What kind of proposition is “One half is greater than ¾.”?


a. True
b. False
c. Either true or false
d. Neither true nor false

3 CO_Q2_General Mathematics SHS


Module 13
Lesson

1 Propositions

Math involves logic. There is the logic in performing mathematical operations, in


playing chess, in making decisions, in communicating with peers and in many other
activities that we do. Actually, we may be using the principles and concepts of logic
in our daily tasks which may be unknowing to us sometimes. That is how the study
of logic may be significant to us.

To commence our discussion on logic, this lesson will introduce the key concepts of
propositions. In the end, learners are expected to demonstrate understanding of
these concepts.

What’s In

Determine what type of sentence is each of the following. On a space provided before
each number, write “D” if it is a declarative sentence, “E” if exclamatory sentence,
“Im” if imperative and, “In” if it is interrogative.

____ 1. Your eye is the lamp of your body.


____ 2. When did you last hug your parents?
____ 3. Admit your fault.
____ 4. Since you are what you eat, then your food can describe your health.
____ 5. Life is more than food, and the body is more than clothing.

4 CO_Q2_General Mathematics SHS


Module 13
What’s New

Fact or Bluff?
Before you go on, pause for a while and play this game with your family member or
peer. All you have to do is to read the listed sentences. The other player will decide
and tell you whether each sentence is a fact or a bluff. To make it more informative,
you may search about each given topic and tell the reasons why it is a fact or a bluff.

Sentences Answer
You are taller in the morning than in the evening. Fact
There are more right-handed people than left-handed ones. Fact
Sourness is a taste in the tip of the tongue. Bluff
There are more Chinese than American. Fact
Melatonin gives color to our skin. Bluff
Did you both enjoy the game? Try it with somebody else.

What is It

Proposition Defined
A Proposition is a declarative sentence that is either true or false, but not both.
A true proposition has a truth value of “true”, otherwise, its truth value is “false”.
Oftentimes, a small letter is used to denote a proposition. That is, the proposition
a: It is getting clearer.
may be read as
a is the proposition “It is getting clearer.”.
Moreover, a sequence of propositions may be denoted by 𝑝𝑝1 , 𝑝𝑝2 , 𝑝𝑝3 , …

Examples
Determine if each sentence is a proposition or not. If it is, give its truth value.
a: Grounding is beneficial to a person.
b: Forgive and forget.
c: Happy birthday!
d: There is no stronger than the heart of a volunteer.

5 CO_Q2_General Mathematics SHS


Module 13
p: 3! = 6/2
q: Never mind, I am just joking.
𝑝𝑝1 : If an individual is great, then there is a teacher behind.
𝑝𝑝2 : Either a person saves before spending, or one spends before saving.
𝑝𝑝3 : It is not shame to greet the utility worker the same way as with the
school principal.
𝑝𝑝4 : If a person is disabled, then he/she is entitled to obtain a PWD ID,
and if a person is entitled to obtain a PWD ID, then he/she is
disabled.

Solution
a: It is a declarative sentence that may either be true or false, so it is a proposition.
And, since research says it’s true, then its truth value is true.
b: It is an imperative sentence. Hence, it is not a proposition.
c: It is an exclamatory sentence which makes it not a proposition also.
d: It is a declarative sentence that may either be true or false. It is a proposition.
Most especially in cases of front liners, this is true. Otherwise proven to be
wrong, then it is false in that specific instance.
p: Translated to the verbal statement, this mathematical sentence is “three factorials
is equal to six divided by two”. It is a declarative sentence, so it is a proposition.
Meanwhile, since 3! = 6 and 6/2 = 3, then the proposition is false.
q: It is a declarative sentence, but be cautious. Notice that the statement cannot be
true nor false. If the statement is true, it cannot be because the speaker is telling
a joke. If it is false, it cannot be either because the person is speaking the truth.
Hence, it is not a proposition.
𝑝𝑝1 : It is a declarative sentence that may either be correct or not. It is therefore a
proposition. Every great person the world has known has a patient teacher or
coach, acknowledged or not, hence the proposition is true.
𝑝𝑝2 : It is a declarative sentence that may either be true or false which tells us that it
is a proposition. It is especially true if one earns money. It may be false if there is
none to spend or save.
𝑝𝑝3 : It is a declarative sentence that is absolutely true. Thus, it is a proposition.
𝑝𝑝4 : It is a declarative sentence which by law is true. It is a proposition.

6 CO_Q2_General Mathematics SHS


Module 13
What’s More

Activity 1.1
Consider the following sentences. Write P if it is a proposition, and NP if not.
1. a: The largest continent is Asia.
2. b: Do I need to wash my hands regularly?
3. c: Occipital is the part of our brain responsible for vision so we have
to be sure that it is taken care of.
4. d: Happy Birthday, Inay!
5. e: The longest bone in the body is the femur.

Activity 1.2
Tell whether each of the propositions below is true or false.
1. f: There are 10 years in a decade and 100 years in a century.
2. g: One of the primary colors is yellow.
3. h: Twenty-five is ¼ of a hundred.
4. i: You will get 15 when you subtract 30 from 50.
5. j: Fish is rich in protein and Vitamin D helps the body effectively absorb
calcium.

What I Have Learned

Fill in the blanks.

1. A proposition is a/an __________________ sentence.


2. It may either be __________________ or __________________, but not both.
3. It is denoted by a/an __________________ letter.
4. A true proposition has a truth value of __________________.
5. A false proposition has a truth value of __________________.

7 CO_Q2_General Mathematics SHS


Module 13
What I Can Do

Around the globe, the human race experienced an unforgettable turning event in the
year 2020. I, you and all the rest out there have in a way or another experienced the
effects of the pandemic COVID-19. Write an essay or propositions about your actual
experiences and your realizations in life while in this crisis.

Note the rubrics shown below which will be used in scoring your essay.
CRITERIA 3 2 1
The essay was The essay was The essay has no
written in an written in an style; no novel
Quality of writing
extraordinary style; interesting style; information and
(score x 3) very informative somewhat not organized
and well-organized informative and
organized
Grammar, usage With no spelling or With some With many spelling
and mechanics grammatical errors spelling or or grammatical
grammatical errors
(score x 2)
errors

8 CO_Q2_General Mathematics SHS


Module 13
Assessment

Choose the letter of the best answer. Write the chosen letter on a separate sheet of
paper.
1. Which of the following may never describe a proposition?
a. either true or false
b. declarative sentence
c. exclamatory sentence
d. a period at the end of the proposition

2. What is the truth value of a true proposition?


a. True
b. False
c. both true and false
d. cannot be determined

3. Which of the following denotes a proposition?


a. a bold letter
b. a small letter
c. a capital letter
d. an italicized letter

4. How is the proposition “f: Time is an independent variable.” correctly read?


a. “Time is an independent variable.”
b. f means “Time is an independent variable.”
c. Proposition f “Time is an independent variable.”
d. f is the proposition “Time is an independent variable.”

For items 5-10, identify the only proposition among the choices.

5.
a. Are there 60 seconds in a minute?
b. Has anybody seen the throne of God?
c. Light travels 186,000 miles per second!
d. Though God may be physically far from us, He is near when His
obedient servant prays

9 CO_Q2_General Mathematics SHS


Module 13
6.
a. Does it break your heart?
b. Be thankful for being alive.
c. Take courage and have a happy heart.
d. The heart pumps blood throughout the body and it keeps us alive.

7.
a. Don’t pollute the air.
b. The brain needs oxygen.
c. Be grateful for the priceless air we take in.
d. Have you thanked God for the air we breathe?

8.
a. Good job students!
b. Be proud of hard work.
c. Are you a working student?
d. When a learner studies well, opportunities knock later.

9.
a. Be truthful.
b. It’s correct!
c. Are you sure?
d. It is wonderful.

10.
a. What is your name?
b. Good day everybody!
c. The Philippines is located in East Asia.
d. Can we kill the virus by just washing our hands regularly?

11. Which makes “Can we eat takoyaki?” not an example of a proposition?


a. It is biased.
b. It is nonsense.
c. It is not declarative.
d. It is a false statement.

12. Which makes “Leap year occurs every five years” a proposition?
a. It involves time.
b. It is a true statement.
c. It is a false statement.
d. It is a simple sentence.

10 CO_Q2_General Mathematics SHS


Module 13
13. Is “Study well!” a proposition?
a. No
b. Yes
c. Sometimes
d. Given is insufficient

14. Which does not make “Do your homework.” a proposition?


a. It is true.
b. It is declarative.
c. It is a statement.
d. It gives a command.

15. What kind of proposition is “There are 15 eggs in a dozen”?


a. True
b. False
c. Either true or false
d. Neither true nor false

11 CO_Q2_General Mathematics SHS


Module 13
Additional Activities

Create an informative poster on healthy lifestyle. The poster should involve at least
three true propositions.

The following rubrics will be used to score your poster:


Criteria 3 2 1
Three or more One or two Common information
Content propositions are propositions are
(score x 4) informative and informative or novel
novel
The design has a Has design but no Has no theme, no
Layout theme; does not spoil clear theme; the text design; the text is
(score x 3) the content; the text is readable not readable
is clear and readable
Reference References are all Some references are No cited reference
(score x 3) cited cited

12 CO_Q2_General Mathematics SHS


Module 13
Module 13
CO_Q2_General Mathematics SHS 13
What I Know What's More Assessment
1. a Activity 1.1 1. c
2. b 1. P 2. a
3. b 2. NP 3. b
4. a 3. P 4. d
5. d 4. NP 5. d
6. d 5. P 6. d
7. d Activity 1.2 7. b
8. d 1. True 8. d
9. c 2. True 9. d
10. d 3. True 10. c
11. c 4. False 11. c
12. a 5. True 12. c
13. a 13. a
14. d 14. d
15. a 15. b
Answer Key
References
Frederick Vuvor, Obed Harrison. 2017. "Biomedical and Biotechnology Research
Journal. A study of the diurnal height changes among sample of adults aged
30 years and above." bmbtrj.org. Accessed February 24, 2019.
https://www.bmbtrj.org/article.asp?issn=2588-
9834;year=2017;volume=1;issue=2;spage=113;epage=119;aulast=Vuvor.

General Mathematics Learner’s Material. First Edition. 2016. p. 240-241

*DepED Material: General Mathematics Learner’s Material

14 CO_Q2_General Mathematics SHS


Module 13
For inquiries or feedback, please write or call:

Department of Education - Bureau of Learning Resources (DepEd-BLR)

Ground Floor, Bonifacio Bldg., DepEd Complex


Meralco Avenue, Pasig City, Philippines 1600

Telefax: (632) 8634-1072; 8634-1054; 8631-4985

Email Address: blr.lrqad@deped.gov.ph * blr.lrpd@deped.gov.ph


General Mathematics
Quarter 2 – Module 14:
Simple and Compound
Propositions

CO_Q2_General Mathematics SHS


Module 14
General Mathematics - Senior High School
Alternative Delivery Mode
Quarter 2 – Module 14: Simple and Compound Propositions
First Edition, 2021

Republic Act 8293, section 176 states that: No copyright shall subsist in any work of
the Government of the Philippines. However, prior approval of the government agency or office
wherein the work is created shall be necessary for exploitation of such work for profit. Such
agency or office may, among other things, impose as a condition the payment of royalties.

Borrowed materials (i.e., songs, stories, poems, pictures, photos, brand names,
trademarks, etc.) included in this module are owned by their respective copyright holders.
Every effort has been exerted to locate and seek permission to use these materials from their
respective copyright owners. The publisher and authors do not represent nor claim ownership
over them.

Published by the Department of Education


Secretary: Leonor Magtolis Briones
Undersecretary: Diosdado M. San Antonio

Development Team of the Module


Writer: Azalea A. Gallano
Editors: Elizabeth D. Lalunio, Anicia J. Villaruel, Roy O. Natividad
Reviewers: Jerry Punongbayan, Diosmar O. Fernandez, Dexter M. Valle, Mark Vincent
Pineda, Rowena M. Liu and Moahna Aura M. Mancenido
Illustrators: Hanna Lorraine Luna, Diane C. Jupiter
Layout Artist: Roy O. Natividad, Sayre M. Dialola, Argie L. Ty and Janeth D. Bendal
Management Team: Francis Cesar B. Bringas
Job S. Zape, Jr.
Ramonito Elumbaring
Reicon C. Condes
Elaine T. Balaogan
Fe M. Ong-ongowan
Hermogenes M. Panganiban
Philip B. Gallendez
Josephine T. Natividad
Anicia J. Villaruel
Dexter M. Valle

Printed in the Philippines by ________________________

Department of Education – Region IV-A CALABARZON

Office Address: Gate 2 Karangalan Village, Barangay San Isidro, Cainta, Rizal
Telefax: 02-8682-5773/8684-4914/8647-7487
E-mail Address: lrmd.calabarzon@deped.gov.ph
General Mathematics
Quarter 2 – Module 14:
Simple and Compound
Propositions
Introductory Message
This Self-Learning Module (SLM) is prepared so that you, our dear learners,
can continue your studies and learn while at home. Activities, questions, directions,
exercises, and discussions are carefully stated for you to understand each lesson.

Each SLM is composed of different parts. Each part shall guide you step-by-
step as you discover and understand the lesson prepared for you.

Pre-tests are provided to measure your prior knowledge on lessons in each


SLM. This will tell you if you need to proceed on completing this module or if you
need to ask your facilitator or your teacher’s assistance for better understanding of
the lesson. At the end of each module, you need to answer the post-test to self-check
your learning. Answer keys are provided for each activity and test. We trust that you
will be honest in using these.

In addition to the material in the main text, Notes to the Teacher are also
provided to our facilitators and parents for strategies and reminders on how they can
best help you on your home-based learning.

Please use this module with care. Do not put unnecessary marks on any part
of this SLM. Use a separate sheet of paper in answering the exercises and tests. And
read the instructions carefully before performing each task.

If you have any questions in using this SLM or any difficulty in answering the
tasks in this module, do not hesitate to consult your teacher or facilitator.

Thank you.
What I Need to Know

This module was prepared with you in mind. This will help you divide into two big
groups the propositions. That is, you will learn the difference between simple and
compound propositions.

The module is composed of one lesson only.


• Lesson 1 – Simple and Compound Propositions

After going through this module, you are expected to:


1. define simple and compound propositions; and
2. distinguish between simple and compound propositions.

What I Know

Choose the letter of the best answer. Write the chosen letter on a separate sheet of
paper.

1. It is a proposition that uses logical connectors.


a. simple proposition
b. logical proposition
c. compound proposition
d. connective proposition

2. Which of the following is not a logical connector?


a. or
b. can
c. and
d. if-then

3. It is a proposition that cannot be broken down into more than one proposition.
a. logical proposition
b. simple proposition
c. compound proposition
d. connective proposition

1 CO_Q2_General Mathematics SHS


Module 14
For numbers 4 to 8, choose the simple proposition among each group of options.
4.
a. Is the laugh too loud?
b. What a bright smile!
c. Brush your teeth after eating.
d. Enamel covers the outer surface of our teeth.
5.
a. Does nature teach us?
b. Try smelling a Sampaguita flower.
c. Sampaguita is our national flower.
d. Don’t pick flowers and follow the rules.
6.
a. Do not ever quit.
b. What does failure teach?
c. Stepping forward is a good start.
d. If rest recharges, then quitting is not an option.
7.
a. That’s wrong!
b. Review your arithmetic.
c. You will get 15 when you subtract 30 from 50.
d. Either a learner review and master the prerequisite skills or not
mastered competencies pile up.
8.
a. That is too much!
b. Do I have to drink my medication?
c. Medication should be taken on time.
d. If one overdoses with drugs, then the body may be harmed.

For numbers 9 to 13, choose the compound proposition.


9.
a. Finish!
b. Water seeks its level.
c. Come and join us or accompany them.
d. Health is wealth and lifestyle defines health.
10.
a. Do you see goodness in others?
b. Your eye is the lamp of your body.
c. Be mindful of the light that is in you.
d. If the eye is good, then the body is enlightened.
11.
a. Initiating to make amends is honorable.
b. Admit your fault and ask for forgiveness.
c. Have you experienced feeling light after swallowing your pride?
d. Either a good person will find a way to say sorry or one will initiate
forgiveness.

2 CO_Q2_General Mathematics SHS


Module 14
12.
a. Honor your parents.
b. Twenty-five is ¼ of a hundred.
c. One of the primary colors is yellow.
d. There are 10 years in a decade and 100 years in a century.
13.
a. Love your parents.
b. When did you last hug your parents?
c. Parents or guardians are a blessing from God.
d. Love of parents prolongs one’s life and disrespecting them shortens
days.

14. Given the simple propositions a: You are what you eat and b: Your food can
describe your health, what compound proposition can be made?
a. You are not what you eat.
b. Your food cannot describe your health.
c. You are what you eat but your food can describe your health.
d. If you are what you eat, then your food can describe your health.

15. What logical connector best fits in merging the simple propositions c: Life is
more than food and d: The body is more than clothing?
a. or
b. not
c. and
d. if then

3 CO_Q2_General Mathematics SHS


Module 14
Lesson
Simple and Compound
1 Propositions
So far, you have initial knowledge of propositions already through Module 13. At this
point, you may want to know more about propositions. To start with, propositions
may be divided into two big groups of simple propositions and compound
propositions. In this lesson, you are expected to demonstrate an understanding of
key concepts simple and compound propositions. You should also be able to
distinguish one from the other.

What’s In

First, let us review what you have learned in the previous module. Answer the
following with all confidence.

Identify each statement below as a simple sentence or compound sentence. Write S


if it a simple sentence and C if it is a compound sentence. If it is a compound
sentence, identify the conjunction used as well.
_________1. Best things in life are free.
_________2. It is not true that a good deed will give a bad result.
_________3. A happy heart is a medicine, but a sorrowful spirit weakens the bones.
_________4. Gone are days that people practice apartheid.
_________5. If bullying introduces a not so good background of the bully, then a
psychosocial intervention should be designed for these bullies.

Notes to the Teacher


Enable learners to perform each task or activity in this module.
Explanation for answers should be provided for exercises which will
not be successfully answered by the learners.

4 CO_Q2_General Mathematics SHS


Module 14
What’s New

Read the famous poem below by Rudyard Kipling, an Indian short story writer, poet
and novelist. See the wisdom in this poem.
If
by: Rudyard Kipling

If you can keep your head when all about you


Are losing theirs and blaming it on you,
If you can trust yourself when all men doubt you,
But make allowance for their doubting too;
If you can wait and not be tired by waiting,
Or being lied about, don’t deal in lies,
Or being hated, don’t give way to hating,
And yet don’t look too good, nor talk too wise:

If you can dream—and not make dreams your master;


If you can think—and not make thoughts your aim;
If you can meet with Triumph and Disaster
And treat those two impostors just the same;
If you can bear to hear the truth you’ve spoken
Twisted by knaves to make a trap for fools,
Or watch the things you gave your life to, broken,
And stoop and build ’em up with worn-out tools:

If you can make one heap of all your winnings


And risk it on one turn of pitch-and-toss,
And lose, and start again at your beginnings
And never breathe a word about your loss;
If you can force your heart and nerve and sinew
To serve your turn long after they are gone,
And so hold on when there is nothing in you
Except the Will which says to them: ‘Hold on!’

If you can talk with crowds and keep your virtue,


Or walk with Kings—nor lose the common touch,
If neither foes nor loving friends can hurt you,
If all men count with you, but none too much;
If you can fill the unforgiving minute
With sixty seconds’ worth of distance run,
Yours is the Earth and everything that’s in it,
And—which is more—you’ll be a Man, my son!
Do you like it? What is the most striking line of the poem for you?
Can you spot and identify as many propositions as you can in the poem?

5 CO_Q2_General Mathematics SHS


Module 14
What is It

Simple and Compound Propositions Defined


A Simple Proposition is a proposition that cannot be broken down into more than
one proposition. Otherwise, it is a Compound Proposition. The latter is a
proposition that is formed by joining simple propositions using logical connectors.
Given propositions p and/or q, some logical connectors may be expressed in terms
of the following:
not p
p and q
p or q
If p, then q
Example 1
In the previous module, we have understood that the following are propositions. This
time let us identify each of these as simple or compound proposition.
a: Grounding is beneficial to a person.
d: There is no stronger than the heart of a volunteer.
p: 3! = 6/2
𝑝1 : If an individual is great, then there is a teacher behind.
𝑝2 : Either a person saves before spending, or one spends before saving.
𝑝3 : It is not a shame to greet the utility worker the same way as with
the school principal.
𝑝4 : If a person is disabled, then he/ she is entitled to obtain a PWD ID,
and if a person is entitled to obtain a PWD ID, then he/ she is
disabled.

Solution:
Notice that among the propositions above, propositions a, d and p are made up of
one declarative sentence each, while propositions 𝑝1 , 𝑝2 , 𝑝3 and 𝑝4 are composed of
more than one declarative sentence.
This informs us that propositions a, d and p are all simple propositions, while
propositions 𝑝1 , 𝑝2 , 𝑝3 and 𝑝4 are compound propositions.

Example 2
Identify the simple components of the compound propositions in Example 1 and the
corresponding symbols using the logical connectors used.

6 CO_Q2_General Mathematics SHS


Module 14
Solution:
Proposition/s with
Proposition Simple Component/s
Logical Connector/s
j: An individual is great.
𝑝1 If j, then x.
x: There is a teacher behind.
o: A person saves before spending.
𝑝2 o or k
k: One spends before saving.
c: It is not a shame to greet the utility
𝑝3 worker the same way as with the school not c
principal.
y: A person is disabled.
𝑝4 z: A person is entitled to obtain a PWD if y then z and if z then y
ID.

What’s More

Activity 1.1
In the blank before each number, write SP, CP, or N if it is a simple proposition,
compound proposition or not a proposition, respectively. Furthermore, identify the
logical connector/s used if it is a compound proposition.

_______________ 1. If bad company ruins good morals, then one should be mindful
in choosing friends.
_______________ 2. Essential oils do not expire.
_______________ 3. Ponder on the sacrifices of your parents.
_______________ 4. If you study hard, then you will have good grades.
_______________ 5. Have you thought of the sick when you are spending too much?
_______________ 6. In tossing a fair coin once you may get a head or a tail.
_______________ 7. There is wisdom in spending for needs and it makes sense to
think at least twice before giving in for wants.
_______________ 8. Two lines are parallel if and only if they are coplanar and do not
intersect.
_______________ 9. Study hard and rest well.
_______________ 10. Camber is not telling the truth.

7 CO_Q2_General Mathematics SHS


Module 14
Activity 1.2
Identify the simple propositions comprising each of the compound propositions
below. Translate it as well into symbols using the logical connector used in each
proposition.

Simple Components Logical


Compound Proposition
(𝒑𝟏 𝒂𝒏𝒅 𝒑𝟐 ) Connector
1. It is not proper to be too
confident.
2. If complacency is deceitful, then
preparation is necessary.
3. Fifty is half of a hundred and a
hundred is a tenth of a
thousand.
4. A person sacrifices before
attaining success or failure
follows one’s slackness.
5. Fish is rich in protein and
Vitamin D helps the body
effectively absorb Calcium.

What I Have Learned

Answer the following questions.


1. What is a simple proposition?
2. What is a compound proposition?
3. Can you name the logical connectors tackled in this lesson?

8 CO_Q2_General Mathematics SHS


Module 14
What I Can Do

The lesson enables you to differentiate a proposition from not. It further teaches you
to categorize a proposition into a simple proposition or compound proposition. You
understood that logical connectors help in forming a compound proposition. In
relation to this, suppose your group is tasked to build a Ferris wheel miniature and
to determine the equation of the function it obeys. As a member of a five-person
group, which of these tasks will you take part in or not? And, why? Will it be doing
the research, contributing to the expenses, buying the materials, designing the
project, constructing the wheel, computing the equation or presenting the output?
Answer in just one sentence. Most possibly, your answer is a compound sentence
with conjunctions or those logical connectors. Write it on another sheet of paper.

Assessment

Choose the letter of the best answer. Write the chosen letter on a separate sheet of
paper.
1. Which describes a compound proposition?
a. It is neither true nor false.
b. It is an imperative sentence.
c. It is made up of simple propositions joined by logical connector/s.
d. It is a proposition that requires the use of prepositions to form logic.

2. Which of the following is not a logical connector?


a. or
b. and
c. would
d. if-then

For numbers 3 to 8, choose the only simple proposition among the options.
3.
a. Climb!
b. Have you climbed a tree?
c. Conserve our coconut trees.
d. The coconut tree is known as the “Tree of life”.

9 CO_Q2_General Mathematics SHS


Module 14
4.
a. It is so blurred!
b. Is that a dog or a cat?
c. Let me ask you for assistance.
d. The normal vision of a person is 20/20.
5.
a. Are there 60 seconds in a minute?
b. Has anybody seen the throne of God?
c. Light travels 186,000 miles per second!
d. Though God may be physically far from us, He is near when His
obedient servant prays.
6.
a. Does it break your heart?
b. It is wise to thank God for being alive.
c. Take courage and have a happy heart.
d. The heart pumps blood throughout the body and it keeps us alive.
7.
a. Don’t pollute the air.
b. The brain needs oxygen.
c. Be grateful for the priceless air we take in.
d. Have you thanked God for the air we breathe?

8.
a. What is your name?
b. Good day everybody!
c. The Philippines is located in East Asia.
d. Can we kill the virus by just washing our hands regularly?

For numbers 9 to 13, choose the only compound proposition among each group of
options.
9.
a. Are you sure?
b. You deserve a good break!
c. Busy, techy and frugal consumers tend to shop online.
d. If one is accountable for own decisions, then one should decide wisely.

10.
a. Time is up!
b. Man can outlive time.
c. Time is an independent variable and distance is a dependent one.
d. The clock keeps on ticking while a couch potato enjoys slackness.

10 CO_Q2_General Mathematics SHS


Module 14
11.
a. Good job students!
b. Be proud of hard work.
c. Are you a working student?
d. If a learner studies well, then opportunities knock later.

12.
a. It’s correct!
b. Are you sure?
c. Be truthful and be trustworthy.
d. It is wonderful and it is a good day.
13.
a. Have you seen it in pictures?
b. Paris’ Eiffel Tower is beautiful!
c. Either watch from afar or climb it.
d. Paris’ Eiffel Tower is named after its maker or it is named after the
place where it stands.

14. Given the simple propositions a: A person prioritizes own spiritual growth and b:
One satisfies own desires. what compound proposition can be made?
a. One does not satisfy own desires.
b. A person does not prioritize own spiritual growth.
c. A person prioritizes own spiritual growth or one satisfies own desires.
d. If a person prioritizes own spiritual growth, then one satisfies own
desires.

15. What logical connector best fits in merging the simple propositions c:
Money is a root of all evil and d: Spending for the needy is wise?
a. or
b. not
c. and
d. if then

11 CO_Q2_General Mathematics SHS


Module 14
Additional Activities

Consider the proposition a: Matmat is telling a joke. Decide whether it is a simple


proposition, compound proposition or neither. Expound your answer.
Criteria 3 2 1
Answer Type of proposition Type of proposition No identified
(score x 2) is correct is incorrect answer
Answer is Answer is
explained explained but some
completely; details are not
manifests touched; manifests
Explanation
understanding of understanding of No explanation
(score x 3)
concepts behind some concepts
the truthfulness of behind
answer truthfulness of
answer

12 CO_Q2_General Mathematics SHS


Module 14
Module 14
CO_Q2_General Mathematics SHS 13
What's More
Activity 1.1 Assessment
What I Know
1. CP, if then
1. c 2. CP, not 1. c
2. b 3. N 2. c
3. b 4. CP, if then 3. d
4. d 4. d
5. N
5. c 5. c
6. CP, or 6. b
6. c
7. CP, and 7. b
7. c
8. c 8. CP, if and only if 8. c
9. d 9. N 9. d
10. d 10. CP,not 10. c
11. d Activity 1.2 11. d
12. d 12. d
Logical 13. d
13. d Simple Components
Connector 14. c
14. d
15. c 𝑝1: It is not proper to 15. d
1. not 𝑝1
be too confident.
𝑝1: Complacency is
deceitful. if 𝑝1, then
2.
𝑝2: Preparation is 𝑝2
necessary.
𝑝1: Fifty is half of a
hundred.
3. 𝑝1 and 𝑝2
𝑝2: A hundred is a
tenth of a thousand.
𝑝1: A person sacrifices
before attaining
4. success. 𝑝1 or 𝑝2
𝑝2: Failure follows
one’s slackness.
𝑝1: Fish is rich in
protein.
5. 𝑝2: Vitamin D helps 𝑝1 and 𝑝2
body effectively absorb
Calcium.
Answer Key
References

Dimasuay, Lynie, Jeric Alcala, and Jane Palacio. General Mathematics: Learner's
Material. Philippines: C & E Publishing, Inc., 2016.

Poem, Family Friends. Family Friends Poem. 2006.


https://www.familyfriendpoems.com (accessed June 13, 2020).
General Mathematics Learner’s Material. First Edition. 2016. p. 242-245

*DepED Material: General Mathematics Learner’s Material

14 CO_Q2_General Mathematics SHS


Module 14
For inquiries or feedback, please write or call:

Department of Education - Bureau of Learning Resources (DepEd-BLR)

Ground Floor, Bonifacio Bldg., DepEd Complex


Meralco Avenue, Pasig City, Philippines 1600

Telefax: (632) 8634-1072; 8634-1054; 8631-4985

Email Address: blr.lrqad@deped.gov.ph * blr.lrpd@deped.gov.ph


General Mathematics
Quarter 2 – Module 15:
Logical Operators

CO_Q2_General Mathematics SHS


Module 15
General Mathematics – Senior High School
Alternative Delivery Mode
Quarter 2 – Module 15: Logical Operators
First Edition, 2021

Republic Act 8293, section 176 states that: No copyright shall subsist in any work of
the Government of the Philippines. However, prior approval of the government agency or office
wherein the work is created shall be necessary for exploitation of such work for profit. Such
agency or office may, among other things, impose as a condition the payment of royalties.

Borrowed materials (i.e., songs, stories, poems, pictures, photos, brand names,
trademarks, etc.) included in this module are owned by their respective copyright holders.
Every effort has been exerted to locate and seek permission to use these materials from their
respective copyright owners. The publisher and authors do not represent nor claim ownership
over them.

Published by the Department of Education


Secretary: Leonor Magtolis Briones
Undersecretary: Diosdado M. San Antonio

Development Team of the Module


Writer: Azalea A. Gallano
Editors: Elizabeth D. Lalunio, Anicia J. Villaruel, Roy O. Natividad
Reviewers: Jerry Punongbayan, Diosmar O. Fernandez, Dexter M. Valle, Daisy Manalo,
Lovely Chrisna A. Palma and Moahna Aura M. Mancenido
Illustrators: Hanna Lorraine Luna, Diane C. Jupiter and Meryll C. Calvendra
Layout Artist: Roy O. Natividad, Sayre M. Dialola, Argie L. Ty and Janeth D. Bendal
Management Team: Francis Cesar B. Bringas
Job S. Zape, Jr.
Ramonito Elumbaring
Reicon C. Condes
Elaine T. Balaogan
Fe M. Ong-ongowan
Hermogenes M. Panganiban
Philip B. Gallendez
Josephine T. Natividad
Anicia J. Villaruel
Dexter M. Valle

Printed in the Philippines by ________________________

Department of Education – Region IV-A CALABARZON

Office Address: Gate 2 Karangalan Village, Barangay San Isidro, Cainta, Rizal
Telefax: 02-8682-5773/8684-4914/8647-7487
E-mail Address: lrmd.calabarzon@deped.gov.ph
General Mathematics
Quarter 2 – Module 15:
Logical Operators
Introductory Message
This Self-Learning Module (SLM) is prepared so that you, our dear learners,
can continue your studies and learn while at home. Activities, questions, directions,
exercises, and discussions are carefully stated for you to understand each lesson.

Each SLM is composed of different parts. Each part shall guide you step-by-
step as you discover and understand the lesson prepared for you.

Pre-tests are provided to measure your prior knowledge on lessons in each


SLM. This will tell you if you need to proceed on completing this module or if you
need to ask your facilitator or your teacher’s assistance for better understanding of
the lesson. At the end of each module, you need to answer the post-test to self-check
your learning. Answer keys are provided for each activity and test. We trust that you
will be honest in using these.

In addition to the material in the main text, Notes to the Teacher are also
provided to our facilitators and parents for strategies and reminders on how they can
best help you on your home-based learning.

Please use this module with care. Do not put unnecessary marks on any part
of this SLM. Use a separate sheet of paper in answering the exercises and tests. And
read the instructions carefully before performing each task.

If you have any questions in using this SLM or any difficulty in answering the
tasks in this module, do not hesitate to consult your teacher or facilitator.

Thank you.
What I Need to Know

In this module, you will know the different operations involved in propositions,
especially in compound propositions. You will learn how to perform each of these
operations. Moreover, this will provide you guidance on the symbols of these
operations, their verbal equivalents, and how they are applied in the real world.

The module is composed of one lesson, namely:


• Lesson 1 – Logical Operators

After going through this module, you are expected to:


1. define the logical operators: negation, conjunction, disjunction, condition, and
biconditional;
2. distinguish the logical operator used in a proposition; and
3. perform different types of operations on propositions.

What I Know

Choose the letter of the best answer. Write the chosen letter on a separate sheet of
paper.
1. Which is not a logical operator?
a. conditional
b. biconditional
c. negation
d. binegation

2. Which of the following is not a logical connector?


a. or
b. can
c. if-then
d. and

3. It is a proposition that can be broken down into more than one proposition.
a. simple proposition
b. compound proposition
c. connective proposition
d. logical proposition

1 CO_Q2_General Mathematics SHS


Module 15
4. Which logical operator uses the connector “or”?
a. negation
b. conjunction
c. disjunction
d. conditional

5. Which denotes disjunction?


a. 𝑝 ∨ 𝑞
b. 𝑝 ∧ 𝑞
c. ~𝑝
d. 𝑝 ↔ 𝑞

For numbers 6 to 10, choose the logical operator used in each given proposition.

6. If a finite won’t fathom the Infinite, then faith must prevail.


a. biconditional
b. conditional
c. conjunction
d. negation

7. Outer beauty does not suggest inner beauty.


a. biconditional
b. conditional
c. disjunction
d. negation

8. Healthy Zade is either a vegan or an organic consumer.


a. biconditional
b. conditional
c. conjunction
d. disjunction

9. Not everyone will love you the way I did.


a. conditional
b. conjunction
c. disjunction
d. negation

10. One has peace of mind if and only if there is contentment.


a. biconditional
b. conditional
c. disjunction
d. negation

2 CO_Q2_General Mathematics SHS


Module 15
For numbers 11 to 15, consider the following propositions:
x: A silent prayer can be heard through a storm.
y: It is a clean heart that counts in prayer.
z: God who is far more than lightyears away listens.

11. Which is the symbol for the proposition “A silent prayer cannot be heard through
a storm”?
a. ∧ 𝑥
b. ~𝑥 → 𝑥
c. ~𝑥
d. ∨ 𝑥

12. Which is the symbol for the proposition “If it is a clean heart that counts
in prayer, then God who is far more than lightyears away listens”?
a. ~𝑧 ∨ 𝑦
b. 𝑦 ∧ 𝑧
c. 𝑦 → 𝑧
d. 𝑧 ↔ ~𝑦

13. Which is the symbol for “Either a silent prayer can be heard through
a storm or God who is far more than lightyears away listens”?
a. (𝑥 ∧ ~) ↔ 𝑧
b. 𝑥 → ~𝑧
c. 𝑥 ∧ ~𝑧
d. 𝑥 ∨ ~𝑧

14. Which is the verbal form of ~𝑦 ∧ ~𝑧?


a. It is a clean heart that counts in prayer and God who is far more than
lightyears away listens.
b. It is a clean heart that counts in prayer or God who is far more than
lightyears away listens.
c. It is not a clean heart that counts in prayer and God who is far more than
lightyears away does not listen.
d. It is not a clean heart that counts in prayer or God who is far more than
lightyears away does not listen.

15. Which is the verbal form of (𝑧 ∧ 𝑥) ↔ 𝑦?


a. God who is far more than lightyears away listens or a silent prayer can be
heard through a storm if and only if it is a clean heart that counts in
prayer.
b. God who is far more than lightyears away listens and silent prayer can be
heard through a storm if and only if it is a clean heart that counts in
prayer.
c. If God who is far more than lightyears away listens and silent prayer can
be heard through a storm, then it is a clean heart that counts in prayer.
d. If God who is far more than lightyears away listens or a silent prayer can
be heard through a storm, then it is a clean heart that counts in prayer.

3 CO_Q2_General Mathematics SHS


Module 15
Lesson

1 Logical Operators

Just like operations with integers, fractions, rational expressions, polynomials,


exponential functions, and the many other types of functions, there are operations
involved in propositions. That is what this module is all about. With knowledge on
propositions, you are now ready to understand the logical operators, their symbols,
and corresponding verbal equivalents. Ultimately, you will learn how to perform the
operations on propositions. Enjoy performing operations on nonnumerical symbols!

What’s In

For each sentence below, identify the conjunction/s used.


_______________1. It is not true that a good deed will give a bad result.
_______________2. A happy heart is a medicine and a sorrowful spirit
weakens the bones.
_______________3. Apartheid is only a thing of the past if and only if dark colored
people are treated the same way as with fair colored race.
_______________4. Every person sides with God or one willingly chooses evil.
_______________5. If bullying introduces a not so good background of the bully, then
a psychosocial intervention should be designed for these bullies.
_______________6. One can serve a master at a time, yet, nobody can serve two
masters simultaneously.
_______________7. Working hard pays off and the satisfaction it gives is priceless.
_______________8. It is a good deed to love friends but, it is divine to love enemies.
_______________9. A wise person cherishes advices and a fool despises corrections.
_______________10. If sharing inspirational stories relaxes a tired spirit, then keep
telling such.

4 CO_Q2_General Mathematics SHS


Module 15
What’s New

Complete the paragraph by changing the underlined letter with the appropriate
logical connector, then write the letter that corresponds to the correct logical
connector on the blanks next page.

Hey, dear student! I know you. Either you


are enrolled in a private school A you
are studying in a public school. I bet, it
does U matter. If you are in a private
school, E you should make the most of the
available resources you have. In like
manner, T you are in a public school, then
you should value the big benefit of enjoying
access to quality education for free. Victory
knows no place, Y any individual can be
successful. A person has to look at the
brighter side B one can grow wherever
he/she is planted. Truly, God is fair.
____ ____ ____ ____ ____ ____
and then or not if if and only if
Did the message sink in? I hope so. Mind you, that will do you good dear learner.
Now, you are very ready to learn a new lesson.

5 CO_Q2_General Mathematics SHS


Module 15
What is It

Logical Operators Enumerated


Let us consider logical connectors we join with propositions. In this part, we shall
deal with logical operators.
Logical operators include negation, conjunction, disjunction, conditional, and
biconditional. The definition of each of the logical operators with the aid of a truth
table and truth values are shown below

Negation Defined
The Negation of a proposition p is denoted by ~p which is read as “not p”, and is
defined through its truth table

𝑝 ~𝑝
T F
F T

Example 1
State the negation of each of the following propositions.
𝑛1 : Quality determines the price.
𝑛2 : A learned is one who is educated.
𝑛3 : 𝑓(𝑥) = 𝑥 2 is a cubic function.
𝑛4 : An obtuse angle measures 180𝑜 .
𝑛5 : A curve is the shortest distance between two points.

Solution:

~𝑛1 : It is not true that quality determines the price or ~𝑛1: Quality does not
determine the price.
~𝑛2 : A learned is not one who is educated.
~𝑛3 : 𝑓(𝑥) = 𝑥 2 is not a cubic function or ~𝑛3 : It is not true that f(x) = x2 is a cubic
function.
~𝑛4 : An obtuse angle does not measure or ~𝑛4 : It is not true that an obtuse angle
measures 180𝑜 .
~𝑛5 : A curve is not the shortest distance between two points.

6 CO_Q2_General Mathematics SHS


Module 15
Conjunction Defined

Another logical operator is the Conjunction of the propositions p and q which is


denoted by 𝑝 ∧ 𝑞 and is read as “p and q”. It is defined through its truth table

p q 𝑝 ∧ 𝑞
T T T
T F F
F T F
F F F

Proposition p and proposition q are called Conjuncts. The conjunction 𝑝 ∧ 𝑞 is true


only when both conjuncts p and q are true.

Example 2
Let d and e be propositions.
d: Leniency is long-suffering.
e: Those who misunderstand it abuse it.

Express the conjunctions below in verbal sentences or in symbols.


1. 𝑑 ∧ 𝑒
2. ~𝑑 ∧ 𝑒
3. “Leniency is long-suffering and it is not true that those who misunderstand it
abuse it.”
4. “Leniency is not long-suffering and those who misunderstand it do not abuse
it.”

Solution:

1. Leniency is long-suffering and those who misunderstand it abuse it.


2. Leniency is not long-suffering and those who misunderstand it abuse it.
3. 𝑑 ∧ (~𝑒)
4. ~𝑑 ∧ (~𝑒)

Example 3
Let r, s and t be propositions.
r: There is a profit in physical exercise.
s: It is best to exercise holiness.
t: A person must be holistically fit.

7 CO_Q2_General Mathematics SHS


Module 15
Express the conjunctions below in verbal sentences or in symbols.
1. There is a profit in physical exercise and it is best to exercise holiness.
2. There is no profit in physical exercise and a person must be holistically fit.

3. 𝑟 ∧ ~𝑡
4. ~𝑟 ∧ 𝑠

Solution:

1. 𝑟 ∧ 𝑠
2. ~𝑟 ∧ 𝑡
3. There is a profit in physical exercise and it is not true that a person must be
holistically fit.
4. There is no profit in physical exercise and it is best to exercise holiness.

Disjunction Defined

Negation and conjunction do not suffice logic. There are logical statements that
connote the disjunction of propositions. The Disjunction of propositions p and q is
denoted by 𝑝 ∨ 𝑞 which read as “𝑝 𝑜𝑟 𝑞”, and defined through its truth table

p q 𝑝 ∨ 𝑞
T T T
T F T
F T T
F F F

Propositions p and q are each called Disjunct. The disjunction 𝑝 ∨ 𝑞 is false only
when both disjuncts p and q are false.

Example 4
Let t, u and y be propositions.
t: He is an old soul.
u: Old songs soothe his ears.
y: Old fashion is inviting to his eyes.

8 CO_Q2_General Mathematics SHS


Module 15
Express the disjunctions below in verbal sentences or in symbols.
1. 𝑢 ∨ 𝑦
2. (~𝑡 ^ (~𝑢)) ∨ 𝑦
3. “Either he is an old soul or old songs soothe his ears and old fashion is inviting
to his eyes.”
4. “He is not an old soul and either old fashion is not inviting to his eyes or old
songs soothe his ears.”

Solution:
1. Old songs soothe his ears or old fashion is inviting to his eyes.
2. Either he is not an old soul and old songs do not soothe his ears or old fashion
is inviting to his eyes.
3. 𝑡 ∨ (𝑢 ∧ 𝑦)
4. ~𝑡 ∧ (~𝑦 ∨ 𝑢)

Example 5
Let j, k, l and m be propositions.
j: Laughter is the best medicine.
k: Grief causes illness.
l: Prayer relieves pain.
m: God takes charge of worries.

Express the disjunctions below in verbal sentences or in symbols.


1. It is either God takes charge of worries and laughter is the best medicine or
prayer relieves pain.
2. Laughter is not the best medicine and God takes charge of worries or prayer
relieves pain.
3. 𝑗 ∨ 𝑙
4. (𝑗 ∧ 𝑘) ∨ (𝑙 ∧ 𝑚)
5. (~𝑘 ∧ ~𝑗) ∨ (𝑙 ∧ 𝑚)

Solution:

1. (𝑚 ∧ 𝑗) ∨ 𝑙
2. ~𝑗 ∧ 𝑚 ∨ 𝑙
3. Laughter is the best medicine or prayer relieves pain.
4. It is either laughter is the best medicine and grief causes illness or prayer
relieves pain and God takes charge of worries.
5. Either grief does not cause illness and laughter is not the best medicine or
prayer relieves pain and God takes charge of worries.

9 CO_Q2_General Mathematics SHS


Module 15
Conditional Defined
Example 6
Let x, y and z be propositions.
x: Dioxins are found almost everywhere.
y: Plastics, bleached paper, and most commodities contain the chemical dioxin.
z: Dioxins are a culprit to many diseases.

Express the conditionals below in verbal sentences or in symbols, as the case may
be.
1. 𝑥 → 𝑧
2. (~𝑦) → (~𝑥 ∧ ~𝑧)
3. “If dioxins are a culprit to many diseases, then they are found almost
everywhere and plastics, bleached paper, and most commodities contain this
chemical.”
4. “If dioxins are not found almost everywhere, then it is not true that either
plastics, bleached paper, and most commodities contain the chemical dioxin,
or dioxins are a culprit to many diseases.”

Solution:

1. If dioxins are found almost everywhere, then they are a culprit to many
diseases.
2. If plastics, bleached paper, and most commodities do not contain the chemical
dioxin, then it is not true that dioxins are found almost everywhere and it is
not true that dioxins are a culprit to many diseases.
3. 𝑧 → (𝑥 ∧ 𝑦)
4. (~𝑥) → ~(𝑦 ∨ 𝑧)

Example 7
Let f, g, and h be propositions.
f: Parabens are harmful to the body.
g: Parabens are found in many products we use.
h: Parabens can affect health.

Express the conditionals below in verbal sentences or in symbols, as the case may
be.

1. 𝑓 → ℎ
2. (~𝑔 ∧∼ 𝑓) → (~ℎ)
3. If parabens are harmful to the body and are found in many products we use,
then they can affect health.
4. If parabens cannot affect health or they are not found in many products we
use, then they are not harmful to the body.

10 CO_Q2_General Mathematics SHS


Module 15
Solution:

1. If parabens are harmful to the body, then they can affect health.
2. If parabens are not found in many products we use and if they are not harmful
to the body, then parabens cannot affect health.
3. (𝑓 ∧ 𝑔) → ℎ
4. (~ℎ ∨∼ 𝑔) → ~𝑓

Biconditional Defined

The last logical proposition that we shall consider is Biconditional. This is denoted
by “𝑝  𝑞” or “p iff q” given propositions or Components p and q and it is read as “p
if and only f q”, and defined through its truth table

p q 𝑝 ↔ 𝑞
T T T
T F F
F T F
F F T
Example 8
Let g, h and i be propositions.
g: Only physically handicapped individuals can be called persons with
disabilities.
h: Psychosocially disabled persons like those with chronic illnesses can also avail
PWD ID.
i: There are numerous benefits that come along with owning a PWD ID.
j: Knowledge of the wide extent of the qualifications for PWD has yet to be spread.

Express the biconditionals below in verbal sentences or in symbols.


1. (~𝑔 ↔ ℎ) ∧ (𝑗 → 𝑖)
2. (𝑔 ∧ (~ℎ)) ↔ (~𝑗)
3. “Knowledge of the wide extent of the qualifications for PWD has yet to be
spread if and only if not only physically handicapped individuals can be called
persons with disabilities.”

11 CO_Q2_General Mathematics SHS


Module 15
Solution:

1. Not only physically handicapped individuals can be called persons with


disabilities if and only if psychosocially disabled persons like those with
chronic illnesses can also avail PWD ID, and if knowledge on the wide extent
of the qualifications for PWD has yet to be spread, then there are numerous
benefits that come along with owning a PWD ID.
2. Only physically handicapped individuals can be called persons with
disabilities and psychosocially disabled persons like those with chronic
illnesses cannot avail PWD ID if and only if it is not true that knowledge on
the wide extent of the qualifications for PWD has yet to be spread.
3. 𝑗  (~𝑔)

What’s More

Activity 1.1

Let a, b, c, and d be propositions.


a: You are what you eat.
b: Your food can describe your health.
c: By nature, human is designed to eat meat.
d: A person should eat healthy food.

Express the following propositions in verbal sentences.


1. (~𝑐) ∧ (𝑏 ↔ 𝑎)
2. (𝑎 ∨ 𝑏) → 𝑑
3. ~𝑑 ↔ (~𝑎 ∧ ~𝑏)

12 CO_Q2_General Mathematics SHS


Module 15
Activity 1.2

Let d, e, f, and g be propositions.


d: There is wisdom in spending on needs.
e: It makes sense to think at least twice before giving in for wants.
f: A good planner saves first before spending.
g: A shopaholic should learn self-control.

Express the following propositions in symbols.


1. If a good planner saves first before spending, then there is wisdom in spending
on needs and it makes sense to think at least twice before giving in for wants.

2. A shopaholic should not learn self-control if and only if a good planner does
not save first before spending and it does not make sense to think at least
twice before giving in for wants.

3. There is wisdom in spending on needs or a shopaholic should not learn self-


control

13 CO_Q2_General Mathematics SHS


Module 15
What I Have Learned

Fill in the blanks.

1. ____________________ of a proposition p is a logical operator denoted by ~p


which is read as “____________________”.

2. Biconditional is a logical operator which is denoted by “_________________” or


“𝑝 iff q” given propositions or components p and q and it is read as
“____________________”.

3. The ____________________ of propositions p and q is denoted by 𝑝 v q which is


read as “p or q”. Propositions p and q are each called ____________________.

4. Another logical operator is conjunction of the propositions p and q which is


denoted by ____________________. It is read as “p and q”. Proposition p and
proposition q are called ____________________.

5. The ____________________ of propositions p and q is denoted by


____________________which is read as “if p, then q” or “p implies q”. Proposition
p is called ____________________, while proposition q is called
____________________.

14 CO_Q2_General Mathematics SHS


Module 15
What I Can Do

Make spoken word poetry on any topic relevant to you. To be sure that it is meaty,
include as many propositions with logical operators as you can. Use a separate sheet
of paper for your poem.

You may improve your output by checking against the following rubric:

Criteria 3 2 1
The poem reflects a The poem reflects The poem reflects no
specific topic; all little of a specific specific topic; very
Content
ideas are consistent topic; some ideas are few ideas are
(score x 4)
and seamless consistent and consistent and
seamless seamless
All lines highlight Some lines highlight Very few lines
rhythm and reflect rhythm and reflect highlight rhythm and
Structure spoken poetry; most spoken poetry; some reflect spoken poetry;
(score x 3) lines include lines include very few lines include
propositions with propositions with propositions with
logical operators logical operators logical operators
Uses clear, rich and Uses imaginative Uses pure everyday
Language
detailed imaginative language language
(score x 3)
language

15 CO_Q2_General Mathematics SHS


Module 15
Assessment

Choose the letter of the best answer. Write the chosen letter on a separate sheet of
paper.

1. It is a logical operator which uses the symbol “v”.


a. negation
b. conjunction
c. disjunction
d. conditional

2. Which logical operator uses the connector “and”?


a. negation
b. conjunction
c. disjunction
d. conditional

3. Which denotes negation?


a. 𝑝 ∨ 𝑞
b. 𝑝 ∧ 𝑞
c. ~𝑝
d. 𝑝 ↔ 𝑞

4. Which denotes biconditional?


a. 𝑝 ∨ 𝑞
b. 𝑝 ∧ 𝑞
c. ~𝑝
d. 𝑝 ↔ 𝑞

5. Which denotes disjunction?


a. 𝑝 ∨ 𝑞
b. 𝑝 ∧ 𝑞
c. ~𝑝
d. 𝑝 ↔ 𝑞

For numbers 6 to 10, identify the logical operator used in each compound
proposition.
6. Competence is not confidence.
a. conditional
b. biconditional
c. negation
d. conjunction

16 CO_Q2_General Mathematics SHS


Module 15
7. You should be wise as a serpent and modest as a dove.
a. conditional
b. disjunction
c. negation
d. conjunction

8. A great man either wins a game with humility or loses a game as a real man
should.
a. biconditional
b. disjunction
c. negation
d. conjunction

9. If a picture paints a thousand words, then I should paint you.


a. biconditional
b. disjunction
c. conditional
d. conjunction

10. Tatay Igo will eat dairy products if and only if they are organic.
a. biconditional
b. disjunction
c. conditional
d. negation

For numbers 11 to 15, consider the following propositions:


j: A soft answer turns away wrath.
k: A harsh word stirs up anger.
l: A wise man listens before speaking.

11. Which is the symbol for the proposition “A soft answer turns away wrath
and a harsh word stirs up anger”?
a. ~𝑗 ∨ 𝑘
b. 𝑗 ∧ 𝑘
c. 𝑗 → 𝑘
d. 𝑗 ↔ ~𝑘

12. Which is the symbol for the proposition “A wise man does not listen before
speaking?
a. ∧ 𝑙
b. ~𝑙 → 𝑙
c. ~𝑙
d. ∨ 𝑙

17 CO_Q2_General Mathematics SHS


Module 15
13. Which is the symbol for “If a soft answer turns away wrath and a harsh
word stirs up anger, then a wise man listens before speaking”?
a. (𝑗 ∧ 𝑘) → 𝑙
b. (𝑗 ∧ 𝑘) ↔ 𝑙
c. (𝑗 ∨ 𝑘) → 𝑙
d. (𝑗 ∨ 𝑘) ↔ 𝑙

14. Which is the verbal statement for ~𝑗 ∧ ~𝑘?


a. A soft answer does not turn away wrath or a harsh word does not stir
up anger.
b. A soft answer does not turn away wrath and a harsh word does not stir
up anger.
c. A soft answer turns away wrath and a harsh word stirs up anger.
d. If a soft answer does not turn away wrath, then a harsh word does not
stir up anger.

15. Which is the verbal statement for 𝑗 → 𝑘?


a. A soft answer turns away wrath or a harsh word stirs up anger.
b. A soft answer turns away wrath and a harsh word does not stir up
anger.
c. If a soft answer turns away wrath, then a harsh word stirs up anger.
d. A soft answer turns away wrath and a harsh word stirs up anger.

18 CO_Q2_General Mathematics SHS


Module 15
Additional Activities

Among the topics listed below, choose one and write a discussion about it. Use
propositions with logical operators to surely add emphasis to important details. Use
the space given to you to write your answer.

a. Combining Fractions (Addition and Subtraction of Fractions)


b. Multiplication of Fractions
c. Division of Fractions
d. Combining Integers
e. Multiplication and Division of Integers
f. Factoring
g. Simplifying Rational Expressions
h. Combining Rational Expressions
i. Multiplication of Rational Expressions
j. Division of Rational Expressions
k. Solving an Equation
l. Solving an Inequality

19 CO_Q2_General Mathematics SHS


Module 15
The following rubric will be used to rate your work:

Criteria 4 3 2 1
The process is The process is Major points of Major points
discussed discussed the process are are not
completely; completely; discussed, but discussed;
with at least without misconception with or
one example; example; may arise without
manifests an manifests an among readers; example;
Content
understanding understanding with or without manifests an
(score x 3)
of all of some example; understanding
prerequisite prerequisite manifests an of no
skills skills understanding prerequisite
of few skill
prerequisite
skills
Consistently Somewhat Inconsistent; Very
logical; aids logical; tends to inconsistent;
clear and easy somewhat complicate an shows no
Structure
understanding aids clear or understanding understanding
(score x 3)
of the topic easy of the topic of the topic
understanding
of the topic
Topic Topic f, g, h, i Topic k or l Topic b, c or e Topic a or d
(score x 2) or j
uses emphatic uses emphatic uses Uses no
statements and statements or propositions logical
Language appropriate appropriate with logical operators
(score x 2) logical logical operators but
operators for operators for inappropriate
propositions propositions sometimes

20 CO_Q2_General Mathematics SHS


Module 15
Module 15
CO_Q2_General Mathematics SHS 21
12. conclusion
11. hypothesis What’s In
10. 𝑝 → 𝑞 1. Not only, but also
9. conditional 2. and
8. conjuncts 3. if and only if
7. 𝑝 ∧ 𝑞 4. or
6. disjuncts 5. if then
5. disjunction 6. yet
4. p if and only if q 7. and
3. p ↔ q 8. but
2. not p 9. and
1. Negation 10. If then
What I Have Learned
What I Know What's More Assessment
1. d Activity 1.1 1. c
2. b 1. By nature, human is not 2. b
3. b designed to eat meat and 3. c
4. d
4. c your food can describe
5. a
5. a your health if and only if 6. c
6. b you are what you eat. 7. d
7. d 2. If you are what you eat or 8. b
8. d your food can describe 9. c
9. d your health, then a 10. a
10. a 11. b
person should eat healthy
12. c
11. c food. 13. a
12. c 3. A person should not eat 14. b
13. d healthy food if and only if 15. c
14. c you are not what you eat
15. b and your food cannot
describe your health.
Activity 1.2
1. 𝑓 → (𝑑 ∧ 𝑒)
2. ~𝑔 ↔ (~𝑓^ ↔ ~𝑒)
3. 𝑑 ∨ ~𝑔
Answer Key
References

Dimasuay, Lynie, Jeric Alcala, and Jane Palacio. General Mathematics. Quezon
City, Philippines: C & E Publishing, Inc., 2016 .

General Mathematics Learner’s Material. First Edition. 2016. p. 246-256

*DepEd Material: General Mathematics Learner’s Material

22 CO_Q2_General Mathematics SHS


Module 15
For inquiries or feedback, please write or call:

Department of Education - Bureau of Learning Resources (DepEd-BLR)

Ground Floor, Bonifacio Bldg., DepEd Complex


Meralco Avenue, Pasig City, Philippines 1600

Telefax: (632) 8634-1072; 8634-1054; 8631-4985

Email Address: blr.lrqad@deped.gov.ph * blr.lrpd@deped.gov.ph


General Mathematics
Quarter 2 – Module 16:
Truth Values of Propositions

CO_Q2_General Mathematics SHS


Module 16
General Mathematics – Senior High School
Alternative Delivery Mode
Quarter 2 – Module 16: Truth Values of Propositions
First Edition, 2020

Republic Act 8293, section 176 states that: No copyright shall subsist in any work of
the Government of the Philippines. However, prior approval of the government agency or office
wherein the work is created shall be necessary for exploitation of such work for profit. Such
agency or office may, among other things, impose as a condition the payment of royalties.

Borrowed materials (i.e., songs, stories, poems, pictures, photos, brand names,
trademarks, etc.) included in this module are owned by their respective copyright holders.
Every effort has been exerted to locate and seek permission to use these materials from their
respective copyright owners. The publisher and authors do not represent nor claim ownership
over them.

Published by the Department of Education


Secretary: Leonor Magtolis Briones
Undersecretary: Diosdado M. San Antonio

Development Team of the Module


Writer: Azalea A. Gallano
Editors: Elizabeth D. Lalunio, Anicia J. Villaruel and Roy O. Natividad
Reviewers: Jerry Punongbayan, Diosmar O. Fernandez, Dexter M. Valle,
Angelo S. Villanueva, Erlene C. Barandino and Moahna Aura M. Mancenido
Illustrators: Hanna Lorraine Luna, Diane C. Jupiter and Michael A. Alonzo
Layout Artists: Roy O. Natividad, Sayre M. Dialola, Argie L. Ty and
Glydel Eveth T. Enriquez
Management Team: Francis Cesar B. Bringas
Job S. Zape, Jr.
Ramonito Elumbaring
Reicon C. Condes
Elaine T. Balaogan
Fe M. Ong-ongowan
Hermogenes M. Panganiban
Philip B. Gallendez
Josephine T. Natividad
Anicia J. Villaruel
Dexter M. Valle

Printed in the Philippines by ________________________

Department of Education – Region 4A CALABARZON

Office Address: Gate 2 Karangalan Village, Brgy. San Isidro, Cainta, Rizal
Telefax: 02-8682-5773/8684-4914/8647-7487
E-mail Address: lrmd.calabarzon@deped.gov.ph
General Mathematics
Quarter 2 – Module 16:
Truth Values of Propositions
Introductory Message

This Self-Learning Module (SLM) is prepared so that you, our dear


learners, can continue your studies and learn while at home. Activities,
questions, directions, exercises, and discussions are carefully stated for you
to understand each lesson.

Each SLM is composed of different parts. Each part shall guide you
step-by-step as you discover and understand the lesson prepared for you.

Pre-tests are provided to measure your prior knowledge on lessons in


each SLM. This will tell you if you need to proceed on completing this module
or if you need to ask your facilitator or your teacher’s assistance for better
understanding of the lesson. At the end of each module, you need to answer
the post-test to self-check your learning. Answer keys are provided for each
activity and test. We trust that you will be honest in using these.

In addition to the material in the main text, Notes to the Teacher are
also provided to our facilitators and parents for strategies and reminders on
how they can best help you on your home-based learning.

Please use this module with care. Do not put unnecessary marks on
any part of this SLM. Use a separate sheet of paper in answering the exercises
and tests. And read the instructions carefully before performing each task.

If you have any questions in using this SLM or any difficulty in


answering the tasks in this module, do not hesitate to consult your teacher
or facilitator.

Thank you.

iii
What I Need to Know

This module will guide you go beyond the surface of propositions. To further enhance
your knowledge on logical operators, we will explore their truth values through this
module. To help you do this, you will learn how to prepare a table of values given a
proposition or a combination of propositions. In turn, this will help you in
determining the truth values.

The module is composed of one lesson, namely:


• Lesson 1 – Truth Values of Propositions

After going through this module, you are expected to:


1. construct truth tables of propositions;
2. determine the truth values of propositions; and
3. distinguish the proposition as tautology or contradiction.

What I Know

Choose the letter of the best answer. Write the chosen letter on a separate sheet of
paper.

1. What is a proposition that is always false?


a. absolute c. negation
b. contradiction d. tautology

For numbers 2 to 4, refer to the following problem:


p and q are both true propositions and r is a false proposition.

2. What is the truth value of ~ 𝑟?


a. true
b. false
c. neither true nor false
d. information is insufficient

3. What is the truth value of 𝑞 ∧ (~ 𝑟)?


a. true
b. false
c. neither true nor false
d. information is insufficient

1 CO_Q2_General Mathematics SHS


Module 16
4. What is the truth value of 𝑝 ∨ (𝑞 ∧ (~ 𝑟))?
a. true
b. false
c. neither true nor false
d. information is insufficient

For numbers 5 to 7, refer to the following problem: p and q are false and r is true.

5. What is the truth value of ~ 𝑝?


a. true
b. false
c. neither true nor false
d. information is insufficient

6. What is the truth value of 𝑞 ↔ 𝑟?


a. true
b. false
c. neither true nor false
d. information is insufficient

7. What is the truth value of (~ 𝑝) ∧ (𝑞 ↔ 𝑟)?


a. true
b. false
c. neither true nor false
d. information is insufficient

For numbers 8 to 10, determine the truth values of the propositions p, q and r that
will make the following statements false.
8. 𝑝 ∧ (𝑞 ∨ 𝑟)
a. p and r are false, while q is true.
b. p, q and r are all true or all false.
c. If p is true, then q and r are both false. If p is false, then each of q and r
may either be true or false.
d. If p is true, then q and r are both true. If p is false, then each of q and r
may either be true or false.

9. (𝑝 ∧ 𝑞) ∨ 𝑟
a. p and q should be false. r may be true or false.
b. q should be false. p and r may either be both true or both false.
c. p should be false. Either q is true and r is false, or vice versa, or both are
true.
d. r should be false. p and q may take any truth value except for both are
true.

10. ~(𝑝 ∧ (~𝑞))


a. p and r should be false, while q should be true.
b. p and q should be false, while r should be true.
c. q should be false, while p and r should be true.
d. q and r should be false, while p should be true.

2 CO_Q2_General Mathematics SHS


Module 16
11. What is the truth value of the conjunction 𝑝 ∧ 𝑞 ∧ (~𝑟) if p and q are both false
propositions and r is a true proposition?
a. true
b. false
c. neither true nor false
d. information is insufficient

12. What is the truth value of the conjunction ~(𝑞 ∧ 𝑝) ∧ 𝑟 if p and q are both false
propositions and r is a true proposition?
a. true
b. false
c. neither true nor false
d. information is insufficient

13. What is the truth value of the disjunction 𝑝 ∨ (~𝑟) if p and q are both true
propositions and r is a false proposition?
a. true
b. false
c. neither true nor false
d. information is insufficient

14. What is the truth value of the disjunction (𝑝 ∨ 𝑟) ∨ (~𝑞) if p and q are both true
propositions and r is a false proposition?
a. true
b. false
c. neither true nor false
d. information is insufficient

15. What is the truth value of the conditional 𝑝 → (𝑞 ∧ 𝑟) if p, q and r are true, false
and true propositions, respectively?
a. true
b. false
c. neither true nor false
d. information is insufficient

3 CO_Q2_General Mathematics SHS


Module 16
Lesson
Truth Values of
1 Propositions
Aside from performing operations on propositions, determining their truth values is
also possible. Given some truth values of its component propositions, the truth value
of a compound proposition may be known. The truth table will systematically guide
you to determine the truth value of a compound proposition.
In the first place, logical operators may be defined by using truth tables. Truth values
of compound propositions will be built on these. To understand the details of this
lesson gradually, read on, dear learner.

What’s In

Complete the following truth tables of logical operators.


1. 2. p q 𝑝 ∧ 𝑞 3. p q 𝑝 ∨ 𝑞
p ~p
T T T T T

F T F T F
F T F T
F F F F

p q 𝑝 → 𝑞 p q 𝑝 ↔ 𝑞
4. T T 5. T T
T F T F
F T F T
F F F F

Take note that the knowledge of truth tables of logical operators is very important
to be successful in this module. As a learner, you should be familiar with the truth
value of each logical connector.

4 CO_Q2_General Mathematics SHS


Module 16
Notes to the Teacher
Enable learners to perform each task or activity in this module.
Remind them to review the truth values of logical operators
introduced in the previous lesson.

What’s New

Hey there! Before you continue reading a new lesson, pause for a while. Have a seat.
Take a deep breath. Now, read the poem word for word silently.

TRUTH THAT VALUES MOST


by: Azalea A. Gallano

Life can teach us good and bad,


While others feel the heat before they see the light,
As for some, a word is enough
And they make it a guide.

Even if one keeps the entire law,


But disobeys one commandment
That person shall be charged guilty
And shall be considered unworthy.

Life can best be spent on service


No one can serve two at the same time, however,
You’ll satisfy and be satisfied with an endeavor you’re in,
But the other will surely be forsaken.

While opportunity lies on your doorsteps, study well


Not all have this opportunity to fondle,
Others give up, yet some chase for their dreams
But you, you have no reason to surrender.

5 CO_Q2_General Mathematics SHS


Module 16
Take time to ponder, however, above all
The main purpose of life is service to the Creator
We fulfill our mission if and only if we serve Him
No more, no less, it is everybody’s calling.

I guess you are done reflecting the message of the poem. Today or later, you will
understand the lines on and in it. For the meantime, can you spot which stanzas
deal with concepts of the logical operators negation, conjunction, disjunction,
conditional and biconditional?

If you analyze the poem, you will identify and understand the differences of these
logical operators. You’ll be able to discover concepts behind each of them.

What is It

Truth Value and Truth Table Defined


Every proposition has two possible truth values. That is, the truth value of a
proposition, be it simple or compound, is either true or false. The TRUTH VALUE,
therefore, refers to the truthfulness of a proposition that may either be true or false.
These can be represented in tabular form. The table that shows the complete possible
truth values of a proposition is called TRUTH TABLE.
A proposition p has the following truth table:

p
T
F

Suppose we are given two propositions p and q. Since there are four possible
combinations of truth values (TT, TF, FT, FF), then their truth table is:

p q
T T
T F
F T
F F

6 CO_Q2_General Mathematics SHS


Module 16
It follows therefore that if there are three propositions p, q and r for example,
their truth table is:

p q r
T T T
T T F
T F T
T F F
F T T
F T F
F F T
F F F
How many possible combinations of truth values do we have if there are three
propositions? Can you name them? Notice that the number of truth-value
combinations increases geometrically as the number of propositions increase. That
is, there are 21 = 2, 22 = 4, 23 = 8 and 24 = 16 such combinations if there are 1, 2, 3
and 4 propositions, respectively. How many truth value combinations shall we expect
if there are n propositions?

In the earlier module, the truth value of logical operators was introduced, but
in this module, you will learn how to find truth table for the compound proposition.

Example 1
Given the statement “A diabetic either takes conventional drugs, natural
medicine or both”, determine the truth value of
𝑝 ∨ (~𝑞 ∧ 𝑟) in each scenario. Let p: A diabetic
takes conventional drugs. Let q: A diabetic
takes natural medicine. And let r: A diabetic
takes both conventional and natural medicines.
Scenario 1: Patient A only eats leaves of
his insulin plant to control his sugar level.
Scenario 2: Patient B consults his doctor
and buys diligently the prescribed conventional
drugs.
Scenario 3: Patient C supplements
conventional drugs with natural medicine he
reads is effective as well for diabetics.

7 CO_Q2_General Mathematics SHS


Module 16
Solution:
Scenario 1: Patient A only eats leaves of his insulin plant to control his sugar level.
The negation of q denoted by ~q states that a diabetic does not take natural
medicine which is false in Scenario 1. Its conjunction with proposition r which is also
false is false as we’ve learned. On the other hand, p is false since Patient A takes
natural medicine only. The disjunction of a false proposition and another false
disjunct is false. Its truth table is illustrated as follows:
𝑝 𝑞 𝑟 ~𝑞 ~𝑞 ∧ 𝑟 𝑝 ∨ (~𝑞 ∧ 𝑟)
F T F F F F

Scenario 2: Patient B consults his doctor and buys diligently the prescribed
conventional drugs.
Since ~q is true in this scenario and r is false, then their conjunction is false.
And, since p is true and (~𝑞 ∧ 𝑟) is false as we have said, then their disjunction is
true. Simply put, since the proposition involves disjunction and one disjunct p is
true, without further analysis we may conclude that the disjunction is true. That is
because disjunction can only be false if both disjuncts are false. Its truth table is
illustrated as follows:
𝑝 𝑞 𝑟 ~𝑞 ~𝑞 ∧ 𝑟 𝑝 ∨ (~𝑞 ∧ 𝑟)
T F F T F T

Scenario 3: Patient C supplements conventional drugs with natural medicine he


reads is effective as well for diabetics.
Both ~q and r are true because Patient C is not taking conventional drugs
only, but instead he takes both, conventional and natural. Thus, the conjunction of
these two propositions is true. Then, although p is false, but since one of the
disjuncts is true, then the disjunction 𝑝 ∨ (~𝑞 ∧ 𝑟) is true. Its truth table is
illustrated as follows:
𝑝 𝑞 𝑟 ~𝑞 ~𝑞 ∧ 𝑟 𝑝 ∨ (~𝑞 ∧ 𝑟)
F F T T T T

8 CO_Q2_General Mathematics SHS


Module 16
Example 2
Let p and q be propositions. Determine the truth value of (𝑝 → 𝑞) → ((~𝑝) ∨
𝑞). Construct its truth table.
Solution:
Since there are two propositions p and q. Thus, the truth table will contain four
rows, the first two columns of which are
p q
T T
T F
F T
F F

Using the truth table for the definition of the conditional, negation and
disjunction statements 𝑝 → 𝑞, ~𝑝 and ~𝑝 ∨ 𝑞 respectively. You need to add three
more columns to indicate the truth values for each statement.

𝑝 → 𝑞 ~𝑝 ~𝑝 ∨ 𝑞
T F T
F F F
T T T
T T T

Lastly, you need to consider the truth value of the proposition


(𝑝 → 𝑞) → (~𝑝 ∨ 𝑞). The truth table of (𝑝 → 𝑞) → ~𝑝 ∨ 𝑞) is as follows:
p q 𝑝 → 𝑞 ~𝑝 ~𝑝 ∨ 𝑞 (𝑝 → 𝑞) → ( ~𝑝 ∨ 𝑞)
T T T F T T
T F F F F T
F T T T T T
F F T T T T

Notice that the proposition (𝑝 → 𝑞) → ((~𝑝) ∨ 𝑞) is always TRUE. Such is


called TAUTOLOGY and is denoted by . It is the opposite of contradiction.

Example 3
Let p and q be propositions. Determine the truth values of (𝑝 ∧ (~ 𝑞)) ∧ (𝑝 ∧ 𝑞).
Construct its truth table.

9 CO_Q2_General Mathematics SHS


Module 16
Solution:
Since there are two propositions p and q. Thus, the truth table will contain
four rows, the first two columns of which are
p q
T T
T F
F T
F F

Using the truth table for the definition of the negation and conjunction
statements ~𝑞, 𝑝 ∧ (∼ 𝑞) and 𝑝 ∧ 𝑞 respectively. You need to add three more columns
to indicate the truth values for each statement.

~𝑞 𝑝 ∧ ~𝑞 𝑝 ∧ 𝑞
F F T
T T F
F F F
T F F

Lastly, you need to consider the truth value of the proposition


(𝑝 ∧ (~ 𝑞)) ∧ (𝑝 ∧ 𝑞). The truth table of (𝑝 ∧ (~ 𝑞)) ∧ (𝑝 ∧ 𝑞) is as follows:

p q ~𝑞 𝑝 ∧ (∼ 𝑞) 𝑝 ∧ 𝑞 (𝑝 ∧ (~ 𝑞)) ∧ (𝑝 ∧ 𝑞)
T T F F T F
T F T T F F
F T F F F F
F F T F F F

Notice that the proposition (𝑝 ∧ (~ 𝑞)) ∧ (𝑝 ∧ 𝑞) is always FALSE. Such is


called CONTRADICTION. Other resources call it a FALLACY. It is denoted by ∅.

10 CO_Q2_General Mathematics SHS


Module 16
What’s More

Activity 1
Determine the truth value of each of the following propositions. Show the truth table
of each as well.
1. ~𝑝 ∧ 𝑞 where p and q are both false propositions
2. (𝑝 ∧ 𝑟) ∧ ~𝑞 where p and q are both false and r is true
3. ~(𝑞 ∨ 𝑝) ∧ 𝑟 where p and q are both true and r is false
4. ~𝑝 → (𝑞  𝑟) where p is true and q and r are both false
5. 𝑝 → (𝑞 ∨ 𝑟) where p and q are both false and r is true

Activity 2
Consider the statement “Lito saves money for organic and natural products and he
buys paraben-free items”. Suppose p: Lito saves money for organic and natural
products. And q: He buys paraben-free items. Decide on the truth value of 𝑝 ∧ 𝑞 in
each of the following scenarios.
1. Scenario A: Lito was not able to save much money to buy natural products.
He was able to purchase items some of which contain parabens.
2. Scenario B: The money that Lito has saved was more than enough for paying
paraben-free items. He even was able to buy organic food.
3. Scenario C: Though Lito has not saved for natural items, he prioritized buying
paraben-free products. He is decided to change his lifestyle.

What I Have Learned

Fill in each blank with “true” or “false”.


1. The truth value of every proposition may be ____________________ or
____________________.
2. Given that p is true, the truth value of the negation of p (~𝑝) is
____________________.
3. The truth value of all conjunction of p and q (𝑝 ∧ 𝑞) is ____________________
except for case where p and q are both ____________________.
4. The truth value of all disjunction of p and q (𝑝 ∨ 𝑞) is ____________________
except for case where p and q are both ____________________.
5. The truth value of all conditional of p and q (𝑝 → 𝑞) is ____________________
except for conditional where p is ____________________ and q is
____________________.

11 CO_Q2_General Mathematics SHS


Module 16
6. The truth value of the biconditional of p and q (𝑝  𝑞) is ____________________
if p and q are both ____________________ or both ____________________.
Otherwise, its truth value is ____________________.
7. A proposition is a tautology if the truth value is always ____________________.
Meanwhile, a proposition is a contradiction or a fallacy if the truth value is
always ____________________.

What I Can Do

Make three simple propositions out of things that matter to you most. What
compound proposition involving logical operators shall represent your plan regarding
these three propositions? Express it in symbols and construct a truth table. What
reflection can you make out of the possible combinations of truth values as you can
see from the truth table? Use a separate sheet of paper for your output.

You may improve your output by checking against the following rubric:

Criteria 4 3 2 1
All meaningful Two are Only one is Not
Propositions
and realistic meaningful and meaningful and meaningful
(score x 2)
realistic realistic and realistic
Complete with Incomplete Complete with Incomplete
component component component with
propositions propositions but propositions component
which gradually gradually which gradually propositions;
complicate from complicate from complicate from truth table
simple to simple to simple to with more
compound compound compound than 3
Truth table proposition proposition proposition incorrect
(score x 3) being asked; being asked; being asked; truth values
truth table with truth table with truth table with
all correct truth all correct truth 1-3 incorrect
values values truth values

12 CO_Q2_General Mathematics SHS


Module 16
Consistently Somewhat Somewhat No textual
logical; aids logical; illogical; tends explanation
Proof
clear and easy somewhat aids to complicate of solution
statement
understanding clear or easy understanding or answer
(score x 3)
of the solution understanding of the solution
of the solution
Very consistent Consistent with Somewhat Inconsistent
with truth truth values; inconsistent with truth
values; uses uses with truth values; uses
Reflection
clear, rich and imaginative values; uses pure
(score x 2)
detailed language imaginative everyday
imaginative language language
language

Assessment

Choose the letter of the best answer. Write the chosen letter on a separate sheet of
paper.
1. What is a proposition that is always true?
a. absolute c. negation
b. contradiction d. tautology

For numbers 2 to 8, consider the following:

In the minutes of the meeting, it is recorded that “All ten members agreed that
there will be a monthly contribution of ₱50.00. Let p be the statement “The collection
for February is ₱500.00”. Decide on the truth value of the negation ~p given the
following scenarios.

2. What is the verbal equivalent of ~p?


a. The collection for February is not ₱500.00.
b. The collection not for February is ₱500.00.
c. All ten members agreed that there will be a monthly contribution of ₱50.00
and the collection for February is ₱500.00.
d. If all ten members agreed that there will be a monthly contribution of
₱50.00, then the collection for February is ₱500.00.

Scenario A: All members of the group paid the monthly contribution.


3. What is the truth value of the negation ~p given Scenario A?
a. true
b. false
c. neither true nor false
d. information is insufficient

13 CO_Q2_General Mathematics SHS


Module 16
4. Which explains the reason behind the truth value of ~p given Scenario A?
a. Despite all members contributed, each may have contributed any amount.
b. Scenario A makes no mention about the contribution of each of ten
members.
c. Given that all ten members of the group contributed, the total collection
for February is ₱500.00.
d. Given that there are ten members of the group, the total collection for
February just like in any month should be ₱500.00.

Scenario B: The treasurer lost the payment of the three members.


5. What is the truth value of the negation ~p given Scenario B?
a. true
b. false
c. neither true nor false
d. information is insufficient

6. Which explains the reason behind the truth value of ~p given Scenario B?
a. Scenario B lacks information.
b. Since three payments were lost, the total collection for February is ₱350.00
only.
c. The other seven members may or may not have paid more than ₱50.00
each.
d. Three payments were lost, but it is a thumbs rule that the one keeping the
collection should make up the lost money.

Scenario C: One member left the group.


7. What is the truth value of the negation ~p given Scenario C?
a. true
b. false
c. neither true nor false
d. information is insufficient

8. Which explains the reason behind the truth value of ~p given Scenario C?
a. Leaving the group does not relate to the total amount collected.
b. Scenario C does not mention whether the member contributed or not
before leaving the group.
c. The remaining nine members divided ₱500.00 equally among themselves
to meet the target amount.
d. With one member withdrawing his/her connection to the group, the
collection shall be lower than ₱500.00.

14 CO_Q2_General Mathematics SHS


Module 16
For numbers 9 to 11, determine all truth values of the propositions p and q that will
make the following statements true.
9. 𝑝 ∧ ~𝑞
a. p and q are both true.
b. p and q are both false.
c. p and q are true and false, respectively.
d. p and q are false and true, respectively.

10. ~(𝑝 ∧ 𝑞)

a. not p and q are false.


b. Both p and q are true.
c. p is true and q is true.
d. At least one of p and q is false.

11. ~(𝑝 ∧ ~𝑞)


a. If p is true, then q should be true. If p is false, then q may either be true or
false.
b. If p is false, then q should be true. If p is true, then q may either be true or
false.
c. If p is true, then q should be false. If p is false, then q may either be true
or false.
d. If p is false, then q should be false. If p is true, then q may either be true
or false.

12. What is the truth value of the conjunction ~𝑝 ∧ (𝑞 ∨ 𝑟) if p and q are both true
propositions and r is a false proposition?
a. true
b. false
c. neither true nor false
d. information is insufficient

13. What is the truth value of the conditional [𝑝 ∧ (𝑞 ∨ 𝑟)] → [(𝑝 ∧ 𝑞) ∨ (𝑝 ∧ 𝑟)] if p
and r are both true propositions and q is a false proposition?
a. true
b. false
c. neither true nor false
d. information is insufficient

14. What is the truth value of the biconditional 𝑝 ↔ (𝑞 ∧ 𝑟) if p is a true proposition


and both q and r are false propositions?
a. true
b. false
c. neither true nor false
d. information is insufficient

15 CO_Q2_General Mathematics SHS


Module 16
15. Which of the following propositions has a truth value of false if p is false and q
is true?
a. 𝑝  (~𝑞)
b. (𝑞 → 𝑝) ∨ (𝑝 ∧ 𝑞)
c. (𝑝 ∧ 𝑞) → (𝑞 ∨ 𝑝)
d. (~𝑝 ∧ 𝑞) ∨ (𝑞  𝑝)

Additional Activities

Assume the propositions


p: Behind Bruce Lee was his wing chun teacher Ip Man.
q: Wing chun boils down to math.
r: It is strength and ability to avoid fight that matter.
s: Wing chun can be learned.

A. Determine the truth value of each proposition below.


Proposition A: If behind Bruce Lee was his wing chun teacher Ip Man, then he must
have lived in simplicity and humility.
Proposition B: If Ip Man is good in math, then wing chun boils down to math.

B. Considering the statement “Wing chun which is about strength and where ability
to avoid fight matters, boils down to math and can be learned”, determine the
truth value of the proposition 𝑞 → (𝑟 ∧ 𝑠) given each scenario:
Scenario A: Matthew upon reading much and correlating wing chun to math in his
research has gradually understood that wing chun is about strength and not fight
and that he can learn it in due time.
Scenario B: Ruth sees math in wing chun. Because of this, she is optimistic that she
can learn it. However, since it is math, she believes it cannot be a good sport in
promoting strength.

16 CO_Q2_General Mathematics SHS


Module 16
The following rubric will be used to rate your work for each of the four problems:
Criteria 4 3 2 1
Complete with incomplete Complete with Incomplete with
component component component component
propositions propositions propositions propositions;
which but gradually which truth table with
gradually complicate from gradually more than 3
complicate from simple to complicate from incorrect truth
Truth table
simple to compound simple to values
(score x 3)
compound proposition compound
proposition being asked; proposition
being asked; truth table with being asked;
truth table with all correct truth truth table with
all correct truth values 1-3 incorrect
values truth values
Consistently Somewhat Somewhat No textual
logical; aids logical; illogical; tends explanation of
Proof
clear and easy somewhat aids to complicate solution or
statement
understanding clear or easy understanding answer
(score x 2)
of the solution understanding of the solution
of the solution

17 CO_Q2_General Mathematics SHS


Module 16
Module 16
CO_Q2_General Mathematics SHS 18
What I Know What's More Assessment
1. b Activity 1 1. d
2. a 1. false 2. a
3. a 2. false 3. b
4. a 3. false 4. c
5. a 4. true 5. a
6. b 5. true 6. b
7. b 7. a
8. c Activity 2 8. d
9. d 1. false 9. c
10. a 2. true 10. d
11. b 3. false 11. c
12. a 12. b
13. a 13. a
14. a 14. b
15. b 15. b
Answer Key
References
General Mathematics Learner’s Material. First Edition. 2016. p. 257-62

*DepED Material: General Mathematics Learner’s Material

Henley, Joe. 2019. "scmp.com." Post magazine. February 17. Accessed February
25, 2019. https://www.scmp.com/magazines/post-magazine/long-
reads/article/2186246/me-and-my-uncle-ip-man-taught-bruce-lee-wing-
chun?module=perpetual_scroll&pgtype=article&campaign=2186246.

19 CO_Q2_General Mathematics SHS


Module 16
For inquiries or feedback, please write or call:

Department of Education - Bureau of Learning Resources (DepEd-BLR)

Ground Floor, Bonifacio Bldg., DepEd Complex


Meralco Avenue, Pasig City, Philippines 1600

Telefax: (632) 8634-1072; 8634-1054; 8631-4985

Email Address: blr.lrqad@deped.gov.ph * blr.lrpd@deped.gov.ph


General Mathematics
Quarter 2 – Module 17:
Logical Equivalence and
Conditional Propositions

CO_Q2_General Mathematics SHS


Module 16
General Mathematics – Senior High School
Alternative Delivery Mode
Quarter 2 – Module 17: Logical Equivalence and Conditional Propositions
First Edition, 2020

Republic Act 8293, section 176 states that: No copyright shall subsist in any work of
the Government of the Philippines. However, prior approval of the government agency or office
wherein the work is created shall be necessary for exploitation of such work for profit. Such
agency or office may, among other things, impose as a condition the payment of royalties.

Borrowed materials (i.e., songs, stories, poems, pictures, photos, brand names,
trademarks, etc.) included in this module are owned by their respective copyright holders.
Every effort has been exerted to locate and seek permission to use these materials from their
respective copyright owners. The publisher and authors do not represent nor claim ownership
over them.

Published by the Department of Education


Secretary: Leonor Magtolis Briones
Undersecretary: Diosdado M. San Antonio

Development Team of the Module


Writer: Azenith G. Mercado
Editors: Elizabeth D. Lalunio, Anicia J. Villaruel and Roy O. Natividad
Reviewers: Jerry Punongbayan, Diosmar O. Fernandez, Dexter M. Valle,
Mary Grace Constantino, Adrian Catapat and Moahna Aura M. Mancenido
Illustrators: Hanna Lorraine Luna, Diane C. Jupiter and Michael A. Alonzo
Layout Artists: Roy O. Natividad, Sayre M. Dialola, Argie L. Ty and
Glydel Eveth T. Enriquez
Management Team: Francis Cesar B. Bringas
Job S. Zape, Jr.
Ramonito Elumbaring
Reicon C. Condes
Elaine T. Balaogan
Fe M. Ong-ongowan
Hermogenes M. Panganiban
Philip B. Gallendez
Josephine T. Natividad
Anicia J. Villaruel
Dexter M. Valle

Printed in the Philippines by ________________________

Department of Education – Region 4A CALABARZON

Office Address: Gate 2 Karangalan Village, Brgy. San Isidro, Cainta, Rizal
Telefax: 02-8682-5773/8684-4914/8647-7487
E-mail Address: lrmd.calabarzon@deped.gov.ph
General Mathematics
Quarter 2 – Module 17:
Logical Equivalence and
Conditional Propositions
Introductory Message

This Self-Learning Module (SLM) is prepared so that you, our dear


learners, can continue your studies and learn while at home. Activities,
questions, directions, exercises, and discussions are carefully stated for you
to understand each lesson.

Each SLM is composed of different parts. Each part shall guide you
step-by-step as you discover and understand the lesson prepared for you.

Pre-tests are provided to measure your prior knowledge on lessons in


each SLM. This will tell you if you need to proceed on completing this module
or if you need to ask your facilitator or your teacher’s assistance for better
understanding of the lesson. At the end of each module, you need to answer
the post-test to self-check your learning. Answer keys are provided for each
activity and test. We trust that you will be honest in using these.

In addition to the material in the main text, Notes to the Teacher are
also provided to our facilitators and parents for strategies and reminders on
how they can best help you on your home-based learning.

Please use this module with care. Do not put unnecessary marks on
any part of this SLM. Use a separate sheet of paper in answering the exercises
and tests. And read the instructions carefully before performing each task.

If you have any questions in using this SLM or any difficulty in


answering the tasks in this module, do not hesitate to consult your teacher
or facilitator.

Thank you.

iii
What I Need to Know
KnowKnowKnow
This module was designed and written with you in mind. It is here to help you master
logical equivalence and conditional propositions. The scope of this module permits it
to be used in many different learning situations. The language used recognizes the
diverse vocabulary level of students. The lessons are arranged to follow the standard
sequence of the course. But the order in which you read them can be changed to
correspond with the textbook you are now using.

After going through this module, you are expected to:


1. illustrate the different forms of conditional propositions;
2. verify the logical equivalences;
3. state the converse, contrapositive, and inverse of a conditional proposition;
and
4. cite a real-life situation using logical equivalence and conditional proposition.

What I Know

Choose the letter of the best answer and write it on a separate sheet of paper.

1. What is the symbol used in referring to logically equivalent propositions?


a. ∧ c. ⟺
b. ∨ d. ↔

For item numbers 2-7, verify ~[𝑝 ∨ (~𝑝 ∧ 𝑞)] ⟺ (~𝑝 ⋀ ~𝑞).

2. Which of the following supports the logical equivalence


~[𝑝 ∨ (~𝑝 ∧ 𝑞)] ⟺ (~𝑝) ∧ ~(~𝑝 ∧ 𝑞)?
a. De Morgan’s Law c. Idempotent Law
b. Identity Law d. Double Negation

3. Which of the following supports the logical equivalence


(~𝑝) ∧ ~(~𝑝 ∧ 𝑞) ⟺ ~𝑝 ∧ [~(~𝑝) ∨ (~𝑞)?
a. Double Negation c. Identity Law
b. Idempotent Law d. De Morgan’s Law

1 CO_Q2_General Mathematics SHS


Module 17
4. Which of the following supports the logical equivalence
~𝑝 ∧ [~(~𝑝) ∨ (~𝑞) ⟺ ~𝑝 ∧ (𝑝 ∨ ~𝑞)?
a. Double Negation c. Identity Law
b. Idempotent Law d. De Morgan’s Law

5. Which of the following supports the logical equivalence


~𝑝 ∧ (𝑝 ∨ ~𝑞) ⟺ (~𝑝 ∧ 𝑝) ∨ (~𝑝 ∧ ~𝑞)?

a. Associative Law c. Commutative Law


b. Distributive Law d. Switcheroo Law

6. Which of the following supports the logical equivalence


(~𝑝 ∧ 𝑝) ∨ (~𝑝 ∧ ~𝑞) ⟺ 𝜙 ∨ (~𝑝 ∧ ~ ∧ 𝑞)?

a. Switcheroo Law c. Domination Law


b. Absorption Law d. Inverse Law

7. Which of the following supports the logical equivalence


𝜙 ∨ (~𝑝 ∧ ~ ∧ 𝑞) ⟺ ~𝑝 ∧ ~𝑞?

a. Domination Law c. Idempotent Law


b. Identity Law d. Double Negation

8. Which of the following logical equivalence is being depicted by


𝑝 ∧ (𝑞 ∧ 𝑟) ⟺ (𝑝 ∧ 𝑞) ∧ 𝑟?

a. Identity Law c. Associative Law


b. Commutative Law d. Distributive Law

9. Which of the following are logically equivalents?


a. converse and contrapositive c. conditional and inverse
b. contrapositive and conditional d. conditional and converse

For item numbers 10-15, refer to the conditional proposition, “If a quadrilateral is a
square, then it has equal sides”.

10. What type of conditionals is being depicted by the conditional statement, “If a
quadrilateral is not a square, then it does not have equal sides.”?
a. converse c. inverse
b. contrapositive d. equivalence

11. Which of the following represents item number 10?


a. ~𝑝 ⟶ ~𝑞 c. ~𝑞 ⟶ ~𝑝
b. 𝑝 ⟶ 𝑞 d. 𝑞 ⟶ 𝑝

2 CO_Q2_General Mathematics SHS


Module 17
12. What type of conditionals is being depicted by the conditional statement, “If a
quadrilateral has equal sides, then it is a ssquare.”?
a. converse c. inverse
b. contrapositive d. equivalence

13. Which of the following represents item number 12?


a. 𝑝 ⟶ 𝑞 c. 𝑞 ⟶ 𝑝
b. ~𝑝 ⟶ ~𝑞 d. ~𝑞 ⟶ ~𝑝

14. What type of conditionals is being depicted by the conditional statement, “If a
quadrilateral does not have equal sides, then it is not a square.”?
a. converse c. inverse
b. contrapositive d. equivalence

15. Which of the following represents item number 14?


a. 𝑝 ⟶ 𝑞 c. 𝑞 ⟶ 𝑝
b. ~𝑝 ⟶ ~𝑞 d. ~𝑞 ⟶ ~𝑝

3 CO_Q2_General Mathematics SHS


Module 17
Lesson
Logical Equivalence and
1 Conditional Propositions
Do you easily believe in everything you hear? How do you know whether a statement
is true or not? It is difficult to claim the truthfulness of one thing without a basis.
That applies to everything. So, the next time you hear a rumor, before believing, one
thing you must consider is to let it undergo a process. Because sometimes, there is
no shortcut.

What’s In

In the past lessons, you have learned the different logical operators, including
negation (~), conjunction (∧), disjunction (∨), conditional (⟶), and biconditional
(⟷).
These logical operators connect propositions and make them compound ones. And
truth values of said propositions can be determined through the use of a truth table.
Let us see whether you can still recall the truth values of logical operators
when used in compound propositions.

Activity 1. Show Me What is True!


Fill in the missing cells of the table.
𝒑 𝒒 𝒑→𝒒 𝒒→𝒑 (𝒑 → 𝒒) ∨ (𝒒 → 𝒑)
T T
T F
F T
F F
Third and fourth columns used conditional. All values for
𝑝 → 𝑞 “p implies q” are true except in the case where p is true and q is false. On the
other hand, all values for 𝑞 → 𝑝 “q implies p” are true except in the case where q is
true and p is false.
The last column involves disjunction
(𝑝 → 𝑞) ∨ (𝑞 → 𝑝) “p implies q” or “q implies p”. As you have noticed with the truth
values you obtained in this column, all are true. And this we call a tautology 𝜏, in
which a proposition is always true.

4 CO_Q2_General Mathematics SHS


Module 17
Activity 2. Prove Me, I’m Wrong!
Complete the following table.
𝒑 𝒒 𝒑→𝒒 ~(𝒑 → 𝒒) 𝒒 ∧ ~(𝒑 → 𝒒)
T T
T F
F T
F F
As recalled above, in the third column 𝑝 → 𝑞 “p implies q”, all are true except in the
case where p is true and q is false. Whereas, the fourth column ~(𝑝 → 𝑞) “negation of
p implies q” denies 𝑝 → 𝑞. Thus, making all that which are true in the third column,
as false; and with that which is the only false in the former as the only true in the
fourth column.
The fifth column involves the conjunction 𝑞 ∧ ~(𝑝 → 𝑞) “q and negation of p implies
q”. How did it differ from the last column in the first table? If you have observed, this
time, all are false. This proposition that always results in false values is a
contradiction 𝜙.

Notes to the Teacher


Guide students in performing the activities. Explain further the
reasons behind the answers in each activity.

5 CO_Q2_General Mathematics SHS


Module 17
What’s New

Activity 3. Complete Me!


Decide for the truth values of the propositions in a truth table below.
𝒑 𝒒 𝒑→𝒒 ~𝒑 (~𝒑) ∨ 𝒒 (𝒑 → 𝒒) ⟷ [(~𝒑) ∨ 𝒒]
T T
T F
F T
F F

Questions:
1. What truth values do you get for 𝑝 → 𝑞?
2. What truth values do you get for ~𝑝?
3. What truth values do you get for (~𝑝) ∨ 𝑞?
4. What have you noticed with every pair of corresponding truth values in the
third and fifth columns?
5. What can you infer from the sixth column?

As mentioned in the other previous activities above, the conditional 𝑝 → 𝑞 results to


all true values except in the case where p is true and q is false. For ~𝑝, values of p
are simply negated. That is if p is true, then ~𝑝 is false, and vice-versa. On the other
hand, for the disjunction (~𝑝) ∨ 𝑞, all are true except when both ~𝑝 and q are false.
Comparing the third and fifth columns, you can easily notice that they have the same
truth values. Moreover, it turns out that when propositions are perfectly alike, their
biconditional will yield values that are all true. Hence, they result in a tautology, as
observed in the last column.

6 CO_Q2_General Mathematics SHS


Module 17
What is It

Two propositions p and q are said to be logically equivalent if they have the same
values in the truth table and if the biconditional 𝑝 ⟷ 𝑞 is a tautology. It is denoted
by 𝑝 ⟺ 𝑞 or 𝑝 ≡ 𝑞.
The above activity shows that (𝑝 → 𝑞) ⟺ [(~𝑝) ∨ 𝑞]. This logical equation is called
Switcheroo Law.

Aside from this, below is a table of various logical equations.


Table of Logical Equivalences
Let p, q, and r be propositions.
Identity Law (𝑝 ∧ 𝜏) ⟺ 𝑝 (𝑝 ∨ 𝜙) ⟺ 𝑝
Domination Law (𝑝 ∨ 𝜏) ⟺ 𝜏 (𝑝 ∧ 𝜙) ⟺ 𝜙
Idempotent Law (𝑝 ∨ 𝑝) ⟺ 𝑝 (𝑝 ∧ 𝑝) ⟺ 𝑝
Inverse Law [𝑝 ∨ (~𝑝)] ⟺ 𝜏 [𝑝 ∧ (~𝑝)] ⟺ 𝜙
Double Negation ~(~𝑝) ⟺ 𝑝
Associative Law 𝑝 ∨ (𝑞 ∨ 𝑟) ⟺ (𝑝 ∨ 𝑞) ∨ 𝑟 𝑝 ∧ (𝑞 ∧ 𝑟) ⟺ (𝑝 ∧ 𝑞) ∧ 𝑟
Commutative Law 𝑝∨𝑞 ⟺𝑞∨𝑝 𝑝∧𝑞⟺𝑞∧𝑝
Distributive Law 𝑝 ∨ (𝑞 ∧ 𝑟) ⟺ (𝑝 ∨ 𝑞) ∧ (𝑝 ∨ 𝑟) 𝑝 ∧ (𝑞 ∨ 𝑟) ⟺ (𝑝 ∧ 𝑞) ∨ (𝑝 ∧ 𝑟)
De Morgan’s Law ~(𝑝 ∨ 𝑞) ⟺ (~𝑝) ∧ (~𝑞) ~(𝑝 ∧ 𝑞) ⟺ (~𝑝) ∨ (~𝑞)
Absorption Law 𝑝 ∨ (𝑝 ∧ 𝑞) ⟺ 𝑝 𝑝 ∧ (𝑝 ∨ 𝑞) ⟺ 𝑝
Switcheroo Law 𝑝 → 𝑞 ⟺ ~𝑝 ∨ 𝑞

Example 1: Using logical equivalences, verify that ~(𝑝 → 𝑞) ⟺ [𝑝 ∧ (~𝑞)].


Proposition Reason
~(𝑝 → 𝑞) Given
⟺ ~[(~𝑝) ∨ 𝑞] Switcheroo Law
⟺ ~(~𝑝) ∧ (~𝑞) De Morgan’s Law
⟺ 𝑝 ∧ (~𝑞) Double Negation

Moreover, the conditional proposition 𝑝 → 𝑞 can be illustrated in three other


conditional statements. They are the following:
1. Converse: 𝑞 → 𝑝
2. Contrapositive: (~𝑞) → (~𝑝)
3. Inverse: (~𝑝) → (~𝑞)
A conditional and its contrapositive are logically equivalent. In the same
manner, based on a given conditional proposition, its converse and inverse are
logically equivalent.

7 CO_Q2_General Mathematics SHS


Module 17
Example 2: Given the following conditional, “If the weather is fine, then I will go to
the beach.”; state its converse, contrapositive and inverse.
1. Converse 𝑞 → 𝑝: If I go to the beach, then the weather is fine.
2. Contrapositive (~𝑞) → (~𝑝): If I do not go to the beach, then the weather is not
fine.
3. Inverse (~𝑝) → (~𝑞): If the weather is not fine, then I will not go to the beach.

What’s More

Activity 1.1
Tell whether the propositions [(𝑝 ⟶ 𝑞) ∧ ~𝑞] and ~(𝑝 ∨ 𝑞) are logically equivalent by
filling in the correct logical equivalence for each statement.
Proposition Reason
[(𝑝 ⟶ 𝑞) ∧ ~𝑞] Given
⟺ (~𝑝 ∨ 𝑞) ∧ ~𝑞
⟺ ~𝑞 ∧ (~𝑝 ∨ 𝑞)
⟺ (~𝑞 ∧ ~𝑝) ∨ (~𝑞 ∧ 𝑞)
⟺ (~𝑞 ∧ ~𝑝) ∨ 𝜙
⟺ (~𝑞 ∧ ~𝑝)
⟺ (~𝑝 ∧ ~𝑞)
⟺ ~(𝑝 ∨ 𝑞)

Activity 1.2
Using the conditional proposition “If I eat foods high in carbohydrates, then I will
gain weight.”, state the following:

1. Converse : _________________________________________________________
2. Contrapositive: ____________________________________________________
3. Inverse : ___________________________________________________________

8 CO_Q2_General Mathematics SHS


Module 17
What I Have Learned

Fill in the blanks with the correct words or phrases to complete the following
statements.

1. Logically equivalent propositions are _______________________________


2. Converse is a conditional statement ________________________________
3. Contrapositive is a conditional statement ___________________________
4. Inverse is a conditional statement __________________________________
5. The pairs of conditional statements which are logically equivalents are
___________________________________________________________________
6. Name and state the laws supporting logical equivalences between
propositions. _______________________________________________________

What I Can Do

This lesson helps you understand propositions which are logically equivalent. You
also have learned the three other forms of conditional propositions. In this
connection, construct a real-life conditional proposition and state its converse,
contrapositive, and inverse. State which pairs are logically equivalent and explain
what make them so.

Try scoring your essay using the rubric below.


Criteria 4 3 2 1
Statements All four Three Two One
(score x 3) conditional conditional conditional conditional
statements are statements are statements are statement is
correct. correct. correct. correct.
Reasoning Very clear and Clear and Clear Vague
(score x 2) very accurate accurate justification justification
justification justification

9 CO_Q2_General Mathematics SHS


Module 17
Assessment

Multiple Choice. Choose the letter of the best answer and write it on a separate
sheet of paper.

1. What do you call two propositions with the same truth values?
a. logically equivalent c. contradictions
b. tautologies d. fallacies

For item numbers 2-7, verify ~(𝑞 → 𝑝) ∨ (𝑝 ∧ 𝑞) ⟺ 𝑞.

2. Which of the following supports the logical equivalence


~(𝑞 → 𝑝) ∨ (𝑝 ∧ 𝑞) ⟺ ~(~𝑞 ∨ 𝑝) ∨ (𝑝 ∧ 𝑞)?
a. Absorption Law c. De Morgan’s Law
b. Commutative Law d. Switcheroo Law

3. Which of the following supports the logical equivalence


~(~𝑞 ∨ 𝑝) ∨ (𝑝 ∧ 𝑞) ⟺ (𝑞 ∧ ~𝑝) ∨ (𝑝 ∧ 𝑞)?
a. Absorption Law and Double Negation
b. Commutative Law and Double Negation
c. De Morgan’s Law and Double Negation
d. Switcheroo Law and Double Negation

4. Which of the following supports the logical equivalence

(𝑞 ∧ ~𝑝) ∨ (𝑝 ∧ 𝑞) ⟺ (𝑞 ∧ ~𝑝) ∨ (𝑞 ∧ 𝑝)?

a. Associative Law c. Distributive Law


b. Commutative Law d. Identity Law

5. Which of the following supports the logical equivalence


(𝑞 ∧ ~𝑝) ∨ (𝑞 ∧ 𝑝) ⟺ 𝑞 ∧ (~𝑝 ∨ 𝑝)?

a. Distributive Law c. Identity Law


b. Idempotent Law d. Switcheroo Law

6. Which of the following supports the logical equivalence


𝑞 ∧ (~𝑝 ∨ 𝑝) ⟺ 𝑞 ∧ 𝜏?
a. Absorption Law c. Identity Law
b. Domination Law d. Inverse Law
7. Which of the following supports the logical equivalence 𝑞 ∧ 𝜏 ⟺ 𝑞?
a. Absorption Law c. Identity Law
b. Domination Law d. Inverse Law

10 CO_Q2_General Mathematics SHS


Module 17
8. Which of the following logical equivalences is being depicted by
𝑝 ∧ (𝑞 ∨ 𝑟) ⟺ (𝑝 ∧ 𝑞) ∨ (𝑝 ∧ 𝑟)?
a. Associative Law c. Distributive Law
b. Commutative Law d. Identity Law

9. Which of the following are logically equivalents?


a. conditional and converse c. converse and contrapositive
b. contrapositive and inverse d. converse and inverse

For item numbers 10-15, refer to the conditional proposition, “If Matmat is a
responsible student, then he will do his assignments”.

10. What type of conditionals is being depicted by the conditional statement, “If
Matmat does his assignments, then he is a responsible student”?
a. converse c. inverse
b. contrapositive d. equivalence

11. Which of the following represents item number 10?


a. 𝑞 ⟶ 𝑝 c. ~𝑞 ⟶ ~𝑝
b. 𝑝 ⟶ 𝑞 d. ~𝑝 ⟶ ~𝑞

12. What type of conditionals is being depicted by the conditional statement, “If
Matmat does not do his assignments, then he is not a responsible student”?
a. converse c. inverse
b. contrapositive d. equivalence

13. Which of the following represents item number 12?


a. 𝑝 ⟶ 𝑞 c. 𝑞 ⟶ 𝑝
b. ~𝑝 ⟶ ~𝑞 d. ~𝑞 ⟶ ~𝑝

14. What type of conditionals is being depicted by the conditional statement, “If
Matmat is not a responsible student, then he will not do his assignments”?
a. converse c. inverse
b. contrapositive d. equivalence

15. Which of the following represents item number 14?


a. 𝑝 ⟶ 𝑞 c. 𝑞 ⟶ 𝑝
b. ~𝑝 ⟶ ~𝑞 d. ~𝑞 ⟶ ~𝑝

11 CO_Q2_General Mathematics SHS


Module 17
Additional Activities

A. Verify the logical equivalence (𝑝 ∧ 𝑞) → 𝑟 ⟺ [(𝑝 → 𝑟) ∨ (𝑞 → 𝑟)].


B. Think of a conditional statement. Cite its converse, contrapositive, and inverse.
Conditional: ___________________________________________________________
Converse: _____________________________________________________________
Contrapositive: ________________________________________________________
Inverse: _______________________________________________________________

12 CO_Q2_General Mathematics SHS


Module 17
Module 17
CO_Q2_General Mathematics SHS 13
What I Know What's More Assessment
1. c Activity 1.1 1. a
2. a Switcheroo Law 2. d
3. d Commutative Law 3. c
4. a
Associative Law 4. b
5. b
6. d Inverse Law 5. a
7. b Identity Law 6. d
8. c Commutative Law 7. c
9. b De Morgan’s law 8. c
10. c 9. d
11. a Activity 1.2 10. a
12. a
1. If I gain weight, then I eat 11. a
13. c
14. b foods high in carbohydrates. 12. b
15. d 2. If I do not gain weight, then 13. d
I do not eat foods high in 14. c
carbohydrates. 15. b
3. If I do not eat foods high in
carbohydrates, then I do not
gain weight.
Answer Key
References

General Mathematics Learner’s Material. First Edition. 2016. P. 263-269

*DepED Material: General Mathematics Learner’s Material

14 CO_Q2_General Mathematics SHS


Module 17
For inquiries or feedback, please write or call:

Department of Education - Bureau of Learning Resources (DepEd-BLR)

Ground Floor, Bonifacio Bldg., DepEd Complex


Meralco Avenue, Pasig City, Philippines 1600

Telefax: (632) 8634-1072; 8634-1054; 8631-4985

Email Address: blr.lrqad@deped.gov.ph * blr.lrpd@deped.gov.ph


General Mathematics
Quarter 2 – Module 18:
Tautologies and Fallacies

CO_Q2_General Mathematic SHS


Module 18
General Mathematics
Alternative Delivery Mode
Quarter 2 – Module 18: Tautologies and Fallacies
First Edition, 2021

Republic Act 8293, section 176 states that: No copyright shall subsist in any work of
the Government of the Philippines. However, prior approval of the government agency or office
wherein the work is created shall be necessary for exploitation of such work for profit. Such
agency or office may, among other things, impose as a condition the payment of royalties.

Borrowed materials (i.e., songs, stories, poems, pictures, photos, brand names,
trademarks, etc.) included in this module are owned by their respective copyright holders.
Every effort has been exerted to locate and seek permission to use these materials from their
respective copyright owners. The publisher and authors do not represent nor claim ownership
over them.

Published by the Department of Education


Secretary: Leonor Magtolis Briones
Undersecretary: Diosdado M. San Antonio

Development Team of the Module


Writers: Baby Jane B. Agudo
Editors: Elizabeth D. Lalunio, Anicia J. Villaruel, Roy O. Natividad
Reviewers: Jerry Punongbayan, Diosmar O. Fernandez, Celestina M. Alba,
Shielamarie E. Arce, Carmela Ana A. Reforma, Rafaela M. Merle
Illustrator: Hanna Lorraine Luna, Diane C. Jupiter
Layout Artist: Roy O. Natividad, Sayre M. Dialola, Argie L. Ty, Glynis P. Aviles
Management Team: Francis Cesar B. Bringas
Job S. Zape, Jr.
Ramonito Elumbaring
Reicon C. Condes
Elaine T. Balaogan
Fe M. Ong-ongowan
Elias A. Alicaya Jr.
Gregorio A. Co Jr.
Gregorio T. Mueco
Herbert D. Perez
Lorena S. Walangsumbat
Jee-Ann O. Borines
Asuncion C. Ilao

Printed in the Philippines by ________________________

Department of Education – Region 4A CALABARZON

Office Address: Gate 2 Karangalan Village, Brgy. San Isidro, Cainta, Rizal
Telefax: 02-8682-5773/8684-4914/8647-7487
E-mail Address: lrmd.calabarzon@deped.gov.ph
General Mathematics
Quarter 2 – Module 18:
Tautologies and Fallacies
Introductory Message
This Self-Learning Module (SLM) is prepared so that you, our dear learners,
can continue your studies and learn while at home. Activities, questions, directions,
exercises, and discussions are carefully stated for you to understand each lesson.

Each SLM is composed of different parts. Each part shall guide you step-by-
step as you discover and understand the lesson prepared for you.

Pre-tests are provided to measure your prior knowledge on lessons in each


SLM. This will tell you if you need to proceed on completing this module or if you
need to ask your facilitator or your teacher’s assistance for better understanding of
the lesson. At the end of each module, you need to answer the post-test to self-check
your learning. Answer keys are provided for each activity and test. We trust that you
will be honest in using these.

In addition to the material in the main text, Notes to the Teacher are also
provided to our facilitators and parents for strategies and reminders on how they can
best help you on your home-based learning.

Please use this module with care. Do not put unnecessary marks on any part
of this SLM. Use a separate sheet of paper in answering the exercises and tests. And
read the instructions carefully before performing each task.

If you have any questions in using this SLM or any difficulty in answering the
tasks in this module, do not hesitate to consult your teacher or facilitator.

Thank you.
What I Need to Know

A proposition is defined earlier as a declarative sentence that is either true or false.


You also learned that we can combine propositions using logical connectors. Are
these compound propositions always true or always false? Think of a sentence that
could never be false? How about a sentence that could never be true? To answer
these questions, this module will help you identify statements that are always true
or always false. This module is written for you to help you understand more, the
different logical statements.

The module is composed of only one lesson:


• Lesson 1 – Tautologies and Fallacies

After going through this module, you are expected to:


1. illustrate different types of tautologies and fallacies; and
2. apply tautologies and fallacies to real-life situations.

What I Know

Choose the letter of the best answer. Write the chosen letter on a separate sheet of
paper.
1. What is a proposition statement that is always true?
a. negation b. tautology c. fallacy d. absolute
2. What is a proposition statement that is always false?
a. negation b. tautology c. fallacy d. absolute

3. Which of the following is NOT a tautology?


a. Merida is brave, or she is not brave.
b. Either you will pass the test, or you will fail on the test.
c. Today is a lucky day and tomorrow is not a lucky day.
d. John loves Math or he does not love Math.
4. Which of the following is a tautology?
a. 𝑝𝑝 and 𝑞𝑞 are both true, thus 𝑝𝑝 ∧ 𝑞𝑞 is true.
b. 𝑝𝑝 is true and proposition 𝑞𝑞 is false, thus 𝑝𝑝 ∧ 𝑞𝑞 is true.
c. 𝑝𝑝 is false and proposition 𝑞𝑞 is true, thus 𝑝𝑝 ∧ 𝑞𝑞 is true.
d. propositions 𝑝𝑝 and 𝑞𝑞 are both false, thus 𝑝𝑝 ∧ 𝑞𝑞 is true.
5. Which of the following is a tautology?
a. It is a sunny day or it is not both hot and sunny day.
b. It is a sunny day or it is hot and sunny day.
c. It is a sunny day and it is not both hot and sunny day.
d. It is not hot and sunny day.

1 CO_Q2_General Mathematic SHS


Module 18
6. Which of the following statements is represented by the logical symbol of the
tautology 𝑝𝑝 ∧ ∼ (p ∨ q)?
a. Beth likes Mathematics but she does not like Math and English
b. Beth likes Mathematics but she does not like Math or English
c. Beth likes Mathematics but she does not like either Math or English
d. Beth likes Mathematics nor she does not like Math or English.
7. Let 𝑝𝑝, and 𝑞𝑞, be the propositions:
𝑝𝑝: Annie has a stomach ache.
𝑞𝑞: Annie misses the exam.
Which of the following illustrate the tautology 𝑝𝑝 ∨ ∼ (p ∧ q)?
a. Annie has a stomach ache or she misses the exam.
b. Annie has a stomach ache and she misses the exam.
c. Annie has a stomach ache or she is not both missing the exam and has
a stomach ache.
d. Annie has a stomach ache and she is either missing the exam or has a
stomach ache.

8. Which of the following is a tautology?


a. [(𝑝𝑝 → 𝑞𝑞) ∧ 𝑝𝑝] → 𝑝𝑝 b. [(𝑝𝑝 → 𝑞𝑞) ∧ 𝑝𝑝] → 𝑞𝑞
c. [~(𝑝𝑝 ∧ 𝑞𝑞) ∧ (~𝑝𝑝)] → 𝑞𝑞 d. [(𝑝𝑝 ∨ 𝑞𝑞) ∧ 𝑝𝑝] → (~𝑞𝑞)

9. Which of the following statements is a fallacy?


a. The dog is either brown, or the dog is not brown.
b. Today is a rainy day or a sunny day.
c. If today is a rainy day, then today is a hot day.
d. Today is a hot day if and only if it is a sunny day.

10. When does 𝑝𝑝 ↔ 𝑞𝑞, become false?


a. when both 𝑝𝑝 and 𝑞𝑞 are true b. when both 𝑝𝑝 and 𝑞𝑞 are false
c. when 𝑝𝑝 is true and 𝑞𝑞 is false d. cannot be determined

11. Which of the following truth table shows that the proposition
𝑝𝑝 → (𝑝𝑝 ∨ 𝑞𝑞) is a tautology?
a. 𝑝𝑝 𝑞𝑞 𝑝𝑝 ∨ 𝑞𝑞 𝑝𝑝 → (𝑝𝑝 ∨ 𝑞𝑞) b. 𝑝𝑝 𝑞𝑞 𝑝𝑝 ∨ 𝑞𝑞 𝑝𝑝 → (𝑝𝑝 ∨ 𝑞𝑞)
T T T T T T F T
T F F T T F F T
F T T T F T T T
F F F T F F F T

c. 𝑝𝑝 𝑞𝑞 𝑝𝑝 ∨ 𝑞𝑞 𝑝𝑝 → (𝑝𝑝 ∨ 𝑞𝑞) 𝑝𝑝 𝑞𝑞 𝑝𝑝 ∨ 𝑞𝑞 𝑝𝑝 → (𝑝𝑝 ∨ 𝑞𝑞)


d.
T T T T T T F T
T F T T T F F T
F T T T F T F T
F F F T F F T T

2 CO_Q2_General Mathematic SHS


Module 18
12. Which of the following truth table shows that the proposition (𝑝𝑝 ∧ (~𝑞𝑞)) ∧ (𝑝𝑝 ∧
𝑞𝑞) is a fallacy?
a. 𝑝𝑝 𝑞𝑞 ~𝑞𝑞 (𝑝𝑝 ∧ (~𝑞𝑞) 𝑝𝑝 ∧ 𝑞𝑞 (𝑝𝑝 ∧ (~𝑞𝑞)) ∧ (𝑝𝑝 ∧ 𝑞𝑞)
T T F F T F
T F T T F F
F T F F F F
F F T F F F
b. 𝑝𝑝 𝑞𝑞 ~𝑞𝑞 (𝑝𝑝 ∧ (~𝑞𝑞) 𝑝𝑝 ∧ 𝑞𝑞 (𝑝𝑝 ∧ (~𝑞𝑞)) ∧ (𝑝𝑝 ∧ 𝑞𝑞)
T T F F T F
T F F F F F
F T F T T F
F F T F F F
𝑝𝑝 𝑞𝑞 ~𝑞𝑞 (𝑝𝑝 ∧ (~𝑞𝑞) 𝑝𝑝 ∧ 𝑞𝑞 (𝑝𝑝 ∧ (~𝑞𝑞)) ∧ (𝑝𝑝 ∧ 𝑞𝑞)
c. T T F T F F
T F F F F F
F T F T T F
F F T F T F
𝑝𝑝 𝑞𝑞 ~𝑞𝑞 (𝑝𝑝 ∧ (~𝑞𝑞) 𝑝𝑝 ∧ 𝑞𝑞 (𝑝𝑝 ∧ (~𝑞𝑞)) ∧ (𝑝𝑝 ∧ 𝑞𝑞)
d.
T T F T F F
T F T T F F
F T F T T F
F F T F T F

For items 13-14, refer to this statement: Charles loves both English and
Mathematics, but he loves neither English nor Filipino.

13. Express the statement to mathematical symbol.


a. (𝑝𝑝 ∧ 𝑞𝑞) ∧ (𝑞𝑞 ∧ 𝑟𝑟) b. (∼ 𝑝𝑝 ∧∼ 𝑞𝑞) ∧ (𝑞𝑞 ∧ 𝑟𝑟)
c. (𝑝𝑝 ∧∼ 𝑝𝑝) ∧ (∼ 𝑞𝑞 ∧ 𝑟𝑟) d. (𝑝𝑝 ∧ 𝑞𝑞) ∧ (∼ 𝑞𝑞 ∧∼ 𝑟𝑟)

14. The given statement is a:


a. tautology b. fallacy
c. both tautology and fallacyd. neither tautology nor fallacy

15. Which of the following is an example of a fallacy of affirming the conclusion?


a. If I will not take a bath, then I cannot go to school.
b. If I will love you, then you will love me too.
c. Assuming that I will not study my lesson, then I will not pass the test.
d. Since I cannot buy a new laptop, then I will not attend online classes.

3 CO_Q2_General Mathematic SHS


Module 18
Lesson

1 Tautologies and Fallacies

Honesty is a value that everyone should possess but we cannot deny the fact that
everything that we say is not always true, sometimes we also say false statements.
In the same manner, not everything that we hear is true, and so we should analyze
first what we heard before we believe it. Mathematics also plays an important role in
analyzing statements, through truth tables we can check whether a statement is
always true (tautology) or always false (fallacy).

You have already learned in module 16 the term tautology, which is true for every
value of the two or more given statements. The contradiction is just the opposite of
tautology or you can contradict the tautology statement. Finding the truth values of
propositions will give you the idea if it is a tautology or a fallacy. In previous modules,
you learned how to construct the truth tables of given propositions or arguments,
while on this module your previous knowledge will be intensified as it applied to real-
life situations. Also, in the previous module, a statement that is always false is called
a contradiction, but since in other sources it is also called a fallacy, we will be using
the word fallacy this time since contradiction is an example of logical fallacy. Also,
the word fallacy will be used in module 20 to find the validity of the arguments. It is
hoped that this module will help identify true statements or analyze statements
before you accept them as true or false.

What’s In

Determine the truth value of each of the following propositions. Show the truth table
of each as well.
1. ~𝑝𝑝 ∧ 𝑞𝑞 where p and q are both true propositions
2. 𝑝𝑝 ∨ ~𝑞𝑞 where p and q are both false propositions
3. 𝑝𝑝 → 𝑞𝑞 where p is true and q is false
4. 𝑝𝑝 ↔ 𝑞𝑞 where p is false and q is true
5. (𝑝𝑝 → 𝑞𝑞) ∧ (𝑞𝑞 → 𝑝𝑝) where p and q are both true propositions
6. (𝑝𝑝 ↔ 𝑞𝑞) ∨ 𝑞𝑞 where p and q are both false
7. ~(𝑞𝑞 ∨ 𝑝𝑝) ∧ 𝑝𝑝 where p and q are both true
8. 𝑝𝑝 ∧ (𝑝𝑝 ↔ 𝑞𝑞) where p is true and q is false
9. 𝑝𝑝 → (𝑞𝑞 ∨ 𝑞𝑞) where p and q are both false
10. ~𝑝𝑝 ∧ (𝑝𝑝 → 𝑞𝑞) where p is false and q is true.

4 CO_Q2_General Mathematic SHS


Module 18
What’s New

Now, that you already know how to perform different types of operations on
propositions, I am confident that you are now ready for the new lesson.

Activity 1:
Determine whether the given statements are always true or just a mistaken belief (false
statement). Write T if the statement is always true or MB if it is a mistaken belief.
1. Today is Monday or today is not Monday.
2. Either Nicco is smart, or he is not smart.
3. If you buy a book then you will read it daily.
4. Assuming that If I plant cactus, then I will get my hands dirty. Since I didn’t
get my hands dirty, therefore I didn’t plant a cactus.
5. If I will study my lessons every day then I will have a passing grade. But, I
study my lessons every day then I will have a passing grade.
6. I love you or I don’t love you.
7. Since I like you, then you will like me too.
9. I can comprehend the writings that I read or I cannot comprehend the
writings that I read.

Activity 2:
Explain whether the given statement is true or false.
1. If I study hard, then I will get an academic award but I will study hard.
Therefore, I will get an academic award.
2. Blessy loves both swimming and running, but she loves neither swimming nor
running.

What is It

A tautology is a compound statement that is true for every value of the individual
statements. The word tautology is derived from a Greek word where ‘tauto’ means
‘same’ and ‘logy’ means ‘logic’.
The simple examples of tautology are:
• Either Mari will buy apples or Mari will not buy apples.
• My pet Yummy is healthy or he is not healthy
• A function is a polynomial function or it is not a polynomial function.

5 CO_Q2_General Mathematic SHS


Module 18
Some important tautologies are presented below:
Laws that illustrates tautologies Possible Statements
Contraposition (𝑝𝑝 → 𝑞𝑞) ↔ (~𝑞𝑞 → ~𝑝𝑝)
Reduction ad absurdum ~(𝑝𝑝 → 𝑞𝑞) ↔ (𝑝𝑝 ∧ ~𝑞𝑞)
~(𝑝𝑝 ∧ 𝑞𝑞) ↔ ( ~𝑝𝑝 ∨ ~𝑞𝑞)
De Morgan’s Laws
~(𝑝𝑝 ∨ 𝑞𝑞) ↔ ( ~𝑝𝑝 ∧ ~𝑞𝑞)
Modus Ponens [(𝑝𝑝 → 𝑞𝑞) ∧ 𝑝𝑝] → 𝑞𝑞
Exportation [𝑝𝑝 → (𝑟𝑟 → 𝑞𝑞)] ↔ [(𝑝𝑝 ∧ 𝑟𝑟) → 𝑞𝑞]
Transitivity (𝑝𝑝 → 𝑟𝑟) ∧ (𝑟𝑟 → 𝑞𝑞) → (𝑝𝑝 → 𝑞𝑞)
Deduction (𝑝𝑝 → 𝑟𝑟) ∧ [(𝑝𝑝 ∧ 𝑟𝑟) → 𝑞𝑞] → (𝑝𝑝 → 𝑞𝑞)

To determine whether a given statement is a tautology, you can use the table of truth
values.

Example:
If I follow the school rules and regulations, then I am a disciplined person, but I
follow the school rules and regulations, therefore I am a disciplined person. Show
that the given statement is a tautology.

Solution:
Step 1: Translate the given statement into symbols.
Let 𝑝𝑝: I follow school rules and regulation.
𝑞𝑞: I am a discipline person.
The statement can be written in symbols as [(𝑝𝑝 → 𝑞𝑞) ∧ 𝑝𝑝] → 𝑞𝑞.

Step 2: Construct the truth table of the given statements.


𝑝𝑝 𝑞𝑞 𝑝𝑝 → 𝑞𝑞 (𝑝𝑝 → 𝑞𝑞) ∧ 𝑝𝑝 [(𝑝𝑝 → 𝑞𝑞) ∧ 𝑝𝑝] → 𝑞𝑞
T T T T T
T F F F T
F T T F T
F F T F T

Step 3: Make a conclusion.


Since the result is all true, it shows that the given statement is a tautology.
Observe also, that the statement illustrates Modus Ponens.

Fallacy is a statement that is always false. (in previous modules it is also a


contradiction)

Examples:
1. Twelve is an odd number.
2. Cats can fly.
3. A triangle has four sides.
4. 𝑎𝑎𝑎𝑎 = 𝑏𝑏 2
5. Today is Monday and Tuesday

6 CO_Q2_General Mathematic SHS


Module 18
Sometimes fallacy is also described as a mistaken idea based on unsound
arguments. Some commonly used fallacies are as follows:
1. Affirming the conclusion – a fallacy of affirming the conclusion is incorrect
reasoning in proving 𝑝𝑝 → 𝑞𝑞 by starting by assuming 𝑞𝑞 and proving 𝑝𝑝.
Example: Assuming that I smile to you, then I am happy to see you.
If you answer all the assignments, then you will learn Math.

2. Denying the hypothesis – a fallacy of denying the hypothesis is incorrect


reasoning in proving 𝑝𝑝 → 𝑞𝑞 by starting with assuming ~𝑝𝑝 and proving ~𝑞𝑞.
Example: Assuming that I will not eat sweet foods, then I will not be a diabetic.
Since I am not hungry, then I will not eat.

3. Circular reasoning – a fallacy in which the reasoner begins with what they are
trying to end with.
Example: A chicken must come from an egg.
But, an egg cannot exist without a chicken laying it.
But, a chicken must come from an egg…

Like tautology, truth table can be also used to show that a statement is a fallacy.
Example: Denise loves both singing and dancing, but she loves neither dancing nor
acting.
Solution:
Step 1: Translate the given statement into symbols.
Let 𝑝𝑝: Denise loves singing.
𝑞𝑞: Denise loves dancing.
𝑟𝑟: Denise loves acting.
The statement can be written in symbols as (𝒑𝒑 ∧ 𝒒𝒒) ∧ (∼ 𝒒𝒒 ∧∼ 𝒓𝒓).

Step 2: Construct the truth table of the given statements.


𝑝𝑝 𝑞𝑞 𝑟𝑟 ∼ 𝑞𝑞 ∼ 𝑟𝑟 𝑝𝑝 ∧ 𝑞𝑞 ∼ 𝒒𝒒 ∧∼ 𝒓𝒓 (𝒑𝒑 ∧ 𝒒𝒒) ∧ (∼ 𝒒𝒒 ∧∼ 𝒓𝒓)
T T T F F T F F
T T F F T T F F
T F T T F F F F
T F F T T F T F
F T T F F F F F
F T F F T F F F
F F T T F F F F
F F F T T F T F

Step 3: Make a conclusion.


Since the result is all false, it shows that the given statement is a fallacy.

7 CO_Q2_General Mathematic SHS


Module 18
What’s More

Activity 1.1
Determine whether the statement is a tautology or fallacy.
1. If today is Saturday, then tomorrow is Monday
2. Assuming that I am a college degree, then I am a teacher.
3. Either Joshua will buy books or Joshua will not buy books.
4. If I will study hard, then I will pass the examination, but I studied hard,
therefore I passed the examination
5. Aldrin loves both Math and Science, but he loves neither Science nor
English

Activity 1.2
Construct a truth table for each of the following to determine whether the given is a
tautology or a fallacy.
1. (𝑝𝑝 → 𝑞𝑞) ↔ (~𝑞𝑞 → ~𝑝𝑝)
2. ~(𝑝𝑝 ∧ 𝑞𝑞) ↔ ( ~𝑝𝑝 ∨ ~𝑞𝑞)
3. ~(𝑝𝑝 → 𝑞𝑞) ↔ (𝑝𝑝 ∧ ~𝑞𝑞)
4. (𝑝𝑝 ∧ 𝑞𝑞) ↔ (𝑝𝑝 → ~𝑞𝑞)
5. 𝑞𝑞 ∧ ~(𝑝𝑝 ∨ 𝑞𝑞)

What I Have Learned

A. Fill in the blanks with the correct term or phrase to complete the sentence.
1. A _____________________ is a statement which is true for every value of the
individual statements.
2. _________________ is statement which is always false.
3. __________________ is an incorrect reasoning in proving 𝑝𝑝 → 𝑞𝑞 by starting by
assuming 𝑞𝑞 and proving 𝑝𝑝.
4. __________________ is an incorrect reasoning in proving 𝑝𝑝 → 𝑞𝑞 by starting with
assuming ~𝑝𝑝 and proving ~𝑞𝑞.
5. ___________________ a fallacy in which the reasoner begins with what they are
trying to end with.

B. How important is your knowledge about tautologies and fallacy in real-life


situations?
______________________________________________________________________________
______________________________________________________________________________

8 CO_Q2_General Mathematic SHS


Module 18
What I Can Do

On your Own!
Make a compound statement which is either a tautology or a fallacy and
write the statement in symbol. Then, construct a truth table to show that the
statement given is a tautology or a fallacy.
The following rubric will be used to rate your work for each of the four problems:
Criteria 4 3 2 1
The statement One of the Two of the Three or more
Translating
is translated logical logical logical
the
into symbols connectors connectors connectors
statements
with correct used is used are used are
into
logical incorrect incorrect incorrect
symbols
connectors.
The truth One of the Two of the Three or more
Accuracy table has values in the values in the of the values
of the complete and truth table is truth table are in the truth
truth table accurate truth incorrect incorrect table are
values incorrect
Consistently Somewhat Somewhat No textual
logical; aids logical; illogical; tends explanation or
clear and easy somewhat to complicate table of values
Proof
understanding aids clear or the of solution or
statement
of the solution easy understanding answer.
understanding of the solution
of the solution

Assessment

Choose the letter of the best answer. Write the chosen letter on a separate sheet of
paper.
1. Which of the following is a tautology?
a. It is a compound statement that is always true.
b. It is a compound statement that is not always true.
c. It is a compound statement that is sometimes true.
d. It is a compound statement that is neither true nor false.

9 CO_Q2_General Mathematic SHS


Module 18
2. Which of the following is a fallacy?
a. It is a compound statement that is always true.
b. It is a compound statement that is not always true.
c. It is a compound statement that is sometimes true.
d. It is a compound statement that is neither true nor false.

3. Which of the following is NOT a tautology?


a. Maria is smart, or she is not smart.
b. Either you will pass the interview, or you will fail in the interview.
c. Anna loves Arts or she does not love Arts.
d. Today is a not lucky day and tomorrow is a lucky day.

4. Which of the following is a tautology?


a. [(𝑝𝑝 → 𝑞𝑞) ∧ 𝑝𝑝] → 𝑝𝑝 b. (𝑝𝑝 → 𝑞𝑞) ↔ (~𝑞𝑞 → 𝑝𝑝)
c. [[𝑝𝑝 → (𝑟𝑟 → 𝑞𝑞)] ↔ [(𝑝𝑝 ∧ 𝑟𝑟) → 𝑞𝑞] d. (𝑝𝑝 → 𝑟𝑟) ∧ [(𝒑𝒑 ↔ 𝒓𝒓) → 𝑞𝑞] → (𝑝𝑝 → 𝑞𝑞)

5. Which of the following is a tautology?


a. Michael is an artist or not an artist.
b. Michael is an artist and a singer.
c. It is not true that Michael is both an artist and a singer
d. Michel is not a singer.

6. Which of the following statements is represented by the logical symbol of the


tautology 𝑝𝑝 ∧ ∼ (p ∨ q)?
a. Jeff likes Music but he does not like Music and Arts
b. Jeff likes Music but he does not like Music or Arts
c. Jeff likes Music nor he does not like Music or Arts.
d. Jeff likes Music but he does not like either Music or Arts

7. Let 𝑝𝑝, and 𝑞𝑞, be the propositions:


𝑝𝑝: Ivy was absent.
𝑞𝑞: Ivy misses the review.
Which of the following illustrates the tautology 𝑝𝑝 ∨ ∼ (p ∧ q)?
a. Ivy was absent or she misses the review.
b. Ivy was absent or she is not both missing the review and she was
absent.
c. Ivy was absent and she misses the review.
d. Ivy was absent and she is either missing the review or she was absent

8. Which of the following is a NOT tautology?


a. [(𝑝𝑝 → 𝑞𝑞) ∧ 𝑝𝑝] → 𝑝𝑝 b. (𝑝𝑝 → 𝑞𝑞) ↔ (~𝑞𝑞 → ~𝑝𝑝)
c. [[𝑝𝑝 → (𝑟𝑟 → 𝑞𝑞)] ↔ [(𝑝𝑝 ∧ 𝑟𝑟) → 𝑞𝑞] d. (𝑝𝑝 → 𝑟𝑟) ∧ [(𝑝𝑝 ∧ 𝑟𝑟) → 𝑞𝑞] → (𝑝𝑝 → 𝑞𝑞

9. Which of the following statement is a fallacy?


a. The dog is either brown, or the dog is not brown.
b. Today is a rainy day or a sunny day.
c. If tomorrow is Monday, then today is Saturday.
d. Today is a hot day if and only if it is a sunny day.

10 CO_Q2_General Mathematic SHS


Module 18
10. Which of the following is a fallacy?
a. (𝑝𝑝 ∧ 𝑞𝑞) ↔ (𝑝𝑝 → ~𝑞𝑞) b. (𝑝𝑝 → 𝑞𝑞) ↔ (~𝑞𝑞 → ~𝑝𝑝)
c. [[𝑝𝑝 → (𝑟𝑟 → 𝑞𝑞)] ↔ [(𝑝𝑝 ∧ 𝑟𝑟) → 𝑞𝑞] d. (𝑝𝑝 → 𝑟𝑟) ∧ [(𝑝𝑝 ∧ 𝑟𝑟) → 𝑞𝑞] → (𝑝𝑝 → 𝑞𝑞

11. Which of the following truth table shows that the proposition 𝑝𝑝 ∨ (𝑞𝑞 → ~𝑝𝑝) is a
tautology?

a. 𝑝𝑝 𝑞𝑞 ~𝑝𝑝 𝑞𝑞 → ~𝑝𝑝 𝑝𝑝 ∨ (𝑞𝑞 → ~𝑝𝑝)


T T F F T
T F F T T
F T T T T
F F T T T
b. 𝑝𝑝 𝑞𝑞 ~𝑝𝑝 𝑞𝑞 → ~𝑝𝑝 𝑝𝑝 ∨ (𝑞𝑞 → ~𝑝𝑝)
T T F F T
T F F F T
F T T T T
F F T T T

𝑝𝑝 𝑞𝑞 ~𝑝𝑝 𝑞𝑞 → ~𝑝𝑝 𝑝𝑝 ∨ (𝑞𝑞 → ~𝑝𝑝)


c.
T T F T T
T F F F T
F T T F T
F F T F T

𝑝𝑝 𝑞𝑞 ~𝑝𝑝 𝑞𝑞 → ~𝑝𝑝 𝑝𝑝 ∨ (𝑞𝑞 → ~𝑝𝑝)


d. T T T F T
T F T T T
F T F T T
F F F T T

12. Which of the following truth table shows that the proposition (𝑝𝑝 ∧ �(𝑞𝑞 ∨ (~𝑝𝑝)� ∧
(~𝑞𝑞)] is a fallacy?

a. 𝑝𝑝 𝑞𝑞 ~𝑝𝑝 ~𝑞𝑞 (𝑞𝑞 ∨ (~𝑝𝑝) �(𝑞𝑞 ∨ (~𝑝𝑝)� ∧ (~𝑞𝑞)] (𝑝𝑝 ∧ �(𝑞𝑞 ∨ (~𝑝𝑝)� ∧ (~𝑞𝑞)]
T T F F F F F
T F F T F F F
F T T F F F F
F F T T T T F

b. 𝑝𝑝 𝑞𝑞 ~𝑝𝑝 ~𝑞𝑞 (𝑞𝑞 ∨ (~𝑝𝑝) �(𝑞𝑞 ∨ (~𝑝𝑝)� ∧ (~𝑞𝑞)] (𝑝𝑝 ∧ �(𝑞𝑞 ∨ (~𝑝𝑝)� ∧ (~𝑞𝑞)]
T T F F T F F
T F F T F F F
F T T F T F F
F F T T T T F

11 CO_Q2_General Mathematic SHS


Module 18
c. 𝑝𝑝 𝑞𝑞 ~𝑝𝑝 ~𝑞𝑞 (𝑞𝑞 ∨ (~𝑝𝑝) �(𝑞𝑞 ∨ (~𝑝𝑝)� ∧ (~𝑞𝑞)] (𝑝𝑝 ∧ �(𝑞𝑞 ∨ (~𝑝𝑝)� ∧ (~𝑞𝑞)]
T T T F T F F
T F T T F F F
F T F F T F F
F F F T T T F
d. 𝑝𝑝 𝑞𝑞 ~𝑝𝑝 ~𝑞𝑞 (𝑞𝑞 ∨ (~𝑝𝑝) �(𝑞𝑞 ∨ (~𝑝𝑝)� ∧ (~𝑞𝑞)] (𝑝𝑝 ∧ �(𝑞𝑞 ∨ (~𝑝𝑝)� ∧ (~𝑞𝑞)]
T T F T T F F
T F F T F F F
F T T F T F F
F F T F T T F

For items 13-14, refer to this statement: Carmela loves both Badminton and
Volleyball, but she loves neither Badminton nor Tennis.

13. Express the statement to a mathematical symbol.


a. (𝑝𝑝 ∧ 𝑞𝑞) ∧ (𝑞𝑞 ∧ 𝑟𝑟) b. (𝑝𝑝 ∧ 𝑞𝑞) ∧ (∼ 𝑞𝑞 ∧∼ 𝑟𝑟)
c. (𝑝𝑝 ∧∼ 𝑝𝑝) ∧ (∼ 𝑞𝑞 ∧ 𝑟𝑟) d. (∼ 𝑝𝑝 ∧∼ 𝑞𝑞) ∧ (𝑞𝑞 ∧ 𝑟𝑟)

14. The given statement is a:


a. tautology b. fallacy
c. both tautology and fallacy d. neither tautology not fallacy

15. Which of the following is an example of a fallacy denying the hypothesis?


a. If I will not take a bath, then I cannot go to school.
b. If I will love you, then you will love me too.
c. Assuming that I will study my lesson, then I will not pass the test.
d. Assuming that I will not go to school every day, then I will not fail.

Additional Activities

Draw a conclusion!

1. If I pass the final exam, I will graduate. I graduated. Therefore…


2. I will go to La Union, I will eat “Halo-Halo” de Iloko. If I eat “Halo-Halo”, I gain
weight. Therefore,…
3. Rachel and Paulo will be at a concert. Rachel was at the concert. Therefore,…
4. If a person is an engineer, then that person has a college degree. Dennis does not
have a college degree. Therefore,…
5. If I am irritated or angry, I cannot concentrate. I can concentrate. Therefore,…

12 CO_Q2_General Mathematic SHS


Module 18
Module 18
CO_Q2_General Mathematic SHS 13
What I Know What's More Assessment
1. b 1. a.
Activity 1.1
2. c 2. b
1. Fallacy 2. Fallacy 3. Tautology
3. c 4. Tautology 5. Fallacy 3. d
4. a 4. c
Activity 1.2
5. a 𝑝𝑝 𝑞𝑞 𝑝𝑝 → 𝑞𝑞 ~𝑝𝑝 ~𝑞𝑞 ~𝑞𝑞 → ~𝑝𝑝 (𝑝𝑝 → 𝑞𝑞) ↔ (~𝑞𝑞 5. a
6. c T T T F F T T 6. d
→ ~𝑝𝑝)
T F F F T F T
7. c F T T T F T T 7. b
F F T T T T T
8. a
8. b Since the result is all true, it shows that the given
statement is a tautology. 9. c
9. c
10. a
10. c
𝑝𝑝 𝑞𝑞 𝑝𝑝 ∧ 𝑞𝑞 ~(𝑝𝑝 ~𝑝𝑝 ~𝑞𝑞 ~𝑝𝑝 ∨ ~𝑞𝑞 ~(𝑝𝑝 ∧ 𝑞𝑞)
T T T F F F F T 11. a
11. c
∧ 𝑞𝑞 ↔ ( ~𝑝𝑝 ∨ ~𝑞𝑞)
T F F T F T T T
F T F T T F T T 12. b
12. a
F F F T T T T T
13. b
13. d Since the result is all true, it shows that the given
statement is a tautology. 14. a
14. b
15. d
15. b
𝑝𝑝 𝑞𝑞 (𝑝𝑝 ~(𝑝𝑝 ~𝑞𝑞 (𝑝𝑝 ~(𝑝𝑝 → 𝑞𝑞)
T T T F F F T
→ 𝑞𝑞) → 𝑞𝑞) ∧ ~𝑞𝑞) ↔ (𝑝𝑝 ∧ ~𝑞𝑞)
T F F T T T T
F T T F F F T
F F T F T F T
Since the result is all true, it shows that the given
statement is a tautology.
𝑝𝑝 𝑞𝑞 ~𝑞𝑞 (𝑝𝑝 (𝑝𝑝 → ~𝑞𝑞) (𝑝𝑝 ∧ 𝑞𝑞)
T T F T F F
∧ 𝑞𝑞) ↔ (𝑝𝑝 → ~𝑞𝑞)
T F T F T F
F T F F T F
F F T F T F
Since the result is all false, it shows that the given
statement is a fallacy.
𝑝𝑝 𝑞𝑞 𝑝𝑝 ∨ 𝑞𝑞 ~( 𝑝𝑝 ∨ 𝑞𝑞) 𝑞𝑞 ∧ ~( 𝑝𝑝
∨ 𝑞𝑞)
T T T F F
T F T F F
F T T F F
F F F T F
Since the result is all false, it shows that the given
statement is a fallacy.
Answer Key
References
Oronce, Orlando. A. General Mathematics. Quezon City: Rex Bookstore, Inc.,2016.
Pp. 304-319

DepEd BLR (2016). General Mathematics, first ed., DepEd Philippines

Tautology in Math. https://tutors.com/math-tutors/geometry-help/tautology- in-


math-definition-examples

Byjus the learning app. https://byjus.com/maths/tautology/

Quipper Tautologies and Fallacy https://link.quipper.com/en/organizations/5468b


d782294ee08a80000ec/curriculum

The FoundationsL Logic and proofshttps://faculty.nps.edu/rgera/current_classes/


MA2025/Section1.6.pdf

14 CO_Q2_General Mathematic SHS


Module 18
For inquiries or feedback, please write or call:

Department of Education - Bureau of Learning Resources (DepEd-BLR)

Ground Floor, Bonifacio Bldg., DepEd Complex


Meralco Avenue, Pasig City, Philippines 1600

Telefax: (632) 8634-1072; 8634-1054; 8631-4985

Email Address: blr.lrqad@deped.gov.ph * blr.lrpd@deped.gov.ph


General Mathematics
Quarter 2 – Module 19:
Validity of Categorical
Syllogisms

CO_Q2_General Mathematic SHS


Module 19
General Mathematics
Alternative Delivery Mode
Quarter 2 – Module 19: Validity of Categorical Syllogisms
First Edition, 2021

Republic Act 8293, section 176 states that: No copyright shall subsist in any work of
the Government of the Philippines. However, prior approval of the government agency or office
wherein the work is created shall be necessary for exploitation of such work for profit. Such
agency or office may, among other things, impose as a condition the payment of royalties.

Borrowed materials (i.e., songs, stories, poems, pictures, photos, brand names,
trademarks, etc.) included in this module are owned by their respective copyright holders.
Every effort has been exerted to locate and seek permission to use these materials from their
respective copyright owners. The publisher and authors do not represent nor claim ownership
over them.

Published by the Department of Education


Secretary: Leonor Magtolis Briones
Undersecretary: Diosdado M. San Antonio

Development Team of the Module


Writers: Nestor N. Sandoval
Editors: Elizabeth D. Lalunio, Anicia J. Villaruel, Roy O. Natividad
Reviewers: Jerry Punongbayan, Diosmar F. Fernandez, Celestina M. Alba,
Shielamarie E. Arce, Carmela Ana A. Reforma, Rafaela M. Merle
Illustrator: Hanna Lorraine Luna, Diane C. Jupiter
Layout Artist: Roy O. Natividad, Sayre M. Dialola, Argie L. Ty, Glynis P. Aviles
Management Team: Francis Cesar B. Bringas
Job S. Zape, Jr.
Ramonito Elumbaring
Reicon C. Condes
Elaine T. Balaogan
Fe M. Ong-ongowan
Elias A. Alicaya Jr.
Gregorio A. Co Jr.
Gregorio T. Mueco
Herbert D. Perez
Lorena S. Walangsumbat
Jee-Ann O. Borines
Asuncion C. Ilao

Printed in the Philippines by ________________________

Department of Education – Region 4A CALABARZON

Office Address: Gate 2 Karangalan Village, Brgy. San Isidro, Cainta, Rizal
Telefax: 02-8682-5773/8684-4914/8647-7487
E-mail Address: lrmd.calabarzon@deped.gov.ph
General Mathematics
Quarter 2 – Module 19:
Validity of Categorical
Syllogisms
Introductory Message
This Self-Learning Module (SLM) is prepared so that you, our dear learners,
can continue your studies and learn while at home. Activities, questions, directions,
exercises, and discussions are carefully stated for you to understand each lesson.

Each SLM is composed of different parts. Each part shall guide you step-by-
step as you discover and understand the lesson prepared for you.

Pre-tests are provided to measure your prior knowledge on lessons in each


SLM. This will tell you if you need to proceed on completing this module or if you
need to ask your facilitator or your teacher’s assistance for better understanding of
the lesson. At the end of each module, you need to answer the post-test to self-check
your learning. Answer keys are provided for each activity and test. We trust that you
will be honest in using these.

In addition to the material in the main text, Notes to the Teacher are also
provided to our facilitators and parents for strategies and reminders on how they can
best help you on your home-based learning.

Please use this module with care. Do not put unnecessary marks on any part
of this SLM. Use a separate sheet of paper in answering the exercises and tests. And
read the instructions carefully before performing each task.

If you have any questions in using this SLM or any difficulty in answering the
tasks in this module, do not hesitate to consult your teacher or facilitator.

Thank you.
What I Need to Know

Thinking and communicating clearly and coherently are ways of logic. It is an


essential area of study which has various applications in your daily life activities.
Your belief can be affected by the way you reason, which ultimately affects the way
you behave and live your life. Therefore, reasoning correctly is of utmost importance.

You encounter arguments in your everyday life, you read them in books and
magazines, and you hear them on television. Even in most examination problems
involving logic are included. Without knowing, you sometimes formulate them when
communicating with other people. Logic will help you to discourse and deal with
others with correct reasoning. Making logic a habit of your thinking makes you
improve your critical thinking skills. As you develop methods and techniques used
to distinguish correct reasoning from incorrect one, you become more confident.

As you may already know the main goal in logic is to determine the validity of
arguments. This module was designed and written to help you know about
determining the validity of categorical syllogisms. It covers varied situations that can
be seen in real life. It is hoped that upon exploring this learning kit you will be eager
and enthusiastic in completing the task required. Although studying logic is
challenging, it is sometimes fun. Good luck!

After going through this module, you are expected to:

1. define categorical syllogism;


2. differentiate Euler’s diagram from Venn diagram;
3. enumerate the different set of rules in categorical syllogism; and
4. determine the validity of categorical syllogism.

1 CO_Q2_General Mathematic SHS


Module 19
What I Know

Let’s find out how far you might already know about this topic! Please take this
challenge! Have Fun!

Choose the letter of the best answer. Write the chosen letter on a separate sheet of
paper.

1. It is the subject term of the conclusion.


a. major term
b. major premise
c. minor term
d. minor premise

2. It has two categorical premises and a conclusion.


a. categorical syllogism
b. conjunctive syllogism
c. hypothetical syllogism
d. logical syllogism

3. It appears in both premises but not in the conclusion.


a. major term
b. minor term
c. middle term
d. minimum term

4. Determine the quantity of the categorical proposition: Some teachers are Math
teachers.
a. affirmative
b. negative
c. particular
d. universal

5. What type of categorical proposition is: No successful persons are lazy.


a. A
b. E
c. I
d. O

2 CO_Q2_General Mathematic SHS


Module 19
For number 6-7, refer to the given categorical syllogism below:

No P are M.
Some M are S.
∴Some S are not P

6. Determine the form of the given standard categorical syllogism.


a. IIE-3
b. EIO-4
c. IOO-1
d. EII-1

7. Which is the correct diagram of the standard categorical syllogism?

8. How many standard forms of categorical syllogisms are there?


a. 256
b. 24
c. 15
d. 9

9. Which of the following letter name is used as symbol for particular negative
categorical syllogism?
a. A
b. E
c. I
d. O

For numbers 10-11, refer to the given categorical syllogism below:

No superb athletes are lazy. No lazy people are disciplined.


Therefore, all superb athletes are disciplined.

10. Which is the major term in the given categorical syllogism?


a. all people
b. lazy people
c. disciplined people
d. superb athletes

3 CO_Q2_General Mathematic SHS


Module 19
11. Which is the correct diagram of the standard categorical syllogism?

(Let S=superb athletes; D=disciplined people and L=lazy people)

For numbers 12-13, refer to the given categorical syllogism below:

All rectangles are parallelograms. All squares are rectangles.


Therefore, all squares are parallelograms.

12. What is the middle term of the given categorical syllogism?


a. parallelogram
b. rectangles
c. rhombus
d. squares

13. Which is the correct Euler’s diagram of the standard categorical syllogism?

(Let S=Squares; R=rectangles and P=parallelograms)

14. Determine the form of the categorical syllogism: Some great teachers are
caring.
All caring people are encouraging. Therefore, some great teachers are
encouraging.
a. IAI-4
b. AII-1
c. AII-3
d. IAI-3

4 CO_Q2_General Mathematic SHS


Module 19
15. Which of the following categorical syllogisms is INVALID?
a. No P are M. Some M are S. Therefore, some S are not P.
b. No superb athletes are lazy. No lazy people are disciplined. Therefore, all
superb athletes are disciplined.
c. All rectangles are parallelogram. All squares are rectangles. Therefore, all
squares are parallelogram.
d. Some great teachers are caring. All caring people are encouraging.
Therefore, some great teachers are encouraging.

5 CO_Q2_General Mathematic SHS


Module 19
Lesson
Validity of Categorical
1 Syllogisms
Deductive and inductive arguments are among the best-known types of arguments.
A categorical syllogism is a deductive argument consisting of exactly three categorical
propositions (two premises and a conclusion). It is said to be valid if and only if the
conjunction of the premises implies the conclusion. There are four ways to determine
the validity of categorical syllogisms: Check the form against the list of valid
arguments; use Venn diagrams; draw a picture of the premises using Euler’s
diagrams and lastly, use a set of rules. You may use any method in determining the
validity of the categorical syllogisms. However, only two methods will be intensively
discussed in this module.

What’s In

Before you proceed with the lesson, you should be able to recall how to construct a
Venn diagram to represent a given statement.

Activity 1: A Visit to Historical Place


Mr. Sandoval wants his children to instill the value of patriotism to his four
children. This summer, he is planning to visit with his family one historical place in
the Philippines. He has not yet decided if they will go to Corregidor Island in Bataan
or will visit the Emilio Aguinaldo Shrine in Kawit, Cavite. To help him decide, he
asked his four children which of the two historical places they want to explore. Their
answers are summarized in a Venn diagram below:

6 CO_Q2_General Mathematic SHS


Module 19
Let B = set of children who want to visit Corregidor Island, Bataan
C = set of children who want to visit Emilio Aguinaldo Shrine, Kawit,Cavite

Who wants to visit:


1. Bataan?
2. Cavite?
3. Bataan only?
4. Cavite only?
5. Cavite and Bataan?
6. Neither of the two spots?

Notes to the Teacher

The students may use any method in determining the validity of the
categorical syllogisms. Emphasize that validity is not determined by
the actual truth values of the premises or the conclusion. It is
determined solely by the relationship between the premises and the
conclusion. It doesn’t matter if the premises are true or false. It is
the matter of structure of this argument. Meanwhile a categorical
syllogism is sound if and only if it is both valid, and all of its
premises are actually true. Soundness of an argument was not
discussed in this module. Also, the distribution of terms was not
also covered here which is a necessary concept to apply the set of
rules specifically rules 2 and 3.

7 CO_Q2_General Mathematic SHS


Module 19
What’s New

Activity 2: What’s that Prove?

Using your reasoning, determine whether the following are VALID or INVALID.
Justify your answers. You may use illustrations to support your claim.

1. God is love; love is blind. Therefore, God is blind.


2. All hardworking students are friendly. No friendly student violates school
policies. Therefore, no violator of school policies is a hardworking student.
3. All monkeys eat banana, Nestor eats banana. Therefore, Nestor is a monkey.
4. All frogs can jump, horses can jump. Therefore, a horse is a frog.
5. All SHS students are honest. Some creative thinkers are SHS students.
Therefore, some creative thinkers are honest.
6. All birds can fly, airplanes can also fly. Therefore, an airplane is a bird.
7. Laila has a heart; banana also has a heart. Therefore, Laila is a banana.
8. All pigs have 4 legs, tables have 4 legs. Therefore, a table is a pig.
9. Angelica is beautiful, a mountain is also beautiful. Therefore, Angelica is a
mountain.
10. All donors for the “Pasko with Ate and Kuya” are successful persons. No
successful persons are lazy persons. Therefore, no lazy persons are donors for
the “Pasko with Ate and Kuya”.

Guide Questions:
1. How did you find the activity?
2. Were you able to determine the validity of each statement?
3. What strategies or methods did you use to justify the validity or invalidity of
each statement?
4. Did you consider the truth value of each proposition in the syllogisms to
determine each validity? Why?
5. How can we determine if the statements just like presented are valid or not?
6. In your own words, what is validity?

8 CO_Q2_General Mathematic SHS


Module 19
What is It

The theory of categorical propositions was originated by Aristotle. It remains


important even today since many of the statements used in ordinary language can
be translated into standard categorical propositions. The method used in logic
provides an approach and framework for some kinds of linguistic usage. In this
lesson, you will learn the methods of how to find valid categorical syllogisms. It
includes using Euler’s and Venn diagram, use of a set of rules and how to look for a
table of valid categorical syllogisms.

Before you begin examining the validity of categorical syllogisms, you must know
first the nature of categorical propositions and syllogisms.

Nature of Terms in Categorical Propositions

A categorical proposition is a proposition that relates two classes or categories. It


is composed of a quantifier, a subject term, a copula and a predicate term.
Quantifiers like ‘‘all,’’ ‘‘no,’’ and ‘‘some’’ specify how much of the subject class is
included in or excluded from the predicate class. Copula like ‘‘are’’ and ‘‘are not’’ link
(or ‘‘couple’’) the subject term with the predicate term.

Example 1. Identify the quantifier, subject, copula and predicate in the categorical
proposition: All cookery students are TVL students.

Answer. The following is an illustration of the quantifier, subject, copula and


predicate of a categorical proposition.

The quantity of a categorical proposition is either universal or particular. Universal


like ‘‘All S are P’’ and ‘‘No S are P’’ assert something about every member of the S
class. Particular like ‘‘Some S are P’’ and ‘‘Some S are not P’’ assert something about
one or more members of the S class.

Example 2. The following are the quantities of the given propositions.


Categorical Propositions Quantifiers Quantity
All Cookery students are TVL students. All universal
No TVL students are academic students. No universal
Some SHS students are TVL students. Some Particular
Some TVL students are not academic students. Some particular

9 CO_Q2_General Mathematic SHS


Module 19
The quality of a categorical proposition is either affirmative or negative. Affirmative
quality like ‘‘All S are P’’ and ‘‘Some S are P’’ affirms class membership. Negative
quality like ‘‘No S are P’’ and ‘‘Some S are not P’’ denies class membership.

Example 3. The following are the qualities of the given propositions.

Quantifiers/
Categorical Propositions Quality
Qualifiers
All Cookery students are TVL students. All, are affirmative
No TVL students are academic students. No, are negative
Some SHS students are TVL students. Some, are affirmative
Some TVL students are not academic students. Some, not negative

Four Basic Types of Categorical Propositions:


The four kinds of categorical propositions have commonly been designated by letter
names corresponding to the first four vowels of the Roman alphabet: A, E, I, O. It
may be presented as follows:

Quantity/Quality Affirmative Negative


A E
Universal
(All S are P.) (No S are P.)
I O
Particular
(Some S are P.) (Some S are not P)

A standard form categorical proposition occurs in one of the following four forms:
All S are P. No S are P. Some S are P. Some S are not P.

Example 4. Express the following into standard categorical propositions.


1. Every TVL student is a SHS student.
2. Most Quezonians are God-fearing.
3. The teacher needs an internet connection.
4. None TVL students are academic student.
5. All great leaders are not arrogant.

Answers.
1. All TVL students are SHS students.
2. Some Quezonians are God-fearing people.
3. All teachers are the person who needs internet connection.
4. No TVL students are academic students.
5. No great leaders are arrogant people.

Remember: Subject and predicate terms need to be expressed as classes of things


(nouns or noun phrases).

10 CO_Q2_General Mathematic SHS


Module 19
Nature of Terms in Categorical Syllogisms

A syllogism is a deductive argument in which a conclusion is inferred from two


premises.

A categorical syllogism is an argument consisting of exactly three categorical


propositions (two premises and a conclusion) in which there appears a total of exactly
three categorical terms, each of which is used exactly twice.

Terms of the Categorical Syllogism

1. Major term is the predicate of the conclusion.


2. Minor term is the subject term of the conclusion.
3. Middle term is the term that appears in both premises but not in the
conclusion.

Parts of the Categorical Syllogism

1. Major premise contains the major term.


2. Minor premise contains the minor term.

A standard form categorical syllogism fulfils all of the following criteria:

1. All three statements are standard-form categorical propositions.


2. The two occurrences of each term are identical.
3. Each term is used in the same sense throughout the argument.
4. The major premise is listed first, the minor premise second, and the conclusion,
last.

Arguments in ordinary language may be offered in a


different arrangement. But just like categorical
propositions, categorical syllogisms need to be written in
standard form. Remember that the validity of a
categorical syllogism depends solely upon its logical
form. Throughout this module, we let S represent the
subject of the conclusion (minor term), P the predicate of
the conclusion (major term), and M the middle term.

Example 5. Express the categorical syllogism below in the standard form.


All cookery students are HE students and all HE students are TVL students.
So, all cookery students are TVL students.

11 CO_Q2_General Mathematic SHS


Module 19
Solution.

Step 1: Identify the conclusion then Step 2: Identify the major and the minor
label the subject with S and the premise. The major premise contains the
predicate with P. Because of the word major term while the minor premise
“so”, the conclusion is the last contains the minor term.
statement.

Focus on the predicate(TVL students),


this is also the major term.
Step 3: Arrange the premises in the correct order. Major premise first followed by
minor premise and then conclusion.

The mood of a categorical syllogism is a series of three letters corresponding to the


type of proposition the major premise, the minor premise and the conclusion. It also
refers to the arrangement of the premises according to quantity (universal or
particular) and quality (affirmative or negative). It consists of the letter names of the
propositions that make it up.

Four Basic Types of Categorical Propositions:

Proposition Letter name Quantity Quality


All S are P. A universal affirmative
No S are P. E universal negative
Some S are P. I particular affirmative
Some S are not P. O particular negative

Example 6: Determine the mood of the categorical syllogism: Some creative thinkers
are SHS students. All SHS students are honest. Therefore, some creative thinkers
are honest.

12 CO_Q2_General Mathematic SHS


Module 19
Solution.

First, write the categorical syllogism in standard form.

Therefore, the mood of the given categorical syllogism is AII.

The figure of a categorical syllogism is determinedby the location of the two


occurrences of the middle term in the premises. Four different arrangements are
possible shown as follows:

Example 7: Determine which figure falls in the categorical syllogism in the previous
example.

Therefore, the categorical syllogism is in the figure 1. Particularly, it is in the form


AII-1.

Since there are 4 figures and 64 moods, there are 4 x 64 = 256 forms of categorical
syllogisms. Only 15 are unconditionally valid and 9 are conditionally valid forms.
UNCONDITIONALLY VALID FORMS
(Valid for both Boolean and Aristotelian standpoints)
Figure 1 Figure 2 Figure 3 Figure 4
AAA EAE IAI AEE
EAE AEE AII IAI
AII EIO OAO EIO
EIO AOO EIO

13 CO_Q2_General Mathematic SHS


Module 19
CONDITIONALLY VALID FORMS Required
(Valid for Aristotelian standpoint only under certain condition) condition
AAI AEO AEO
S exists
EAO EAO
AAI EAO
M exists
EAO
AAI P exists

Once the mood and figure of a syllogism are known, the validity of the syllogism can
be determined by checking the mood and figure against a list of valid syllogistic forms
above. For instance, the categorical syllogism in the form AOO-2 is unconditionally
valid while EAO-3 is conditionally valid.

Determining the Validity of Categorical Syllogisms Using Euler’s


Diagram

Euler’s Diagram can also be used to determine the validity of categorical syllogism.
Consider the Euler’s Circles and representation of the four propositions.
Proposition Pictorial representation

(A)
All S are P

(E)
No S are P

Some S are not P or All S are P or All P are S

(I)
Some S are P

Some S are P or All P are S or No S are P

(O)
Some S are not P

14 CO_Q2_General Mathematic SHS


Module 19
Example 8: Determine the validity of the categorical syllogisms below using the
Euler’s diagram.

Solution:

Given: Given:
All Quezonians are Filipinos. All SHS students are honest.
Nestor is a Quezonian. Some creative thinkers are SHS students.
Therefore, Nestor is a Filipino. Therefore some creative thinkers are honest.

Solution: Solution:
Let S=SHS students’ class, H=honest people’s
class and C=creative thinker people’s class
then we will have the following Euler’s
diagram:

An argument is valid if it is
impossible for the conclusion to
be false given that the premises
are true while it is invalid if it is
possible for the conclusion to be
false given that the premises are In the three figures, the conclusion “some
true. Therefore, the given creative thinkers are honest” is TRUE.
categorical syllogism is VALID. Therefore, we have shown that the given
categorical syllogism is VALID.

Determining the Validity of Categorical Syllogisms Using Venn


Diagram

Venn diagram uses different ways of illustrating propositions from Euler’s diagram.
Both methods require a little practice before it can be done with the facility. John
Venn used two overlapping circles to represent the relationship between two classes.

The shaded portion represents a class that has no members. It is empty and cannot
hold a value. The circle with an “x” signifies that the class has at least one member.

15 CO_Q2_General Mathematic SHS


Module 19
Use shading in diagramming universal Put an arbitrary x in diagramming
propositions. particular propositions to denote the
possible position of an element.
All S are P Some S are P

A I

No S are P Some S are Not P

E O

Here is the three-step procedure to assess the validity of categorical syllogisms using
Venn diagram:

1. Draw three interlocking circles and label them with S, P and M to represent
the minor, major, and middle terms of the syllogism respectively.

2. Draw the diagram to represent each of the two premises but not the conclusion.
Remember:

• Diagram the premises, just as you would when you diagrammed the
premises alone.
• If an argument has universal and particular premises, draw the universal
premise first.
• Use an X to indicate “at least one”, and place it on the line between two
areas if it isn’t clear which side of the line it should be.
• When diagramming two universal premises, make the lines go opposite
directions when filling in the circle (makes it easier for your reader to "see"
each premise)

3. After diagramming the premises, see if the conclusion is consistent with the
drawing. If it is, the syllogism is valid. If not, the syllogism is invalid.

16 CO_Q2_General Mathematic SHS


Module 19
Example 9. Determine the validity of the following using Venn Diagram
All P are M.
No S are M.
∴No S are P.

Step 1. Draw three interlocking Step 2.1. Draw the diagram to represent
circles and label them with S, P each of the two premises but not the
and M to represent the minor, conclusion. Since both premises are
major, and middle terms of the universal, you may draw either of the two
syllogism respectively first.

In this case, All P are M was drawn.


Step 2.2. Draw the other premise. Step 3. After diagramming the premises, see
if the conclusion is consistent with the
drawing. If it is, the syllogism is valid. If not,
the syllogism is invalid.

No S are M.
As you can see, the intersection between S
and P is shaded. It means that the
conclusion No S is P is true. Hence, the
categorical syllogism is VALID.

Example 10. Determine the validity of the following using Venn Diagram
Some P are not M.
No M are S. .
∴Some S are P.

Step 1. Draw three Step 2.1. Draw the diagram to represent each
interlocking circles and label of the two premises but not the conclusion.
them with S, P and M to Draw the universal premise first.
represent the minor, major,
and middle terms of the
syllogism respectively.

No M are S.

17 CO_Q2_General Mathematic SHS


Module 19
Step 2.2. Draw the other Step 3. After diagramming the premises, see if
premise. the conclusion is consistent with the drawing.

Some P are not M.


Put an X on the curve line
between two areas as shown
above. This is to denote that an The above illustrations show that the conclusion
element can be in either areas. could be true or could be false. Therefore, the
syllogism is INVALID.

Example 11: Determine the validity of the categorical syllogism:


Some creative thinkers are SHS students.
All SHS students are honest.
Therefore, some creative thinkers are honest.
Solution. Write the categorical syllogism in the standard form.
All SHS students are honest students.
Some creative thinker students are SHS students.
∴ Some creative thinker students are honest students.

Assign a letter to each class.

Draw the Venn diagram of the standard categorical syllogism.

Step 1. Draw three interlocking Step 2.1. Draw the diagram to represent each
circles and label them with S, P of the two premises but not the conclusion.
and M to represent the minor, Draw the universal premise first.
major, and middle terms of the
syllogism, respectively.

All SHS students are honest students

18 CO_Q2_General Mathematic SHS


Module 19
Step 2.2. Draw the other Step 3. After diagramming the premises, see if
premise. the conclusion is consistent with the drawing.
If it is, the syllogism is valid. If not, the
syllogism is invalid.

Some creative thinker students


are SHS students.
The illustration above shows that the
Place an X on the respective conclusion is true. Therefore, the categorical
unshaded area as shown above. syllogism is VALID.

Determining the Validity of Categorical Syllogism Using Set of Rules

The method of Venn and Euler’s diagram maybe check against the set of rules. The
syllogism is invalid if any one of these rules is violated. Accordingly, a specific formal
fallacy is committed.

Rule 1: There should be three terms in the syllogism each of which is used in the
same sense.
Fallacy: Fallacy of four terms
Example: God is love, love is blind therefore God is blind.

Rule 2: The middle term must be distributed in at least one premise. (At least one of
the middle terms must be universal.)
Fallacy: Undistributed middle.
Example: All teachers are human beings. All gentlemen are human beings.
Therefore, all teachers are gentlemen.

Rule 3: If a term is distributed in the conclusion, then it must be distributed in a


premise. (The major and the minor terms should only be universal in the
conclusion if they are universal in the premises.)
Fallacy: Illicit major and Illicit minor.
Example: All teachers are human beings. Some gentlemen are not teachers.
Therefore, some gentlemen are not human beings.

Rule 4: No conclusion drawn from two negative premises. Likewise, If the premises
are affirmative, then the conclusion must be affirmative.
Fallacy: Exclusive premises
Example: No teachers are gentlemen. Some Humans are not teachers. Therefore,
Some gentlemen are not humans.

19 CO_Q2_General Mathematic SHS


Module 19
Rule 5: If one premise is affirmative and the other is negative, then the conclusion
must be negative.
Fallacy: Drawing an affirmative conclusion from a negative premise, or drawing a
negative conclusion from an affirmative premise
Example: Some students are kind. Some students are not brave. Therefore, some
kind students are brave.

Rule 6: If both premises are universal, the conclusion cannot be particular. Also
there is no conclusion that can be from two particular premises.
One premise at least must be universal and if one premise is particular, then
the conclusion must be particular.
Fallacy: Existential fallacy
Example: No teachers are gentlemen. All human beings are gentlemen. Therefore,
some human beings are not teachers.

What’s More

Activity 1.1
Write the following categorical proposition in standard form
1. Filipinos are Asians.
2. A reptile is cold-blooded.
3. Many SHS students are with honors.
4. Most introverts aren’t optimistic.
5. All successful people are not cheaters.

Activity 1.2
Determine the validity of the following categorical syllogisms using the Venn diagram.
1. All M are P. 4. Some P are M.
Some S are M. All M are S. .
∴Some S are P. ∴No S are P.

2. No P are M. 5. All P are M.


Some S are M. . No M are S. .
∴Some S are not P. ∴No S are P.
3. Some M are not P.
All M are S. .
∴Some S are not P.

20 CO_Q2_General Mathematic SHS


Module 19
Activity 1.3

Identify the terms, mood and figure then determine its validity by looking at the table
of valid categorical syllogisms. Write your answer in the table and use the format
given below:

Kinds of term Term Mood Figure Validity


Major term
Minor term
Middle term

1. Some of the wastes collected in factories are radioactive. No wastes collected in


factories are dumped into the ocean. Therefore, some radioactive wastes are
dumped into the ocean.

2. All pliers are tools and all screwdrivers are tools. Therefore, all pliers are
screwdrivers.

3. All contagious diseases should be subject to mandatory testing. COVID-19 is


contagious. Therefore, testing for COVID-19 should be mandatory.

4. All working students have no laptops and all successful students are persons
who have laptops. Thus, all working students are successful students.

5. All sea vessels that sunk are unsafe vessels. All well-maintained vessels are safe
vessels. Therefore, no vessels that sunk are well-maintained.

Activity 1.4
Reconstruct the syllogistic forms from the following combinations of mood and figure.

1. OAE-3 6. EIA-4
2. AII-3 7. IAE-1
3. AOO-2 8. EAO-4
4. AAA-1 9. EAO-2
5. OEI-3 10. OEA-4

Activity 1.5
Determine the validity of the following categorical syllogisms using any method.
1. All educational games should be encouraged. Not all games are educational
games. Therefore, not all games should be encouraged.
2. All leaders are good communicators. All good communicator people are creative.
Therefore, all creative people are leaders.
3. All good students show love for country. Some students who show love for
country are respectful. Therefore, all respectful students are good students.

21 CO_Q2_General Mathematic SHS


Module 19
4. All prisms are plane figures. All cubes are prisms. Therefore, all cubes are plane
figures.
5. No Quezonians are lazy. Most lazy people have low income. Therefore, no
Quezonians have low income.

What I Have Learned

A. Please read the sentences carefully and fill in the missing word/s by writing your
answer on the line/s provided.
1. A ________________ is a proposition that relates two classes or categories.
2. A ________________ is a deductive argument in which a conclusion is inferred
from two premises.
3. A ________________ is an argument consisting of exactly three categorical
propositions (two premises and a conclusion) in which there appear a total of
exactly three categorical terms, each of which is used exactly twice.
4. The ________________of a categorical proposition is either universal or
particular
5. The ________________of a categorical proposition is either affirmative or
negative
6. Terms of the Categorical Syllogism
a. ________________is the predicate of the conclusion.
b. ________________is the subject term of the conclusion.
c. ________________is the term that appears in both premises but not in
the conclusion.
7. Parts of the Categorical Syllogism
a. ________________contains the major term.
b. ________________contains the minor term.
8. The ________________of a categorical syllogism consists of the letter
names of the propositions that make it up. Moods are defined as the
arrangement of the premises according to quantity (universal or
particular) and quality (affirmative or negative). In other words, we can
say that mood is determined by the type of standard form categorical
propositions of the syllogism contains.
9. The ________________of a categorical syllogism is determined by the location
of the two occurrences of the middle term in the premises. Four different
arrangements are possible. If we let S represent the subject of the conclusion
(minor term), P the predicate of the conclusion (major term), and M the middle
term, and leave out the quantifiers and copulas, the four possible
arrangements may be illustrated as follows:
10. Since there are 4 figures and 64 moods, there are
_____________________categorical syllogisms. Only ___________________are
unconditionally valid and ________________are conditionally valid forms.

22 CO_Q2_General Mathematic SHS


Module 19
What I Can Do

A. Construct your own syllogisms that satisfy the following conditions:

Major term Minor term Middle term Form


1 TVL students Good performing ACP club members
AAA-3
students
2 fathers Married people Drivers IAI-3
3 SMAW students Students who are Students who
good in history know the pen name EIO-4
subject of Jose Rizal
4 Transmissible Airborne diseases Contagious
AAA-1
diseases diseases
5 Students who Students who cheat Students who
achieved career expelled from EII-1
goals college

1. An AII-3categorical syllogism with these terms: major: creative people; minor:


brave people; middle: musicians.
2. An unconditionally valid syllogism in the first figure with a universal negative
conclusion and these terms: major: Academic students; minor: ICT students;
middle: TVL students.
3. An unconditionally valid syllogism in the fourth figure having two universal
premises and these terms: major: Filipinos; minor: selfish; middle: hospitable.
4. A valid syllogism having mood AOO and these terms: major: entrepreneurs;
minor: risk-taker people; middle: persistent people.
5. A valid syllogism in the first figure having a particular negative conclusion and
these terms: major: SHS subjects; minor: fun and challenging subjects; middle:
research subjects

23 CO_Q2_General Mathematic SHS


Module 19
Assessment

Choose the letter of the best answer. Write the chosen letter on a separate sheet of paper.

1. It is the predicate term of the conclusion.


a. major term
b. major premise
c. minor term
d. minor premise

2. It is a proposition that relates two classes or categories.


a. categorical proposition
b. conjunctive proposition
c. c. hypothetical proposition
d. d. logical proposition

3. The copula in the categorical proposition “All STEM students are students with
scientific attitude” is
a. all
b. are
c. STEM
d. scientific

4. Determine the quality of the categorical proposition: Some teachers are not
Math teachers.
a. affirmative
b. b. particular
c. c. negative
d. d. universal

5. If we will change the categorical proposition, “All successful people are not lazy”
in standard form, which of the following is the correct proposition?
a. All S are P.
b. b. No S are P.
c. c. Some S are P.
d. d. Some S are not P.

6. How many standard forms of categorical syllogisms are unconditionally valid?


a. 256
b. 24
c. 15
d. 9

24 CO_Q2_General Mathematic SHS


Module 19
For number 7-8, refer to the given categorical syllogism below:

All M are P. Some M are S. Therefore, some S are P.

7. Determine the form of the given standard categorical syllogism


a. IIE-3
b. AII-3
c. IOO-1
d. EII-1

8. Which is the correct diagram of the standard categorical syllogism?

9. Which of the following letter name is used as symbol for universal negative
categorical syllogism?
a. A
b. E
c. I
d. O

For numbers 10-11, refer to the given categorical syllogism below:


Some jokes are stupid. All jokes are hilarious. Therefore, some stupid things are
hilarious.

10. Which is the major term in the given categorical syllogism?


a. jokes
b. hilarious things
c. stupid things
d. stupid jokes

11. Which is the correct diagram of the standard categorical syllogism?

25 CO_Q2_General Mathematic SHS


Module 19
For number 12-13, refer to the given categorical syllogism below:
Some dedicated people are teachers. All teachers are public servants. Therefore,
some dedicated people are public servants.

12. What is the minor premise of the given categorical syllogism?


a. All teachers are public servants.
b. Some dedicated people are teachers.
c. All public servants are dedicated people.
d. Some dedicated people are public servants.

13. Which is the correct diagram of the given categorical syllogism?

14. Determine the form of the categorical syllogism:


All competent teachers are experienced teachers.
Some competent teachers are efficient teachers.
Therefore, some experienced teachers are not efficient teachers.
a. AII-1
b. b. AIO-2
c. c. IAO-3
d. d. IAI-4

15. Which of the following categorical syllogisms is INVALID?


a. All M are P. Some M are S. Therefore, some S are P.
b. Some jokes are stupid. All jokes are hilarious.
Therefore, some stupid things are hilarious.
c. Some dedicated people are teachers. All teachers are public servants.
Therefore, some dedicated people are public servants.
d. All competent teachers are experienced teachers.
Some competent teachers are efficient teachers.
Therefore, some experienced teachers are not efficient teachers.

26 CO_Q2_General Mathematic SHS


Module 19
Additional Activities

I. For each of the following syllogisms, complete the following steps:

A. Put the syllogism into standard form, symbolizing all the statements
appropriately (use S to indicate the minor term, P the major term, and M
the middle term)
B. Identify the mood and figure of the syllogism.
C. Draw a Venn diagram representing the syllogism, making sure to label the
circles.
D. Determine whether the argument is valid or invalid, based on the diagram.

1. Some proposed researches are expensive study, because all experimental


researches are expensive study, and some proposed researches are
experimental studies.

2. All island municipalities are home of peace-loving people. Some peace-


loving people municipalities are 5th class municipalities. Therefore, some
5thclass municipalities are island municipalities

3. All STEM students are honest. All honest students are persons genuinely
interested in others. Therefore, some persons genuinely interested in
others are STEM students.

II. Write the conclusion to make the following premises valid. If no conclusion can be
validly drawn, write ‘‘no conclusion.’’

1. Some M are P.
All M are S.

2. No M are P.
Some S are M.

3. Some P are M.
No S are M.

4. All P are M.
No M are S.

5. No P are M.
Some M are S.

27 CO_Q2_General Mathematic SHS


Module 19
Answer Key

28 CO_Q2_General Mathematic SHS


Module 19
29 CO_Q2_General Mathematic SHS
Module 19
References

Books

Howe, Karin. n.d. https://www.pdfdrive.com/ (accessed June 12, 2020).

Hurley, Patrick J. A Concise Introduction to Logic. 7th. California: Wadsworth


Publishing, 2001.

Kreeft, Peter. Socratic Logic. 3rd. South Bend, Indiana: St. Augustine's Press, 2008.

Kundan, K. n.d. Syllogism.https://www.pdfdrive.com/ (accessed June 12, 2020).

Sirswal, Desh Raj. n.d. Categorical Syllogism. https://www.pdfdrive.com/ (accessed


June 12, 2020).

Electronic References

n.d. https://www.pdfdrive.com/ (accessed June 12, 2020).

n.d. https://www.youtube.com/watch?v=KcNESCrkIiQ&t=140s (accessed June 12,


2020).

n.d. https://www.youtube.com/watch?v=to6kEzy17cM (accessed June 12, 2020).

n.d. https://www.youtube.com/watch?v=fp3-DWh24eI (accessed June 12, 2020).

n.d. https://www.youtube.com/watch?v=ngUr6WrLzQ8 (accessed June 12, 2020).

30 CO_Q2_General Mathematic SHS


Module 19
For inquiries or feedback, please write or call:

Department of Education - Bureau of Learning Resources (DepEd-BLR)

Ground Floor, Bonifacio Bldg., DepEd Complex


Meralco Avenue, Pasig City, Philippines 1600

Telefax: (632) 8634-1072; 8634-1054; 8631-4985

Email Address: blr.lrqad@deped.gov.ph * blr.lrpd@deped.gov.ph


General Mathematics
Quarter 2 – Module 20:
Valid Arguments and Fallacies

CO_Q2_General Mathematics SHS


Module 20
General Mathematics – Senior High School
Alternative Delivery Mode
Quarter 2 – Module 20: Valid Arguments and Fallacies
First Edition, 2020

Republic Act 8293, section 176 states that: No copyright shall subsist in any work of
the Government of the Philippines. However, prior approval of the government agency or office
wherein the work is created shall be necessary for exploitation of such work for profit. Such
agency or office may, among other things, impose as a condition the payment of royalties.

Borrowed materials (i.e., songs, stories, poems, pictures, photos, brand names,
trademarks, etc.) included in this module are owned by their respective copyright holders.
Every effort has been exerted to locate and seek permission to use these materials from their
respective copyright owners. The publisher and authors do not represent nor claim ownership
over them.

Published by the Department of Education


Secretary: Leonor Magtolis Briones
Undersecretary: Diosdado M. San Antonio

Development Team of the Module


Writer: Azenith G. Mercado
Editors: Elizabeth D. Lalunio, Anicia J. Villaruel and Roy O. Natividad
Reviewers: Jerry Punongbayan, Diosmar O. Fernandez, Dexter M. Valle, Arvin Asnan,
Cherish Mae U. Buenviaje and Moahna Aura M. Mancenido
Illustrators: Hanna Lorraine G. Luna and Diane C. Jupiter
Layout Artists: Roy O. Natividad, Sayre M. Dialola, Argie L. Ty and
Glydel Eveth T. Enriquez
Management Team: Francis Cesar B. Bringas
Job S. Zape, Jr.
Ramonito Elumbaring
Reicon C. Condes
Elaine T. Balaogan
Fe M. Ong-ongowan
Hermogenes M. Panganiban
Philip B. Gallendez
Josephine T. Natividad
Anicia J. Villaruel
Dexter M. Valle

Printed in the Philippines by ________________________

Department of Education – Region 4A CALABARZON

Office Address: Gate 2 Karangalan Village, Brgy. San Isidro, Cainta, Rizal
Telefax: 02-8682-5773/8684-4914/8647-7487
E-mail Address: lrmd.calabarzon@deped.gov.ph
General Mathematics
Quarter 2 – Module 20:
Valid Arguments and Fallacies
Introductory Message

This Self-Learning Module (SLM) is prepared so that you, our dear


learners, can continue your studies and learn while at home. Activities,
questions, directions, exercises, and discussions are carefully stated for you
to understand each lesson.

Each SLM is composed of different parts. Each part shall guide you
step-by-step as you discover and understand the lesson prepared for you.

Pre-tests are provided to measure your prior knowledge on lessons in


each SLM. This will tell you if you need to proceed on completing this module
or if you need to ask your facilitator or your teacher’s assistance for better
understanding of the lesson. At the end of each module, you need to answer
the post-test to self-check your learning. Answer keys are provided for each
activity and test. We trust that you will be honest in using these.

In addition to the material in the main text, Notes to the Teacher are
also provided to our facilitators and parents for strategies and reminders on
how they can best help you on your home-based learning.

Please use this module with care. Do not put unnecessary marks on
any part of this SLM. Use a separate sheet of paper in answering the exercises
and tests. And read the instructions carefully before performing each task.

If you have any questions in using this SLM or any difficulty in


answering the tasks in this module, do not hesitate to consult your teacher
or facilitator.

Thank you.

iii
What I Need to Know
KnowKnowKnow
This module was designed and written with you in mind. It is here to help you master
logical equivalence and conditional propositions. The scope of this module permits it
to be used in many different learning situations. The language used recognizes the
diverse vocabulary level of students. The lessons are arranged to follow the standard
sequence of the course. But the order in which you read them can be changed to
correspond with the textbook you are now using.

After going through this module, you are expected to:


1. establish the validity and falsity of real-life arguments using logical
propositions, syllogisms, and fallacies;
2. determine the rules of inferences and fallacies that apply to the given
arguments; and
3. manifest cautiousness in accepting issues and principles in life.

What I Know

Choose the letter of the best answer. Write the chosen letter on a separate sheet of
paper.
1. What refers to the set of propositions formed by premises supporting the
conculsion?
a. argument
b. tautology
c. fallacy
d. inference

2. How do you call 𝑝1 and 𝑝2 in the propositional form (𝑝1 ∧ 𝑝2 ) → 𝑞?


a. tautology
b. fallacy
c. premise
d. conclusion

3. How do you call 𝑞 in the propositional form (𝑝1 ∧ 𝑝2 ) → 𝑞?


a. conclusion
b. premise
c. fallacy
d. tautology

1 CO_Q2_General Mathematics SHS


Module 20
4. What is true about a valid argument?
a. It is a premise.
b. It is a conclusion.
c. It is a fallacy.
d. It is a tautology.

5. Which of the following is a rule of inference?


a. Affirming the Disjunct
b. Modus Tollens
c. Improper Transposition
d. Denying a Conjunct

6. Which of the following is true with the argument, “If you are a STEM student, then
you have Basic Calculus subject. You are a STEM student. Hence, you have Basic
Calculus subject.”?
a. It is a fallacy by Fallacy of the Converse.
b. Is is a fallacy by Denying a Conjunct.
c. It is valid by Rule of Disjunctive Syllogism.
d. It is valid by Modus Ponens.

7. Which of the following propositional forms supports the argument in item number
6?
a. [(𝑝 ∨ 𝑞) ∧ (~𝑝)] → 𝑞
b. [(𝑝 → 𝑞) ∧ 𝑝] → 𝑞
c. [(𝑝 → 𝑞) ∧ 𝑞] → 𝑝
d. [~(𝑝 ∧ 𝑞) ∧ (~𝑝)] → 𝑞

8. Which of the following is true with the argument, “If I have higher allowance, then
I can buy my needs and wants. Therefore, if I can buy my needs and wants, then
I have higher allowance.”?
a. It is a fallacy by Improper Transposition.
b. Is is a fallacy by Fallacy of the Consequent.
c. It is valid by Rule of Proof by Cases.
d. It is valid by Rule of Addition.

9. Which of the following propositional forms supports the argument in item number
8?
a. (𝑝 → 𝑞) → (𝑞 → 𝑝)
b. (𝑝 → 𝑞) → [(~𝑝) → (~𝑞)]
c. 𝑝 → (𝑝 ∨ 𝑞)
d. [(𝑝 → 𝑟) ∧ (𝑞 → 𝑟)] → [(𝑝 ∨ 𝑞) → 𝑟]

2 CO_Q2_General Mathematics SHS


Module 20
10. Which of the following is true with the argument, “If it rains, then Rhea is not
going to the party. Rhea is not going to the party. Therefore, it rains.”?
a. It is valid by Modus Tollens.
b. It is valid by Rule of Simplification.
c. It is a fallacy by Fallacy of the Consequent.
d. It is a fallacy by Fallacy of the Converse.

11. Which of the following propositional forms supports the argument in item
number 10?
a. (𝑝 ∧ 𝑞) → 𝑝
b. [(𝑝 → 𝑞) ∧ (~𝑞) → (~𝑝)
c. [(𝑝 → 𝑞) ∧ 𝑞] → 𝑝
d. (𝑝 → 𝑞) → (𝑞 → 𝑝)

12. Which of the following is true with the argument, “Azel does not eat deep fried
and junk foods. Azel does not eat deep fried foods. Thus, she eats junk foods.”?
a. It is valid by Law of Syllogism.
b. It is valid by Rule of Disjunctive Syllogism.
c. It is a fallacy by Denying a Conjunct.
d. It is a fallacy by Affirming the Disjunct.

13. Which of the following propositional forms supports the argument in item
number 12?
a. [(𝑝 ∨ 𝑞) ∧ (~𝑝)] → 𝑞
b. [(𝑝 → 𝑞 ) ∧ (𝑞 → 𝑟)] → (𝑝 → 𝑟)
c. [(𝑝 ∨ 𝑞) ∧ 𝑝] → (~𝑞)
d. [~(𝑝 ∧ 𝑞) ∧ (~𝑝)] → 𝑞

14. Which of the following is true with the argument, “Featured delicacy at Rod and
Baby’s Online Pasalubong Store is either the best seller or the most affordable.
This month’s featured delicacy is the best seller. It follows that this delicacy is
not the most affordable.”?
a. It is a fallacy by Affirming the Disjunct.
b. It is a fallacy by Fallacy of the Inverse.
c. It is valid by Rule of Simplification.
d. It is valid by Rule of Contradiction.

15. Which of the following propositional forms supports the argument in item
number 14?
a. [(𝑝 ∨ 𝑞) ∧ 𝑝] → (~𝑞)
b. [(𝑝 → 𝑞) ∧ (~𝑝)] → (~𝑞)
c. (𝑝 ∧ 𝑞) → 𝑝
d. [(~𝑝) → 𝜙] → 𝑝

3 CO_Q2_General Mathematics SHS


Module 20
Lesson
Valid Arguments and
1 Fallacies
“All butterflies love flowers. Concon loves flowers. Concon is a butterfly.” Do they
make sense? Are they interconnected? Given three ideas, you infer the agreement or
disagreement between the first two ideas based on their agreement or disagreement
with the third idea. So, do you agree that Concon is a butterfly? Well, take a look
again at your previous lesson.

What’s In

Activity 1
Indicate the correct categorical proposition for each of the following.

P
S P S x P S P
S
x

(1) (2) (3) (4)

A categorical proposition expresses the relationship between two categories or sets.


In better understanding the relationship between and among the sets in the
propositions, we make use of Venn diagrams as in the figures above. In naming the
said figures, let us recall the four standard categorical propositions. For (1) we have,
All S are P. Then for (2), No S are P. Some S are P is represented by x in (3), whereas
x in (4) represents Some S are not P.
All and no are universal quantifiers referring to every element of a set. Some, on the
other hand, is an existential qualifier pertaining to at least one element of the set.
You have learned in your previous lesson that a syllogism is a deductive argument
consisting of two premises and one conclusion. And a syllogism formed by categorical
propositions only, is called as a categorical syllogism. To determine its validity, the
Venn diagram is used.
Consider the following:
“All above 18 years of age can have a driver’s license. Christian has a driver’s license.
Thus, Christian is above 18 years of age.”
Is this valid?

4 CO_Q2_General Mathematics SHS


Module 20
Let A = set of people who are above 18 years of age
A D = set of those who can have a driver’s license
C = set containing a single element which is Christian
D Premise 1 becomes all D are A. Premise 2 becomes all C are D.
C Therefore, all C are A. It can be verified that this argument is valid
through the Venn diagram on the left.

Notes to the Teacher


Lead students in understanding the concept of Venn Diagram.
This will make them better understand categorical syllogism.

What’s New

Activity 2
Consider the compound propositions (1) and (2) below, given that:
hypothesis p: I was able to buy suman at Infanta’s agri-booth.
conclusion q: I was in Lucena City during the Niyogyugan Festival.

(1) If I was able to buy suman at Infanta’s agri-booth, then I was in Lucena City
during the Niyogyugan Festival. I was able to buy suman at Infanta’s agri-booth.
Therefore, I was in Lucena City during the Niyogyugan Festival.
(2) If I was able to buy suman at Infanta’s agri-booth, then I was in Lucena City
during the Niyogyugan Festival. I was in Lucena City during the Niyogyugan
Festival. Therefore, I was able to buy suman at Infanta’s agri-booth.
Questions:
1. How would you write the first compound proposition in propositional form?
2. How would you write the second compound proposition in propositional
form?
3. What similarities and differences have you observed between the two forms?
4. Which do you think is acceptable between the two? Why do you say so?

5 CO_Q2_General Mathematics SHS


Module 20
Niyogyugan Festival is held annually during August in Lucena City, the capital of
Quezon Province. All municipalities in the province gather around the Quezon
Provincial Capitol located in Lucena City, and put up their respective agri-booth
featuring and selling their own local products. It is also held online especially during
the pandemic. Nevertheless, it continues to display the creativity and
resourcefulness of Quezonians. It includes virtual trade fairs and shopping featuring
various local products in the province.
In the given above, the first compound proposition is expressed as [(𝑝 → 𝑞) ∧ 𝑝] → 𝑞,
while the second one’s propositional form is [(𝑝 → 𝑞) ∧ 𝑞] → 𝑝.
The two both involve conditionals and conjunction. Their hypotheses are almost the
same, both with 𝑝 → 𝑞 as part of conjunctions. However, some parts of the
conjunctions vary. That is, (1) contains p; while (2) has q. Also, the conclusions are
different. (1) includes q, whereas (2) involves p.
Going back to the two compound propositions stated above, considering that both
state that 𝑝 → 𝑞, then it is an acceptable idea that being able to buy suman at
Infanta’s agri-booth, basically means that you are in Lucena City during the
Niyogyugan Festival. Now, compound proposition (1) states that I was able to buy
suman at Infanta’s agri-booth, then it is only right to say that I was in Lucena City
witnessing the festival at that time.
However, (2) states that I was in Lucena City when the festival happened. Does it
follow that I was able to witness it? Or does it mean that I got to see all the agri-
booth? How if even I saw Infanta’s booth, I was not able to buy suman because the
booth run out of it due to its high demand among festival-goers? Following the
established idea involving 𝑝 → 𝑞, being in Lucena City during the festival does not
necessarily mean that I was able to buy suman. Hence, the compound proposition
(1) is more acceptable.

What is It

An argument is a set of propositions formed by premises supporting the conclusion.


It can be written in the propositional form (𝑝1 ∧ 𝑝2 ∧ … ∧ 𝑝𝑛 ) → 𝑞 or in standard form:
𝑝1
𝑝2

𝑝𝑛
∴𝑞

𝑝1 , 𝑝2 , … , 𝑝𝑛 are the premises of the argument, while q is the conclusion.


Example 1: Let us consider arguments G and G’ represented by the compound
propositions in the previous activity where:
p: I was able to buy suman at Infanta’s agri-booth.
q: I was in Lucena City during the Niyogyugan Festival.

6 CO_Q2_General Mathematics SHS


Module 20
𝐺 𝑝→𝑞 If I was able to buy suman at 𝐺′ 𝑝 → 𝑞 If I was able to buy suman at
Infanta’s agri-booth, then I Infanta’s agri-booth, then I
was in Lucena City during was in Lucena City during
the Niyogyugan Festival. the Niyogyugan Festival.

𝑝 I was able to buy suman at 𝑞 I was in Lucena City during


Infanta’s agri-booth. the Niyogyugan Festival.

∴𝑞 Therefore, I was in Lucena ∴𝑝 Therefore, I was able to buy


City during the Niyogyugan suman at Infanta’s agri-
Festival. booth.

Which is valid between the two given arguments?

Solution:

For an argument to be valid when the premises are true, the conclusion must
be true as well. In argument G, the premises are 𝑝 → 𝑞 and p, and the conclusion is
q. It can be seen in the truth table that it is possible that when the premises 𝑝 → 𝑞
and p are both true, the conclusion q is true. Hence argument G is a valid argument.
On the other hand, for argument G’, when the premises 𝑝 → 𝑞 and q are both true, it
is possible to get a false conclusion. Hence G’ is not a valid argument.
Moreover, an argument is valid if conditional (𝑝1 ∧ 𝑝2 ∧ … ∧ 𝑝𝑛 ) → 𝑞 is a
tautology.

𝒑 𝒒 𝒑→𝒒 (𝒑 → 𝒒) ∧ 𝒑 [(𝒑 → 𝒒) ∧ 𝒑] → 𝒒 (𝒑 → 𝒒) ∧ 𝒒 [(𝒑 → 𝒒) ∧ 𝒒] → 𝒑


T T T T T T T
T F F F T F T
F T T F T T F
F F T F T F T
Since [(𝑝 → 𝑞) ∧ 𝑞] → 𝑝 is not a tautology, then it is not a valid argument. And
since [(𝑝 → 𝑞) ∧ 𝑝] → 𝑞 is a tautology, then the argument is valid. This is known as
Modus Ponens (or Rule of Detachment). Yet being valid does not guarantee that the
conclusions are true. Instead, the validity of an argument implies that the conclusion
logically follows the premises.

7 CO_Q2_General Mathematics SHS


Module 20
Rules of Inference
Let 𝑝, 𝑞, and r be propositions.
Rules of Inference Propositional Form Standard Form
Rule of 𝑝∧𝑞
(𝑝 ∧ 𝑞) → 𝑝
Simplification ∴ 𝑝
𝑝
Rule of Addition 𝑝 → (𝑝 ∨ 𝑞)
∴𝑝 ∨ 𝑞
𝑝
Rule of 𝑞
(𝑝 ∧ 𝑞) → (𝑝 ∧ 𝑞)
Conjunction
∴𝑝 ∧ 𝑞
𝑝→𝑞
Modus Ponens [(𝑝 → 𝑞) ∧ 𝑝] → 𝑞 𝑝
𝑞
𝑝→𝑞
Modus Tollens [(𝑝 → 𝑞) ∧ (~𝑞)] → (~𝑝) ~𝑞
∴ ~𝑝
𝑝→𝑞
Law of Syllogism [(𝑝 → 𝑞) ∧ (𝑞 → 𝑟)] → (𝑝 → 𝑟) 𝑞→𝑟
∴𝑝 →𝑟
𝑝∨𝑞
Rule of Disjunctive
[(𝑝 ∨ 𝑞) ∧ (~𝑝)] → 𝑞 ~𝑝
Syllogism
∴ 𝑞
Rule of (~𝑝) → 𝜙
[(~𝑝) → 𝜙] → 𝑝
Contradiction ∴ 𝑝
𝑝→𝑟
Rule of Proof by 𝑞→𝑟
[(𝑝 → 𝑟) ∧ (𝑞 → 𝑟)] → [(𝑝 ∨ 𝑞) → 𝑟]
Cases
∴ (𝑝 ∨ 𝑞) → 𝑟
Example 2: Determine which rule is manifested in this argument.
Buko pie and yema cake are well-known delicacies in Quezon Province.
Therefore, buko pie is a well-known delicacy in Quezon Province.
Solution:
Let 𝑝: “Buko pie is a well-known delicacy in Quezon Province.”
𝑞: “Yema cake is a well-known delicacy in Quezon Province.”
𝑝∧𝑞
Then the argument is in the form (𝑝 ∧ 𝑞) → 𝑝 or . This is a valid
∴ 𝑝
argument by the Rule of Simplification.
On the other hand, an argument that is not valid is called a fallacy. Hence in
a fallacy, the premises may be true, while the conclusion is false. In a truth table, we
call such set as a counterexample. Consequently, the conditional (𝑝1 ∧ 𝑝2 ∧ … ∧ 𝑝𝑛 ) →
𝑞 is not a tautology. For instance, argument G ' is a fallacy as can be seen in the first
truth table presented in this lesson. In standard form it is

𝑝→𝑞
𝑞
∴ 𝑝
This argument is a fallacy of the converse.

8 CO_Q2_General Mathematics SHS


Module 20
Common Fallacies in Logic
Let 𝑝, 𝑞, and r be propositions.
Common Fallacies in
Propositional Form Standard Form
Logic
𝑝→𝑞
Fallacy of the Converse [(𝑝 → 𝑞) ∧ 𝑞] → 𝑝 𝑞
∴ 𝑝
𝑝→𝑞
Fallacy of the Inverse [(𝑝 → 𝑞) ∧ (~𝑝)] → (~𝑞) ~𝑝
∴ ~𝑞
𝑝∨𝑞
Affirming the Disjunct [(𝑝 ∨ 𝑞) ∧ 𝑝] → (~𝑞) 𝑝
∴ ~𝑞
Fallacy of the 𝑝→𝑞
(𝑝 → 𝑞) → (𝑞 → 𝑝)
Consequent ∴𝑞 → 𝑝
~(𝑝 ∧ 𝑞)
Denying a Conjunct [~(𝑝 ∧ 𝑞) ∧ (~𝑝)] → 𝑞 ~𝑝
∴ 𝑞
𝑝→𝑞
Improper Transposition (𝑝 → 𝑞) → [(~𝑝) → (~𝑞)]
∴ (~𝑝) → (~𝑞)
Example 3: Determine which fallacy is manifested in this argument.
If Neth cooks, then she has something to eat.
Therefore, if Neth has something to eat, then she cooked.
Solution:
Let 𝑝: “Neth cooks.”
𝑞: “Neth has something to eat.”
In propositional and standard forms, the argument is expressed as (𝑝 → 𝑞) →
𝑝→𝑞
(𝑞 → 𝑝) and , respectively. This is a Fallacy of the Consequent.
∴𝑞 → 𝑝

Example 4: Determine whether the given is a valid argument or a fallacy, by


identifying the rule manifested in this argument.

If you study hard, then you will get high scores in summative
assessments. If you get scores in summative assessments, then you will
receive high grades. Therefore, if you study hard, then you will receive
high grades.
Solution:
Let 𝑝: “You study hard.”
𝑞: “You get high scores in summative assessments.”
𝑟: “You receive high grades.”
In propositional and standard forms, the argument is expressed as [(𝑝 → 𝑞) ∧
𝑝→𝑞
(𝑞 → 𝑟)] → (𝑝 → 𝑟) and 𝑞 → 𝑟 , respectively.
∴𝑝 → 𝑟
Therefore, by the Law of Syllogism, the argument is valid.

9 CO_Q2_General Mathematics SHS


Module 20
What’s More

Activity 1.1
Write the propositional form for each item. Decide whether each of the following
arguments is valid or not. If it is valid, identify the rule of inference justifying the
validity. If not, write the type of fallacy used.

Propositional Rule of Inference/


Argument
Form Type of Fallacy

___________________ ___________________ 1. Fruits are sources of vitamins.


Therefore, fruits or vegetables are
sources of vitamins.
___________________ ___________________ 2. If the motorcycle does not have
gasoline, then it will not run. The
motorcycle did not run. Therefore, it
had no gasoline.
___________________ ___________________ 3. Regan does not speak French and
German. Regan does not speak
French. Therefore, he speaks
German.
___________________ ___________________ 4. On my birthday, I plan to have a
party or to plant a tree. I did not
have a party on my birthday.
Therefore, I planted a tree, instead.
___________________ ___________________ 5. If I patronize my country’s own
products, then I will prefer eating
Filipino delicacies. I do not prefer
eating Filipino delicacies. Therefore,
I do not patronize my country’s own
products.
Activity 1.2
1. Given the following propositions, construct the required argument using Rule of
Proof by Cases as a rule of inference.
p: There is a city ordinance not allowing the use of items made from plastic.
q: Plastic-free items are beneficial to health.
r: I will encourage others to use eco-friendly products.
2. Given the propositions below, construct the required argument using Fallacy of
Inverse as a fallacy in logic.
p: It rains.
q: Rhea will use her umbrella.

10 CO_Q2_General Mathematics SHS


Module 20
What I Have Learned

Fill in the blanks with the correct words or phrases to complete the following
statements.

1. An argument is
_____________________________________________________________________________
_____________________________________________________________________________

2. A valid argument is
_____________________________________________________________________________
_____________________________________________________________________________

3. An argument is not valid when


_____________________________________________________________________________
_____________________________________________________________________________

4. Rules of Inferences are used to


_____________________________________________________________________________
_____________________________________________________________________________

5. Common Fallacies in Logic are used to


_____________________________________________________________________________
_____________________________________________________________________________

6. Five of the nine rules of inferences are


_____________________________________________________________________________
_____________________________________________________________________________

7. Three of the five common fallacies are


_____________________________________________________________________________
_____________________________________________________________________________

11 CO_Q2_General Mathematics SHS


Module 20
What I Can Do

You have learned when an argument can be considered as valid or just a fallacy. But
it must also be noted that though valid, an argument can either be sound or bad.
Hence, we must be very careful in believing and in accepting issues or principles in
life. Now, suppose you are from a poor family and one day you happen to pass by a
note saying, “If you were born poor, then you have no better future ahead of you.”
You know you are poor. So does it mean you have no chance to have a bright future?
The next question you asked yourself was, “Is this valid?” If so, “Is it a sound or a
bad argument?” Share your sentiments about this.

Try scoring your essay using the rubric below.


Criteria 4 3 2 1
Very relative Somewhat Less relative Not relative to
Idea to the topic relative to the to the topic the topic and
(score x 3) and well- topic and and poorly very poorly
organized organized organized organized
No spelling, Very few Several Many spelling,
Spelling,
punctuation spelling, spelling, punctuation,
Grammar,
or punctuation, punctuation, and
and
grammatical and and grammatical
Punctuations
errors grammatical grammatical errors
(score x 2)
errors errors

12 CO_Q2_General Mathematics SHS


Module 20
Assessment

Choose the letter of the best answer. Write the chosen letter on a separate sheet of
paper.
1. What is true about an argument?
a. It is always valid.
b. It can never be a fallacy.
c. It is formed by a conclusion supporting the premises.
d. It is formed by premises supporting the conclusion.

For item numbers 2 to 5, refer to the argument, “If you exercise daily, then you have
higher chance to become healthy. You exercise daily. Therefore, you have higher
chance to become healthy.”

2. How would you represent the first premise?


a. 𝑞 → 𝑝
b. 𝑝 → 𝑞
c. 𝑝
d. 𝑞

3. How would you represent the second premise?


a. 𝑞
b. 𝑝
c. 𝑝 ∧ 𝑞
d. 𝑝 ∨ 𝑞

4. How would you represent the conclusion?


a. 𝑝 ∧ 𝑞
b. 𝑝 ∨ 𝑞
c. 𝑞
d. 𝑝

5. Which of the following is true with the given argument?


a. It is valid by Modus Ponens.
b. It is valid by Modus Tollens.
c. It is a fallacy by Improper Transposition.
d. It is a fallacy by Denying a Conjunct.

13 CO_Q2_General Mathematics SHS


Module 20
6. Which of the following is true with the argument, “If the weather is fair, then there
are visible stars at night. There are no visible stars at night. Therefore, the
weather is not fair.”?
a. It is a fallacy by Fallacy of the Consequent.
b. It is a fallacy by Affirming the Disjunct.
c. It is valid by Modus Tollens.
d. It is valid by Modus Ponens.

7. Which of the following propositional forms supports the argument in item number
6?
a. [(𝑝 ∨ 𝑞) ∧ 𝑝] → (~𝑞)
b. (𝑝 → 𝑞) → (𝑞 → 𝑝)
c. [(𝑝 → 𝑞) ∧ 𝑝] → 𝑞
d. [(𝑝 → 𝑞) ∧ (~𝑞) → (~𝑝)

8.Which of the following is true with the argument, “If the weather is fair, then there
are visible stars at night. There are visible stars at night. Therefore, the weather is
fair.”?
a. It is valid by Modus Tollens.
b. It is valid by Modus Ponens.
c. It is a fallacy by Fallacy of the Converse.
d. It is a fallacy by Improper Transposition.

9. Which of the following propositional forms supports the argument in item number
8?
a. (𝑝 → 𝑞 ) → [(~𝑝) → (~𝑞)]
b. [(𝑝 → 𝑞) ∧ 𝑞] → 𝑝
c. [(𝑝 → 𝑞) ∧ 𝑝] → 𝑞
d. [(𝑝 → 𝑞) ∧ (~𝑞) → (~𝑝)

10. Which of the following is true with the argument, “If you make it a habit to do
good, then you eventually possess a happy heart. If you possess a happy heart,
then your body and soul become healthier. Hence, if you make it a habit to do
good, then your body and soul become healthier.”?
a. It is a fallacy by Affirming a Disjunct.
b. It is a fallacy by Denying a Conjunct.
c. It is valid by Rule of Contradiction.
d. It is valid by Law of Syllogism.

14 CO_Q2_General Mathematics SHS


Module 20
11. Which of the following propositional forms supports the argument in item
number 10?
a. [~(𝑝 ∧ 𝑞) ∧ (~𝑝)] → 𝑞
b. [(𝑝 ∨ 𝑞) ∧ 𝑝] → (~𝑞)
c. [(𝑝 → 𝑞 ) ∧ (𝑞 → 𝑟)] → (𝑝 → 𝑟)
d. [(~𝑝) → 𝜙] → 𝑝

12. Which of the following is true with the argument, “If you exercise regularly, then
your blood pressure becomes manageable. Your blood pressure becomes
manageable. Therefore, you exercise regularly.”?
a. It is a fallacy by Fallacy of the Converse.
b. It is a fallacy by Fallacy of the Inverse.
c. It is valid by Rule of Addition.
d. It is valid by Rule of Simplification.

13. Which of the following propositional forms supports the argument in item
number 12?
a. (𝑝 ∧ 𝑞) → 𝑝
b. 𝑝 → (𝑝 ∨ 𝑞)
c. [(𝑝 → 𝑞) ∧ (~𝑝)] → (~𝑞)
d. [(𝑝 → 𝑞) ∧ 𝑞] → 𝑝

14. Which of the following is true with the argument, “If you are a licensed teacher,
then you are a professional. You are not a licensed teacher. Therefore, you are
not a professional.”?
a. It is a fallacy by Fallacy of the Inverse.
b. It is a fallacy by Fallacy of the Converse.
c. It is valid by Modus Tollens.
d. It is valid by Modus Ponens.

15. Which of the following propositional forms supports the argument in item
number 14?
a. [(𝑝 → 𝑞) ∧ (~𝑝)] → (~𝑞)
b. [(𝑝 → 𝑞) ∧ 𝑞] → 𝑝
c. [(𝑝 → 𝑞) ∧ (~𝑞) → (~𝑝)
d. [(𝑝 → 𝑞) ∧ 𝑝] → 𝑞

15 CO_Q2_General Mathematics SHS


Module 20
Additional Activities

A. Construct a valid argument justified by Rule of Proof by Cases.

B. Construct an invalid argument using Improper Transposition.

16 CO_Q2_General Mathematics SHS


Module 20
Module 20
CO_Q2_General Mathematics SHS 17
What I What's More Assessment
Know Activity 1.1 1. d
1. a 2. b
2. c 1.𝑝 → (𝑝 ∨ 𝑞) Rule of Addition 3. b
3. a 2.[(𝑝 → 𝑞) ∧ 𝑞] → 𝑝 Fallacy of the Converse 4. c
4. d 3.[~(𝑝 ∧ 𝑞) ∧ Denying a Conjunct 5. a
5. b (~𝑝)] → 𝑞 6. c
6. d 4.[(𝑝 ∨ 𝑞) ∧ (~𝑝)] → Rule of Disjunctive 7. d
7. b 8. b
𝑞 Syllogism
8. b 9. c
9. a 5.[(𝑝 → 𝑞) ∧ (~𝑞) → Modus Tollens
10. d
10. d (~𝑝) 11. c
11. c 12. a
12. c 13. d
13. d Activity 1.2 14. a
14. a 1.If there is a City Ordinance not allowing the use 15. a
15. a of items made from plastic, then I will encourage
others to use eco-friendly products. If plastic-free
items are beneficial to health, then I will encourage
others to use eco-friendly products. Therefore, if
there is a City Ordinance not allowing the use of
items made from plastic or if plastic-free items are
beneficial to health, then I will encourage others to
use eco-friendly products.
2.If it rains, then Rhea will use her umbrella. It does
not rain. Therefore, Rhea will not use her umbrella.
Answer Key
References

Dimasuy, Lynie, Alcala, Jeric, Palacio, Jane. General Mathematics. Quezon City,
Philippines: C & E Publishing, Inc. 2017. Pp. 183-184, 191

General Mathematics Learner’s Material. First Edition. 2016. pp. 270-282


*DepED Material: General Mathematics Learner’s Material

18 CO_Q2_General Mathematics SHS


Module 20
For inquiries or feedback, please write or call:

Department of Education - Bureau of Learning Resources (DepEd-BLR)

Ground Floor, Bonifacio Bldg., DepEd Complex


Meralco Avenue, Pasig City, Philippines 1600

Telefax: (632) 8634-1072; 8634-1054; 8631-4985

Email Address: blr.lrqad@deped.gov.ph * blr.lrpd@deped.gov.ph

19 CO_Q2_General Mathematics SHS


Module 20

You might also like